You are on page 1of 278

B ĐỀ THI TH THPT QUỐC GIA

2020 MÔN TIẾNG ANH CÁC TRƯỜNG


TRONG CẢ NƯỚC

(CÓ ĐÁP ÁN VÀ GI I CHI TI T)


SỞ GD & ĐT ĐỀ KIỂM TRA CHẤT LƯỢNG LẦN 1 – KHỐI 12
TRƯỜNG THPT HÀN THUYÊN NĂM HỌC: 2019 – 2020
50 câu trắc nghiệm MÔN: TIẾNG ANH
Thời gian làm bài: 60 phút; không kể thời gian phát đề

Blacken the letter A, B, C, or D on your answer sheet to indicate the correct answer to each of the
following questions.
Câu 1: Do you know that ________ longest river in ________ world is ________ Nile?
A. the/the/the B. a/the/the C. x/the/a D. the/the/x
Câu 2: Lightweight luggage enables you to manage easily even when fully ________.
A. crowded B. loaded C. carried D. packed
Câu 3: The revolving door remained ________ because Posey was pushing on it the wrong way.
A. station B. stationed C. stationary D. stationery
Câu 4: ________ at the airport more early, he would have met his friend.
A. Then he arrived B. Had he arrived C. When he arrived D. He has arrived
Câu 5: You must have forgotten to send the email, ________ there’s nothing in my inbox.
A. because of B. for C. due to D. owning to
Câu 6: The wine had made him a little ________ and couldn’t control his movement.
A. light-headed B. narrow-minded C. light-hearted D. light-footed
Câu 7: My little girl is tired now because she ________ all day.
A. studies B. studied C. has been studying D. has studied
Câu 8: The new director has really got things ________.
A. flying B. running C. jogging D. moving
Câu 9: ________________________, he left the hall quickly.
A. Not be rewarded with a smile B. Not rewarding with a smile
C. Not having rewarded with a smile D. Not being rewarded with a smile
Câu 10: We were ________ a mile of our destination when we ran out of petrol.
A. hardly B. only C. within D. inside
Câu 11: The forecast has revealed that the world’s reserves of fossil fuel will have ________ by 2015.
A. taken over B. caught up C. run out D. used off
Câu 12: I usually buy my clothes ________. It’s cheaper than going to the dress maker.
A. on the shelf B. on the house C. in public D. off the peg
Câu 13: As ________ in Greek and Roman mythology, harpies were frightful monsters that were half
woman and half bird.
A. description B. describing C. to describe D. described

Trang 1
Câu 14: Scientists have invented walls and windows that can block out the noise, which allows
individuals to focus on their work without ________.
A. be disturbed B. disturbing C. being disturbed D. being disturbing
Blacken the letter A, B, C, or D on your answer sheet to show the underlined part that needs correction
Câu 15: Televisions are now an everyday feature of most households in the United States, and television
viewing is the number of one activity leisure.
A. everyday B. activity leisure C. most households D. television viewing
Câu 16: New sources of energy have been looking for as the number of fossil fuels continues to decrease.
A. New sources B. been looking C. fossil D. continues
Câu 17: Almost poetry is more enjoyable when it is read aloud.
A. is more B. Almost C. it is D. enjoyable
Read the following passage and blacken the letter A, B, C, or D on your answer sheet to indicate the
correct answer to the following questions.
Foot racing is a popular activity in the United States. It is seen not only as a competitive sport but
also as a way to exercise, to enjoy the camaraderie of like-minded people, and to donate money to a
good cause. Though serious runners may spend months training to compete, other runners and walkers
might not train at all. Those not competing to win might run in an effort to beat their own time or simply
to enjoy the fun and exercise. People of all ages, from those of less than one year (who may be pushed in
strollers) to those in their eighties, enter into this sport. The races are held on city streets, on college
campuses, through parks, and in suburban areas, and they are commonly 5 to 10 kilometers in length.
The largest foot race in the world is the 12-kilometer Bay to Breakers race that is held in San Francisco
every spring. This race begins on the east side of the city near San Francisco Bay and ends on the west
side at the Pacific Ocean. There may be 80,000 or more people running in this race through the streets and
hills of San Francisco. In the front are the serious runners who compete to win and who might finish in as
little as 34 minutes. Behind them are the thousands 20 who take several hours to finish. In the back of the
race are those who dress in costumes and come just for fun. One year there was a group of men who
dressed like Elvis Presley, and another group consisted of firefighters who were tied together in a long
line and who were carrying a fire hose. There was even a bridal party, in which the bride was dressed in a
long white gown and the groom wore a tuxedo. The bride and groom threw flowers to bystanders, and
they were actually married at some point along the route.
Câu 18: Which of the following best describes the organization of this passage?
A. chronological order B. cause and result
C. statement and example D. specific to general
Câu 19: The main purpose of this passage is to ________________________.
A. describe a popular activity B. give reasons for the popularity of the foot races
C. make fun of runners in costume D. encourage people to exercise

Trang 2
Câu 20: In what sentence(s) does the author give reasons for why people enter foot races?
A. Foot racing … and exercise. B. Behind them … a fire horse.
C. The largest … 34 minutes. D. People of all ages … in length.
Câu 21: Which of the following is NOT implied by the author?
A. Foot races appeal to a variety of people
B. Walkers can compete for prizes
C. Entering a race is a way to give support to an organization
D. Running is a good way to strengthen the heart
Câu 22: The word “bystander” refers to __________.
A. a spectator B. a judge C. a participant D. a walker
Câu 23: The phrase “to a good cause” could be best replaced by which of the following?
A. to reward the winner B. to protect a wise investment
C. for a good purpose D. for an award
Câu 24: Which of the following is NOT mentioned in this passage?
A. Some runners were ready to put out a fire
B. Some runners looked like Elvis Presley
C. Some runners were participating in a wedding
D. Some runners were serious about winning
Câu 25: The word “camaraderie” as used in line 2 could be best replaced by which of the following
_______.
A. views B. companionship C. games D. jokes
Blacken the letter A, B, C, or D on your answer sheet to indicate the word or phrase that is OPPOSITE
in meaning to the underlined part in each of the following questions.
Câu 26: I think it’s impossible to abolish school examinations. They are necessary to evaluate students’
progress.
A. continue B. extinguish C. organize D. stop
Câu 27: He revealed his intentions of leaving the company to the manager during the office dinner party.
A. concealed B. disclosed C. misled D. influenced
Read the following passage and blacken the letter A, B, C, or D on your answer sheet to indicate the
correct word for each of the blanks in the following questions.
ABROAD WITH US
The company study abroad was originally founded in 1991 for students who wished to study French
in France. Its success led to the establishment of other schools around the world.
Learning the language in the country in which it is spoken is, of course, a (28) ________ more
effective and faster process than studying in your own country. It is also an opportunities to mix with
local habitants and (29) ________ a greater and deeper (30) ________ of the people whose language you

Trang 3
are studying. This makes the whole experience much more enjoyable. You may wish to attend an Italian
language and a cooking course in Italy or combine a Portuguese course with a sport. You should look at
all the possibilities and (31) ________ the best choice. In order to choose the (32) ________ course and
location for you, we suggest you contact us to discuss your particular requirements. From the information
you give us, we can make some recommendation.
Câu 28: A. lots B. far C. so D. very
Câu 29: A. acquire B. require C. inquire D. enquire
Câu 30: A. know B. knowledgeable C. knowledge D. knowledgeably
Câu 31: A. do B. make C. take D. get
Câu 32: A. exact B. right C. precise D. fit
Blacken the letter A, B, C, or D on your answer sheet to indicate the sentence that is closest in meaning
to each of the following questions.
Câu 33: “I’m sorry I gave you the wrong number”, said Paul to Susan.
A. Paul apologized to Susan for having given the wrong number.
B. Paul accused Susan of having given him the wrong number.
C. Paul thanked to Susan for giving the wrong number.
D. Paul denied giving Susan the wrong number.
Câu 34: I needn’t have watered the garden because it came down in torrents after that.
A. If only I had watered the garden before it came down in torrents.
B. It’s a pity I had watered the garden before it came down in torrents.
C. I regret having watered the garden before it came down in torrents.
D. If it had came down in torrents, I wouldn’t have watered the garden.
Câu 35: The students complained that the teacher was inexperienced.
A. The teacher was popular despite his inexperience.
B. The teacher was favored because of his inexperience.
C. The students praised the teacher for his inexperience.
D. The teacher was not supported for his inexperience.
Blacken the letter A, B, C, or D on your answer sheet to indicate the sentence that best combines each
pair of sentences in the following questions.
Câu 36: We gain more knowledge about how to stay safe online. We worry about the threats of
cybercrime less.
A. The more knowledge about how to stay safe online we gain, the more we worry about the threats of
cybercrime.
B. The more we stay online to gain safety knowledge, the less we worry about the threats of
cybercrime.

Trang 4
C. The more knowledge about how to stay safe online we gain, the less we worry about the threats of
cybercrime.
D. The more we know about how to stay safe online, we worry about the threats of cybercrime less.
Câu 37: We arrived at the airport. We realized our passports were still at home.
A. Not until we arrived at the airport, did we realize that our passports were still at home.
B. We arrived at the airport and realized that our passports are still at home.
C. Not until had we arrived at the airport, we realized our passports were still at home.
D. It was until we arrived at the airport that we realize our passports were still at home.
Blacken the letter A, B, C, or D on your answer sheet to indicate the word or phrase that is CLOSEST
in meaning to the underlined part in each of the following questions.
Trả lời cho các câu 38, 39 dưới đây:
Câu 38: He was sacked from his job after the manager discovered that he had stolen some money from
his colleagues.
A. dismissed B. ejected C. evicted D. dropped
Câu 39: She simply took it for granted that the check was good and did not ask him any questions about
it.
A. objected to it B. permitted it
C. looked it over D. accepted it without investigation
Blacken the letter A, B, C, or D on your answer sheet to indicate the word with the main stress
different from that of the other three words in each question.
Câu 40: A. escape B. before C. enough D. welfare
Câu 41: A. situation B. examination C. generation D. education
Read the following passage and blacken the letter A, B, C, or D on your answer sheet to indicate the
correct answer to the following questions.
Very few people in the modern world obtain their food supply by hunting and gathering in the
natural environment surrounding their homes. This method of harvesting from nature’s provision is the
oldest known subsistence strategy and has been practised for at least the last two million years. It was,
indeed, the only way to obtain food until rudimentary farming and the domestication of wild animals
were introduced about 10,000 years ago.
Because hunter-gatherers have fared poorly in comparison with their agricultural cousins, their
numbers have dwindled, and they have been forced to live in marginal environments, such as deserts and
arctic wastelands. In higher latitudes, the shorter growing seasons have restricted the availability of plant
life. Such conditions have caused a greater dependence on hunting, and on fishing along the coasts and
waterways. The abundance of vegetation in the lower latitudes of the tropics, on the other hand, has
provided a greater opportunity for gathering a variety of plants. In short, the environmental differences
have restricted the diet and have limited possibilities for the development of subsistence societies.

Trang 5
Contemporary hunter-gatherers may help us understand our prehistoric ancestors. We know from
the observation of modern hunter-gatherers in both Africa and Alaska that a society based on hunting and
gathering must be very mobile. While the entire community camps in a central location, a smaller party
harvests the food within a reasonable distance from the camp. When the food in the area has become
exhausted, the community moves on to exploit another site. We also notice seasonal migration patterns
evolving for most hunter-gatherers, along with a strict division of labor between the sexes. These patterns
of behavior may be similar to those practised by mankind during the Paleolithic Period.
Câu 42: The word “domestication” in the first paragraph mostly means ________.
A. making wild animals used to living with and working for humans
B. hatching and raising new species of wild animals in the home
C. teaching animals to do a particular job or activity in the home
D. adapting animals to suit a new working environment
Câu 43: According to the passage, subsistence societies depend mainly on ________.
A. hunter-gatherers’ tools B. agricultural products
C. farming methods D. nature’s provision
Câu 44: In the lower latitudes of the tropics, hunter-gatherers ________.
A. can free themselves from hunting B. live along the coasts and waterways for fishing
C. have better food gathering from nature D. harvest shorter seasonal crops
Câu 45: typical feature of both modern and prehistoric hunter-gatherers is that ________.
A. they live in the forests for all their life
B. they don’t have a healthy and balanced diet
C. they don’t have a strong sense of community
D. they often change their living places
Câu 46: Which of the following would serve as the best title of the passage?
A. Evolution of Humans’ Farming Methods B. Hunter-gatherers and Subsistence Societies
C. A Brief History of Subsistence Farming D. Hunter-gatherers: Always on the Move
Blacken the letter A, B, C, or D on your answer sheet to indicate the word whose underlined part is
pronounced differently from the other three in each question.
Câu 47: A. wicked B. beloved C. learned D. cooked
Câu 48: A. shame B. drank C. cable D. blaze
Blacken the letter A, B, C, or D on your answer sheet to indicate the correct answer to each of the
following questions.
Câu 49: Joanne: “You should have flown with the earlier flight!”
Alex: “____________.”
A. Why not? B. Oh, I’m sorry to hear that
C. It was fully booked D. Yes. I did

Trang 6
Câu 50: Helen and Mary are talking about Mary’s result at school.
Helen: “Your parents must be proud of your result at school!”
Mary: “____________.”
A. Of course B. I’m glad you like it
C. Sorry to hear that D. Thanks. It’s certainly encouraging
Đáp án
1-D 2-D 3-C 4-B 5-B 6-A 7-C 8-D 9-D 10-C
11-C 12-D 13-D 14-C 15-B 16-B 17-B 18-C 19-A 20-A
21-D 22-A 23-C 24-A 25-B 26-A 27-A 28-B 29-A 30-C
31-B 32-B 33-A 34-C 35-D 36-C 37-A 38-A 39-D 40-D
41-B 42-A 43-D 44-C 45-D 46-B 47-D 48-B 49-C 50-D

LỜI GIẢI CHI TIẾT


Câu 1: Đáp án D
Phương pháp giải:
Kiến thức: Mạo từ
Giải chi tiết:
Mạo từ “the” dùng trong cấu trúc:
- So sánh nhất: the + adj-est/ most adj
- the + N (được cho là duy nhất): the world, the sun, the moon, …
Không dùng mạo từ trước tên núi, hồ, đường phố.
Tạm dịch: Bạn có biết rằng con sông dài nhất thế giới là sông Nin không?
Câu 2: Đáp án D
Phương pháp giải:
Kiến thức: Từ vựng
Giải chi tiết:
A. crowded (adj): đông đúc, chật ních
B. loaded (adj): nặng nề, gay go
C. carried (adj): được mang theo
D. packed (adj): nhồi nhét
Tạm dịch: Hành lý nhẹ cho phép bạn quản lý dễ dàng ngay cả khi nó được nhồi nhét đầy đồ.
Câu 3: Đáp án C
Phương pháp giải:
Kiến thức: Từ loại
Giải chi tiết:
A. station (n): trạm, điểm, đồn

Trang 7
B. stationed => không tồn tại
C. stationary (adj): đứng ở một chỗ, không chuyển động
D. stationery (n): đồ dùng văn phòng
Dấu hiệu: sau động từ “remained” (vẫn) cần một tính từ
Tạm dịch: Cánh cửa quay vẫn đứng yên vì Posey đang đẩy nó sai cách.
Câu 4: Đáp án B
Phương pháp giải:
Kiến thức: Đảo ngữ câu điều kiện loại 3
Giải chi tiết:
- Dấu hiệu: mệnh đề chính chia “would have + P2”
- Cách dùng: Câu điều kiện loại 3 dùng để diễn tả một giả thiết trái ngược với thực tế đã xảy ra ở quá khứ
- Công thức: If S + had + Ved/ V3 + O, S + would/ could + have + Ved/ V3 + O.
- Đảo ngữ câu điều kiện loại 3: Had + S + (not) + Ved/ V3, S + would/ could have + Ved/ V3.
Tạm dịch: Nếu anh ấy đến sân bay sớm hơn, thì anh ấy đã gặp bạn anh ấy rồi.
Câu 5: Đáp án B
Phương pháp giải:
Kiến thức: Liên từ
Giải chi tiết:
A. because of + N/ Ving: bởi vì
B. for S + V: vì, bởi vì
C. due to + N/ Ving: bởi vì
D. owning to + N/ Ving: bởi vì
“there’s nothing in my box” là một cụm S – V => loại A, C, D
Tạm dịch: Bạn chắc hẳn là đã quên gửi thư điện tử, vì không có gì trong hộp thư đến của tôi.
Câu 6: Đáp án A
Phương pháp giải:
Kiến thức: Từ vựng
Giải chi tiết:
A. light-headed (adj): không tỉnh táo
B. narrow-minded (adj): hẹp hòi, nhỏ nhen
C. light-hearted (adj): vui vẻ, vô tư
D. light-footed (adj): nhanh chân, nhanh nhẹn
Tạm dịch: Rượu đã làm cho anh ta mất tỉnh táo và không thể tự kiểm soát chuyển động của chính mình.
Câu 7: Đáp án C
Phương pháp giải:
Kiến thức: Thì hiện tại hoàn thành tiếp diễn

Trang 8
Giải chi tiết:
- Dấu hiệu: “all day” (suốt cả ngày)
- Cách dùng: Thì hiện tại hoàn thành tiếp diễn được sử dụng để nói về những hành động xảy ra trong quá
khứ nhưng vẫn còn tiếp tục ở hiện tại - nhấn mạnh tính liên tục của hành động - và có thể vẫn còn tiếp
diễn trong tương lai.
- Công thức: S + have/ has been + Ving.
Tạm dịch: Cô bé của tôi bây giờ mệt mỏi vì học suốt cả ngày.
Câu 8: Đáp án D
Phương pháp giải:
Kiến thức: Thành ngữ
Giải chi tiết:
A. flying (adj): bay, biết bay, mau chóng
B. running (adj): tiến hành trong lúc chạy, di động
C. jogging (n): sự đẩy nhẹ, sự lắc nhẹ
D. moving (adj): cảm động, làm mủi lòng
=> get things moving: tạo ra sự tiến bộ
Tạm dịch: Giám đốc mới đã thực sự tạo ra những sự tiến bộ.
Câu 9: Đáp án D
Phương pháp giải:
Kiến thức: Rút gọn mệnh đề trạng ngữ
Giải chi tiết:
Khi có 2 chủ ngữ giống nhau => rút gọn:
- dùng Ving: khi mệnh đề mang nghĩa chủ động
- dùng Not being V_ed/V cột 3: khi mệnh đề mang nghĩa bị động (đứng đầu câu)
Câu đầy đủ: When he wasn’t rewarded with a smile, he left the hall quickly.
Câu rút gọn: Not being rewarded with a smile, he left the hall quickly.
Tạm dịch: Không được thưởng với một nụ cười, anh nhanh chóng rời khỏi hội trường.
Câu 10: Đáp án C
Phương pháp giải:
Kiến thức: Từ vựng
Giải chi tiết:
A. hardly (adv): khắc nghiệt, khó khăn, vừa mới, hầu như không => không phù hợp nghĩa câu
B. only (adv): chỉ => không phù hợp nghĩa câu
C. within (adj): trong vòng
D. inside (adj): ở trong, trong nội bộ => không phù hợp nghĩa câu
Tạm dịch: Chúng tôi đã đi trong vòng một dặm đến đích đến khi hết xăng.

Trang 9
Câu 11: Đáp án C
Phương pháp giải:
Kiến thức: Cụm động từ
Giải chi tiết:
A. taken over: chịu trách nhiệm điều hành
B. caught up: làm bù để bắt kịp công việc đã lỡ, bắt kịp, đuổi kịp ai
C. run out: hết, cạn sạch
D. used off => không tồn tại
Tạm dịch: Dự báo đã tiết lộ rằng trữ lượng nhiên liệu hóa thạch trên thế giới sẽ hết vào năm 2015.
Câu 12: Đáp án D
Phương pháp giải:
Kiến thức: Thành ngữ
Giải chi tiết:
A. on the shelf : xếp xó, bỏ đi
B. on the house: miễn phí, không tính tiền
C. in public: giữa công chúng, công khai
D. off the peg: may sẵn (quần áo)
Tạm dịch: Tôi thường mua quần áo may sẵn. Nó rẻ hơn so với việc đến chỗ may đo quần áo.
Câu 13: Đáp án D
Phương pháp giải:
Kiến thức: Rút gọn mệnh đề trạng ngữ
Giải chi tiết:
Khi có 2 chủ ngữ giống nhau => rút gọn:
- dùng Ving: khi mệnh đề mang nghĩa chủ động
- dùng V_ed/V cột 3: khi mệnh đề mang nghĩa bị động
Và giữ lại trạng từ.
Câu đầy đủ: As harpies were described in Greek and Roman mythology, they were frightful monsters
that were half woman and half bird.
Câu rút gọn: As described in Greek and Roman mythology, they were frightful monsters that were half
woman and half bird.
Tạm dịch: Theo được mô tả trong thần thoại Hy Lạp và La Mã, những con yêu quái là những con quái
vật đáng sợ là một nửa phụ nữ và một nửa con chim.
Câu 14: Đáp án C
Phương pháp giải:
Kiến thức: to V/ Ving
Giải chi tiết:

Trang 10
without + V_ing: không làm gì
without + being Ved/ V cột 3: không bị làm gì
Tạm dịch: Các nhà khoa học đã phát minh ra các bức tường và cửa sổ có thể ngăn chặn tiếng ồn, cho
phép các cá nhân tập trung vào công việc của họ mà không bị làm phiền.
Câu 15: Đáp án B
Phương pháp giải:
Kiến thức: Từ vựng
Giải chi tiết:
leisure activity: hoạt động lúc rảnh rỗi
Sửa: activity leisure => leisure activity
Tạm dịch: Ti vi là một đặc trưng hàng ngày của hầu hết các hộ gia đình ở Hoa Kỳ và xem truyền hình là
một trong những hoạt động giải trí.
Câu 16: Đáp án B
Phương pháp giải:
Kiến thức: Câu bị động
Giải chi tiết:
- Dấu hiệu: “New sources of energy” (Các nguồn năng lượng mới) chịu tác động của hành động “look”
(tìm thấy)
- Câu bị động thì hiện tại hoàn thành: S + have/ has been + Ved/ V3.
Sửa: been looking => been looked
Tạm dịch: Các nguồn năng lượng mới đã được tìm kiếm vì số lượng nhiên liệu hóa thạch tiếp tục giảm.
Câu 17: Đáp án B
Phương pháp giải:
Kiến thức: Từ loại
Giải chi tiết:
- Dấu hiệu: trước danh từ “poetry” (thơ ca) cần một tính từ
- Almost (adv): hầu như, gần như => bổ nghĩa cho động từ, tính từ hoặc một trạng từ khác.
Almost thường đi với: anybody, anything, no one, nobody, all, everybody, every.
- Most (adj): hầu hết, phần lớn, đa số => chỉ bổ nghĩa cho danh từ
Most + N (không xác định)
Sửa: Almost => Most
Tạm dịch: Hầu hết thơ ca đều thú vị hơn khi được đọc to.
Câu 18: Đáp án C
Phương pháp giải:
Kiến thức: Đọc hiểu
Giải chi tiết:

Trang 11
Điều nào sau đây mô tả đúng nhất về tổ chức của đoạn văn này?
A. thứ tự thời gian
B. nguyên nhân và kết quả
C. trình bày và ví dụ
D. cụ thể cho chung
Thông tin: Foot racing is a popular activity in the United States… The largest foot race in the world is
the 12-kilometer Bay to Breakers race that is held in San Francisco every spring.
Tạm dịch: Chạy đua là một hoạt động phổ biến ở Hoa Kỳ… Cuộc chạy đua lớn nhất thế giới là cuộc đua
Bay tới Breakers dài 12 km được tổ chức tại San Francisco vào mỗi mùa xuân.
Câu 19: Đáp án A
Phương pháp giải:
Kiến thức: Đọc hiểu
Giải chi tiết:
Mục đích chính của đoạn văn này là để _______________.
A. mô tả một hoạt động phổ biến
B. đưa ra lý do cho sự phổ biến của các cuộc chạy đua
C. tạo niềm vui cho người chạy bộ mặc trang phục (theo phong cách nào đó)
D. khuyến khích mọi người tập thể dục
Thông tin: Foot racing is a popular activity in the United States.
Tạm dịch: Chạy đua là một hoạt động phổ biến ở Hoa Kỳ.
Câu 20: Đáp án A
Phương pháp giải:
Kiến thức: Đọc hiểu
Giải chi tiết:
Trong câu nào tác giả đưa ra lý do tại sao mọi người tham gia cuộc chạy đua?
A. Chạy đua … và tập thể dục.
B. Đằng sau họ … một con ngựa lửa.
C. Trận đấu lớn nhất … 34 phút.
D. Mọi người ở mọi lứa tuổi … chiều dài.
Thông tin: Foot racing is a popular activity in the United States. It is seen not only as a competitive sport
but also as a way to exercise, to enjoy the camaraderie of like-minded people, and to donate money to a
good cause. Though serious runners may spend months training to compete, other runners and walkers
might not train at all. Those not competing to win might run in an effort to beat their own time or simply
to enjoy the fun and exercise.
Tạm dịch: Chạy đua là một hoạt động phổ biến ở Hoa Kỳ. Nó không chỉ được coi là một môn thể thao
cạnh tranh mà còn là một cách để tập thể dục, để tận hưởng tình bạn của những người cùng chí hướng và

Trang 12
quyên góp tiền cho một mục đích chính đáng. Mặc dù những người chạy bộ nghiêm túc có thể dành nhiều
tháng tập luyện để thi đấu, nhưng cũng có những người chạy bộ và đi bộ khác có thể không tập luyện gì
cả. Những người không cạnh tranh để giành chiến thắng có thể chạy trong một nỗ lực để đánh bại thời
gian của riêng họ hoặc chỉ đơn giản là để tận hưởng niềm vui và tập thể dục.
Câu 21: Đáp án D
Phương pháp giải:
Kiến thức: Đọc hiểu
Giải chi tiết:
Điều nào sau đây KHÔNG được tác giả ngụ ý?
A. Cuộc chạy đua thu hút nhiều người
B. Người đi bộ có thể cạnh tranh để giành giải thưởng
C. Tham gia một cuộc đua là một cách để hỗ trợ cho một tổ chức
D. Chạy là một cách tốt để giữ cho một trái tim khỏe
Thông tin: It is seen not only as a competitive sport but also as a way to exercise, to enjoy the
camaraderie of like-minded people, and to donate money to a good cause… People of all ages, from those
of less than one year (who may be pushed in strollers) to those in their eighties, enter into this sport.
Tạm dịch: Nó không chỉ được coi là một môn thể thao cạnh tranh mà còn là một cách để tập thể dục, để
tận hưởng tình bạn của những người cùng chí hướng và quyên góp tiền cho một mục đích chính đáng…
Mọi người ở mọi lứa tuổi, từ những bé dưới một tuổi (có thể được đẩy trong xe đẩy) đến những người ở
độ tuổi tám mươi, tham gia môn thể thao này.
Câu 22: Đáp án A
Phương pháp giải:
Kiến thức: Đọc hiểu
Giải chi tiết:
Từ “bystander” chỉ __________.
bystander (n): khán giả, người đứng xem
A. a spectator: một khán giả
B. a judge: một thẩm phán
C. a participant: một người tham gia
D. a walker: một người đi bộ
Thông tin: The bride and groom threw flowers to bystanders, and they were actually married at some
point along the route.
Tạm dịch: Cô dâu và chú rể ném hoa cho người đứng xem, và họ thực sự đã kết hôn tại một số điểm dọc
theo tuyến đường.
Câu 23: Đáp án C
Kiến thức: Đọc hiểu

Trang 13
Giải chi tiết:
Cụm từ “to a good cause” có thể được thay thế bằng cái nào sau đây?
to a good cause: cho một mục đích chính đáng
A. để thưởng cho người chiến thắng
B. để bảo vệ một khoản đầu tư khôn ngoan
C. cho một mục đích tốt
D. cho một giải thưởng
Thông tin: It is seen not only as a competitive sport but also as a way to exercise, to enjoy the
camaraderie of like-minded people, and to donate money to a good cause.
Tạm dịch: Nó không chỉ được coi là một môn thể thao cạnh tranh mà còn là một cách để tập thể dục, để
tận hưởng tình bạn của những người cùng chí hướng và quyên góp tiền cho một mục đích chính đáng.
Câu 24: Đáp án A
Kiến thức: Đọc hiểu
Giải chi tiết:
Điều nào sau đây KHÔNG được đề cập trong đoạn văn này?
A. Một số vận động viên đã sẵn sàng để dập lửa
B. Một số vận động viên trông giống như Elvis Presley
C. Một số vận động viên đang tham gia một đám cưới
D. Một số vận động viên nghiêm túc về chiến thắng
Thông tin: In the front are the serious runners who compete to win and who might finish in as little as 34
minutes… ne year there was a group of men who dressed like Elvis Presley, and another group consisted
of firefighters who were tied together in a long line and who were carrying a fire hose. There was even a
bridal party, in which the bride was dressed in a long white gown and the groom wore a tuxedo. The bride
and groom threw flowers to bystanders, and they were actually married at some point along the route.
Tạm dịch: Ở phía trước là những vận động viên nghiêm túc thi đấu để giành chiến thắng và có thể hoàn
thành trong ít nhất 34 phút… Một năm nọ, có một nhóm những người đàn ông ăn mặc như Elvis Presley
và một nhóm khác gồm các nhân viên cứu hỏa bị trói chặt vào nhau thành một hàng dài và đang mang
theo vòi cứu hỏa. Thậm chí còn có một bữa tiệc cô dâu, trong đó cô dâu mặc một chiếc váy dài màu trắng
và chú rể mặc một bộ tuxedo. Cô dâu và chú rể ném hoa cho những người đứng xem, và họ thực sự đã kết
hôn tại một số điểm dọc theo tuyến đường.
Câu 25: Đáp án B
Kiến thức: Đọc hiểu
Giải chi tiết:
Từ “camaraderie” được sử dụng trong dòng 2 có thể được thay thế tốt nhất bằng __________ .
camaraderie (n): tình bạn
A. views (n): tầm nhìn, quan điểm

Trang 14
B. companionship (n): tình bạn
C. games (n): trò chơi
D. jokes (n): chuyện cười, chuyện đùa
Thông tin: It is seen not only as a competitive sport but also as a way to exercise, to enjoy the
camaraderie of like-minded people, and to donate money to a good cause.
Tạm dịch: Nó không chỉ được coi là một môn thể thao cạnh tranh mà còn là một cách để tập thể dục, để
tận hưởng tình bạn của những người cùng chí hướng và quyên góp tiền cho một mục đích chính đáng.
Câu 26: Đáp án A
Kiến thức: Từ trái nghĩa
Giải chi tiết:
abolish (v): thủ tiêu, bãi bỏ, hủy bỏ
A. continue (v): tiếp tục, duy trì
B. extinguish (v): dập tắt, làm tiêu tan
C. organize (v): tổ chức
D. stop (v): ngừng, dừng lại
=> abolish >< continue
Tạm dịch: Tôi nghĩ rằng không thể hủy bỏ các kỳ thi ở trường. Chúng cần thiết để đánh giá sự tiến bộ
của sinh viên.
Câu 27: Đáp án A
Giải chi tiết:
reveal - revealed - revealed: bộc lộ, tiết lộ
A. conceal - concealed - concealed: giấu giếm, che đậy
B. disclose - disclosed - disclosed: vạch trần, phơi bày
C. mislead - misled - misled: làm cho lạc đường, làm cho lạc lối
D. influence - influenced - influenced: ảnh hưởng, tác động
=> reveal >< conceal
Tạm dịch: Anh ta tiết lộ ý định rời công ty cho người quản lý trong bữa tiệc tối tại văn phòng.
Câu 28: Đáp án B
Kiến thức: So sánh hơn
Giải chi tiết:
Công thức nhấn mạnh trong so sánh hơn: far/ much + more adj/ adj-er
Learning the language in the country in which it is spoken is, of course, a (28) far more effective and
faster process than studying in your own country.
Tạm dịch: Tất nhiên, học ngôn ngữ ở quốc gia nơi nó được nói là một quá trình hiệu quả và nhanh hơn
nhiều so với học tập tại đất nước của bạn.
Câu 29: Đáp án A

Trang 15
Kiến thức: Từ vựng
Giải chi tiết:
A. acquire (v): đạt được, giành được
B. require (v): đòi hỏi, yêu cầu, quy định
C. inquire (v): điều tra, hỏi thăm, hỏi mua
D. enquire (v): điều tra, hỏi thăm, hỏi mua
It is also an opportunities to mix with local habitants and (29) acquire
Tạm dịch: Đây cũng là một cơ hội để hòa hợp với các cư dân địa phương và có được
Câu 30: Đáp án C
Kiến thức: Từ loại
Giải chi tiết:
Dấu hiệu: sau tính từ ở dạng so sánh hơn “greater and deeper” (sâu rộng hơn) cần một danh từ
A. know (v): biết, hiểu
B. knowledgeable (adj): thông thạo, thành thạo
C. knowledge (n): kiến thức, sự hiểu biết
D. knowledgeably (adv): thông thạo, thành thạo
It is also an opportunities to mix with local habitants and acquire a greater and deeper (30) knowledge of
the people whose language you are studying.
Tạm dịch: Đây cũng là một cơ hội để hòa hợp với các cư dân địa phương và có được kiến thức sâu rộng
hơn về những người mà bạn đang học ngôn ngữ.
Câu 31: Đáp án B
Kiến thức: Từ vựng
Giải chi tiết:
A. do (v): làm, hành động
B. make (v): chế tạo, sản xuất
C. take (v): cầm, lấy
D. get (v): có được, lấy được
=> make choice: lựa chọn
You should look at all the possibilities and (31) make the best choice.
Tạm dịch: Bạn nên xem xét tất cả các khả năng và đưa ra lựa chọn tốt nhất.
Câu 32: Đáp án B
Kiến thức: Từ vựng
Giải chi tiết:
A. exact (adj): chính xác (trong mọi chi tiết)
B. right (adj): thích hợp
C. precise (adj): rõ ràng và chính xác

Trang 16
D. fit (adj): phù hợp (về kích cỡ, loại)
In order to choose the (32) right course and location for you, we suggest you contact us to discuss your
particular requirements.
Tạm dịch: Để chọn khóa học và địa điểm thích hợp cho bạn, chúng tôi khuyên bạn nên liên hệ với chúng
tôi để thảo luận về các yêu cầu cụ thể của bạn.
Câu 33: Đáp án A
Kiến thức: Câu tường thuật
Giải chi tiết:
Câu trực tiếp: “I’m sorry + I + V_ed/V cột 2”, S1 said to S2.
Câu gián tiếp: S1 apologized (to S2) for having + Ved/ V cột 3: Ai đó xin lỗi ai vì đã làm gì
Tạm dịch: “Tôi xin lỗi tôi đã đưa nhầm số cho bạn”, Paul nói với Susan.
= Paul xin lỗi Susan vì đã đưa nhầm số.
B. Paul buộc tội Susan đã đưa nhầm số cho anh ta. => sai về nghĩa
C. Paul cảm ơn Susan vì đã đưa nhầm số. => sai về nghĩa
D. Paul từ chối đưa cho Susan số sai. => sai về nghĩa
Câu 34: Đáp án C
Kiến thức: Cấu trúc đồng nghĩa
Giải chi tiết:
- Câu ước ở quá khứ dùng để diễn tả những mong ước về một sự việc không có thật ở quá khứ, hay giả
định một điều ngược lại so với thực tại đã xảy ra ở quá khứ, thường là để diễn tả sự nuối tiếc với tình
huống ở quá khứ.
Công thức chung: S + wish(es)/ If only + S + had + Ved/ PII + O.
- regret + Ving/ having Ved/ V3: hối tiếc vì đã làm gì
Tạm dịch: Tôi đáng lẽ không cần tưới nước cho khu vườn bởi vì trời đổ mưa ngay sau đó.
= Tôi rất tiếc vì đã tưới nước cho khu vườn trước khi trời đổ mưa.
A. Tôi ước tôi đã tưới nước cho khu vườn trước khi trời đổ mưa. => sai về nghĩa
B. Sai thì. Sau “it’s a pity” (thật tiếc) không dùng thì quá khứ hoàn thành
D. Nếu trời đã đổ mưa, thì tôi sẽ không phải tưới nước cho khu vườn. => sai về nghĩa
Câu 35: Đáp án D
Kiến thức: Cấu trúc đồng nghĩa
Giải chi tiết:
complain (v): phàn nàn
= tobe not supported: không được ủng hộ
Tạm dịch: Học sinh phàn nàn rằng giáo viên thiếu kinh nghiệm.
= Giáo viên không được ủng hộ vì thiếu kinh nghiệm.
A. Giáo viên nổi tiếng mặc dù thiếu kinh nghiệm. => sai về nghĩa

Trang 17
B. Giáo viên được ưa thích vì thiếu kinh nghiệm. => sai về nghĩa
C. Các sinh viên khen ngợi giáo viên về sự thiếu kinh nghiệm. => sai về nghĩa
Câu 36: Đáp án C
Kiến thức: So sánh kép
Giải chi tiết:
Công thức: The + more (adj/ adj-er/N) + S1 + V1, the + more (adj/ adj-er/N) + S2 + V2. càng… càng
Tạm dịch: Chúng tôi có thêm kiến thức về cách giữ an toàn trực tuyến. Chúng tôi lo lắng về các mối đe
dọa của tội phạm mạng ít hơn.
= Chúng tôi càng có nhiều kiến thức về cách giữ an toàn trực tuyến, chúng tôi càng ít lo lắng về các mối
đe dọa của tội phạm mạng.
A. Chúng tôi càng có nhiều kiến thức về cách giữ an toàn trực tuyến, chúng tôi càng lo lắng về các mối đe
dọa của tội phạm mạng. => sai về nghĩa
B. Chúng ta càng trực tuyến để có được kiến thức về an toàn, chúng ta càng ít lo lắng về các mối đe dọa
của tội phạm mạng. => sai về nghĩa
D. Sai công thức.
Câu 37: Đáp án A
Kiến thức: Đảo ngữ
Giải chi tiết:
- Công thức: It is/ was not until + trạng từ chỉ thời gian/mệnh đề + that... + S + V: Mãi cho đến….. thì…
- Đảo ngữ: Not until + S + V/ trạng từ chỉ thời gian + V(trợ) + S + V(chính): Mãi cho đến khi…
Tạm dịch: Chúng tôi đến sân bay. Chúng tôi nhận ra hộ chiếu của chúng tôi vẫn ở nhà.
= Mãi cho đến khi chúng tôi đến sân bay thì chúng tôi mới nhận ra rằng hộ chiếu của chúng tôi vẫn ở nhà.
B. Sai thì động từ: “are”
C. Sai thì động từ: “had we arrived”
D. Sai công thức.
Câu 38: Đáp án A
Kiến thức: Từ đồng nghĩa
Giải chi tiết:
sack - sacked - sacked: sa thải, cách chức
A. dismiss - dismissed - dismissed: giải tán, sa thải
B. eject - ejected - ejected: đuổi ra, tống ra
C. evict - evicted - evicted: đuổi
D. drop - dropped - dropped: rớt xuống, gục xuống
=> sack (v) = dismiss (v): sa thải, đuổi việc
Tạm dịch: Anh ta bị đuổi việc sau khi người quản lý phát hiện ra rằng anh ta đã ăn cắp một số tiền từ các
đồng nghiệp của mình.

Trang 18
Câu 39: Đáp án D
Kiến thức: Thành ngữ
Giải chi tiết:
took it for granted: cho điều gì là đúng, cho là hiển nhiên
A. objected to it: phản đối nó
B. permitted it: cho phép nó
C. looked it over: điều tra nó
D. accepted it without investigation: chấp nhận nó mà không cần điều tra
=> took it for granted = accepted it without investigation: cho điều gì là đúng, cho là hiển nhiên
Tạm dịch: Cô ấy đơn giản cho rằng việc kiểm tra là tốt và không hỏi anh ta bất kỳ câu hỏi nào về việc
đó.
Câu 40: Đáp án D
Kiến thức: Trọng âm từ có 2 âm tiết
Giải chi tiết:
A. escape /ɪˈskeɪp/
B. before /bɪˈfɔːr/
C. enough /ɪˈnʌf/
D. welfare /ˈwelfer/
Quy tắc:
- Những động từ có 2 âm tiết thường có trọng âm rơi vào âm tiết thứ hai.
- Những danh từ, tính từ có 2 âm tiết thường có trọng âm rơi vào âm tiết thứ nhất.
Trọng âm đáp án D rơi vào âm tiết thứ nhất, còn lại là âm thứ hai
Câu 41: Đáp án B
Kiến thức: Trọng âm từ có 4, 5 âm tiết
Giải chi tiết:
A. situation /sɪtʃuˈeɪʃn/
B. examination /ɪɡzæmɪˈneɪʃn/
C. generation /dʒenəˈreɪʃn/
D. education /edʒuˈkeɪʃn/
Quy tắc: Những từ có tận cùng là đuôi “-ion” có trọng âm rơi vào âm tiết đứng ngay trước nó.
Trọng âm đáp án B rơi vào âm tiết thứ tư, còn lại là âm thứ ba
Câu 42: Đáp án A
Kiến thức: Đọc hiểu
Giải chi tiết:
Từ “domestication” trong đoạn đầu tiên có nghĩa là ___________.
domestication (n): sự thuần hóa

Trang 19
A. làm cho động vật hoang dã quen với việc sống và làm việc cho con người
B. ấp nở và nuôi các loài động vật hoang dã mới trong nhà
C. dạy động vật làm một công việc hoặc hoạt động cụ thể trong nhà
D. huấn luyện các con vật để làm việc trong môi trường mới
Thông tin: It was, indeed, the only way to obtain food until rudimentary farming and the domestication
of wild animals were introduced about 10,000 years ago.
Tạm dịch: Thực sự, đó là cách duy nhất để có được thức ăn cho đến khi canh tác nông nghiệp và thuần
hóa động vật hoang dã đã ra đời cách đây khoảng 10.000 năm trước.
Câu 43: Đáp án D
Kiến thức: Đọc hiểu
Giải chi tiết:
Theo đoạn văn, các phương kế sinh nhai khác phụ thuộc chủ yếu vào ________.
A. công cụ của người săn bắn, hái lượm
B. nông sản
C. phương pháp canh tác
D. nguồn cung cấp tự nhiên
Thông tin: This method of harvesting from nature’s provision is the oldest known subsistence strategy
and has been practised for at least the last two million years.
Tạm dịch: Phương pháp thu hoạch này từ nguồn cung cấp tự nhiên là phương kế sinh nhai lâu đời nhất
được biết đến và đã được thực hiện trong ít nhất hai triệu năm qua.
Câu 44: Đáp án C
Kiến thức: Đọc hiểu
Giải chi tiết:
Ở các vĩ độ thấp hơn của vùng nhiệt đới, người săn bắt hái lượm ________.
A. có thể tự thoát khỏi việc săn bắn
B. sống dọc theo bờ biển và kênh rạch để câu cá
C. thu thập lương thực tốt hơn từ thiên nhiên
D. thu hoạch mùa vụ ngắn hơn
Thông tin: The abundance of vegetation in the lower latitudes of the tropics, on the other hand, has
provided a greater opportunity for gathering a variety of plants.
Tạm dịch: Mặt khác, sự phong phú của thảm thực vật ở những vùng vĩ độ thấp của vùng nhiệt đới lại
cung cấp một cơ hội lớn hơn cho việc hái lượm nhiều loại thực vật khác nhau.
Câu 45: Đáp án D
Kiến thức: Đọc hiểu
Giải chi tiết:
Một đặc điểm điển hình của cả những người săn bắn hái lượm thời tiền sử và hiện đại là ________.

Trang 20
A. họ sống trong rừng suốt đời
B. họ không có chế độ ăn uống cân bằng và lành mạnh
C. họ không có tính gắn kết cộng đồng
D. họ thường thay đổi nơi ở
Thông tin: We know from the observation of modern hunter-gatherers in both Africa and Alaska that a
society based on hunting and gathering must be very mobile. … When the food in the area has become
exhausted, the community moves on to exploit another site. We also notice seasonal migration patterns
evolving for most hunter-gatherers, along with a strict division of labor between the sexes.
Tạm dịch: Chúng ta biết điều này từ việc quan sát những người săn bắn hái lượm hiện đại ở cả Châu Phi
và Alaska rằng một xã hội dựa trên săn bắn và hái lượm phải rất linh động. … Khi thực phẩm trong khu
vực đã cạn kiệt, họ chuyển sang khai thác ở một vùng khác. Chúng ta cũng nhận ra những người săn bắn,
hái lượm thường di cư theo mùa, cùng với sự phân công lao động nghiêm ngặt giữa hai giới.
Câu 46: Đáp án B
Kiến thức: Đọc hiểu
Giải chi tiết:
Điều nào sau đây sẽ là tiêu đề phù hợp nhất của bài đọc?
A. Sự phát triển phương pháp canh tác của loài người
B. Người săn bắt hái lượm và phương kế sinh nhai
C. Sơ lược về lịch sử sinh hoạt nông nghiệp
D. Người săn bắt hái lượm: Luôn luôn di chuyển
Câu 47: Đáp án D
Kiến thức: Phát âm “-ed”
Giải chi tiết:
A. wicked /ˈwɪkɪd/
B. beloved /bɪˈlʌvɪd/
C. learned /ˈlɜːrnɪd/
D. cooked /kʊkt/
Quy tắc:
Cách phát âm đuôi “ed”:
- Đuôi “ed” được phát âm là /ɪd/ khi động từ có phát âm kết thúc là /t/ hay /d/
- Đuôi “ed” được phát âm là /t/ khi động từ có phát âm kết thúc là /s/,/f/,/p/,/ʃ/,/tʃ/,/k/
- Đuôi “ed” được phát âm là /d/ với các trường hợp còn lại
Lưu ý: Một số từ kế thúc bằng “-ed” được dùng làm tính từ, đuôi “-ed” được phát âm là /ɪd/: aged,
blessed crooked, dogged, naked, learned, ragged, wicked, wretched, beloved, …
Phần gạch chân đáp án D phát âm là /t/, còn lại là /ɪd/
Câu 48: Đáp án B

Trang 21
Kiến thức: Phát âm “a”
Giải chi tiết:
A. shame /ʃeɪm/
B. drank /dræŋk/
C. cable /ˈkeɪbl/
D. blaze /bleɪz/
Phần gạch chân đáp án B phát âm là /æ/, còn lại là /eɪ/
Câu 49: Đáp án C
Kiến thức: Ngôn ngữ giao tiếp
Giải chi tiết:
Joanne: “Bạn đang lẽ ra nên bay chuyến sớm hơn!”
Alex: “____________.”
A. Tại sao không?
B. Ồ, tôi xin lỗi khi nghe điều đó
C. Nó đã hết chỗ
D. Có. Tôi đã làm vậy
Các phương án A, B, D không phù hợp với ngữ cảnh
Câu 50: Đáp án D
Kiến thức: Ngôn ngữ giao tiếp
Giải chi tiết:
Helen và Mary đang nói về kết quả của Mary ở trường.
- Helen: “Bố mẹ của bạn chắc là tự hào về kết quả của bạn ở trường lắm!”
- Mary: “____________.”
A. Tất nhiên rồi
B. Tôi rất vui vì bạn thích nó
C. Rất tiếc khi nghe điều đó
D. Cảm ơn. Điều đó chắc chắn rất là khích lệ đấy
Các phương án A, B, C không phù hợp với ngữ cảnh

Trang 22
SỞ GD & ĐT VĨNH PHÚC ĐỀ KIỂM TRA CHẤT LƯỢNG LẦN 2 – KHỐI 12
TRƯỜNG THPT BÌNH SƠN NĂM HỌC: 2019 – 2020
ĐỀ GỐC SỐ 1 MÔN: TIẾNG ANH
Thời gian làm bài: 60 phút; không kể thời gian phát đề

Mark the letter A, B, C, or D on your answer sheet to indicate the word whose underlined part differs
from the other three in pronunciation in each of the following questions.
Câu 1 (NB): A. approached B. noticed C. supported D. finished
Câu 2 (NB): A. them B. threaten C. through D. thunder
Mark the letter A, B, C, or D on your answer sheet to indicate the word that differs from the rest in the
position of the primary stress in each of the following questions.
Câu 3 (NB): A. centralise B. achievement C. combustion D. diversity
Câu 4 (NB): A. relevant B. assistant C. argument D. attitude
Mark the letter A, B, C, or D on your answer sheet to indicate the correct answer to each of the following
questions.
Câu 5 (VD): I finally __________ the cold that I had had all week.
A. pass out B. get over C. come down with D. pull through
Câu 6 (VD): My college graduation was a real__________ day for my whole life.
A. red letter B. red ink C. red tape D. red brick
Câu 7 (VD): The weather is going to change soon; I feel it in my __________
A. body B. legs C. bones D. skin
Câu 8 (TH): My parents lent me the money. _________, I couldn’t have afforded the trip.
A. Therefore B. However C. Only if D. Otherwise
Câu 9 (TH): They _________ enthusiastically when their teacher ___________in.
A. will discuss/ will come B. will have discussed/ comes
C. were discussing/ came D. discuss/ comes
Câu 10 (TH): The earthquake, and tsunami that followed, has killed over 1300 people, the majority of
________were in the city of Palu in Central Sulawesi, Indonesia.
A. which B. them C. whom D. that
Câu 11 (TH): There is no excuse for your late submission! You _________ the report by last Monday.
A. needn’t have finished B. mightn’t have finished
C. should have finished D. must have finished
Câu 12 (TH): I don’t think Jill would be a good teacher. She’s got little patience, __________?
A. hasn’t she B. does she C. doesn’t she D. has she
Câu 13 (NB): There are billions of stars in_________ space.
A. a B. an C. the D. x

Trang 23
Câu 14 (TH): _________ ten minutes earlier, you would have got a better seat.
A. If you arrived B. Had you arrived C. Were you arrived D. If you hadn’t arrived
Câu 15 (TH): The boys proposed that their group leader__________ a camping trip.
A. organize B. organized C. organizes D. organizing
Câu 16 (TH): In some parts of the country, prices are__________ than in others.
A. the highest B. higher and higher C. highest D. more and more high
Câu 17 (TH): I deeply regret __________ to her so severely last night. She was badly hurt.
A. to speak B. to be speaking C. being spoken D. having spoken
Câu 18 (TH): No sooner ________ than he begins to be washed.
A. a human being born B. a human being had been born
C. was a human being born D. is a human being born
Mark the letter A, B, C, or D on your answer sheet to indicate the most suitable response to complete each
of the following exchanges.
Câu 19 (NB): Ensoleill and Sunny are talking about Ted’s accident last week.
Ensoleill: “A motor bike knocked Ted down”.
Sunny: “______________”
A. How terrific! B. Poor Ted! C. What it is now? D. What a motor bike!
Câu 20 (TH): Mary and her friend, Ensoleill, are in a coffee shop.
Mary: “Would you like Matcha ice–cream or Caramen with jam?”
Ensoleill: “ ____________________”
A. Yes, I’d love two. B. I like eating them all.
C. It doesn’t matter. D. Neither is fine. They are good.
Mark the letter A, B, C, or D on your answer sheet to indicate the word(s) CLOSET in meaning to the
underlined word(s) in each of the following questions.
Câu 21 (VDC): You’re not trying to suggest I should turn a blind eye and forget all about it?
A. criticize for B. look into C. wink at D. worried about
Câu 22 (VD): If that was done on a national scale, we would wipe out this infectious disease.
A. establish B. retain C. eliminate D. maintain
Mark the letter A, B, C, or D on your answer sheet to indicate the word(s) OPPOSITE in meaning to the
underlined word(s) in each of the following questions.
Câu 23 (TH): Vietnamese have a strong sense of hospitality and feel embarrassed if they cannot show
their guests full respect by preparing for their interval.
A. difference B. unfriendliness C. generosity D. politeness
Câu 24 (VDC): We run a very tight ship here, and we expect all our employees to be at their desks by
eight o’clock and take good care of their own business.
A. have a good voyage B. organize things inefficiently

Trang 24
C. run faster than others D. manage an inflexible system
Read the following passage and mark the letter A, B, C, or D on your answer sheet to indicate the correct
word or phrase that best fits each of the numbered blanks.
Just like any institution, a family also needs rules. Rules are very important because they keep things in
(25) ________. For a family, rules are as necessary as food and clothing. The rules are very important
because they remain peace and order in the family. For example, children should respect and listen to
their parents.
If there are (26) _______to be made, for examples, it is not only the parents who are going to decide
especially if the children are already grown up. It would be better if everybody is involved in making the
decision of important matter.
Rules are needed to maintain a harmonious relationship (27)_________ family members. Parents are
there the pillars of the family and guide children to be responsible and practice good values.
Rules teach children to become more responsible and have discipline not only at home but especially
outside of the home. When there are rules to follow, children will know (28)_______ they should do or
should not do. Rules help to avoid conflict and misunderstanding that may lead to fights and aggression
especially among children. Parents should set rules for the children to follow to avoid quarrels and fights.
There are times when the children fight over simple matters and this happens if the parents do or not
interfere but when parents set the rule and let the children follow, there will be (29)_______conflicts.
Câu 25 (TH): A. need B. order C. time D. shape
Câu 26 (TH): A. calls B. problems C. changes D. decisions
Câu 27 (NB): A. in B. between C. inside D. among
Câu 28 (TH): A. which B. that C. how D. what
Câu 29 (TH): A. fewer B. more C. little D. less
Read the passage and mark the letter A, B, C, or D on your answer sheet to indicate the correct answer to
each of the questions .
Sometimes you know things about people the first time you see them, for example, that you want to be
friends with them or that you don’t trust them. But perhaps this kind of intuition isn’t as hard to explain as
it may seem. For instance, people give out their body language signals all the time. The way you hold
your body, head and arms tells people about your mood. If you hold your arms tightly at your sides, or
fold them across your chest, people will generally feel that you are being defensive. Holding your head to
one side shows interest in the other, while an easy, open posture indicates that you are self- confident. All
this affects the way you feel about someone.
Also, a stranger may remind you of a meeting with someone. This may be because of something as
simple as the fact that he or she is physically similar to someone who treated you well or badly. Your
feelings about a stranger could be influenced by a smell in the air that brings to mind a place where you

Trang 25
were happy as a child. Since even a single word can bring back a memory such as that, you may never
realize it is happening.
Câu 30 (TH): What does the word “open” in the passage most closely mean?
A. unlimited B. enlarged C. relaxed D. not shut
Câu 31 (TH): What influences your impression of a person you meet in the first time?
A. familiarity B. intuition C. feeling D. knowledge
Câu 32 (TH): What one feels about a stranger may be influenced by something that ________.
A. revives one’s past memories B. points to one’s childhood
C. strengthen one’s past behaviors D. reminds one of one’s past treatment
Câu 33 (TH): What does the second paragraph discuss?
A. Meaning of signals one implies toward a stranger
B. Factors that cause people to act differently
C. How people usually behave to a stranger
D. Factors that may influence one’s feeling about a stranger
Câu 34 (TH): Intuition described in the passage can be explained by means of __________.
A. signals B. languages C. behaviors D. styles
Read the following passage and mark the letter A, B, C, or D on your answer sheet to indicate the correct
answer to each of the questions.
The Arts and Crafts Movement in the United States was responsible for sweeping changes in attitudes
toward the decorative arts, then considered the minor or household arts. Its focus on decorative arts
helped to induce United States museums and private collectors to begin collecting furniture, glass,
ceramics, metalwork, and textiles in the late nineteenth and early twentieth centuries. The fact that
artisans, who were looked on as mechanics or skilled workers in the eighteenth century, are frequently
considered artists today is directly attributable to the Arts and Crafts Movement of the nineteenth century.
The importance now placed on attractive and harmonious home decoration can also be traced to this
period, when Victorian interior arrangements were revised to admit greater light and more freely flowing
spaces.
The Arts and Crafts Movement reacts against mechanized processes that threatened handcrafts and
resulted in cheapened, monotonous merchandise. Founded in the late nineteenth century by British social
critics John Ruskin and William Morris, the movement revered craft as a form of art. In a rapidly
industrializing society, most Victorians agreed that art was an essential moral ingredient in the home
environment, and in many middle- and working-class homes craft was the only form of art. Ruskin and
his followers criticized not only the degradation of artisans reduced to machine operators, but also the
impending loss of daily contact with handcrafted objects, fashioned with pride, integrity, and attention to
beauty.

Trang 26
In the United States as well as in Great Britain, reformers extolled the virtues of handcrafted objects:
simple, straightforward design; solid materials of good quality; and sound, enduring construction
techniques. These criteria were interpreted in a variety of styles, ranging from rational and geometric to
romantic or naturalistic. Whether abstract, stylized, or realistically treated, the consistent theme in
virtually all Arts and Crafts design is nature.
The Arts and Crafts Movement was much more than a particular style; it was a philosophy of domestic
life. Proponents believed that if simple design, high-quality materials, and honest construction were
realized in the home and its appointments, then the occupants would enjoy moral and therapeutic effects.
For both artisan and consumer, the Arts and Crafts doctrine was seen as a magical force against the
undesirable effects of industrialization.
Câu 35 (VD): The passage primarily focuses on nineteenth century arts and crafts in terms of which of the
following?
A. Their naturalistic themes. B. Their importance in museum collections.
C. Their British origin. D. Their role in an industrialized society.
Câu 36 (TH): According to the passage, before the nineteenth century, artisans were thought to be
________.
A. defenders of moral standards B. creators of cheap merchandise
C. skilled workers D. talented artists
Câu 37 (TH): It can be inferred from the passage that the Arts and Crafts Movement would have
considered all of the following to be artists EXCEPT _________.
A. creators of textile designs
B. people who produce handmade glass objects
C. operators of machines that automatically cut legs, for furniture
D. metalworkers who create unique pieces of jewelry
Câu 38 (TH): The word “revered” in the second paragraph is closest in meaning to _________.
A. respected B. described C. dubbed D. created
Câu 39 (TH): According to paragraph 2, the handcrafted objects in the homes of middle and working-
class families usually were _________.
A. made by members of the family B. the least expensive objects in their homes
C. regarded as being morally uplifting D. thought to symbolize progress
Câu 40 (TH): The word “extolled” in the third paragraph is closest in meaning to _________.
A. exposed B. praised C. believed D. accepted
Câu 41 (TH): According to the passage, which of the following changes occurred at the same time as the
Arts and Crafts Movement?
A. The creation of brighter and more airy spaces inside homes.
B. The rejection of art that depicted nature in a realistic manner.

Trang 27
C. A decline of interest in art museum collections.
D. An increase in the buying of imported art objects.
Câu 42 (NB): The word “it” in the last paragraph refers to ________.
A. the Arts and Crafts design B. nature
C. the Arts and Crafts Movement D. a particular style
Mark the letter A, B, C, or D on your answer sheet to indicate the underlined part that needs correction in
each of the following questions.
Câu 43 (TH): Having found guilty of racketeering, the mobster was sentenced to a number of years in
prison.
A. Having found B. of C. was sentenced D. a number
Câu 44 (TH): Most people consider it women's responsible to take care of children and do housework.
A. Most B. of C. responsible D. do
Câu 45 (NB): They have made a lot of progress until the country became independent.
A. until B. a lot of C. independent D. became
Mark the letter A, B, C or D to indicate the sentence that is closest in meaning to each of the following
questions.
Câu 46 (VDC): Mary was very disappointed. However, she tried to keep calm.
A. Mary was too disappointed to keep calm.
B. Mary lost her temper because of her disappointment.
C. Disappointed as she was, Mary tried to keep calm.
D. Feeling disappointed, Mary tried to keep calm, but she failed.
Câu 47 (VDC): She gave a great performance at the festival. We now know she has artistic talent.
A. But for her great performance at the festival, we wouldn't know about her artistic talent now.
B. Amazing as her artistic talent is, we don't know about her great performance at the festival.
C. Although she gave a great performance at the festival, we know she has artistic talent.
D. Hardly had we know about her artistic talent when she gave a great performance at the festival.
Mark the letter A, B, C, or D on your answer sheet to indicate the sentence that is closest in meaning to
each of the following questions.
Câu 48 (VD): Be careful or you may get lost and run out of money.” She said.
A. She advised me to be careful or I might get lost and run out of money.
B. She warned me to be careful or I had to get lost and run out of money.
C. She ordered me to be careful and I might get lost and run out of money.
D. She told me to be careful if I got lost and run out of the money.
Câu 49 (VD): It isn’t necessary for us to get a visa for Singapore.
A. We mustn’t get a visa for Singapore. B. We needn’t get a visa for Singapore.
C. We mayn’t get a visa for Singapore. D. We shouldn’t get a visa for Singapore.

Trang 28
Câu 50 (VD): The last time I went to the museum was a year ago.
A. I have not been to the museum for a year. B. A year ago, I often went to the museum.
C. My going to the museum lasted a year. D. At last I went to the museum after a year.

Đáp án
1-C 2-A 3-A 4-B 5-B 6-A 7-C 8-D 9-C 10-C
11-C 12-D 13-D 14-B 15-A 16-B 17-D 18-D 19-B 20-C
21-C 22-C 23-B 24-B 25-B 26-D 27-D 28-D 29-A 30-C
31-B 32-A 33-D 34-A 35-D 36-C 37-C 38-A 39-C 40-B
41-A 42-C 43-A 44-C 45-A 46-C 47-A 48-A 49-B 50-A

LỜI GIẢI CHI TIẾT


Câu 1: Đáp án C
Phương pháp giải:
Kiến thức: Phát âm “ed”
Giải chi tiết:
A. approached /əˈprəʊtʃt/
B. noticed /ˈnəʊtɪst/
C. supported /səˈpɔːtɪd/
D. finished /ˈfɪnɪʃt/
Quy tắc:
Cách phát âm đuôi “-ed”:
- Đuôi “-ed” được phát âm là /ɪd/ khi động từ có phát âm kết thúc là /t/ hay /d/
- Đuôi “-ed” được phát âm là /t/ khi động từ có phát âm kết thúc là /s/,/f/,/p/,/ʃ/,/tʃ/,/k/
- Đuôi “-ed” được phát âm là /d/ với các trường hợp còn lại
Phần gạch chân phương án C được phát âm là /ɪd/, còn lại là /t/
Câu 2: Đáp án A
Phương pháp giải:
Kiến thức: Phát âm “th”
Giải chi tiết:
A. them /ðəm/
B. threaten /ˈθretn/
C. through /θruː/
D. thunder /ˈθʌndə(r)/
Phần gạch chân phương án A được phát âm là /ð/, còn lại là /θ/
Câu 3: Đáp án A

Trang 29
Phương pháp giải:
Kiến thức: Trọng âm của từ có 3, 4 âm tiết
Giải chi tiết:
A. centralise /ˈsentrəlaɪz/
B. combustion /kəmˈbʌstʃən/
C. achievement /əˈtʃiːvmənt/
D. diversity /daɪˈvɜːsəti/
Phương án A trọng âm rơi vào âm tiết thứ nhất, còn lại là âm thứ 2
Câu 4: Đáp án B
Kiến thức: Trọng âm của từ có 3 âm tiết
Giải chi tiết:
A. relevant /ˈreləvənt/
B. assistant /əˈsɪstənt/
C. argument /ˈɑːɡjumənt/
D. attitude /ˈætɪtjuːd/
Phương án B trọng âm rơi vào âm tiết thứ 2, còn lại là âm thứ nhất
Câu 5: Đáp án B
Kiến thức: Cụm động từ
Giải chi tiết:
A. pass out: bất tỉnh
B. get over: vượt qua
C. come down with: mắc, nhiễm
D. pull through: sống trở lại
Tạm dịch: Tôi cuối cùng đã vượt qua bệnh cảm lạnh mà tôi đã mắc phải suốt cả tuần.
Câu 6: Đáp án A
Kiến thức: Sự kết hợp từ
Giải chi tiết:
“red letter day”: một ngày đặc biệt, đáng nhớ

Tạm dịch: Ngày lễ tốt nghiệp đại học của tôi đã thực sự là một ngày đặc biệt trong suốt cuộc đời tôi.
Câu 7: Đáp án C
Kiến thức: Thành ngữ
Giải chi tiết:
“feel one’s bones”: có trực giác/linh cảm về điều gì

Tạm dịch: Thời tiết sẽ thay đổi sớm; tôi có linh cảm về điều đó.

Trang 30
Câu 8: Đáp án D
Kiến thức: Liên từ + trạng từ
Giải chi tiết:
A. Therefore: Do đó
B. However: Tuy nhiên
C. Only if: Nếu chỉ
D. Otherwise (adv): Nếu không thì
Tạm dịch: Bố mẹ tôi đã cho tôi mượn tiền. Nếu không thì tôi đã không thể chi trả cho chuyến đi.
Câu 9: Đáp án C
Kiến thức: Thì quá khứ tiếp diễn
Giải chi tiết:
- Cách dùng: thì quá khứ tiếp diễn diễn tả hành động đang diễn ra trong quá khứ (chia quá khứ tiếp diễn)
thì có hành động khác cắt ngang (chia quá khứ đơn)
- Công thức: S + was/were +V_ing when + S + V_ed/V cột 2
Tạm dịch: Họ đang thảo luận nhiệt tình khi giáo viên bước vào lớp.
Câu 10: Đáp án C
Kiến thức: Đại từ quan hệ
Giải chi tiết:
Trong mệnh đề quan hệ:
- which: cái mà => thay thế cho từ chỉ người (có thể đứng sau giới từ)
- whom: người mà => thay thế cho từ chỉ người (có thể đứng sau giới từ)
- that: cái mà, người mà => thay thế cho từ chỉ vật, chỉ người hoặc hỗn hợp người, vật (không đứng sau
giới từ)
“the majority of them were …” => kết hợp tạo thành một câu mới, không phải một mệnh đề quan hệ =>
loại
“the majority of whom” (phần lớn trong số họ - ý nói những người chết do động đất, sóng thần) => thay
thế cho “1300 people” (1300 người) là cụm từ chỉ người, bổ sung nghĩa cho nó
Tạm dịch: Trận động đất và sóng thần xảy ra sau đó đã giết chết hơn 1300 người, phần lớn trong số họ ở
thành phố Palu ở miền Trung Sulawesi, Indonesia.
Câu 11: Đáp án C
Kiến thức: Câu phỏng đoán
Giải chi tiết:
needn’t have done sth: lẽ ra không cần làm gì (nhưng đã làm)
mightn’t have done sth: có thể đã không làm gì
should have done sth: lẽ ra nên làm gì (nhưng đã không làm)
must have done sth: chắc hẳn là đã làm gì

Trang 31
Tạm dịch: Đừng lấy cớ cho việc trễ hạn nộp của bạn! Bạn lẽ ra nên hoàn thành báo cáo trước thứ Hai tuần
trước.
Câu 12: Đáp án D
Kiến thức: Câu hỏi đuôi
Giải chi tiết:
Vế trước có “little” (hầu như không) mang ý nghĩa phủ định => câu hỏi đuôi sẽ là khẳng định
Động từ vế trước chia thì hiện tại hoàn thành => câu hỏi đuôi: “has”
Chủ ngữ là “she” => câu hỏi đuôi: “she”
Tạm dịch: Tôi không nghĩ rằng Jill sẽ là một giáo viên giỏi. Cô ấy gần như không có tí kiên nhẫn nào,
phải không?
Câu 13: Đáp án D
Kiến thức: Mạo từ
Giải chi tiết:
a/an + danh từ số ít, chưa xác định
the + danh từ đã được xác định rõ
“space” ( không gian vũ trụ) => không cần mạo từ
“in space”: trong không gian
Tạm dịch: Có hàng tỉ vì sao trong không gian.
Câu 14: Đáp án B
Kiến thức: Đảo ngữ trong câu điều kiện loại 3
Giải chi tiết:
- Dấu hiệu: mệnh đề chính “would have got”
- Câu điều kiện loại 3 diễn tả một điều kiện không có thực trong quá khứ dẫn đến kết quả trái với sự thật
trong quá khứ.
- Công thức: If + S + had(not) + P2, S + would + have + P2
- Dạng đảo ngữ: Had + S + (not) + P2, S + would + have + P2
Tạm dịch: Nếu bạn đã đến sớm hơn 10 phút, bạn có lẽ đã có một chỗ ngồi tốt hơn.
Câu 15: Đáp án A
Kiến thức: Thức giả định
Giải chi tiết:
Câu giả định thường dùng với một số động từ: propose, advise, recommend, suggest,…
Công thức: S1 + động từ giả định + that + S2 + V_nguyên thể
Tạm dịch: Các bạn nam đã đề xuất nhóm trưởng của họ tổ chức một buổi cắm trại.
Câu 16: Đáp án B
Kiến thức: Cấu trúc tăng tiến
Giải chi tiết:

Trang 32
- Cấu trúc: S + V + adv/adj-er + and + adv/adj-er: Cái gì càng ngày càng thế nào
- “high” là tính từ ngắn => higher
Tạm dịch: Ở một vài nơi trên đất nước, giá cả ngày càng cao hơn ở những nơi khác.
Câu 17: Đáp án D
Kiến thức: to V/ V_ing
Giải chi tiết:
- Dấu hiệu: “last night” (tối qua)
- regret + V_ing: hối hận đã làm gì
Tạm dịch: Tôi rất hối hận khi đã nói nặng lời với cô ấy tối hôm qua. Cô ấy đã rất tổn thương.
Câu 18: Đáp án D
Kiến thức: Đảo ngữ
Giải chi tiết:
Cấu trúc đảo ngữ câu bị động:
No sooner + tobe + S + P2 than S + V: Ngay sau khi cái gì được làm gì thì …
Động từ “begins” vế sau chia hiện tại đơn => vế trước cũng chia ở hiện tại
Tạm dịch: Ngay sau khi một con người được sinh ra thì cậu ta bắt đầu được tắm rửa sạch sẽ.
Câu 19: Đáp án B
Kiến thức: Ngôn ngữ giao tiếp
Giải chi tiết:
Ensoleill và Sunny đang nói về vụ tai nạn của Ted tuần trước.
Ensoleill: “Một chiếc xe gắn máy đã đâm vào Ted.”
Tim: “_________."
A. Thật tuyệt!
B. Tội nghiệp Ted!
C. Bây giờ là cái gì nào?
D. Thật là một cái xe gắn máy!
Các phản hồi A, C, D không phù hợp ngữ cảnh.
Câu 20: Đáp án C
Kiến thức: Ngôn ngữ giao tiếp
Giải chi tiết:
Mary và bạn của mình Ensoleill đang ở trong một quán cà phê.

Mary: “Bạn muốn dùng kem trà xanh hay caramen với mứt?”
Ensoleill: “________.”
A. Có, tớ thích hai.
B. Tớ thích ăn tất cả.

Trang 33
C. Vị nào cũng được. (Không quan trọng đâu)
D. Không dùng vị nào cũng được. Chúng đều ngon.
Các phản hồi A, B, D không phù hợp ngữ cảnh.
Câu 21: Đáp án C
Kiến thức: Thành ngữ
Giải chi tiết:
turn a blind eye (idiom): cố phớt lờ, bỏ qua
A. criticize for (v): chỉ trích
B. look into (v): điều tra
C. wink at (v): vờ như không thấy
D. worried about (v): lo lắng về
=> turn a blind eye = wink at
Tạm dịch: Không phải bạn đang cố gắng khuyên tôi nên vờ như không thấy và quên hết về nó đâu đúng
không?
Câu 22: Đáp án C
Kiến thức: Cụm động từ
Giải chi tiết:
wipe out (phr. verb): quét sạch, xóa bỏ
A. establish (v): thiết lập
B. retain (v): tiếp tục duy trì
C. eliminate (v): xóa bỏ
D. maintain (v): duy trì
=> wipe out = eliminate
Tạm dịch: Nếu việc đó được thực hiện trên phạm vi cả nước, chúng ta sẽ loại bỏ được căn bệnh truyền
nhiễm này.
Câu 23: Đáp án B
Kiến thức: Từ vựng, từ trái nghĩa
Giải chi tiết:
hospitality (n): sự thân thiện, hiếu khách
A. difference (n): sự khác biệt
B. unfriendliness (n): sự không thân thiện
C. generosity (n): sự hào phóng, rộng lượng
D. politeness (n): sự lịch sự
=> hospitality >< unfriendliness
Tạm dịch: Người Việt Nam giàu lòng hiếu khách và cảm thấy xấu hổ nếu họ không thể bày tỏ hết sự tôn
trọng dành cho khách mời bằng việc chuẩn bị cho giờ nghỉ của họ.

Trang 34
Câu 24: Đáp án B
Kiến thức: Từ vựng, từ trái nghĩa
Giải chi tiết:
run a very tight ship (idiom): điều hành rất tốt
A. have a good voyage (v): có một chuyến đi biển vui vẻ
B. organize things inefficiently (v): tổ chức mọi thứ không hiệu quả
C. run faster than others (v): chạy nhanh hơn người khác
D. manage an inflexible system (v): quản lí một hệ thống không linh hoạt
=> run a very tight ship >< organize things inefficiently
Tạm dịch: Chúng tôi điều hành mọi thứ ở đây rất tốt, và chúng tôi hi vọng tất cả các nhân viên của mình
sẽ vào bàn làm việc lúc 8 giờ và làm tốt công việc của họ.
Câu 25: Đáp án B
Kiến thức: Từ vựng
Giải chi tiết:
in need: cần thiết
in order: theo thứ tự
in time: kịp giờ
in shape: vừa vặn
=> keep sth in order: giữ thứ gì đúng theo đúng trật tự
Rules are very important because they keep things in (25) order.
Tạm dịch: Quy tắc thì rất quan trọng bởi vì chúng giữ mọi thứ theo trật tự.
Câu 26: Đáp án D
Kiến thức: Từ vựng
Giải chi tiết:
calls (n): các cuộc gọi
problems (n): các vấn đề
changes (n): các thay đổi
decisions (n): các quyết định
“make decisions”: đưa ra các quyết định
If there are (26) decisions to be made, for examples, it is not only the parents who are going to decide
especially if the children are already grown up.
Tạm dịch: Ví dụ, nếu có những quyết định cần được đưa ra thì không chỉ cha mẹ người mà sẽ có quyền
quyết định mà còn có cả con cái nếu chúng đã trưởng thành.
Câu 27: Đáp án D
Kiến thức: Giới từ
Giải chi tiết:

Trang 35
in (prep): trong
between (prep): ở giữa (2 người hoặc vật)
inside (prep): ở bên trong
among (prep): ở giữa (từ 3 người hoặc vật trở lên)
“family members” (các thành viên trong gia đình) => nhiều hơn 2 người
Rules are needed to maintain a harmonious relationship (27) among family members.
Tạm dịch: Quy tắc thì cần thiết để duy trì một mối quan hệ hài hòa giữa các thành viên trong gia đình.
Câu 28: Đáp án D
Kiến thức: Mệnh đề quan hệ
Giải chi tiết:
Trong mệnh đề quan hệ
A. which: cái mà => dùng làm chủ ngữ, thay thế cho danh từ chỉ vật: ...N(thing), which + V
B. that: cái mà, người mà => dùng làm chủ ngữ, thay thế cho danh từ chỉ người hoặc vật: ...N(peroson,
thing), that + V
C. how (pronoun): như thế nào
D. what = the things that : cái mà … => ...what + S + V
When there are rules to follow, children will know (28) what they should do or should not do.
Tạm dịch: Khi có những quy tắc phải theo, trẻ em sẽ biết cái gì mà chúng nên làm hoặc không nên làm.
Câu 29: Đáp án A
Kiến thức: So sánh hơn
Giải chi tiết:
“little” (rất ít) và “less” (ít hơn - dạng so sánh hơn của “little”) đi với danh từ không đếm được
“fewer” (rất ít) đi với danh từ số nhiều đếm được
“more” (nhiều – dạng so sánh hơn của “many” hoặc “much”) đi với cả danh từ số nhiều đếm được và
không đếm được
“conflicts” là danh từ số nhiều đếm được => dùng “fewer” phù hợp với ngữ cảnh nhất
There are times when the children fight over simple matters and this happens if the parents do or not
interfere but when parents set the rule and let the children follow, there will be (29) fewer conflicts.
Tạm dịch: Có những lúc khi trẻ xung đột vì những vấn đề đơn giản và điều này xảy ra nếu cha mẹ có
hoặc không can thiệp nhưng khi cha mẹ đặt ra những luật lệ và để trẻ tuân theo, sẽ có ít các cuộc xung đột
hơn.
Câu 30: Đáp án C
Kiến thức: Đọc hiểu
Giải chi tiết:
Từ “open” trong đoạn văn gần nghĩa nhất với từ nào?
A. unlimited (adj): không giới hạn

Trang 36
B. enlarged (adj): nới rộng
C. relaxed (adj): thoải mái
D. not shut (v): không đóng
open = relaxed (adj): thoải mái, cởi mở
Thông tin: Holding your head to one side shows interest in the other, while an easy, open posture
indicates that you are self- confident.
Tạm dịch: Giữ đầu của bạn nghiêng về một phía thể hiện sự hứng thú với người khác, trong khi đó, một
cử chỉ dễ dàng, cởi mở cho thấy bạn đang tin vào bản thân.
Câu 31: Đáp án B
Kiến thức: Đọc hiểu
Giải chi tiết:
Cái gì ảnh hưởng đến ấn tượng của bạn về một người khi bạn gặp lần đầu tiên?
A. familiarity (n): sự thân thuộc
B. intuition (n): trực giác
C. feeling (n): cảm giác
D. knowledge (n): kiến thức
Thông tin: Sometimes you know things about people the first time you see them, for example, that you
want to be friends with them or that you don’t trust them. But perhaps this kind of intuition isn’t as hard
to explain as it may seem.
Tạm dịch: Đôi khi bạn biết được nhiều điều về người mà bạn mới gặp lần đầu, ví dụ, rằng bạn muốn làm
bạn với họ hoặc bạn không tin tưởng họ. Nhưng có lẽ kiểu trực giác này thì dường như không khó để giải
thích.
Câu 32: Đáp án A
Kiến thức: Đọc hiểu
Giải chi tiết:
Điều mà một người cảm nhận về một người lạ có thẻ bị ảnh hưởng bởi thứ gì đó _________.
A. revives one’s past memories: làm sống lại những kí ức của ai đó
B. points one’s childhood: cho thấy thời thơ ấu của ai đó
C. strengthen one’s past behaviors: kích thích hành vi của ai đó trong quá khứ
D. reminds one of one’s past treatment: nhắc nhở ai đó về cách đối xử của ai trong quá khứ
Thông tin: Your feelings about a stranger could be influenced by a smell in the air that brings to mind a
place where you were happy as a child. Since even a single word can bring back a memory such as that,
you may never realize it is happening.
Tạm dịch: Cảm giác của bạn về một người lạ cũng có thể bị ảnh hưởng bởi mùi hương trong không khí
cái mà làm bạn nhớ lại một nơi bạn đã từng vui vẻ khi còn nhỏ. Thậm chí từng từ ngữ cũng có thể mang
kí ức trở lại như vậy, bạn có thể không bao giờ nhận ra rằng nó đang xảy ra.

Trang 37
Câu 33: Đáp án D
Kiến thức: Đọc hiểu
Giải chi tiết:
Đoạn văn thứ hai đề cập đến cái gì?
A. Ý nghĩa của các tín hiệu mà một người ám chỉ với một người lạ
B. Các nhân tố khiến con người hành động khác nhau
C. Mọi người thường cư xử với một người lạ như thế nào
D. Các nhân tố có thể ảnh hưởng đến cảm giác của ai đó về một người lạ
Thông tin: This may be because of something as simple as the fact that he or she is physically similar to
someone who treated you well or badly. Your feelings about a stranger could be influenced by a smell in
the air …
Tạm dịch: Điều này có thể là bởi vì một vài thứ đơn giản như là anh hoặc cô ấy có ngoại hình khá giống
với ai đó người mà đã từng đối xử tốt hoặc tệ với bạn. Cảm giác của bạn về một người lạ cũng có thể bị
ảnh hưởng bởi mùi hương trong không khí…
Câu 34: Đáp án A
Kiến thức: Đọc hiểu
Giải chi tiết:
Trực giác được miêu tả trong đoạn văn có thể được giải thích bằng ________.
A. signals (n): các kí hiệu
B. languages (n): ngôn ngữ
C. behaviors (n): hành vi
D. styles (n): phong cách
Thông tin: But perhaps this kind of intuition isn’t as hard to explain as it may seem. For instance, people
give out their body language signals all the time.
Tạm dịch: Nhưng có lẽ kiểu trực giác này thì dường như không khó để giải thích. Ví dụ, con người thể
hiện kí hiệu ngôn ngữ cơ thể mọi lúc.
Câu 35: Đáp án D
Kiến thức: Đọc hiểu
Giải chi tiết:
Đoạn văn chủ yếu tập trung vào khía cạnh nào dưới đây của nghệ thuật và thủ công trong thế kỉ 19?
A. Các chủ đề thực tế của chúng.
B. Tầm quan trọng của việc sưu tầm trong bảo tàng.
C. Nguồn gốc Anh của chúng.
D. Vai trò của chúng trong một xã hội công nghiệp hóa.
Câu 36: Đáp án C
Kiến thức: Đọc hiểu

Trang 38
Giải chi tiết:
Theo đoạn văn, trước thế kỉ 19, các nghệ nhân được coi như là_______.
A. người bảo vệ các tiêu chuẩn đạo đức
B. người tạo ra những món hàng rẻ tiền
C. người công nhân lành nghề
D. những nghệ sĩ tài năng
Thông tin: The fact that artisans, who were looked on as mechanics or skilled workers in the eighteenth
century,...
Tạm dịch: Việc các nghệ nhân, những người được coi là thợ cơ khí hoặc công nhân lành nghề trong thế
kỷ thứ 18,...
Câu 37: Đáp án C
Kiến thức: Đọc hiểu
Giải chi tiết:
Có thể suy ra từ đoạn văn rằng phong trào nghệ thuật và thủ công có lẽ đã coi những người nào dưới đây
là nghệ sĩ trừ ________.
A. những người tạo ra thiết kế may dệt
B. người mà sản xuất các đồ bằng kính thủ công
C. những người điều hành máy móc mà tự động cắt chân cho các đồ nội thất
D. công nhân luyện kim người mà tạo ra những mảnh trang sức độc đáo
Thông tin: Its focus on decorative arts helped to induce United States museums and private collectors to
begin collecting furniture, glass, ceramics, metalwork, and textiles in the late nineteenth and early
twentieth centuries. The fact that artisans, who were looked on as mechanics or skilled workers in the
eighteenth century, are frequently considered artists today is directly attributable to the Arts and Crafts
Movement of the nineteenth century.
Tạm dịch: Sự tập trung của nó vào nghệ thuật trang trí đã giúp thúc đẩy các viện bảo tàng của Mĩ và nhà
sưu tập tư nhân bắt đầu thu thập đồ nội thất, thủy tinh, gốm sứ, kim loại và các sản phẩm dệt vào cuối thế
kỷ 19 và đầu thế kỷ 20. Việc các nghệ nhân, những người được coi là thợ cơ khí hoặc công nhân lành
nghề trong thế kỷ thứ mười tám, thường được coi là nghệ sĩ ngày nay thì trực tiếp liên quan đến phong
trào nghệ thuật và thủ công của thế kỷ 19.
Chỉ có đáp án C không được đề cập
Câu 38: Đáp án A
Kiến thức: Đọc hiểu
Giải chi tiết:
Từ “revered” trong đoạn 2 gần nghĩa nhất với______.
revered: được tôn thờ
A. respected: được tôn trọng

Trang 39
B. described: được miêu tả
C. dubbed: được gọi tên
D. created: được sáng tạo
=> revered = respected: được tôn trọng, tôn thờ
Thông tin: Founded in the late nineteenth century by British social critics John Ruskin and William
Morris, the movement revered craft as a form of art.
Tạm dịch: Được thành lập vào cuối thế kỷ XIX bởi các nhà phê bình xã hội người Anh John Ruskin và
William Morris, phong trào này tôn sùng thủ công như một hình thức nghệ thuật.
Câu 39: Đáp án C
Kiến thức: Đọc hiểu
Giải chi tiết:
Theo đoạn 2, các sản phẩm thủ công ở các gia đình thuộc tầng lớp trung lưu và lao động thường là
_______.
A. được làm bởi các thành viên trong gia đình
B. các đồ vật ít dắt tiền nhất trong nhà họ
C. được coi như là việc nâng cao các giá trị đạo đức
D. được nghĩ như là quá trình biểu tượng hóa
Thông tin: In a rapidly industrializing society, most Victorians agreed that art was an essential moral
ingredient in the home environment, and in many middle- and working-class homes craft was the only
form of art.
Tạm dịch: Trong một xã hội công nghiệp hóa nhanh chóng, hầu hết người dân Victoria đều đồng ý rằng
nghệ thuật là một thành phần thiết yếu hình thành đạo đức trong gia đình, và ở tầng lớp trung lưu và lao
động, thủ công gia đình là hình thức nghệ thuật duy nhất.
Câu 40: Đáp án B
Kiến thức: Đọc hiểu
Giải chi tiết:
Từ “extolled” trong đoạn 3 gần nghĩa nhất với _______
extolled: được ca tụng
A. exposed: được tiếp xúc
B. praised: được ca ngợi
C. believed: được tin tưởng
D. accepted: được chấp nhận
=> extolled = praised: được ca ngợi
Thông tin: In the United States as well as in Great Britain, reformers extolled the virtues of handcrafted
objects: …
Tạm dịch: Ở Mĩ cũng như Anh, các nhà cải cách đã ca ngợi những điểm sáng của các đồ vật thủ công: …

Trang 40
Câu 41: Đáp án A
Kiến thức: Đọc hiểu
Giải chi tiết:
Theo bài văn, những thay đổi nào dưới đây đã xảy ra đồng thời với phong trào nghệ thuật và thủ công?
A. Việc tạo ra không gian sáng và thoáng đãng hơn bên trong các ngôi nhà.
B. Sự chối bỏ của nghệ thuật cái mà mô tả thiên nhiên bằng phương thức hiện thực
C. Sự giảm về hứng thú với việc sưu tập nghệ thuật bảo tàng
D. Sự gia tăng trong việc bán các đồ nghệ thuật nhập khẩu
Thông tin: ...to the Arts and Crafts Movement of the nineteenth century. The importance now placed on
attractive and harmonious home decoration can also be traced to this period, when Victorian interior
arrangements were revised to admit greater light and more freely flowing spaces.
Tạm dịch: ... đến phong trào nghệ thuật và thủ công của thế kỷ 19. Tầm quan trọng bây giờ được đặt vào
việc trang trí nhà sao cho hấp dẫn và hài hòa cũng có thể được bắt nguồn từ thời kỳ này, khi việc sắp xếp
nội thất của Victoria đã được sửa đổi để nhận nguồn sáng lớn hơn và không gian phóng khoáng hơn.
Câu 42: Đáp án C
Kiến thức: Đọc hiểu
Giải chi tiết:
Từ “it” trong đoạn cuối đề cập đến__________
A. thiết kế về nghệ thuật và thủ công
B. thiên nhiên
C. phong trào nghệ thuật và thủ công
D. một phong cách cụ thể
Thông tin: The Arts and Crafts Movement was much more than a particular style; it was a philosophy of
domestic life.
Tạm dịch: Phong trào nghệ thuật và thủ công không chỉ là một phong cách cụ thể; nó còn là một triết lý
của cuộc sống trong nước.
Câu 43: Đáp án A
Kiến thức: Mệnh đề rút gọn
Giải chi tiết:
Hai mệnh đề cùng chủ ngữ => có thể rút gọn mệnh đề
Câu mang nghĩa bị động, hành động vế đầu xảy ra trước hành động vế sau
=> rút gọn theo công thức: Having + been + P2
Sửa: “Having found” => “Having been found”
Tạm dịch: Bị phát hiện ra tội lừa đảo, tên cướp đã bị tuyên án một vài năm tù.
Câu 44: Đáp án C
Kiến thức: Từ loại

Trang 41
Giải chi tiết:
Sau tính từ sở hữu “women’s” cần một danh từ số nhiều để chỉ chung
responsible (adj): có trách nhiệm
responsibility (n): trách nhiệm
Sửa: responsible => responsibilities
Tạm dịch: Phần lớn mọi người đều coi trách nhiệm của phụ nữ là chăm con và làm việc nhà.
Câu 45: Đáp án A
Kiến thức: Giới từ
Giải chi tiết:
Vế trước động từ chia thì hiện tại hoàn thành: “have made”
=> giới từ thường được sử dụng là “since + mốc thời gian” hoặc “for + khoảng thời gian”
“the country became independent” (đất nước độc lập) => mốc thời gian
Sửa: until => since
Tạm dịch: Họ đã đạt được nhiều tiến bộ kể từ khi đất nước độc lập.
Câu 46: Đáp án C
Kiến thức: Câu đồng nghĩa, đảo ngữ với “as”
Giải chi tiết:
Cấu trúc đảo ngữ với “as”: Adj/Adv + S + V, S + V
Tạm dịch:
Mary đã quá thất vọng. Tuy nhiên, cô ấy đã cố gắng giữ bình tĩnh.
A. Mary đã quá thất vọng đến nỗi cô ấy không thể giữ bình tĩnh. => sai nghĩa
B. Mary mất bình tĩnh bởi vì sự thất vọng của cô ấy. => sai nghĩa
C. Mặc dù cô ấy thất vọng, Mary vẫn cố gắng giữ bình tĩnh.
D. Cảm thấy thất vọng, Mary đã cố giữ bình tĩnh, nhưng cô ấy đã thất bại. => sai nghĩa
Câu 47: Đáp án A
Kiến thức: Cấu trúc đồng nghĩa
Giải chi tiết:
Câu điều kiện hỗn hợp diễn tả điều kiện không có thật ở quá khứ dẫn đến kết quả trái với hiện tại
Công thức: If + S + had (not) + P2, S + would + V_nguyên thể
But for = If … not : Nếu không nhờ
Tạm dịch:
Cô ấy đã có một màn trình diễn tuyệt vời ở lễ hội. Bây giờ chúng tôi biết được tài năng nghệ thuật của cô
ấy.
A. Nếu không nhờ màn trình diễn tuyệt vời của cô ấy ở lễ hội, bây giờ chúng tôi đã không biết về tài năng
nghệ thuật của cô ấy.

Trang 42
B. Tuy tài năng nghệ thuật của cô ấy thì tuyệt vời, nhưng chúng tôi đã không biết về màn trình diễn của
cô ấy ở lễ hội. => sai nghĩa
C. Mặc dù cô ấy đã có một màn trình diễn tuyệt vời ở lễ hội, chúng tôi biết về tài năng nghệ thuật của cô
ấy. => sai nghĩa
D. Ngay sau khi chúng tôi biết tài năng nghệ thuật của cô ấy thì cô ấy đã có màn trình diễn tuyệt vời ở lễ
hội. => sai nghĩa
Câu 48: Đáp án A
Kiến thức: Lời nói gián tiếp
Giải chi tiết:
“Be careful or …” : Hãy cẩn thận nếu không thì …
= S + advised + S + to V_nguyên thể: Ai đó đã khuyên ai làm gì
warn sb of/about doing sth: cảnh báo ai làm gì
Tạm dịch:
“Hãy cẩn thận hoặc bạn có thể bị lạc và hết sạch tiền” Cô ấy đã nói
A. Cô ấy đã khuyên tôi nên cẩn thận nếu không tôi có thể bị lạc và hết sạch tiền.
B. Cô ấy đã cảnh cáo tôi phải cẩn thận hoặc tôi phải bị lạc và hết sạch tiền. => sai nghĩa, sai cấu trúc
C. Cô ấy đã yêu cầu tôi phải cẩn thận và tôi có thể bị lạc và hết sạch tiền. => sai nghĩa
D. Cô ấy đã bảo tôi phải cẩn thận nếu tôi bị lạc và hết sạch tiền. => sai nghĩa
Câu 49: Đáp án B
Kiến thức: Động từ khuyết thiếu
Giải chi tiết:
mustn’t: không được phép làm gì
needn’t: không cần làm gì
mayn’t: không thể làm gì
shouldn’t: không nên làm gì
Tạm dịch:
Thật không cần thiết cho chúng tôi khi lấy visa sang Singapore.
A. Chúng tôi không được phép nhận visa sang Singapore. => sai nghĩa
B. Chúng tôi không cần phải lấy visa sang Singapore.
C. Chúng tôi có thể không lấy visa sang Singapore. => sai nghĩa
D. Chúng tôi không nên lấy visa sang Singapore. => sai nghĩa
Câu 50: Đáp án A
Kiến thức: Thì hiện tại hoàn thành
Giải chi tiết:
- Cách dùng: Thì hiện tại hoàn thành diễn tả sự việc bắt đầu trong quá khứ, kéo dài đến hiện tại và có thể
tiếp tục ở tương lai. Với cách dùng này thường có “since + mốc thời gian” hoặc “for + khoảng thời gian”

Trang 43
- Công thức: S + have/has (not) + P2 + for + khoảng thời gian/since + mốc thời gian
“a year” (một năm) là khoảng thời gian => dùng “for”
Tạm dịch: Lần cuối tôi đi đến viện bảo tàng là một năm trước.
A. Tôi đã không đi đến viện bảo tàng trong một năm rồi.
B. Một năm trước, tôi thường đi đến viện bảo tàng. => sai nghĩa
C. Chuyến đi đến viện bảo tàng của tôi đã kéo dài một năm. => sai nghĩa
D. Cuối cùng tôi đã đi đến viện bảo tàng sau một năm. => sai nghĩa

SỞ GD & ĐT HƯNG YÊN KỲ KHẢO SÁT CHẤT LƯỢNG LỚP 12 THPT NĂM 2019
Mã đề 816 MÔN: TIẾNG ANH
Thời gian làm bài: 60 phút; không kể thời gian phát đề

Mark the letter A, B, C, or D on your answer sheet to indicate the most suitable response to complete each
of the following exchanges.
Câu 1 (TH): Christina and John are in the English evening class.
Christina: “Why do you think the most people learn English?”
John: “……………………………………………………….”
A. Very often it’s to get a better job B. All of them are
C. Because I like it D. I heard it was very good
Câu 2 (TH): John and Mary are having dinner at her house.
John: “This dish is really delicious!”
Mary: “……………. . It’s called Yakitori, and it’s made with chicken livers.”
A. I guess you’re right. B. It’s my pleasure.
C. Sure, I’ll be glad to. D. I’m glad you like it.
Read the following passage and mark the letter A, B, C, or D on your answer sheet to indicate the correct
answer to each of the questions.
Instructors at American colleges and universities use many different teaching methods. Some instructors
give assignments every day. They grade homework. Students in their classes have to take many quizzes, a
midterm exam, and a final test. Other instructors give only writing assignments. Some teachers always
follow a course outline and usually use the textbooks. Others send students to the library for assignments.
The atmosphere in some classrooms is very formal. Students call their instructors “Professor Smith”,
“Mrs. Jones”, and so on. Some teachers wear business clothes and give lectures. Other classrooms have
an informal atmosphere. Students and teachers discuss their ideas. Instructors dress informally, and
students call them by their first names. American teachers are not alike in their teaching styles.
At most American colleges and universities, facilities for learning and recreation are available to students.
Students can often use recorders, video machines, and computers at libraries and learning centres. They

Trang 44
can buy books, notebooks, and other things at campus stores. They can get advice on their problems from
counsellors and individual help with their classes from tutors. Students can relax and have fun on campus,
too. Some schools have swimming pools and tennis courts. Most have snack bars or cafeterias.
(Source. Adapted from http://www.ukedu.org)
Câu 3 (TH): What is the main idea of the first paragraph?
A. Ways of teaching B. Ways of giving assignments
C. Ways of using the textbook D. Ways of taking an exam
Câu 4 (TH): What does the phrase “business clothes” in paragraph 2 mean?
A. trendy clothes B. casual clothes
C. formal clothes D. clothes for business people
Câu 5 (TH): Where do students and teachers discuss their idea?
A. At learning centers B. In classrooms with formal atmosphere
C. In classrooms with informal atmosphere D. At libraries
Câu 6 (TH): What can’t students do at most American colleges and universities?
A. They can’t buy anything at campus stores.
B. They can’t use the computers that are linked to libraries.
C. They can’t ask their counselors and tutors for advice.
D. They can’t have tutors and counselors solved their problems.
Câu 7 (TH): Which of the following statements is NOT true about schools in America?
A. They offer sports and leisure facilities for students.
B. They have no recreation facilities.
C. They are well-equipped.
D. They have stores on campus.
Read the following passage and mark the letter A, B, C, or D on your answer sheet to indicate the correct
answer to each of the questions.
There is a common trend for people to indulge in leisure activities. Did you ever watch a video on the
Internet? Maybe you used YouTube. YouTube is a Web site where people can share their video. Today,
YouTube is an important part of the Internet. However, that wasn’t always true.
YouTube started with a young man named Jawed Karim and two friends. One day, Karim was on the
Internet. He wanted information about the 2004 tsunami in Southeast Asia. He found news stories about
it, but he couldn’t find any videos. This gave Karin an idea. He wanted to help people put video on the
Internet. Karim told his friends about this idea. Together, they created a company – YouTube.
YouTube become a global success. Millions of people around the world visited the Web site. It was clear
to Google, another Internet company, that YouTube had a lot of value. Google made a deal. It bought
YouTube for 1.65 USD. As a result, YouTube investors and its employees made a lot of money. The

Trang 45
three friends who started YouTube were very big investors. Therefore, they made an enormous amount of
money.
Karim became very rich, and he continued to work toward his PhD. There was something else he wanted
to do. He wanted to help young people go into business. He used money and experience to start a new
company called Youniversity Ventures. This company helps young people who have good business ideas.
It gives them advice and money to start Internet businesses. Milo is one business that students started with
the help of Youniversity Ventures.
Milo is a shopping Web site. It helps people find products in stores near their homes. Another example is
AirBoB. This Web site helps people find for video conferences. People in different places can use this site
to have business meetings.
Karim has some advice for students who want to start business. First, find a successful company. Do a lot
of research about the company and the top people in the company. There, copy the way they do things.
For students who wants to start Internet business, Karim is probably a very good example to copy.
(Source. Adapted from http://www.grid.unep.ch/waste/download/waste1213.pdf)
Câu 8 (VD): What can be the best title for the passage?
A. To be successful on the Internet B. Support from Youniversity Ventures
C. Sharing Success on the Internet D. The best videos from YouTube
Câu 9 (NB): What does the word “This” in paragraph 2 refer to?
A. the information of the 2004 tsunami B. the 2004 tsunami in Southeast Asia
C. that he could find no videos D. the news stories he could find
Câu 10 (TH): How did the YouTube investors make a lot of money?
A. Google bought their YouTube company. B. They invested a lot of money in Google.
C. They sold things on Milo Website. D. They made a profit from Youniversity Ventures.
Câu 11 (TH): Which of the following is NOT mentioned about Karim?
A. He earned a lot of money from YouTube. B. He is the owner of Youniversity Ventures.
C. He had the original idea of YouTube. D. He created a lot of sample videos for YouTube.
Câu 12 (TH): Which of the following can replace the word “enormous” in paragraph 3?
A. considerable B. favourable C. huge D. increasing
Câu 13 (TH): What does Youniversity Ventures do?
A. It buys Internet companies such as Milo.
B. It helps people find inexpensive vacation places worldwide.
C. It gives people advice and money to start business.
D. It helps people to get PhDs at famous Universities.
Câu 14 (TH): What kind of Web site is AirBoB?
A. It helps people search vacation places. B. It helps people find successful businesses.
C. It helps people held business meetings. D. It helps people buy goods and services.

Trang 46
Câu 15 (TH): What is Karim’s advice for students who want to start business?
A. Work for successful people and companies.
B. Copy successful people and companies.
C. Give money to Youniversity Ventures.
D. Get a PhD degree from famous university.
Mark the letter A, B, C, or D on your answer sheet to indicate the word that differs from the rest in the
position of the primary stress in each of the following questions.
Câu 16 (NB): A. attract B. divide C. finish D. invite
Câu 17 (NB): A. economics B. entertainment C. radiation D. geography
Mark the letter A, B, C, or D on your answer sheet to indicate the sentence that is closest in meaning to
each of the following questions.
Câu 18 (VD): Were it not for the money, this job wouldn’t be worthwhile.
A. Although the salary is poor, the job is worthwhile.
B. The only thing that makes this job worthwhile is the money.
C. This job is not rewarding at all, so everyone wants to get it.
D. This job offers a poor salary, so it is worthwhile.
Câu 19 (VD): The burglar was caught red-handed by the police when he broke into the flat.
A. The police caught the burglar to break into the flat.
B. The police caught the burglar breaking into the flat.
C. The police caught the burglar when breaking into the flat.
D. Then the burglar had broken into the flat, the police caught him at one.
Câu 20 (VD): He didn’t pay attention to what I said.
A. He had no intention of talking to me. B. He didn’t hear me even though I was saying to
him.
C. He took no notice of my words. D. He didn’t pay attention because I didn’t say a word.
Mark the letter A, B, C, or D on your answer sheet to indicate the correct answer to each of the following
questions.
Câu 21 (TH): Such characters as fairies or witches in Walt Disney animated cartoons are purely
_________
A. imaginable B. imaginative C. imagining D. imaginary
Câu 22 (NB): Neither Mary nor her brothers _________at the party yet. They may be getting stuck in the
traffic.
A. are arriving B. have arrived C. has arrived D. is arriving
Câu 23 (TH): The old market is said _________in a fire two years ago.
A. to be destroyed B. to have destroyed
C. to have been destroyed D. to be destroying

Trang 47
Câu 24 (TH): We should participate in the movement _________to conserve the natural environment.
A. which organized B. organizing C. to organize D. organized
Câu 25 (TH): I have decided to buy that house. I won’t change my mind _________what you say.
A. no matter B. although C. because D. whether
Câu 26 (NB): An endangered species is the one _________ population is so small that it is in danger of
becoming extinct.
A. which B. what C. whose D. who
Câu 27: I don’t understand why the students didn’t keep silent while the speech _________.
A. was being made B. was made C. has been made D. would be made
Câu 28 (VDC): The jokes Jack tells are as old as _________.
A. the hills B. the mountains C. the oceans D. the earth
Câu 29 (VDC): When finding a new house, parents should _________ all the conditions for their
children’s education and entertainment.
A. keep pace with B. make room for C. get rid of D. take into account
Câu 30 (TH): Ms. Brown asked me _________ in my class.
A. how many students there were B. were there how many students
C. how many students were there D. there were how many students
Câu 31 (NB): After David _________ his homework, he went straight to bed.
A. had finished B. has finished C. was finished D. finished
Câu 32 (TH): _________, we would have been forced to sack him.
A. Were he not to resign B. If he resigned
C. Had he not resigned D. If he had resigned
Câu 33 (TH): Students also have the opportunity to choose from a wide range of _________ courses in
the university.
A. optional B. unique C. compulsory D. limited
Câu 34 (TH): I’d rather _________ to the party with my parents because there was nothing interesting
there.
A. hadn’t been invited B. not have been invited
C. haven’t been invited D. not be invited
Mark the letter A, B, C, or D on your answer sheet to indicate the sentence that best combines each pair
of sentences in the following questions.
Câu 35 (VD): You have just passed your exam. This makes your parents happy.
A. You have just passed your exam makes your parents happy.
B. That you have passed your exam makes your parents happy.
C. You have just passed your exam which it makes your parents happy.
D. Having just passed your exam making your parents happy.

Trang 48
Câu 36 (VD): She raised her hand high. She wanted to attract her teacher’s attention.
A. Because her teacher attracted her, she raised her hand high.
B. She had such a high raising of hand that she attracted her teacher’s attention.
C. Though she raised her hand high, she could not attract her teacher’s attention.
D. To attract her teacher’s attention, she raised her hand high.
Mark the letter A, B, C, or D on your answer sheet to indicate the word(s) CLOSEST in meaning to the
underlined word(s) in each of the following questions.
Câu 37 (VD): Not until all their demands had been turned down did the workers decide to go on strike for
more welfare.
A. detected B. reviewed C. sacked D. rejected
Câu 38 (NB): She was brought up in a well-off family. She can’t understand the problems we are facing.
A. poor B. wealthy C. broke D. kind
Mark the letter A, B, C, or D on your answer sheet to indicate the word(s) OPPOSITE in meaning to the
underlined word(s) in each of the following questions.
Câu 39 (TH): The palace was badly damaged by fire, but was eventually restored to its original splendor.
A. renovated B. refurbished C. strengthened D. devastated
Câu 40 (NB): All children can attend without paying fees at state schools.
A. secondary schools B. independent schools
C. primary schools D. high schools
Mark the letter A, B, C, or D on your answer sheet to indicate the word whose underlined part differs
from the other three in pronunciation in each of the following questions.
Câu 41 (NB): A. bags B. days C. speeds D. dates
Câu 42 (NB): A. searched B. cooked C. described D. developed
Mark the letter A, B, C, or D on your answer sheet to show the underlined part that needs correction in
each of the following questions.
Câu 43 (TH): Some snakes have hollow teeth they are called fans that they use to poison their victims.
A. Some B. they are called C. use D. their
Câu 44 (NB): What the woman was saying were so important that I asked everyone to stop talking and
listen.
A. What B. was saying C. were D. to stop taking
Câu 45 (NB): George is not enough intelligent to pass this economics class without help.
A. enough intelligent B. to pass C. economics D. help
Read the following passage and mark the letter A, B, C, or D on your answer sheet to indicate the correct
word or phrase that best fits each of the numbered blanks.
EXAM ADVICE

Trang 49
In Part Three of the Speaking Section you work together with a partner. You have to do a (46) _____ task
which usually lasts about 3 minutes. One possible task is “problem solving”, which means you have to
look at some (47) _____ information and then discuss the problem with your partner. You may be shown
photos, drawings, diagrams, maps, plans, advertisements or computer graphics and it is (48) ________
that you study them carefully. If necessary, check you know exactly what to do by politely asking the
examiner to repeat the instruction or make them clearer.
While you are doing the task, the examiner will probably say very little and you should ask your partner
questions and make (49) ________ if he or she is not saying much. If either of you have any real
difficulties, the examiner may decide to step in and help. Normally, however, you will find plenty to say,
which helps the assessor to give you a fair mark. This mark depends on your success in doing the task by
competing with your partner, which includes taking (50) ____ in giving opinions and replying
appropriately, although in the end it may be possible to “agree to disagree”.
(Source. Adapted from http://www.grid.unep.tech)
Câu 46 (VD): A. single B. scarce C. lonely D. unique
Câu 47 (TH): A. noticeable B. visual C. optical D. obvious
Câu 48 (TH): A. helpful B. essential C. probable D. successful
Câu 49 (TH): A. statements B. suggestions C. speeches D. ideas
Câu 50 (VD): A. changes B. sentences C. turns D. sides

Đáp án
1-A 2-D 3-A 4-C 5-C 6-D 7-B 8-C 9-C 10-A
11-D 12-C 13-C 14-C 15-B 16-C 17-D 18-B 19-B 20-C
21-D 22-B 23-C 24-D 25-A 26-C 27-A 28-A 29-D 30-A
31-A 32-C 33-A 34-B 35-B 36-D 37-D 38-B 39-D 40-B
41-D 42-C 43-B 44-C 45-A 46-A 47-B 48-B 49-B 50-C

LỜI GIẢI CHI TIẾT


Câu 1: Đáp án A
Kiến thức: Ngôn ngữ giao tiếp
Giải chi tiết:
Christina và John đang ở trong lớp học tiếng Anh buổi tối.
Christina: Cậu nghĩ tại sao nhiều người học tiếng Anh?
A. Thường là để có được công việc tốt hơn
B. Tất cả những thứ đó
C. Bởi vì mình thích
D. Mình nghe thấy rằng nó rất tốt

Trang 50
Các phản hồi B, C, D không phù hợp.
Tạm dịch: Christina và John đang ở trong lớp học tiếng Anh buổi tối.
Christina: Cậu nghĩ tại sao nhiều người học tiếng Anh?
John: Thường là để có công việc tốt hơn.
Câu 2: Đáp án D
Kiến thức: Ngôn ngữ giao tiếp
Giải chi tiết:
John và Mary đang ăn tối ở nhà cô ấy (nhà của Mary).
John: “Món này rất ngon.”
Mary: “…….. . Nó có tên là Yakitori, và nó được làm từ gan gà.”
A. Mình đoán là cậu đúng rồi
B. Thật hân hạnh/Không có gì đâu
C. Chắc chắn rồi, mình cũng sẽ vui
D. Mình vui vì cậu thích nó
Các phản hồi A, B, C không phù hợp.
Tạm dịch: John và Mary đang ăn tối ở nhà cô ấy (nhà của Mary).
John: “Món này rất ngon.”
Mary: “Mình vui vì cậu thích nó. Nó có tên là Yakitori, và nó được làm từ gan gà.”
Câu 3: Đáp án A
Kiến thức: Đọc hiểu
Giải chi tiết:
Ý chính của đoạn văn thứ nhất là gì?
A. Các cách dạy học
B. Các cách giao bài tập
C. Các cách sử dụng sách giáo khoa
D. Các cách làm bài thi
Thông tin: Instructors at American colleges and universities use many different teaching methods.
Tạm dịch: Những giảng viên ở trường cao đẳng và đại học ở Mỹ sử dụng nhiều phương pháp giảng dạy
khác nhau.
Câu 4: Đáp án C
Kiến thức: Đọc hiểu
Giải chi tiết:
Cụm từ “business clothes” trong đoạn 2 có nghĩa là gì?
A. quần áo theo trend (xu hướng)
B. quần áo bình thường (mặc hàng ngày)
C. quần áo lịch sự, trang trọng

Trang 51
D. quần áo cho người làm kinh doanh
Thông tin: Some teachers wear business clothes and give lectures.
Tạm dịch: Một vài giảng viên mặc quần áo rất trang trọng và giảng bài.
Câu 5: Đáp án C
Kiến thức: Đọc hiểu
Giải chi tiết:
Sinh viên và giảng viên trao đổi ý kiến ở đâu?
A. Tại trung tâm học tập
B. Trong lớp học với không khí trang trọng
C. Trong lớp học với không khí không trang trọng
D. Ở thư viện
Thông tin: Other classrooms have an informal atmosphere. Students and teachers discuss their ideas.
Instructors dress informally, and students call them by their first names.
Tạm dịch: Những lớp học khác có bầu không khí không trang trọng. Sinh viên và giảng viên trao đổi
những ý tưởng. Giảng viên ăn mặc không trang trọng, và sinh viên có thể gọi họ bằng tên.
Câu 6: Đáp án D
Kiến thức: Đọc hiểu
Giải chi tiết:
Sinh viên không thể làm gì ở đa số trường cao đẳng và đại học ở Mỹ?
A. Họ không thể mua bất cứ thứ gì tại các cửa hàng trong khuôn viên trường.
B. Họ không thể sử dụng các máy tính được liên kết với thư viện.
C. Họ không thể nhờ cố vấn và gia sư cho lời khuyên.
D. Họ không thể để gia sư và cố vấn giải quyết vấn đề cho họ.
Chỉ có phương án D không được nhắc đến.
Thông tin: Students can often use recorders, video machines, and computers at libraries and learning
centres. They can buy books, notebooks, and other things at campus stores. They can get advice on their
problems from counsellors and individual help with their classes from tutors.
Tạm dịch: Sinh viên thường có thể sử dụng máy ghi âm, máy quay phim và máy vi tính ở thư viện và
trung tâm học tập. Họ có thể mua sách, vở, và những thứ khác ở cửa hàng khuôn viên trường. Họ có thể
được nhận lời khuyên về vấn đề của mình từ các nhà cố vấn và vấn đề cá nhân trong lớp học từ gia sư.
Câu 7: Đáp án B
Kiến thức: Đọc hiểu
Giải chi tiết:
Khẳng định nào dưới đây không đúng về các trường học ở Mỹ?
A. Chúng cung cấp các cơ sở thể thao và giải trí cho sinh viên.
B. Chúng không có phương tiện giải trí.

Trang 52
C. Chúng được trang bị tốt.
D. Chúng có cửa hàng trong khuôn viên trường.
Thông tin:
At most American colleges and universities, facilities for learning and recreation are available to students.
Students can relax and have fun on campus, too. Some schools have swimming pools and tennis courts.
Most have snack bars or cafeterias.
Tạm dịch:
Ở hầu hết các trường cao đẳng và đại học ở Mỹ, cơ sở vật chất để học và giải trí luôn có sẵn cho sinh
viên.
Sinh viên có thể thư giãn và giải trí ở khuôn viên trường. Một số trường có hồ bơi và sân quần vợt. Hầu
hết đều có cửa hàng thức ăn nhanh hay căng – tin.
Câu 8: Đáp án C
Kiến thức: Đọc hiểu
Giải chi tiết:
Cái nào có thể là tiêu đề tốt nhất cho đoạn văn?
A. Để thành công trên Internet => đoạn văn không nói về các cách để thành trên mạng
B. Hỗ trợ từ Youniversity Ventures => ý nhỏ trong bài (đoạn 4)
C. Chia sẻ thành công trên Internet
D. Những video hay nhất từ YouTube => không phù hợp
Câu 9: Đáp án C
Kiến thức: Đọc hiểu
Giải chi tiết:
Từ “This” trong đoạn 2 ám chỉ cái gì?
A. thông tin về trận sóng thần năm 2004
B. trận sóng thần năm 2004 ở Đông Nam Á
C. việc anh ta không thể tìm thấy video
D. những câu chuyện tin tức anh ấy có thể tìm thấy
Thông tin: He found news stories about it, but he couldn’t find any videos. This gave Karin an idea.
Tạm dịch: Anh ta tìm thấy những câu chuyện tin tức về nó, nhưng anh ta không thể tìm thấy bất kỳ video
nào. Điều này đã nảy ra cho Karin một ý tưởng.
Câu 10: Đáp án A
Kiến thức: Đọc hiểu
Giải chi tiết:
Bằng cách nào những nhà đầu tư YouTube đã kiếm được nhiều tiền?
A. Google đã mua công ty YouTube của họ.
B. Họ đã đầu tư rất nhiều tiền vào Google.

Trang 53
C. Họ đã bán những thứ trên trang web Milo.
D. Họ đã kiếm được lợi nhuận từ Youniversity Ventures.
Thông tin: Google made a deal. It bought YouTube for 1.65 USD. As a result, YouTube investors and its
employees made a lot of money.
Tạm dịch: Google đã thực hiện một thỏa thuận. Nó đã mua YouTube với giá 1,65 đô la Mỹ. Do đó, các
nhà đầu tư YouTube và nhân viên của công ty đã kiếm được rất nhiều tiền.
Câu 11: Đáp án D
Kiến thức: Đọc hiểu
Giải chi tiết:
Câu nào dưới đây về Karim không được nhắc đến?
A. Anh ấy kiếm được rất nhiều tiền từ YouTube.
B. Anh ấy là chủ sở hữu của Youniversity Ventures.
C. Anh ấy đã có ý tưởng ban đầu về YouTube.
D. Anh ấy đã tạo ra rất nhiều video mẫu cho YouTube.
Phương án D không được nhắc đến.
Thông tin:
Karim told his friends about this idea. Together, they created a company – YouTube.
As a result, YouTube investors and its employees made a lot of money.
He used money and experience to start a new company called Youniversity Ventures.
Tạm dịch:
Karim nói với bạn bè của mình về ý tưởng này. Họ cùng nhau tạo ra một công ty - YouTube.
Do đó, các nhà đầu tư YouTube và nhân viên của công ty đã kiếm được rất nhiều tiền.
Anh ấy đã sử dụng tiền và kinh nghiệm để thành lập một công ty mới có tên Youniversity Ventures.
Câu 12: Đáp án C
Kiến thức: Đọc hiểu
Giải chi tiết:
Từ nào dưới đây có thể thay thế từ “enormous” trong đoạn 3?
A. considerable (adj): to tát
B. favourable (adj): tốt, thuận tiện
C. huge (adj): rất lớn
D. increasing (adj): tăng
enormous = huge (adj): rất lớn, cực kì lớn
Thông tin: Therefore, they made an enormous amount of money.
Tạm dịch: Do đó, họ đã kiếm được một số tiền rất lớn.
Câu 13: Đáp án C
Kiến thức: Đọc hiểu

Trang 54
Giải chi tiết:
Youniversity Ventures làm gì?
A. Nó mua các công ty Internet như Milo.
B. Nó giúp mọi người tìm thấy những nơi nghỉ mát rẻ tiền trên toàn thế giới.
C. Nó cho mọi người lời khuyên và tiền để bắt đầu kinh doanh.
D. Nó giúp mọi người có được bằng tiến sĩ tại các trường đại học nổi tiếng.
Thông tin: It gives them advice and money to start Internet businesses.
Tạm dịch: Công ty cho họ lời khuyên và tiền để bắt đầu kinh doanh trên mạng.
Câu 14: Đáp án C
Kiến thức: Đọc hiểu
Giải chi tiết:
AirBoB là loại trang web nào?
A. Nó giúp mọi người tìm kiếm nơi nghỉ mát.
B. Nó giúp mọi người tìm thấy các doanh nghiệp thành công.
C. Nó giúp mọi người tổ chức các cuộc họp kinh doanh.
D. Nó giúp mọi người mua hàng hóa và dịch vụ.
Thông tin: Another example is AirBoB. This Web site helps people find for video conferences. People in
different places can use this site to have business meetings.
Tạm dịch: Một ví dụ khác là AirBoB. Trang web này giúp mọi người tìm kiếm cho các hội nghị video. .
Mọi người ở những nơi khác nhau có thể sử dụng trang web này để có các cuộc họp kinh doanh.
Câu 15: Đáp án B
Kiến thức: Đọc hiểu
Giải chi tiết:
Lời khuyên Karim cho các sinh viên muốn bắt đầu kinh doanh là gì?
A. Làm việc cho những người và các công ty thành công.
B. Học hỏi theo những người và các công ty thành công.
C. Đưa tiền cho liên Youniversity Ventures.
D. Học để lấy bằng tiến sĩ từ trường đại học nổi tiếng.
Thông tin: First, find a successful company. Do a lot of research about the company and the top people in
the company. There, copy the way they do things.
Tạm dịch: Đầu tiên, tìm một công ty thành công. Nghiên cứu thật nhiều về công ty và những người hàng
đầu trong công ty. Học theo cách mà họ làm việc.
Câu 16: Đáp án C
Kiến thức: Trọng âm từ có 2 âm tiết
Giải chi tiết:
A. attract /əˈtrækt/

Trang 55
B. divide /dɪˈvaɪd/
C. finish /ˈfɪnɪʃ/
D. invite /ɪnˈvaɪt/
Quy tắc: Hầu hết động từ có 2 âm tiết, trọng âm rơi vào âm tiết thứ 2. Ngoại lệ: ‘finish
Phương án C trọng âm rơi vào âm tiết thứ 1, còn lại rơi vào âm tiết thứ 2.
Câu 17: Đáp án D
Kiến thức: Trọng âm từ có 4 âm tiết
Giải chi tiết:
A. economics /ˌekəˈnɒmɪks/
B. entertainment /ˌentəˈteɪnmənt/
C. radiation /ˌreɪdiˈeɪʃn/
D. geography /dʒiˈɒɡrəfi/
Quy tắc:
- Những từ tận cùng bằng –ic, -tion, -graphy thường có trọng âm rơi vào âm tiết ở ngay trước các đuôi
này.
- Hậu tố -ment không làm thay đổi trọng âm của từ gốc.
Phương án D trọng âm rơi vào âm tiết thứ 2, còn lại rơi vào âm tiết thứ 3
Câu 18: Đáp án B
Kiến thức: Câu đồng nghĩa
Giải chi tiết:
Đảo ngữ câu điều kiện loại 2: Were + S + O, S + would (not) + V_nguyên thể
A. Mặc dù mức lương kém, công việc thật đáng để làm.
B. Điều duy nhất khiến cho công việc này đáng để làm là tiền.
C. Công việc này hoàn toàn không bổ ích, vì vậy mọi người đều muốn có được nó.
D. Công việc này cung cấp một mức lương kém, vì vậy nó đáng để làm.
Các phương án A, C, D không cùng nghĩa với câu gốc.
Tạm dịch: Nếu không phải vì tiền thì công việc này sẽ không đáng để làm.
= Điều duy nhất khiến cho công việc này đáng để làm là tiền.
Câu 19: Đáp án B
Kiến thức: Câu đồng nghĩa
Giải chi tiết:
catch sb doing sth: bắt gặp ai đang làm gì
= be catch red-handed: bị bắt quả tang (đang làm gì)
catch sb do sth: gặp ai làm gì (quan sát hành động từ khi nó bắt đầu đến khi nó kết thúc)
A. Cảnh sát bắt được tên trộm đột nhập vào căn hộ. (sai cấu trúc với “catch”)
B. Cảnh sát bắt được tên trộm đột nhập vào căn hộ.

Trang 56
C. Cảnh sát bắt được tên trộm khi anh ta (anh cảnh sát) đột nhập vào căn hộ. => sai nghĩa
D. Sau đó, tên trộm đã đột nhập vào căn hộ, cảnh sát bắt được anh ta ngay lập tức. => sai nghĩa
Tạm dịch: Tên trộm đã bị cảnh sát bắt quả tang khi anh ta đột nhập vào căn hộ.
= Cảnh sát bắt được tên trộm đột nhập vào căn hộ.
Câu 20: Đáp án C
Kiến thức: Câu đồng nghĩa
Giải chi tiết:
not pay attention to sth: chú tâm, để ý, lắng nghe kĩ,…
= take no notice of sth
A. Anh ấy không có ý định nói chuyện với tôi.
B. Anh ấy không nghe thấy tôi mặc dù tôi đang nói với anh ấy.
C. Anh ấy không chú ý đến lời nói của tôi.
D. Anh ấy không chú ý vì tôi đã không nói một lời nào.
Các phương án A, B, D không cùng nghĩa với câu gốc.
Tạm dịch: Anh không chú ý đến những gì tôi nói.
Câu 21: Đáp án D
Kiến thức: Từ loại, từ vựng
Giải chi tiết:
Sau tobe “are” cần một tính từ.
A. imaginable (adj): giàu trí tưởng tượng
B. imaginative (adj): có thể tưởng tượng
C. imagining (V_ing): tưởng tượng
D. imaginary (adj): tưởng tượng, hư không
Tạm dịch: Những nhân vật như tiên nữ hay phù thủy trong phim hoạt hình Walt Disney hoàn toàn là
tưởng tượng.
Câu 22: Đáp án B
Kiến thức: Sự hòa hợp giữa chủ ngữ và động từ
Giải chi tiết:
Neither S1 nor S2 + V_chia theo S2: Không … cũng không …
S2 là “her brothers” (những người anh của cô ấy), ngôi thứ 3 số nhiều => loại C, D
Cuối câu có “yet” (chưa) => dấu hiệu thì hiện tại hoàn thành
Công thức: S + have + P2
Tạm dịch: Cả Mary và anh trai cô đều vẫn chưa đến bữa tiệc. Họ có thể đang bị kẹt xe.
Câu 23: Đáp án C
Kiến thức: Câu tường thuật đặc biệt
Giải chi tiết:

Trang 57
Dấu hiệu: “two years ago” (2 năm trước)
Công thức: S + am/is/are + said + to have + P2
Chủ ngữ “The old market” (Chợ cũ), động từ “destroy” (phá hủy) => câu bị động
Công thức dạng bị động: S + am/is/are + said + to have been + P2
Tạm dịch: Chợ cũ được cho là đã bị phá hủy trong một vụ hỏa hoạn hai năm trước.
Câu 24: Đáp án D
Kiến thức: Rút gọn mệnh đề quan hệ
Giải chi tiết:
“the movement” (phong trào), động từ “organize” (tổ chức) => câu mang nghĩa bị động
Rút gọn mệnh đề quan hệ mang nghĩa bị động => dùng P2
Câu đầy đủ: We should participate in the movement which is organized to conserve the natural
environment.
Câu rút gọn: We should participate in the movement organized to conserve the natural environment.
Tạm dịch: Chúng ta nên tham gia vào phong trào được tổ chức để bảo tồn môi trường tự nhiên.
Câu 25: Đáp án A
Kiến thức: Từ vựng
Giải chi tiết:
A. no matter: bất kể là, dù cho gì đi chăng nữa
B. although: mặc dù
C. because: bởi vì
D. whether: liệu
Tạm dịch: Tôi đã quyết định mua căn nhà đó. Tôi sẽ không thay đổi suy nghĩ cho dù bạn có nói gì đi nữa.
Câu 26: Đáp án C
Kiến thức: Đại từ quan hệ
Giải chi tiết:
Trong mệnh đề quan hệ:
which + S + V / V : cái mà => thay cho từ chỉ vật phía trước
what + S + V : cái mà
whose + N … : của … => chỉ sở hữu
who + V : người mà … => thay cho từ chỉ người phía trước
population (n): dân số => dùng với “whose”
Tạm dịch: Một loài có nguy cơ tuyệt chủng là loài có dân số quá nhỏ đến mức có nguy cơ bị biến mất.
Câu 27: Đáp án A
Kiến thức: Thì quá khứ tiếp diễn
Giải chi tiết:

Trang 58
Cách dùng: Thì quá khứ tiếp diễn diễn tả một sự việc đang diễn ra (chia quá khứ tiếp diễn) thì bị một
hành động khác cắt ngang (chia quá khứ đơn)
Chủ ngữ “the speech” (bài phát biểu), động từ “make” => câu bị động
Công thức câu bị động thì quá khứ tiếp diễn: S + V_quá khứ đơn while + S + was/were + being + P2
make (nguyên thể) => made (quá khứ) => made (P2)
Tạm dịch: Tôi không thể hiểu tại sao sinh viên không giữ trật tự khi bài phát biểu đang được nói.
Câu 28: Đáp án A
Kiến thức: Thành ngữ
Giải chi tiết:
be as old as the hills: rất cũ, cổ xưa
Tạm dịch: Những câu đùa mà Jack nói thì nó rất là cũ rồi.
Câu 29: Đáp án D
Kiến thức: Thành ngữ
Giải chi tiết:
A. keep pace with: bắt kịp, theo kịp
B. make room for: nhường chỗ cho
C. get rid of: tránh khỏi
D. take into account: chú ý đến, lưu tâm đến, suy xét đến
Tạm dịch: Khi tìm một ngôi nhà mới, cha mẹ nên suy xét đến tất cả các điều kiện cho giáo dục và giải trí
của con họ.
Câu 30: Đáp án A
Kiến thức: Câu tường thuật
Giải chi tiết:
Công thức tường thuật câu hỏi Wh-: S + asked + O + wh-word + S + V_lùi thì
Câu trực tiếp: “How many students are there in your class?”
Sang câu gián tiếp: are there => there were; your => my
Tạm dịch: Cô Brown hỏi tôi xem lớp tôi có bao nhiêu học sinh.
Câu 31: Đáp án A
Kiến thức: Thì quá khứ hoàn thành
Giải chi tiết:
Cách dùng: Thì quá khứ hoàn thành diễn tả sự việc xảy ra và kết thúc trước một sự việc khác trong quá
khứ.
Công thức: S + had + P2
Tạm dịch: Sau khi David đã làm xong bài tập về nhà, cậu ấy lên giường đi ngủ luôn.
Câu 32: Đáp án C
Kiến thức: Đảo ngữ câu điều kiện loại 3

Trang 59
Giải chi tiết:
Dấu hiệu: mệnh đề chính chia “would have + been + P2”
Cách dùng: Diễn tả điều kiện không có thật ở quá khứ dẫn đến kết quả trái với quá khứ
Công thức câu điều kiện loại 3: If + S + had (not) + P2, S + would (not) + have + (been) + P2
Đảo ngữ: Had + S + (not) + P2, S + would (not) + have + (been) + P2
Tạm dịch: Nếu anh ta không từ chức, chúng tôi sẽ buộc phải sa thải anh ta.
Câu 33: Đáp án A
Kiến thức: Từ vựng
Giải chi tiết:
A. optional (adj): tùy chọn, không bắt buộc
B. unique (adj): độc đáo, duy nhất
C. compulsory (adj): bắt buộc
D. limited (adj): hạn chế, giới hạn
Tạm dịch: Sinh viên cũng có cơ hội lựa chọn trong một loạt các khóa học không bắt buộc ở trường đại
học.
Câu 34: Đáp án B
Kiến thức: Cấu trúc với “would rather”
Giải chi tiết:
Dấu hiệu: “there was nothing interesting there” => quá khứ đơn
Cấu trúc: S + would rather + (not) have + P2 … : Ai đó thích là/thà rằng … (trong quá khứ)
Dạng bị động: S + would rather + (not) have + been + P2 …
Tạm dịch: Tôi thà không được mời đến bữa tiệc với bố mẹ còn hơn vì không có gì thú vị ở đó.
Câu 35: Đáp án B
Kiến thức: Mệnh đề danh từ
Giải chi tiết:
Công thức: That + S + V_theo thì + V_chia theo chủ ngữ số ít: Việc mà … thì …
Phương án A sai vì thiếu đại từ quan hệ hoặc chưa đúng dạng rút gọn MĐ quan hệ mang nghĩa chủ động
Phương án C sai vì thừa “it”
Phương án D sai vì thiếu chủ ngữ
Tạm dịch: Việc bạn đã vượt qua kỳ thi của bạn làm cho cha mẹ của bạn hạnh phúc.
Câu 36: Đáp án D
Kiến thức: Cụm từ chỉ mục đích
Giải chi tiết:
to + V-nguyên thể: để … => chỉ mục đích
Cô giơ tay cao. Cô muốn thu hút sự chú ý của giáo viên.
A. Vì cô giáo thu hút cô ấy, cô ấy giơ tay cao.

Trang 60
B. Cô ấy giơ tay cao đến mức thu hút sự chú ý của giáo viên.
C. Mặc dù cô ấy giơ tay cao, cô ấy không thể thu hút sự chú ý của giáo viên.
D. Để thu hút sự chú ý của giáo viên, cô ấy giơ tay cao.
Các phương án A, B, C không cùng nghĩa với câu gốc
Tạm dịch: Cô giơ tay cao. Cô muốn thu hút sự chú ý của giáo viên.
= Để thu hút sự chú ý của giáo viên, cô ấy giơ tay cao.
Câu 37: Đáp án D
Kiến thức: Cụm động từ
Giải chi tiết:
turn down: từ chối
A. detected: phát hiện
B. reviewed: xem xét
C. sacked: sa thải
D. rejected: từ chối
be turned down = be rejected: bị từ chối
Tạm dịch: Mãi cho đến khi tất cả các yêu cầu của họ đã bị từ chối thì các công nhân mới quyết định đình
công để có thêm phúc lợi.
Câu 38: Đáp án B
Kiến thức: Từ vựng
Giải chi tiết:
well-off (adj): giàu có
A. poor (adj): nghèo
B. wealthy (adj): giàu có
C. broke (V2): đổ, vỡ
D. kind (adj): tử tế
Tạm dịch: Cô ấy được nuôi dưỡng trong một gia đình khá giả. Cô ấy không thể hiểu những vấn đề chúng
ta đang phải đối mặt đâu.
Câu 39: Đáp án D
Kiến thức: Từ vựng
Giải chi tiết:
be restored (v): được tu sửa, khôi phục
A. renovated: được cải tạo
B. refurbished: được tân trang
C. strengthened: được làm cho kiên cố, vững chắc
D. devastated: bị phá hủy
=> be restored >< be devastated

Trang 61
Tạm dịch: Cung điện đã bị hư hại nặng nề bởi vụ cháy, nhưng cuối cùng đã được khôi phục lại vẻ tráng lệ
ban đầu.
Câu 40: Đáp án B
Kiến thức: Từ vựng
Giải chi tiết:
state school: trường công lập, trường nhà nước
secondary school: trường trung học, cấp 2
independent school: trường dân lập, trường tư thục
primary school: trường tiểu học, cấp 1
high school: trường phổ thông, cấp 3
=> state schools >< independent schools
Tạm dịch: Tất cả trẻ em đều có thể theo học mà không cần trả học phí tại trường công lập.
Câu 41: Đáp án D
Kiến thức: Phát âm “_s”
Giải chi tiết:
A. bags /bæɡz/
B. days /deɪz/
C. speeds /spiːdz/
D. dates /deɪts/
Quy tắc:
Có 3 cách phát âm phụ âm cuối “s”:
- /s/: Khi từ có tận cùng là các phụ âm vô thanh /p/, k/, /f/, /ð/, /t/
- /ɪz/: Khi từ có tận cùng là các âm /s/, /ʃ/, /tʃ/, /z/, /ʒ/, /dʒ/ (thường có tận cùng là các chữ cái ce, x, z, sh,
ch, s, ge, ss)
- /z/: Khi từ có tận cùng là các nguyên âm và các phụ âm còn lại.
Phần gạch chân phương án D được phát âm là /s/, còn lại là /z/
Câu 42: Đáp án C
Kiến thức: Phát âm “_ed”
Giải chi tiết:
A. searched /sɜːtʃt/
B. cooked /kʊkt/
C described /dɪˈskraɪbd/
D. developed /dɪˈveləpt/
Quy tắc:
Cách phát âm đuôi “-ed”:
- Đuôi “-ed” được phát âm là /ɪd/ khi động từ có phát âm kết thúc là /t/ hay /d/

Trang 62
- Đuôi “-ed” được phát âm là /t/ khi động từ có phát âm kết thúc là /s/,/f/,/p/,/ʃ/,/tʃ/,/k/
- Đuôi “-ed” được phát âm là /d/ với các trường hợp còn lại
Phần gạch chân phương án C được phát âm là /d/, còn lại là /t/
Câu 43: Đáp án B
Kiến thức: Rút gọn mệnh đề quan hệ
Giải chi tiết:
Dùng phân từ (V_ed/V cột 3) để rút gọn mệnh đề quan hệ mang nghĩa bị động. Khi đó, ta bỏ đại từ quan
hệ, bỏ tobe và chỉ giữ lại V_ed/V3
Câu đầy đủ: Some snakes have hollow teeth which/that are called fans that they use to poison their
victims.
Câu rút gọn: Some snakes have hollow teeth called fans that they use to poison their victims.
Sửa: they are called => called hoặc which/that are called
Tạm dịch: Một số loài rắn có hàm răng rỗng được gọi là quạt mà chúng dùng để đầu độc nạn nhân.
Câu 44: Đáp án C
Kiến thức: Sự hòa hợp giữa chủ ngữ và động từ
Giải chi tiết:
What + S + V…: Điều mà … => giữ vai trò như một cụm danh từ => động từ theo sau chia số ít
Sửa: were => was
Tạm dịch: Điều mà người phụ nữ đang nói quan trọng đến mức tôi yêu cầu mọi người ngừng nói chuyện
để lắng nghe cô ấy nói.
Câu 45: Đáp án A
Kiến thức: Cấu trúc với “enough”
Giải chi tiết:
Cấu trúc: S + be + (not) + adj + enough + to V_nguyên thể: Ai đó (không) đủ cái gì để làm gì
intelligent (adj): thông minh => đứng trước “enough”
Sửa: enough intelligent => intelligent enough
Tạm dịch: George không đủ thông minh để đỗ vào lớp kinh tế học này mà không cần giúp đỡ.
Câu 46: Đáp án A
Kiến thức: Sự kết hợp từ
Giải chi tiết:
do a single task: làm một nhiệm vụ
scarce (adj): khan hiếm, hiếm có
lonely (adj): cô đơn
unique (adj): độc đáo, độc nhất
Tạm dịch: Bạn phải làm một nhiệm vụ mà thường kéo dài khoảng 3 phút.
Câu 47: Đáp án B

Trang 63
Kiến thức: Từ vựng
Giải chi tiết:
optical (adj): thuộc về mắt
visual (adj): trực quan
noticeable (adj): đáng chú ý
obvious (adj): rõ ràng
Tạm dịch: Một nhiệm vụ có thể là giải quyết vấn đề, có nghĩa là bạn phải xem xét một số thông tin trực
quan và sau đó thảo luận vấn đề với cộng sự của mình.
Câu 48: Đáp án B
Kiến thức: Thức giả định
Giải chi tiết:
Công thức: It is essential that + S + V_nguyên thể: Cần thiết để ai làm gì
helpful (adj): hữu ích
probable (adj): có lẽ đúng
successful (adj): thành công
Tạm dịch: Bạn có thể được cho xem hình ảnh, bản vẽ, sơ đồ, bản đồ, kế hoạch, quảng cáo hoặc đồ họa
máy tính và điều cần thiết là bạn nên nghiên cứu chúng một cách cẩn thận.
Câu 49: Đáp án B
Kiến thức: Sự kết hợp từ
Giải chi tiết:
make suggestions: đề xuất, đựa ra gợi ý, đề nghị
statement (n): khẳng định, bài phát biểu
speech (n): bài phát biểu
idea (n): ý tưởng
Tạm dịch: Trong khi bạn đang thực hiện nhiệm vụ, giám khảo có thể sẽ nói rất ít và bạn nên hỏi cộng sự
của mình và đưa ra gợi ý nếu người đó không nói nhiều.
Câu 50: Đáp án C
Kiến thức: Thành ngữ
Giải chi tiết:
take turns (in something/to do something): làm gì đó theo thứ tự
Tạm dịch: bao gồm việc luân phiên nhau đưa ra ý kiến và trả lời một cách thích hợp, mặc dù cuối cùng,
có thể có thể đồng ý với điều đó không đồng ý.

SỞ GD & ĐT VĨNH PHÚC ĐỀ KIỂM TRA CHẤT LƯỢNG LẦN 1 – KHỐI 12


TRƯỜNG THPT ĐỘI CẤN NĂM HỌC: 2019 – 2020
Mã đề 132 MÔN: TIẾNG ANH

Trang 64
Thời gian làm bài: 60 phút; không kể thời gian phát đề

Mark the letter A, B, C, or D on your answer sheet to indicate the sentence that best combines each pair
of sentences in the following questions.
Câu 1 (VD): The proposal seemed like a good idea. The manager refused it.
A. The manager didn't like to proposal because it didn't seem a good idea.
B. Since the proposal seemed like a good idea, the manager refused it.
C. The manager refused the proposal although it seemed like a good idea.
D. The proposal didn't seem like a good idea, so the manager didn't accept it.
Câu 2 (VD): Sports clear my mind. They also help me maintain a healthy lifestyle.
A. Not only do sports clear my mind, but they also help me maintain a healthy lifestyle.
B. Neither do sports clear my mind nor help me maintain a healthy lifestyle.
C. Sports clear my mind so that I can maintain a healthy lifestyle.
D. All that sports can do is to clear my mind instead of maintaining a healthy lifestyle.
Mark the letter A, B, C, or D on your answer sheet to indicate the word that differs from the rest in the
position of the primary stress in each of the following questions.
Câu 3 (NB): A. argument B. admission C. domestic D. acquaintance
Câu 4 (NB): A. precede B. confide C. modern D. remind
Mark the letter A, B, C, or D on your answer sheet to indicate the most suitable response to complete each
of the following exchanges.
Câu 5 (TH): Teacher: “Jenny, you’ve made a good job on the project”.
Jenny: “ _____________”
A. Not really. I didn’t. B. Oh. I guess so. C. Never mention it. D. Thank you!
Câu 6 (TH): Mark is talking to his brother about his favorite color.
Mark: “Why do you like red color?”
Mark’s brother: “ ____________________”
A. You make me feel more confident. B. It’s a good idea, thank you.
C. It’s the color of love and passion. D. You didn’t do anything for me.
Read the following passage and mark the letter A, B, C, or D on your answer sheet to indicate the correct
word or phrase that best fits each of the numbered blanks.
Monkeys are similar to humans in many ways. For example, the (7)__________in a monkey family, such
as between brother and sister, are often very close. A team of researchers studied a pair of bonobo
monkeys called Kanzi and Panbanisha. (8)___________brother and sister team had learned how to make
knives from stone. So the researchers decided to record how good they were. The researchers put a
banana inside a box. Then they gave the bonobos what they needed to make knife. The (9)__________of
this knife was to cut open the box to get the banana. Kanzi made a very good knife, but his sister

Trang 65
Panbanisha could not. Kanzi saw his sister was feeling (10)___________and so he tried to give his knife
to her. However, the scientists did not let him. Even in this situation, Kanzi knew what to do. When no
one was looking, he put his knife (11)__________ his sister could easily find it, and she finally got her
banana. To researchers, it was obvious from Kanzi's behavior that he really wanted to help his sister.
Câu 7 (TH): A. relatives B. relation C. relate D. relationships
Câu 8 (NB): A. Those B. The C. These D. A
Câu 9 (TH): A. performance B. usage C. function D. application
Câu 10 (TH): A. disappointing B. disappointed C. disappointment D. disappoint
Câu 11 (TH): A. where B. how C. that D. what
Mark the letter A, B, C, or D on your answer sheet to indicate the word(s) OPPOSITE in meaning to the
underlined word(s) in each of the following questions.
Câu 12 (NB): Some people think their abilities have limitations but with encouragement they can perform
much better than expected.
A. certainties B. boundaries C. possibilities D. advantages
Câu 13 (TH): Many women prefer to use cosmetics to enhance their beauty and make them look younger.
A. improve B. worsen C. maximize D. enrich
Read the passage and mark the letter A, B, C, or D on your answer sheet to indicate the correct answer to
each of the questions.
In today's competitive world, what responsible parents would not want to give their children the best
possible start in life? For this reason, many parents want their children, often as young as ten months old,
to become familiar with computers. They seem to think that if their children grow up with computers,
they will be better equipped to face the challenges of the future.
No one has proved that computers make children more creative or more intelligent. The truth may even
be the opposite. Educational psychologists claim that too much exposure to computers, especially for the
very young, may negatively affect normal brain development. Children gain valuable experience of the
world from their interaction with physical objects. Ten-month-old babies may benefit more from bumping
their heads or putting various objects in their mouths than they will from staring at eye-catching cartoons.
A four-year-old child can improve hand-eye coordination and understand cause and effect better by
experimenting with a crayon than by moving a cursor around a computer screen. So, as educational
psychologists suggest, instead of government funding going to more and more computer classes, it might
be better to devote resources to music and art programs.
It is ludicrous to think that children will fall behind if they are not exposed to computers from an early
age. Time is too precious to spend with a "mouse". Now is the time when they should be out there
learning to ride a bike. There will be time later on for them to start banging away at keyboards.
Câu 14 (TH): Children who spend a lot of time on their computers _________.
A. will suffer from brain damage

Trang 66
B. do not necessarily make more progress than those who don't
C. tend to have more accidents than those who don't
D. tend to like music and art more than those who don't
Câu 15 (VD): What would be an appropriate title for this passage?
A. More computers mean brighter future B. Never too early to start
C. Computers in schools D. Let kids be kids
Câu 16 (NB): The pronoun "they" in paragraph 2 refers to________.
A. heads B. mouths C. babies D. objects
Câu 17 (TH): What is TRUE according to the passage?
A. It is better for children to take computer lessons than art lessons.
B. Parents should not put off buying a computer for their children.
C. There is no evidence that children who use computers are cleverer than those who do not.
D. Computers seriously harm children's eyesight.
Câu 18 (TH): The word "ludicrous" paragraph 3 mostly means _________.
A. sensible B. ridiculous C. humorous D. ironic
Mark the letter A, B, C, or D on your answer sheet to indicate the word whose underlined part differs
from the other three in pronunciation in each of the following questions.
Câu 19 (NB): A. blood B. food C. mood D. boot
Câu 20 (NB): A. missed B. hoped C. washed D. removed
Mark the letter A, B, C, or D on your answer sheet to indicate the word(s) CLOSEST in meaning to the
underlined word(s) in each of the following questions.
Câu 21 (VDC): Every time he opens his mouth, he immediately regrets what he said. He is always putting
his foot in his mouth.
A. speaking indirectly B. saying embarrassing things
C. doing things in the wrong order D. making a mistake
Câu 22 (TH): In spite of her embarrassment before Rodya's urgent and challenging look, she could not
deny herself that satisfaction.
A. hard B. curious C. demanding D. difficult
Read the following passage and mark the letter A, B, C, or D on your answer sheet to indicate the correct
answer to each of the questions.
Culture is a word in common use with complex meanings, and is derived, like the term broadcasting,
from the treatment and care of the soil and of what grows on it. It is directly related to cultivation and the
adjectives cultural and cultured are part of the same verbal complex. A person of culture has identifiable
attributes, among them a knowledge of and interest in the arts, literature, and music. Yet the word culture
does not refer solely to such knowledge and interest nor, indeed, to education. At least from the 19th
century onwards, under the influence of anthropologists and sociologists, the word culture has come to be

Trang 67
used generally both in the singular and the plural (cultures) to refer to a whole way of life of people,
including their customs, laws, conventions, and values.
Distinctions have consequently been drawn between primitive and advanced culture and cultures,
between elite and popular culture, between popular and mass culture, and most recently between national
and global cultures. Distinctions have been drawn too between culture and civilization; the latter is a word
derived not, like culture or agriculture, from the soil, but from the city. The two words are sometimes
treated as synonymous. Yet this is misleading. While civilization and barbarism are pitted against each
other in what seems to be a perpetual behavioral pattern, the use of the word culture has been strongly
influenced by 6 conceptions of evolution in the 19th century and of development in the 20th century.
Cultures evolve or develop. They are not static. They have twists and turns. Styles change. So do
fashions. There are cultural processes. What, for example, the word cultured means has changed
substantially since the study of classical (that is, Greek and Roman) literature, philosophy, and history
ceased in the 20th century to be central to school and university education. No single alternative focus
emerged, although with computers has come electronic culture, affecting kinds of study, and most
recently digital culture. As cultures express themselves in new forms not everything gets better or more
civilized.
The multiplicity of meanings attached to the word made and will make it difficult to define. There is no
single, unproblematic definition, although many attempts have been made to establish one. The only non-
problematic definitions go back to agricultural meaning (for example, cereal culture or strawberry
culture) and medical meaning (for example, bacterial culture or penicillin culture). Since in anthropology
and sociology we also acknowledge culture clashes, culture shock, and counter-culture, the range of
reference is extremely wide.
Câu 23 (VD): It can be inferred from the passage that since the 20th century ________.
A. all schools and universities have taught classical literature, philosophy, and history
B. classical literature, philosophy, and history have been considered as core subjects
C. classical literature, philosophy, and history have not been taught as core subjects
D. schools and universities have not taught classical literature, philosophy, and history
Câu 24 (NB): The word "It" in paragraph 1 refers to ________.
A. broadcasting B. the soil C. culture D. the treatment and care
Câu 25 (VD): The passage mainly discusses ________.
A. the derivatives of the word culture B. the distinction between culture and civilization
C. the figurative meanings of the word culture D. the multiplicity of meanings of the word
culture
Câu 26 (TH): Which of the following is NOT true about the word culture?
A. Its use has been considerably changed. B. It is a word that cannot be defined.
C. It differs from the word civilization. D. It evolves from agriculture.

Trang 68
Câu 27 (TH): The author remarks that culture and civilization are the two words that ________.
A. share the same word formation pattern
B. have nearly the same meaning
C. are both related to agriculture and cultivation
D. do not develop from the same meaning
Câu 28 (TH): The word "attributes" in paragraph 1 most likely means ________.
A. qualities B. aspects C. fields D. skills
Câu 29 (TH): The word "static" in paragraph 2 could best be replaced by _________.
A. dense B. balanced C. unchanged D. regular
Câu 30 (TH): It is stated in paragraph 1 that a cultured person _______.
A. does a job relevant to education
B. has knowledge of arts, literature, and music
C. has a job related to cultivation
D. takes care of the soil and what grows on it
Mark the letter A, B, C, or D on your answer sheet to indicate the underlined part that needs correction in
each of the following questions.
Câu 31 (NB): Sleeping, resting, and to drink fruit juice are the best ways to cure for a cold.
A. Sleeping B. to drink C. juice D. best ways
Câu 32 (NB): The novel was such interesting that I read it from the beginning to the end in 4 hours.
A. such B. read C. the beginning D. in
Câu 33 (NB): The result of the studies have had a strong impact on future developments.
A. of B. have had C. strong D. on
Mark the letter A, B, C, or D on your answer sheet to indicate the sentence that is CLOSEST in meaning
to each of the following questions.
Câu 34 (VD): The news surprised everyone in the family.
A. Everyone in the family found the news surprising.
B. The news in the family made everyone surprisingly.
C. The news made everyone surprised in the family.
D. Everyone was surprised by the news in the family.
Câu 35 (VDC): People say that Cameron was the best director of his time.
A. Cameron is said to be best director of his time.
B. Cameron was said to have been the best director of his time.
C. It was said that Cameron was the best director of his time.
D. Cameron is said to have been the best director of his time.
Câu 36 (VD): "Be sorry for sending the wrong information, Kate" said Rita.
A. Kate sent the wrong information, and she was sorry.

Trang 69
B. Rita apologized to Kate for sending her the wrong information.
C. Rita was sorry about Kate's sending the wrong information.
D. Rita was sorry for not sending Kate the information.
Mark the letter A, B, C, or D on your answer sheet to indicate the correct answer to each of the following
questions.
Câu 37 (NB): If you ________a wallet in the street, what would you do with it?
A. found B. have found C. find D. had found
Câu 38 (TH): Please accept our ________apology for the inconvenience this delay is causing all the
passengers here at Pearson International Airport.
A. completed B. original C. estimated D. sincere
Câu 39 (TH): While Peter ________the rose bush in the back yard, the phone rang.
A. were watering B. has watered C. watered D. was watering
Câu 40 (TH): Some modern couples organize their marriage and work out the tasks and duties, which
may gradually turn their marriage into a business or ________relationship.
A. contract B. contractually C. contractual D. contracting
Câu 41 (TH): You look so tired! You _______out too late last night.
A. must have been B. had been C. had to be D. should have been
Câu 42 (VD): I wonder if you could _______me a favour, Jackon.
A. give B. bring C. do D. make
Câu 43 (VD): The song _______by our listeners as their favorite of the week is "Goodbye Baby" by
Tunesmiths.
A. was chosen B. is chosen C. chosen D. choosing
Câu 44 (NB): E-mail allows people _______ in touch, regardless of distance.
A. staying B. to stay C. stay D. to staying
Câu 45 (TH): I'm really sleepy today. I wish I _______ Bob to the airport last night.
A. hadn't had to take B. didn't have to take C. didn't take D. weren't taking
Câu 46 (TH): Finding a job in this time of economic crisis is becoming ________.
A. more than difficult B. most and most difficult
C. more and more difficult D. more difficult than
Câu 47 (TH): ________as the most important crop in Hawaii is sugar cane.
A. It is ranked B. That ranks C. It ranks D. What ranks
Câu 48 (TH): Sometimes at weekends, my father helps my mom with _______meals.
A. keeping B. preparing C. making D. arranging
Câu 49 (VD): Nobody took any _______of the warning and they went swimming in the contaminated
water.
A. regard B. recognition C. notice D. attention

Trang 70
Câu 50 (VD): As Joe's roommate, I find him a fairly nice fellow, even if at times it is not easy to
______his noisy behavior.
A. put up with B. look up to C. get on with D. catch up with

Đáp án
1-C 2-A 3-A 4-C 5-D 6-C 7-D 8-B 9-C 10-B
11-A 12-D 13-B 14-A 15-D 16-C 17-C 18-B 19-A 20-D
21-B 22-B 23-C 24-C 25-D 26-B 27-D 28-A 29-C 30-B
31-B 32-A 33-B 34-A 35-D 36-B 37-A 38-D 39-D 40-C
41-A 42-C 43-C 44-B 45-A 46-C 47-D 48-B 49-C 50-A

LỜI GIẢI CHI TIẾT


Câu 1: Đáp án C
Kiến thức: Câu đồng nghĩa, liên từ
Giải chi tiết:
because: bởi vì
since: bởi vì
although: mặc dù
so: vì vậy, vì thế
Tạm dịch: Đề xuất có vẻ là một ý tưởng tốt. Giám đốc đã từ chối nó.
A. Giám đốc không thích đề xuất bởi vì nó dường như không phải là một ý tưởng hay. => sai nghĩa
B. Bởi vì đề xuất có vẻ là một ý tưởng hay, giám đốc đã từ chối nó. => sai nghĩa
C. Giám đốc đã từ chối đề xuất mặc dù nó có vẻ là một ý tưởng tốt.
D. Đề xuất có vẻ không giống một ý tưởng hay, vì vậy giám đốc đã không chấp nhận nó. => sai nghĩa
Câu 2: Đáp án A
Kiến thức: Câu đồng nghĩa, đảo ngữ với “Not only...but also...”
Giải chi tiết:
Cấu trúc: Not only + trợ V + S + V_nguyên thể, but + S + also + V

Tạm dịch: Thể thao khiến đầu óc tôi thư giãn. Chúng cũng giúp tôi duy trì một lối sống lành mạnh.
A. Thể thao không chỉ khiến đầu óc tôi thư giãn, mà chúng còn giúp tôi duy trì một lối sống lành mạnh.
B. Thể thao không khiến đầu óc tôi thư giãn cũng không giúp tôi duy trì một lối sống lành mạnh. => sai
nghĩa
C. Thể thao khiến đầu óc tôi thư giãn để mà tôi có thể duy trì một lối sống lành mạnh. => sai nghĩa
D. Tất cả điều mà thể thao có thể làm là khiến đầu óc tôi thư giãn thay vì duy trì một lối sống lành mạnh.
=> sai nghĩa

Trang 71
Câu 3: Đáp án A
Kiến thức: Trọng âm của từ có 3 âm tiết
Giải chi tiết:
A. argument /ˈɑːɡjumənt/
B. admission /ədˈmɪʃn/
C. domestic /dəˈmestɪk/
D. acquaintance /əˈkweɪntəns/
Phương án A trọng âm rơi vào âm tiết thứ nhất, còn lại là âm thứ 2
Câu 4: Đáp án C
Kiến thức: Trọng âm của từ có 2 âm tiết
Giải chi tiết:
A. precede /prɪˈsiːd/
B. confide /kənˈfaɪd/
C. modern /ˈmɒdn/
D. remind /rɪˈmaɪnd/
Phương án C trọng âm rơi vào âm tiết thứ nhất, còn lại là âm thứ 2
Câu 5: Đáp án D
Kiến thức: Ngôn ngữ giao tiếp
Giải chi tiết:
Tạm dịch:
Teacher: “Jenny, em đã làm rất tốt dự án”
Jenny: “_________."
A. Không thực sự. Em đã không làm.
B. Ồ. Em đoán vậy.
C. Đừng bao giờ nhắc đến nó.
D. Cảm ơn cô!
Các phản hồi A, B, C không phù hợp ngữ cảnh.
Câu 6: Đáp án C
Kiến thức: Ngôn ngữ giao tiếp
Giải chi tiết:
Tạm dịch:
Mark đang nói chuyện với anh trai mình về màu yêu thích của anh ấy.
Mark: “Tại sao anh lại thích màu đỏ?”
Anh trai Mark: “_________."
A. Em khiến anh thấy tự tin.
B. Đó là một ý kiến hay, cảm ơn.

Trang 72
C. Nó là màu của tình yêu và sự đam mê.
D. Em đã không làm được gì cho anh.
Các phản hồi A, B, D không phù hợp ngữ cảnh.
Câu 7: Đáp án D
Kiến thức: Từ loại, từ vựng
Giải chi tiết:
A. relatives (n): những người họ hàng
B. relation (n): sự liên quan
C. relate (v): liên quan
D. relationships (n): mối quan hệ
Sau mạo từ “the” cần một danh từ => loại C
For example, the (7) relationships in a monkey family, such as between brother and sister, are often very
close.
Tạm dịch: Ví dụ, các mối quan hệ trong một gia đình nhà khỉ, như là giữa anh trai và em gái, thì thường
rất gần gũi.
Câu 8: Đáp án B
Kiến thức: Mạo từ
Giải chi tiết:
Danh từ “team” ở dạng số ít => trước nó cần mạo từ hoặc đại từ chỉ định đi với danh từ số ít
Those/These + N số nhiều => Loại A, C
A + N số ít, chưa xác định
The + N đã xác định
Danh từ “team” đã xác định, đã được đề cập trong câu trước => dùng “the”
(8) The brother and sister team had learned how to make knives from stone.
Tạm dịch: Cặp anh trai và em gái này đã học cách làm dao từ đá.
Câu 9: Đáp án C
Kiến thức: Từ vựng
Giải chi tiết:
A. performance (n): màn trình diễn
B. usage (n): cách sử dụng
C. function (n): chức năng
D. application (n): sự ứng dụng
The (9) function of this knife was to cut open the box to get the banana.
Tạm dịch: Chức năng của con dao này là để cắt mở chiếc hộp để lấy quả chuối.
Câu 10: Đáp án B
Kiến thức: Từ loại

Trang 73
Giải chi tiết:
Sau động từ tri giác “feel” cần một tính từ chỉ cảm giác
A. disappointing (adj): thất vọng (bản chất của thứ gì đó)
B. disappointed (adj): thất vọng (cảm giác về cái gì)
C. disappointment (n): sự thất vọng
D. disappoint (v): làm cho thất vọng
Kanzi saw his sister was feeling (10) disappointed ...
Tạm dịch: Kanzi đã thấy em gái mình cảm thấy thất vọng...
Câu 11: Đáp án A
Kiến thức: Đại từ quan hệ
Giải chi tiết:
A. where = in/at which: nơi mà
B. how: như thế nào
C. that: cái mà, người mà
D. what = the things that: cái mà
When no one was looking, he put his knife (11) where his sister could easily find it, and she finally got
her banana.
Tạm dịch: Khi không có ai nhìn, nó đã đặt con dao của mình ở nơi mà em gái nó có thể dễ dàng tìm thấy,
và cuối cùng em gái nó có thể lấy được quả chuối của mình.
Câu 12: Đáp án D
Kiến thức: Từ vựng, từ trái nghĩa
Giải chi tiết:
limitations (n): những hạn chế
A. certainties (n): sự chắc chắn
B. boundaries (n): những ranh giới
C. possibilities (n): những khả năng, điều có thể
D. advantages (n): những thuận lợi
=> limitations >< advantages
Tạm dịch: Một vài người nghĩ khả năng của họ có những hạn chế nhưng với sự khích lệ, họ có thể biểu
hiện tốt hơn mong đợi nhiều.
Câu 13: Đáp án B
Kiến thức: Từ vựng, từ trái nghĩa
Giải chi tiết:
enhance (v): nâng cao
A. improve (v): cải thiện
B. worsen (v): tệ hơn

Trang 74
C. maximize (v): tối ưu hóa
D. enrich (v): làm giàu hơn
=> enhance >< worsen
Tạm dịch: Nhiều phụ nữ thích sử dụng mĩ phẩm hơn để nâng cao vẻ đẹp của họ và khiến họ trông trẻ hơn.
Câu 14: Đáp án A
Kiến thức: Đọc hiểu
Giải chi tiết:
Trẻ em người mà dành nhiều thời gian vào máy tính ________.
A. sẽ bị tổn thương não
B. không đạt được nhiều tiến bộ cần thiết hơn những đứa trẻ khác không dùng máy tính
C. có xu hướng có nhiều tai nạn hơn những đứa trẻ không dùng máy tính
D. có xu hướng thích âm nhạc và mĩ thuật hơn những đứa trẻ không dùng máy tính
Thông tin: Educational psychologists claim that too much exposure to computers, especially for the very
young, may negatively affect normal brain development.
Tạm dịch: Các nhà tâm lí về giáo dục khẳng định rằng việc tiếp xúc quá nhiều với máy tính, đặc biệt là
với trẻ quá nhỏ, có thể gây tác động tiêu cực tới sự phát triển bình thường của não bộ.
Câu 15: Đáp án D
Kiến thức: Đọc hiểu
Giải chi tiết:
Câu nào sẽ là một nhan đề phù hợp cho bài đọc này?
A. Nhiều máy tính hơn có nghĩa là tương lai tươi sáng hơn
B. Không bao giờ quá sớm để bắt đầu
C. Máy tính ở các trường học
D. Hãy để trẻ em là trẻ em
Thông tin:
- So, as educational psychologists suggest, instead of government funding going to more and more
computer
classes, it might be better to devote resources to music and art programs.
- It is ludicrous to think that children will fall behind if they are not exposed to computers from an early
age.
Tạm dịch:
- Vì vậy, khi những nhà tâm lí học giáo dục đề nghị, thay vì chính phủ sẽ gây quĩ cho nhiều các lớp học
máy tính, nó có thể tốt hơn khi tận dụng nguồn tài nguyên đó vào các chương trình nghệ thuật và âm
nhạc.
- Thật lố bịch khi nghĩ rằng trẻ em sẽ bị bỏ lại phía sau nếu không được tiếp xúc với máy tính từ sớm.
Các phương án A, B, C không phù hợp

Trang 75
Câu 16: Đáp án C
Kiến thức: Đọc hiểu
Giải chi tiết:
Đại từ “they” trong đoạn 2 đề cập đến_________.
A. những cái đầu
B. những cái miệng
C. những đứa trẻ
D. những đồ vật
Thông tin: Ten-month-old babies may benefit more from bumping their heads or putting various objects
in their mouths than they will from staring at eye-catching cartoons.
Tạm dịch: Trẻ 10 tháng tuổi có thể đạt được lợi từ việc va đầu chúng vào đâu đó hoặc cho những đồ vật
khác nhau vào trong miệng hơn là chúng đạt được từ việc nhìn chằm chằm vào các bộ phim hoạt hình bắt
mắt.
Câu 17: Đáp án C
Kiến thức: Đọc hiểu
Giải chi tiết:
Cái nào dưới đây đúng theo đoạn văn?
A. Tốt hơn cho trẻ khi tham gia các tiết học máy tính thay vì các tiết học mĩ thuật
B. Cha mẹ không nên trì hoãn việc mua máy tính cho trẻ.
C. Không có bằng chứng rằng trẻ em mà sử dụng máy tính thì thông minh hơn những đứa trẻ không sử
dụng.
D. Máy tính gây hại đến thị lực của trẻ một cách nghiêm trọng.
Thông tin: No one has proved that computers make children more creative or more intelligent.
Tạm dịch: Không một ai đã chứng minh rằng máy tính khiến trẻ thông minh hoặc sáng tạo hơn.
Câu 18: Đáp án B
Kiến thức: Đọc hiểu
Giải chi tiết:
Từ “ludicrous” trong đoạn 3 gần nghĩa nhất với _______.
ludicrous (adj): lố bịch
A. sensible (adj): hợp lí
B. ridiculous (adj): ngớ ngẩn
C. humorous (adj): hài hước
D. ironic (adj): trớ trêu
=> ludicrous = ridiculous: lố bịch, ngớ ngẩn
Câu 19: Đáp án A
Kiến thức: Phát âm “oo”

Trang 76
Giải chi tiết:
A. blood /blʌd/
B. food /fuːd/
C. mood /muːd/
D. boot /buːt/
Phần gạch chân phương án A được phát âm là /ʌ/, còn lại là /uː/
Câu 20: Đáp án D
Kiến thức: Phát âm “ed”
Giải chi tiết:
A. missed /mɪst/
B. hoped /həʊpt/
C. washed /wɒʃt/
D. removed /rɪˈmuːvd/
Quy tắc:
Cách phát âm đuôi “-ed”:
- Đuôi “-ed” được phát âm là /ɪd/ khi động từ có phát âm kết thúc là /t/ hay /d/
- Đuôi “-ed” được phát âm là /t/ khi động từ có phát âm kết thúc là /s/,/f/,/p/,/ʃ/,/tʃ/,/k/
- Đuôi “-ed” được phát âm là /d/ với các trường hợp còn lại
Phần gạch chân phương án D được phát âm là /d/, còn lại là /t/
Câu 21: Đáp án B
Kiến thức: Thành ngữ
Giải chi tiết:
putting his foot in his mouth (idiom): nói những điều ngu ngốc, gây lúng túng (không nên nói)

A. speaking directly (v): nói trực tiếp


B. saying embarrassing things (v): nói những điều gây lúng túng
C. doing things in wrong order (v); làm mọi thứ sai trật tự
D. making a mistake (v): mắc lỗi
=> putting his foot in his mouth = saying embarrassing things: nói những điều gây lúng túng
Tạm dịch: Mỗi khi anh ấy mở miệng, anh ấy ngay lập tức nói hối hận với những gì anh ấy đã nói. Anh ấy
luôn nói những điều gây lúng túng.
Câu 22: Đáp án B
Kiến thức: Từ vựng, từ đồng nghĩa
Giải chi tiết:
challenging (adj): thách thức, khiêu khích
A. hard (adj): khó khăn

Trang 77
B. curious (adj): tò mò
C. demanding (adj): khắc nghiệt
D. difficult (adj): khó khăn
=> challenging = curious: tò mò, thách thức
Tạm dịch: Mặc dù bối rối trước cái nhìn hối thúc và đầy thách thức của Rodya, cô ấy cũng không thể phủ
nhận sự thỏa mãn đó.
Câu 23: Đáp án C
Kiến thức: Đọc hiểu
Giải chi tiết:
Có thể suy ra suy ra từ đoạn văn rằng từ thế kỉ 20 _________.
A. tất cả các trường học và đại học đã dạy văn học cổ điển, triết học và lịch sử
B. văn học cổ điển, triết học và lịch sử được coi như là các môn học chính
C. văn học cổ điển, triết học và lịch sử không được dạy như là các môn học chính
D. các trường học và đại học đã không dạy văn học cổ điển, triết học và lịch sử
Thông tin: What, for example, the word cultured means has changed substantially since the study of
classical (that is, Greek and Roman) literature, philosophy, and history ceased in the 20th century to be
central to school and university education.
Tạm dịch: Ví dụ, từ có nghĩa là văn hóa đã thay đổi đáng kể từ khi nghiên cứu về văn học, triết học và
lịch sử cổ điển (đó là Hy Lạp và La Mã) đã dừng lại ở thế kỷ 20 để trở thành trọng tâm của giáo dục
trường học và đại học.
Câu 24: Đáp án C
Kiến thức: Đọc hiểu
Giải chi tiết:
Từ “It” trong đoạn 1 đề cập đến __________.
A. việc phát sóng
B. đất
C. văn hóa
D. cách đối xử và chăm sóc
Thông tin: Culture is a word in common use with complex meanings, and is derived, like the term
broadcasting, from the treatment and care of the soil and of what grows on it. It is directly related to
cultivation and the adjectives cultural and cultured are part of the same verbal complex.
Tạm dịch: Văn hóa là một từ thông dụng sử dụng với nhiều ý nghĩa phức tạp, và được bắt nguồn, giống
như khái niệm phát sóng, từ việc xử lý và chăm sóc đất và những gì nuôi trồng trên đó. Nó liên quan trực
tiếp đến việc canh tác và các tính từ liên quan đến văn hóa và có văn hóa là một phần của cùng một phức
hợp bằng lời nói.
Câu 25: Đáp án D

Trang 78
Kiến thức: Đọc hiểu
Giải chi tiết:
Bài văn chủ yếu đề cập đến __________.
A. nguồn gốc của từ văn hóa
B. sự khác biệt giữa văn hóa và văn minh
C. những ý nghĩa điển hình của từ văn hóa
D. tính đa nghĩa của từ văn hóa
Thông tin: Culture is a word in common use with complex meanings,... The multiplicity of meanings
attached to the word made and will make it difficult to define.
Tạm dịch: Văn hóa là một từ thông dụng sử dụng với nhiều ý nghĩa phức tạp,... Tính đa nghĩa của từ gắn
liền với từ được tạo ra và sẽ gây khó khăn cho việc định nghĩa.
Câu 26: Đáp án B
Kiến thức: Đọc hiểu
Giải chi tiết:
Cái nào dưới đây không đúng với từ văn hóa?
A. Tính ứng dụng của nó đã thay đổi đáng kể.
B. Nó là một từ không thể định nghĩa
C. Nó khác với từ văn minh
D. Nó phát triển từ nông nghiệp
Thông tin:
- Culture is a word in common use with complex meanings, and is derived, like the term broadcasting,
from the treatment and care of the soil and of what grows on it.
- Distinctions have been drawn too between culture and civilization...
- ...the word cultured means has changed substantially,...
Tạm dịch:
- Văn hóa là một từ thông dụng sử dụng với nhiều ý nghĩa phức tạp, và được bắt nguồn, giống như khái
niệm phát sóng, từ việc xử lý và chăm sóc đất và những gì nuôi trồng trên đó.
- Sự khác biệt đã được rút ra giữa văn hóa và nền văn minh...
- ... từ văn hóa có nghĩa đã thay đổi đáng kể...
Các phương án A, C, D đều được nhắc đến.
Câu 27: Đáp án D
Kiến thức: Đọc hiểu
Giải chi tiết:
Tác giả ghi lại rằng văn hóa và văn minh là 2 từ mà ________.
A. có cùng mẫu hình thành từ
B. có nghĩa gần như nhau

Trang 79
C. cả hai đều liên quan đến nông nghiệp và canh tác
D. không phát triển từ cùng một ý nghĩa
Thông tin: Distinctions have been drawn too between culture and civilization; the latter is a word derived
not, like culture or agriculture, from the soil, but from the city.
Tạm dịch: Sự khác biệt đã được rút ra giữa văn hóa và nền văn minh; từ sau là một từ có nguồn gốc,
giống như văn hóa hoặc nông nghiệp, không phải từ đất, mà là từ thành phố.
Câu 28: Đáp án A
Kiến thức: Đọc hiểu
Giải chi tiết:
Từ “attributes” trong đoạn 1 gần nghĩa nhất với _______
attributes (n): các thuộc tính
A. qualities (n): những tính chất
B. aspects (n): các khía cạnh
C. fields (n): các lĩnh vực
D. skills (n): các kĩ năng
=> attributes = qualities: thuộc tính, tính chất
Câu 29: Đáp án C
Kiến thức: Đọc hiểu
Giải chi tiết:
Từ “static” trong đoạn 2 có thể thay thể bằng _______
static (adj): yên, đứng yên
A. dense (adj): đông đúc
B. balanced (adj): cân bằng
C. unchanged (adj): không thay đổi
D. regular (adj): thường xuyên
=> static = unchanged: yên, không thay đổi
Câu 30: Đáp án B
Kiến thức: Đọc hiểu
Giải chi tiết:
Được đề cập trong đoạn 1 rằng một người có văn hóa _________.
A. làm việc liên quan đến giáo dục
B. có kiến thức về mĩ thuật, văn học, và âm nhạc
C. có công việc liên quan đến canh tác
D. chăm sóc đất và những gì trồng trên đó
Thông tin: A person of culture has identifiable attributes, among them a knowledge of and interest in the
arts, literature, and music.

Trang 80
Tạm dịch: Một người có văn hóa có những đặc tính có thể xác nhận được, trong số đó có kiến thức và
quan tâm đến nghệ thuật, văn học và âm nhạc.
Câu 31: Đáp án B
Kiến thức: Cấu trúc song hành
Giải chi tiết:
Liên từ “and” kết nối các từ cùng tính chất
“sleeping”, “resting” ở dạng V_ing => động từ sau “and” cũng phải ở dạng _ing
Sửa: “to drink” => “drinking”
Tạm dịch: Ngủ, nghỉ ngơi và uống nước ép trái cây là những cách tốt nhất để trị cảm lạnh.
Câu 32: Đáp án A
Kiến thức: Cấu trúc “so...that…; such…that…”
Giải chi tiết:
Cấu trúc: S1 + be + so + adj + that + S2 + V: Cái gì quá làm sao đến nỗi mà ai đó làm gì
= S1 + be + such + a/an + adj + N + that S2 + V
“interesting” là tính từ, sau nó không có danh từ
Sửa: “such” => “so”
Tạm dịch: Cuốn tiểu thuyết quá thú vị đến nỗi mà tôi đã đọc nó từ đầu đến cuối trong 4 tiếng.
Câu 33: Đáp án B
Kiến thức: Sự hòa hợp giữa chủ ngữ và động từ
Giải chi tiết:
Chủ ngữ là “The result” => động từ chia theo chủ ngữ ngôi thứ 3 số ít
Sửa: “have had” => “has had”
Tạm dịch: Kết quả của các cuộc nghiên cứu đã có ảnh hưởng mạnh mẽ đến sự phát triển tương lai.
Câu 34: Đáp án A
Kiến thức: Câu đồng nghĩa
Giải chi tiết:
make sb + adj: khiến ai đó thế nào
surprised (adj): cảm thấy ngạc nhiên
surprising (adj): ngạc nhiên => chỉ đặc điểm, tính chất của sự vật
Tạm dịch: Tin tức đã gây ngạc nhiên cho mọi người trong gia đình.
A. Mọi người trong nhà đã cảm thấy tin tức đáng ngạc nhiên.
B. Sai ngữ pháp vì “made + sb + adj”
C. Tin tức khiến mọi người ngạc nhiên trong gia đình. => sai nghĩa
D. Mọi người bị ngạc nhiên bởi tin tức trong gia đình. => sai nghĩa
Câu 35: Đáp án D
Kiến thức: Câu bị động gián tiếp

Trang 81
Giải chi tiết:
People say that S + V_ed.
= It is said that S + V_ed.
= S + is/ am/ are said to + have + P2.
Động từ 1 (say) chia hiện tại đơn, động từ 2 (was) chia quá khứ đơn => dùng “to have + P2”
Tạm dịch:
Mọi người nói rằng Cameron đã là đạo diễn xuất sắc nhất ở thời đại anh ấy.
A. Sai ngữ pháp “be” => “have been”, “best => the best”
B. Sai thì. “was” => “is” (vì “say” chia hiện tại đơn)
C. Sai thì. “was” => “is”
D. Cameron được cho là người đạo diễn xuất sắc nhất ở thời đại của anh ấy.
Câu 36: Đáp án B
Kiến thức: Lời nói gián tiếp
Giải chi tiết:
“Be sorry for V_ing …” : Rất xin lỗi vì đã làm gì …
= S + apologized to + O + for V_ing: Ai đó đã xin lỗi ai vì đã làm gì
Tạm dịch: “Rất xin lỗi vì đã gửi sai thông tin, Kate” Rita đã nói
A. Kate đã gửi sai thông tin, và cô ấy đã xin lỗi => sai nghĩa
B. Rita đã xin lỗi Kate vì đã gửi cho cô ấy sai thông tin.
C. Rita rất xin lỗi về việc Kate gửi sai thông tin. => sai nghĩa
D. Rita rất xin lỗi vì đã không gửi thông tin cho Kate. => sai nghĩa
Câu 37: Đáp án A
Kiến thức: Câu điều kiện loại 2
Giải chi tiết:
- Dấu hiệu: có mệnh đề chứa “if”, mệnh đề chính “would + V_nguyên thể”
- Câu điều kiện loại 2 diễn tả một điều kiện không có thực ở hiện tại dẫn đến kết quả trái với hiện tại.
- Công thức câu nghi vấn: If + S + V_ed , wh_word + would + S + V_nguyên thể?
find => found
Tạm dịch: Nếu bạn tìm thấy một cái ví trên phố, bạn sẽ làm gì với nó?
Câu 38: Đáp án D
Kiến thức: Từ vựng
Giải chi tiết:
A. competed (adj): đã hoàn thành
B. original (adj): nguyên bản
C. estimated (adj): được ước tính
D. sincere (adj): chân thành

Trang 82
Tạm dịch: Hãy chấp nhận lời xin lỗi chân thành của chúng tôi cho sự bất tiện vì sự trì hoãn này đang
khiến tất cả các hành khách ở đây tại sân bay quốc tế Pearson.
Câu 39: Đáp án D
Kiến thức: Thì quá khứ tiếp diễn
Giải chi tiết:
- Cách dùng: thì quá khứ tiếp diễn diễn tả hành động đang diễn ra trong quá khứ (chia quá khứ tiếp diễn)
thì có hành động khác cắt ngang (chia quá khứ đơn)
- Công thức: While S + was/were +V_ing, S + V_ed/V cột 2
Tạm dịch: Trong khi Peter đang tưới cho bụi hoa hồng ở sân phía sau, thì điện thoại kêu.
Câu 40: Đáp án C
Kiến thức: Từ loại
Giải chi tiết:
Trước danh từ “relationship” cần một tính từ
A. contract (n): hợp đồng
B. contractually (adv): một cách có tính hợp đồng
C. contractual (adj): có tính hợp đồng
D. contracting (v_ing): kí hợp đồng
Tạm dịch: Một vài cặp đôi hiện đại tổ chức hôn lễ của họ và thực hiện các nhiệm vụ, nghĩa vụ, cái mà có
thể dần biến cuộc hôn nhân thành một cuộc giao dịch hoặc mối quan hệ hợp đồng.
Câu 41: Đáp án A
Kiến thức: Câu phỏng đoán
Giải chi tiết:
must have been: chắc hẳn đã
should have been: lẽ ra nên đã (nhưng đã không làm)
Tạm dịch: Bạn trông rất mệt mỏi! Bạn chắc hẳn đã ở bên ngoài rất muộn tối hôm qua.
Câu 42: Đáp án C
Kiến thức: Từ vựng
Giải chi tiết:
“do sb a favor”: giúp ai đó một việc
Tạm dịch: Tôi băn khoăn rằng liệu bạn có thể giúp tôi làm một việc không nhỉ Jackson.
Câu 43: Đáp án C
Kiến thức: Rút gọn mệnh đề quan hệ
Giải chi tiết:
Ta dùng phân từ bị động để rút gọn mệnh đề quan hệ mang nghĩa bị động.
Dạng đầy đủ: The song which/that was chosen by our listeners as their favorite of the week
Rút gọn: The song chosen by our listeners as their favorite of the week

Trang 83
Tạm dịch: Bài hát được chọn bởi người nghe của chúng ta như là bài hát yêu thích của tuần là “Goodbye
Baby” của Tunesmiths.
Câu 44: Đáp án B
Kiến thức: to V/ V_ing
Giải chi tiết:
allow + sb + to V_nguyên thể: cho phép ai đó làm gì
Tạm dịch: Thư điện tử cho phép con người giữ liên lạc, bất kể khoảng cách.
Câu 45: Đáp án A
Kiến thức: Câu ước ở quá khứ
Giải chi tiết:
Dấu hiệu: “I wish” (Tôi ước), “last night” (đêm qua)
Cách dùng: diễn tả điều ước, mong muốn trái với kết quả nào đó trong quá khứ
Cấu trúc: S + wish + S + had (not) + P2
have to => had to
Tạm dịch: Hôm nay tôi thực sự buồn ngủ. Tôi ước rằng tối qua tôi đã không phải đưa Bob đến sân bay.
Câu 46: Đáp án C
Kiến thức: Cấu trúc tăng tiến
Giải chi tiết:
- “difficult” là tính từ dài => more difficult
- Cấu trúc: S + V + more and more + adj/adv dài: Cái gì càng ngày càng thế nào
Tạm dịch: Tìm một công việc trong thời kì khủng hoảng kinh tế này thì đang càng ngày càng trở nên khó
khăn.
Câu 47: Đáp án D
Kiến thức: Mệnh đề quan hệ
Giải chi tiết:
Trong mệnh đề quan hệ:
- that: cái mà, người mà => thay thế cho từ chỉ vật, chỉ người hoặc hỗn hợp người, vật
- what = the thing that: cái mà => dùng làm chủ ngữ
Tạm dịch: Cái xếp hạng là vụ mùa quan trọng nhất ở Hawaii là mía.
Câu 48: Đáp án B
Kiến thức: Từ vựng
Giải chi tiết:
A. keeping (v): giữ
B. preparing (v): chuẩn bị
C. making (v): làm (make a meal)
D. arranging (v): sắp đặt

Trang 84
Tạm dịch: Thỉnh thoảng vào những ngày cuối tuần, bố tôi giúp mẹ tôi chuẩn bị các bữa ăn.
Câu 49: Đáp án C
Kiến thức: Sự kết hợp từ
Giải chi tiết:
“take notice of sth”: chú ý đến cái gì
Tạm dịch: Không ai đã chú ý đến cảnh báo và họ đã đi bơi ở chỗ nước bị ô nhiễm.
Câu 50: Đáp án A
Kiến thức: Cụm động từ
Giải chi tiết:
A. put up with: chịu đựng
B. look up to: kính trọng
C. get on with: hòa thuận với
D. catch up with: theo kịp
Tạm dịch: Là bạn cùng phòng của Joe, tôi cảm thấy anh ấy là một người bạn khá tốt, ngay cả khi có đôi
lúc không dễ để chịu đựng cách hành xử ồn ào của anh ấy.

SỞ GD & ĐT VĨNH PHÚC ĐỀ THI KSCL LẦN 1 NĂM HỌC 2019 – 2020
TRƯỜNG THPT ĐỒNG DẬU MÔN: TIẾNG ANH 12
Thời gian làm bài: 60 phút; không kể thời gian phát đề

Mark the letter A, B, C, or D on your answer sheet to indicate the word that differs from the other three in
the position of primary stress in each of the following questions from 1 to 2:
Câu hỏi 1 (NB): A. farm B. stayed C. garden D. harm
Câu hỏi 2 (NB): A. achieved B. wanted C. required D. replied
Mark the letter A, B, C, or D on your answer sheet to indicate the word whose underlined part differs
from the other three in pronunciation in each of the following questions from 3 to 4:
Câu hỏi 3 (NB): A. knowledge B. careful C. arrive D. happen
Câu hỏi 4 (NB): A. solidarity B. determination C. organization D. consideration
Choose the option that best completes each of the following exchanges from 5 to 6:
Câu hỏi 5 (NB): Jim: “This dictionary is for you. I hope you will find it useful.”
Mai: “__________.”
A. No problem! B. Thanks. I’ll do it C. Thanks. It’s very kind of you D. Yes, please
Câu hỏi 6 (NB): Kate: “How lovely your cats are!”
David: “__________.”
A. I love them, too. B. Thank you, it is nice of you to say so.
C. Can you say it again? D. Really? They are

Trang 85
Mark the letter A, B, C, or D on your answer sheet to indicate the word(s) CLOSEST in meaning to the
underlined word(s) in each of the following questions from 7 to 8:
Câu hỏi 7 (TH): There was a long period without rain in the countryside last year so the harvest was poor.
A. famine B. flood C. epidemic D. drought
Câu hỏi 8 (VD): How many countries took part in the last Olympic Games?
A. participated B. performed C. succeeded D. hosted
Mark the letter A, B, C, or D on your answer sheet to indicate the word(s) OPPOSITE in meaning to the
underlined word(s) in each of the following questions from 7 to 8:
Câu hỏi 9 (TH): “That is a well-behaved boy whose behaviour has nothing to complain about.”
A. behaving cleverly B. good behaviour C. behaving nice D. behaving improperly
Câu hỏi 10 (VDC): When he passes the entrance exam, his parents will be walking on the air.
A. extremely happy B. feeling extremely unhappy
C. feeling extremely airy D. extremely light
Choose the correct answer to each of the following questions from 11 to 24:
Trả lời cho các câu 11, 12, 13, 14, 15, 16, 17, 18, 19, 20, 21, 22, 23, 24 dưới đây:
Câu hỏi 11 (TH): The boy waved his hands to his mother, who was standing at the school gate, to
___________ her attention.
A. pull B. follow C. tempt D. attract
Câu hỏi 12 (NB): He ___________ to the cinema yesterday evening.
A. has gone B. goes C. went D. had gone
Câu hỏi 13 (NB): Claire wanted to know what time ___________.
A. the banks closed B. the banks would close
C. did the banks close D. do the banks close
Câu hỏi 14 (NB): You stayed at home last night, ___________?
A. didn’t you B. had you C. did you D. would you
Câu hỏi 15 (NB): Sometimes she does not agree ___________ her husband about child reading but they
soon find the solutions.
A. with B. for C. on D. of
Câu hỏi 16 (NB): In my free-time, I often help mom with ___________ the house.
A. clean B. to clean C. cleans D. cleaning
Câu hỏi 17 (NB): I have lived here ___________ 5 years.
A. on B. for C. since D. from
Câu hỏi 18 (NB): This is ___________ most beautiful song I’ve ever listened to.
A. no article B. an C. a D. the
Câu hỏi 19 (NB): Jim ___________ a book at the moment.
A. is reading B. had read C. reads D. has read

Trang 86
Câu hỏi 20 (VD): My responsibility is to ___________ my little brothers.
A. take over B. join hands C. work together D. take care of
Câu hỏi 21 (NB): I asked him ___________, but he said nothing.
A. what’s the matter was B. what was the matter
C. what the matter was D. the matter was what
Câu hỏi 22 (TH): He advised them ___________ in class.
A. don’t talk B. to not talk C. not to talk D. to talk not
Câu hỏi 23 (TH): My sister is very fond ___________ eating chocolate candy.
A. about B. of C. at D. with
Câu hỏi 24 (TH): We enjoy ___________ time together in the evening when the family members gather
in the living room after a day of working hard.
A. caring B. spending C. doing D. taking
Choose the correct word or phrase that best fits each of the numbered blanks from 25 to 29:
Earth is the only place we know of in the universe that can support human life. ____25____ human
activities are making the planet less fit to live on. As the western world ____26____ on consuming two-
thirds of the world's resources while half of the world's population do so just to stay alive we are rapidly
destroying the lonely resource we have by which all people can survive and prosper. Everywhere fertile
soil is ____27____ built on or washed into the sea. Renewable resources are exploited so much that they
will never be able to recover ____28____. We discharge pollutants into the atmosphere without any
thought of the consequences. As a result, the planet's ability to support people is being reduced at the very
time when rising human numbers and consumption are ____29____ increasingly heavy demands on it.
Câu hỏi 25 (TH): A. Although B. Still C. Yet D. Despite
Câu hỏi 26 (VD): A. continues B. carries C. follows D. repeats
Câu hỏi 27 (TH): A. sooner B. neither C. rather D. either
Câu hỏi 28 (TH): A. greatly B. utterly C. completely D. quite
Câu hỏi 29 (VD): A. making B. doing C. taking D. having
Read the following passage and mark the letter A, B, C, or D on your answer sheet to incorrect answer to
each of the questions from 30 to 34:
The Singapore Science Centre is located on a six-hectare site in Jurong. At the centre, we can discover the
wonders of science and technology in a fun way. Clap your hands and colorful bulbs will light up. Start a
wheel spinning and it will set off a fan churning. It is a place to answer our curiosity and capture our
imagination.
The centre features over four hundred exhibits covering topics like solar radiation, communication,
electronics, mathematics, nuclear energy and evolution. It aims to arouse interest in science and
technology among us and the general public. The centre is the first science one to be established in South
East Asia. It was opened in 1977 and it now receives an average of one thousand, two hundred visitors a

Trang 87
day. The exhibits can be found in four exhibition galleries. They are the Lobby, Physical Sciences, Life
Sciences and Aviation. These exhibits are renewed annually so as to encourage visitors to make return
visits to the centre.
Instead of the usual “Hands off” notices found in exhibition halls, visitors are invited to touch and feel the
exhibits, push the buttons, turn the cranks or pedals. This is an interesting way to learn science even if
you hate the subject. A Discovery Centre was built for children between the ages of three and twelve.
This new exhibition gallery was completed in 1985. Lately this year a stone-age exhibit was built. It
shows us about the animals and people which were extinct.
Câu hỏi 30 (TH): What can be the best title of the passage?
A. Singapore Science Centre B. Science Centre
C. Discovery Centre D. Physical Sciences
Câu hỏi 31 (NB): The word “It” in paragraph 2 refers to ________ .
A. the centre B. the general public C. evolution D. solar radiation
Câu hỏi 32 (TH): According to the paragraph 2, which of the following is NOT true about the Singapore
Science Centre?
A. Visitors are encouraged to return to the centre.
B. The centre was not opened until 1977.
C. The exhibits in the centre cover a wide range of topics.
D. The centre is the first one established in the world.
Câu hỏi 33 (TH): It is stated in paragraph 2 that __________.
A. the science centre makes people interested in science and technology
B. only students can visit the science centre
C. visitors don’t want to come back to the science centre
D. there are only several exhibits in the science centre
Câu hỏi 34 (TH): The author mentions all of the following in the passage EXCEPT _________.
A. The exhibits are renewed every year.
B. The centre is located in Jurong.
C. There are four exhibition galleries in the centre.
D. The centre is the biggest in Asia.
Read the following passage and mark the letter A, B, C, or D on your answer sheet to incorrect answer to
each of the questions from 35 to 42:
Smart cards and mobile phones are becoming an increasingly popular way to make all sorts of payments.
Even now, in Japan thousands of transactions, from paying rail tickets to picking up the groceries, take
place every day with customers passing their handsets across a small flat-screen device. And predictions
in the world of finance reckon that payments using mobile phones will have risen to more than $50 billion
in the very near future.

Trang 88
What's the appeal of e-cash? Compared to cheque or credit cards, it offers the speed of cash, but more so.
It takes just one tenth of a second to complete most transactions and as no change is required, errors in
counting are eliminated. Fraud and theft are also reduced and for the retailer, it reduces the cost of
handling money. Sony's vision of having a chip embedded in computers, TVs and games consoles means
that films, music and games can be paid for easily without having to input credit card details. And what
about the future of the banks? Within their grip on the market, banks and creditcard firms want to be in a
position to collect most of the fees from the users of mobile and contactless-payment systems. But the
new system could prove to be a "disruptive technology" as far as the banks are concerned. If payments for
a few coffees, a train ticket and a newspaper are made every day by a commuter with a mobile, this will
not appear on their monthly credit card statements but on their mobile phone statements. And having
spent fortunes on branding, credit-card companies and banks do not want to see other payment systems
gaining popularity. It's too early to say whether banks will miss out and if so, by how much. However,
quite a few American bankers are optimistic. They feel there is reason to be suspicious of those who
predict that high-street banks may be a thing of the past. They point out that Internet banking did not
result in the closure of their high-street branches as was predicted. On the contrary, more Americans than
ever are using local branches. So, whether we'll become a totally cash-free society remains open to
contention.
Câu hỏi 35 (VD): What is the main idea of the first paragraph?
A. Predictions of future payment methods.
B. The increasing popularity of new payment methods.
C. The absence of traditional payment methods.
D. Japan's advanced forms of payment.
Câu hỏi 36 (TH): Why does the author mention "a small flat-screen device" in the first paragraph?
A. to exemplify the e-cash system B. to praise the e-cash system C. to criticize the e-
cash system D. to inform the e-cash system
Câu hỏi 37 (TH): Which of the following is NOT true about the strong point of e-cash?
A. reduced cost B. no fraud C. fewer mistakes D. faster speed
Câu hỏi 38 (TH): The word "embedded” in the second paragraph is closest in meaning to ________.
A. isolated B. manufactured C. generated D. integrated
Câu hỏi 39 (TH): The word "grip" in the third paragraph is closest in meaning to ________.
A. power B. success C. wealth D. range of branches
Câu hỏi 40 (TH): The author mentions the case of commuters in the third paragraph to illustrate
________.
A. the banks' cooperation with credit-card companies
B. a possible drawback of the system
C. the modern technology of the e-cash system

Trang 89
D. the transferability of the system
Câu hỏi 41 (TH): The word "their” in the third paragraph refers to ________.
A. credit cards B. high-street banks C. American bankers D. Internet banking
Câu hỏi 42 (VD): How does the writer seem to feel about the future of banks?
A. neutral B. optimistic C. uncertain D. pessimistic
Mark the letter A, B, C, or D on your answer sheet to indicate the sentence that best combines each pair
of sentences in the following questions from 43 to 44:
Câu hỏi 43 (VDC): It isn’t just that the level of education of this school is high. It’s that it’s also been
consistent for years.
A. It isn’t fair to deny that this school is successful, as it has had the same high standards for many
years now.
B. The standard of education is not high in this school, but at least all the students are at the same.
C. The level of education in this school, which is usually quite high, shows only slight variations
from year to year.
D. Not only are the standards of education good in this school, but it has maintained those
standards over the years.
Câu hỏi 44 (VD): No one but the experts was able to realize that the painting was an imitation. It greatly
resembled the original.
A. It was obvious that only a person with great talent could fake a painting so successfully.
B. It was almost impossible for amateurs to realize that the painting was not authentic, though the
experts could judge it quite easily.
C. The painting looked so much like the authentic one that only the experts could tell it wasn’t
genuine.
D. It was hard for ordinary people to judge between the fake painting and the real one, but not for
the experts.
Mark the letter A, B, C, or D on your answer sheet to indicate the sentence that is closest in meaning to
each of the following questions from 45 to 47:
Câu hỏi 45 (TH): Your coffee is not as good as mine.
A. Mine is better than yours. B. My coffee is more good than yours.
C. My coffee is better than your. D. My coffee is better than yours.
Câu hỏi 46 (VD): “You’re always making terrible mistakes”, said the teacher.
A. The teacher realized that his students always made terrible mistakes.
B. The teacher complained about his students making terrible mistakes.
C. The teacher asked his students why they always made terrible mistakes.
D. The teacher made his students not always make terrible mistakes.
Câu hỏi 47 (VD): Difficult though the exam was, he still passed it with flying colours.

Trang 90
A. Because the exam was difficult, he passed it with flying colours.
B. Although the exam was difficult, but he still passed it with flying colours.
C. No matter how difficult the exam was, he still passed it with flying colours.
D. The exam was not too difficult, so he could pass it with flying colours.
Mark the letter A, B, C, or D on your answer sheet to indicate the underlined part that needs correction in
each of the following questions from 48 to 50:
Câu hỏi 48 (NB): Tom is one of the students which get the worst result in the latest exam.
A. the latest exam B. worst result C. one of the students D. which
Câu hỏi 49 (TH): No sooner had the wind stopped blowing when it started to rain heavily.
A. heavily B. when C. stopped blowing D. had
Câu hỏi 50 (NB): Although they always argue with each other, but they are good friends.
A. with B. always C. friends D. but

Đáp án
1-B 2-B 3-C 4-A 5-C 6-B 7-D 8-A 9-D 10-B
11-D 12-C 13-A 14-A 15-A 16-D 17-B 18-D 19-A 20-D
21-C 22-C 23-B 24-B 25-C 26-B 27-D 28-C 29-A 30-A
31-A 32-D 33-A 34-D 35-B 36-A 37-B 38-D 39-A 40-B
41-C 42-C 43-D 44-C 45-D 46-B 47-C 48-D 49-B 50-D

LỜI GIẢI CHI TIẾT


Câu 1: Đáp án B
Kiến thức: Phát âm “-a”
Giải chi tiết:
A. farm /fɑːm/
B. stayed /steɪd/
C. garden /ˈɡɑːdn/
D. harm /hɑːm/
Phần gạch chân đáp án B phát âm là /eɪ/, còn lại là /ɑː/
Câu 2: Đáp án B
Kiến thức: Phát âm “-ed”
Giải chi tiết:
A. achieved /əˈtʃiːvd/
B. wanted /ˈwɒntɪd/
C. required /rɪˈkwaɪəd/
D. replied /rɪˈplaɪd/

Trang 91
Quy tắc:
Cách phát âm đuôi “ed”:
- Đuôi “ed” được phát âm là /ɪd/ khi động từ có phát âm kết thúc là /t/ hay /d/
- Đuôi “ed” được phát âm là /t/ khi động từ có phát âm kết thúc là /s/,/f/,/p/,/ʃ/,/tʃ/,/k/
- Đuôi “ed” được phát âm là /d/ với các trường hợp còn lại
Phần gạch chân đáp án B phát âm là /ɪd/, còn lại là /d/
Câu 3: Đáp án C
Kiến thức: Trọng âm từ có 2 âm tiết
Giải chi tiết:
A. knowledge /ˈnɒlɪdʒ/
B. careful /ˈkeəfl/
C. arrive /əˈraɪv/
D. happen /ˈhæpən/
Quy tắc:
- Những động từ có 2 âm tiết thường có trọng âm rơi vào âm tiết thứ hai.
- Những danh từ, tính từ có 2 âm tiết thường có trọng âm rơi vào âm tiết thứ nhất.
Trọng âm đáp án C rơi vào âm tiết thứ hai, còn lại là âm thứ nhất
Câu 4: Đáp án A
Kiến thức: Trọng âm từ có 5 âm tiết
Giải chi tiết:
A. solidarity /sɒlɪˈdærəti/
B. determination /dɪtɜːmɪˈneɪʃn/
C. organization /ɔːɡənaɪˈzeɪʃn/
D. consideration /kənsɪdəˈreɪʃn/
Quy tắc: Những từ có tận cùng là đuôi “-ion”, “-ity” có trọng âm rơi vào âm tiết đứng ngay trước nó.
Trọng âm đáp án A rơi vào âm tiết thứ ba, còn lại là âm thứ tư
Câu 5: Đáp án C
Kiến thức: Ngôn ngữ giao tiếp
Giải chi tiết:
Jim: “Từ điển này là dành cho cậu. Mình hy vọng rằng cậu sẽ thấy nó hữu ích.”
Mai: “__________.”
A. Không có gì đâu!
B. Cảm ơn. Mình sẽ làm nó.
C. Cảm ơn. Cậu thật sự rất tốt bụng.
D. Vâng, làm ơn.
Các phản hồi A, B, D không phù hợp với ngữ cảnh.

Trang 92
Câu 6: Đáp án B
Kiến thức: Ngôn ngữ giao tiếp
Giải chi tiết:
Kate: “Những con mèo của bạn thật đáng yêu!”
David: “__________.”
A. Mình cũng yêu chúng.
B. Cảm ơn, bạn thật tốt khi nói như vậy.
C. Bạn có thể nói lại không?
D. Thật sao? Chúng đấy
Các phản hồi A, C, D không phù hợp với ngữ cảnh.
Câu 7: Đáp án D
Kiến thức: Từ vựng
Giải chi tiết:
long period without rain: khoảng thời gian dài không có mưa
A. famine (n): nạn đói kém
B. flood (n): lũ lụt, nạn lụt
C. epidemic (n): bệnh dịch
D. drought (n): hạn hán
=> long period without rain = drought (n): hạn hán
Tạm dịch: Có một khoảng thời gian dài không có mưa ở vùng nông thôn vào năm ngoái nên sản lượng
thu hoạch rất kém.
Câu 8: Đáp án A
Kiến thức: Từ vựng
Giải chi tiết:
took part in: đã tham gia
A. participate - participated - participated: tham gia
B. perform - performed - performed: biểu diễn, trình diễn
C. succeed - succeeded - succeeded: thành công, thịnh vượng
D. host - hosted - hosted: đăng cai tổ chức
=> took part in = participated: đã tham gia
Tạm dịch: Có bao nhiêu quốc gia tham gia Thế vận hội Olympic vừa qua?
Câu 9: Đáp án D
Kiến thức: Từ vựng
Giải chi tiết:
well-behaved (adj): có cư xử đúng, có giáo dục
A. behaving cleverly: cư xử khéo léo

Trang 93
B. good behaviour: cư xử đúng mực
C. behaving nice: cư xử tử tế
D. behaving improperly: cư xử không đúng mực
=> well-behaved (adj): có cư xử đúng, có giáo dục >< behaving improperly: cư xử không đúng mực
Tạm dịch: Đây là một cậu bé cư xử tốt người mà không có hành vi gì để phàn nàn.
Câu 10: Đáp án B
Kiến thức: Thành ngữ
Giải chi tiết:
walking on the air: rất vui, rất hạnh phúc
A. extremely happy: vô cùng hạnh phúc
B. feeling extremely unhappy: cảm thấy vô cùng bất hạnh
C. feeling extremely airy: cảm giác cực kỳ nhẹ nhàng
D. extremely light: cực kỳ nhẹ nhàng
=> walking on the air: rất vui, rất hạnh phúc >< feeling extremely unhappy: cảm thấy vô cùng bất hạnh
Tạm dịch: Khi anh ấy vượt qua kỳ thi tuyển sinh, bố mẹ anh ấy sẽ rất hạnh phúc.
Câu 11: Đáp án D
Kiến thức: Từ vựng
Giải chi tiết:
A. pull (v): lôi, kéo, giật
B. follow (v): đi theo, theo sau
C. tempt (v): cám dỗ, quyến rũ
D. attract (v): thu hút, lôi cuốn
attract one’s attention: thu hút sự chú ý của ai
Tạm dịch: Cậu bé vẫy tay với mẹ, người đang đứng ở cổng trường, để thu hút sự chú ý của bà ấy.
Câu 12: Đáp án C
Kiến thức: Thì quá khứ đơn
Giải chi tiết:
Dấu hiệu: “yesterday evening” (tối hôm qua)
Cách dùng: Thì quá khứ đơn dùng để diễn tả hành động xảy ra và kết thúc trong quá khứ.
Công thức chung: S + Ved.
go => went
Tạm dịch: Anh ấy đi xem phim tối hôm qua.
Câu 13: Đáp án A
Kiến thức: Câu tường thuật
Giải chi tiết:
Công thức tường thuật câu hỏi “Wh-question”: S1 wanted to know + wh-word + S2 + V(lùi 1 thì)

Trang 94
Câu trực tiếp: “What time do the banks close?” (thì hiện tại đơn)
Câu tường thuật: what time the banks closed (thì quá khứ đơn)
Tạm dịch: Claire muốn biết mấy giờ các ngân hàng đóng cửa.
Câu 14: Đáp án A
Kiến thức: Câu hỏi đuôi
Giải chi tiết:
Vế trước câu hỏi đuôi ở dạng khẳng định => câu hỏi đuôi dạng phủ định
Vế trước: S + Ved => câu hỏi đuôi: didn’t + S?
Tạm dịch: Bạn đã ở nhà tối qua, đúng không?
Câu 15: Đáp án A
Kiến thức: Giới từ
Giải chi tiết:
agree with sbd about sth: đồng ý với ai về điều gì

Tạm dịch: Đôi khi cô không đồng ý với chồng về việc đọc sách của con họ nhưng họ sớm tìm ra giải
pháp.
Câu 16: Đáp án D
Kiến thức: to V/ Ving
Giải chi tiết:
Sau giới từ => động từ để dạng V_ing
Tạm dịch: Trong thời gian rảnh, tôi thường giúp mẹ dọn dẹp nhà cửa.
Câu 17: Đáp án B
Kiến thức: Giới từ
Giải chi tiết:
Dấu hiệu: have lived => thì hiện tại hoàn thành
Trong thì hiện tại hoàn thành: for + khoảng thời gian, since + mốc thời gian
“5 years” (5 năm) là một khoảng thời gian => dùng “for”
Công thức: S + have/ has + PII + for + khoảng thời gian
Tạm dịch: Tôi đã sống ở đây 5 năm.
Câu 18: Đáp án D
Kiến thức: Mạo từ
Giải chi tiết:
Mạo từ “the” dùng trong câu so sánh hơn nhất: the + adj-est/ most adj
Tạm dịch: Đây là bài hát hay nhất mà tôi đã từng nghe.
Câu 19: Đáp án A
Kiến thức: Thì hiện tại tiếp diễn

Trang 95
Giải chi tiết:
Dấu hiệu: “at the moment” (ngay lúc này)
Cách dùng: Thì hiện tại tiếp diễn diễn tả một hành động đang diễn ra tại thời điểm nói.
Công thức: S + am/is/are + Ving.
Tạm dịch: Lúc này, Jim đang đọc một cuốn sách.
Câu 20: Đáp án D
Kiến thức: Cụm động từ
Giải chi tiết:
A. take over: tiếp quản
B. join hands: chung tay cùng làm
C. work together: làm cùng nhau
D. take care of: chăm sóc
Tạm dịch: Trách nhiệm của tôi là chăm sóc các em trai của tôi.
Câu 21: Đáp án C
Kiến thức: Câu tường thuật
Giải chi tiết:
Công thức tường thuật câu hỏi “Wh-question”: S1 asked S2 + wh-word + S2 + V(lùi 1 thì)
Câu trực tiếp: “What is the matter?” (thì hiện tại đơn)
Câu tường thuật: what the matter was (thì quá khứ đơn)
Tạm dịch: Tôi hỏi vấn đề của anh ta là gì, nhưng anh ta không nói gì.
Câu 22: Đáp án C
Kiến thức: to V/ V_ing
Giải chi tiết:
advise sb (not) to V_nguyên thể: khuyên ai (không) làm gì
Tạm dịch: Anh ấy khuyên họ không nên nói chuyện trong lớp.
Câu 23: Đáp án B
Kiến thức: Giới từ
Giải chi tiết:
to be fond of: mến, yêu thích
Tạm dịch: Em gái tôi rất thích ăn kẹo sô cô la.
Câu 24: Đáp án B
Kiến thức: Từ vựng
Giải chi tiết:
A. care - caring: trông nom, chăm sóc
B. spend - spending: sử dụng, chi tiêu, dành
C. do - doing: làm, hành động

Trang 96
D. take - taking: cầm, lấy
Tạm dịch: Chúng tôi thích dành thời gian bên nhau vào buổi tối khi các thành viên trong gia đình quây
quần trong phòng khách sau một ngày làm việc vất vả.
Câu 25: Đáp án C
Kiến thức: Liên từ
Giải chi tiết:
A. Although S + V: mặc dù
B. Still: ấy thế mà, vẫn
C. Yet S + V: tuy nhiên
D. Despite + N/Ving: dù, mặc dù
Earth is the only place we know of in the universe that can support human life. (25) Yet human activities
are making the planet less fit to live on.
Tạm dịch: Trái Đất là nơi duy nhất chúng ta biết đến trong vũ trụ này mà có thể nuôi sống loài người. Tuy
nhiên hoạt động của con người đang khiến cho Trái Đất trở thành một khắc nghiệt.
Câu 26: Đáp án B
Kiến thức: Cụm động từ
Giải chi tiết:
carry on: tiếp tục
A. continues (v): tiếp tục
C. follows (v): đi theo, theo sau
D. repeats (v): nhắc lại, lặp lại
As the western world (26) carries on consuming two-thirds of the world's resources …
Tạm dịch: Vì thế giới phương Tây tiếp tục tiêu thụ 2/3 tài nguyên của Trái Đất …
Câu 27: Đáp án D
Kiến thức: Cấu trúc “either … or …”
Giải chi tiết:
Dấu hiệu: Sau chỗ trống có “or”
Cấu trúc: S + V + either … or …. : … hoặc … hoặc …
Các phương án khác:
no sooner … than ….: vừa mới …. thì …
neither …. nor … : không … cũng không ….
rather …. than …: … hơn là …
Everywhere fertile soil is (27) either built on or washed into the sea.
Tạm dịch: Ở khắp mọi nơi, đất màu mỡ hoặc được trồng trọt hoặc bị thải ra biển.
Câu 28: Đáp án C
Kiến thức: Từ vựng

Trang 97
Giải chi tiết:
A. greatly (adv): rất, lắm
B. utterly (adv): hoàn toàn, chắc chắn
C. completely (adv): hoàn toàn, đầy đủ, trọn vẹn
D. quite (adv): khá
Renewable resources are exploited so much that they will never be able to recover (28) completely.
Tạm dịch: Các nguồn tài nguyên có thể phục hồi thì bị khai thác quá mức đến nỗi chúng không bao giờ
khôi phục lại hoàn toàn được nữa.
Câu 29: Đáp án A
Kiến thức: Sự kết hợp từ
Giải chi tiết:
A. make - making: chế tạo, sản xuất
B. do - doing: làm, hành động
C. take - taking: cầm, lấy
D. have - having: có
=> make demand on: đòi hỏi, yêu cầu
As a result the planet's ability to support people is being reduced at the very time when rising human
numbers and consumption are (29) making increasingly heavy demands on it.
Tạm dịch: Kết quả là khả năng nuôi sống loài người của hành tinh này đang suy giảm cùng lúc khi mà
dân cư thế giới đang tăng và nhu cầu tiêu dùng đang cần rất nhiều nguồn tài nguyên từ Trái Đất.
Câu 30: Đáp án A
Kiến thức: Đọc hiểu
Giải chi tiết:
Tiêu đề phù hợp nhất cho bài đọc là gì?
A. Trung tâm khoa học Singapore
B. Trung tâm khoa học
C. Trung tâm khám phá
D. Khoa học vật lý
Thông tin: The Singapore Science Centre is located on a six-hectare site in Jurong.
Tạm dịch: Trung tâm Khoa học Singapore nằm trên một khu đất rộng sáu héc ta ở Jurong.
Câu 31: Đáp án A
Kiến thức: Đọc hiểu
Giải chi tiết:
Từ “It” trong đoạn 2 thay thế cho __________.
A. trung tâm
B. công chúng nói chung

Trang 98
C. tiến hóa
D. bức xạ mặt trời
Thông tin: The centre features over four hundred exhibits covering topics like solar radiation,
communication, electronics, mathematics, nuclear energy and evolution. It aims to arouse interest in
science and technology among us and the general public.
Tạm dịch: Trung tâm có hơn bốn trăm triển lãm bao gồm các chủ đề như bức xạ mặt trời, truyền thông,
điện tử, toán học, năng lượng hạt nhân và tiến hóa. Nó nhằm mục đích khơi dậy sự quan tâm đến khoa
học và công nghệ giữa chúng ta và công chúng.
Câu 32: Đáp án D
Kiến thức: Đọc hiểu
Giải chi tiết:
Theo đoạn 2, điều nào sau đây KHÔNG đúng với Trung tâm Khoa học Singapore?
A. Du khách được khuyến khích quay trở lại trung tâm.
B. Trung tâm không được mở cho đến năm 1977.
C. Các cuộc triển lãm ở trung tâm bao gồm một loạt các chủ đề.
D. Trung tâm là nơi đầu tiên được thành lập trên thế giới.
Thông tin: The centre is the first science one to be established in South East Asia.
Tạm dịch: Đó là trung tâm khoa học đầu tiên được thành lập ở Đông Nam Á.
Câu 33: Đáp án A
Kiến thức: Đọc hiểu
Giải chi tiết:
Được khẳng định trong đoạn 2 rằng __________.
A. trung tâm khoa học làm cho mọi người quan tâm đến khoa học và công nghệ.
B. chỉ sinh viên mới có thể đến trung tâm khoa học
C. du khách không muốn quay lại trung tâm khoa học
D. chỉ có một số triển lãm trong trung tâm khoa học
Thông tin: It aims to arouse interest in science and technology among us and the general public.
Tạm dịch: Nó nhằm mục đích khơi dậy sự quan tâm đến khoa học và công nghệ giữa chúng ta và công
chúng.
Câu 34: Đáp án D
Kiến thức: Đọc hiểu
Giải chi tiết:
Tác giả đề cập đến tất cả những điều sau đây trong đoạn văn NGOẠI TRỪ _________.
A. Các cuộc triển lãm được đổi mới hàng năm.
B. Trung tâm nằm ở Jurong
C. Có bốn phòng triển lãm ở trung tâm.

Trang 99
D. Trung tâm là lớn nhất ở châu Á.
Thông tin:
These exhibits are renewed annually so as to encourage visitors to make return visits to the centre.
The Singapore Science Centre is located on a six-hectare site in Jurong…
The exhibits can be found in four exhibition galleries…
Tạm dịch:
Những triển lãm này được đổi mới hàng năm để khuyến khích du khách thực hiện các chuyến thăm quan
trở lại trung tâm.
Trung tâm Khoa học Singapore nằm trên một khu đất rộng sáu ha ở Jurong…
Các triển lãm có thể được tìm thấy trong bốn phòng…
Câu 35: Đáp án B
Kiến thức: Đọc hiểu
Giải chi tiết:
Ý chính của đoạn đầu tiên là gì?

A. Dự đoán các phương thức thanh toán trong tương lai.


B. Sự phổ biến ngày càng tăng của các phương thức thanh toán mới.
C. Sự vắng mặt của các phương thức thanh toán truyền thống.
D. Hình thức thanh toán tiên tiến của Nhật Bản.
Thông tin: Smart cards and mobile phones are becoming an increasingly popular way to make all sorts of
payments.
Tạm dịch: Thẻ thông minh và điện thoại di động đang trở thành một cách ngày càng phổ biến để thực
hiện tất cả các loại thanh toán.
Câu 36: Đáp án A
Kiến thức: Đọc hiểu
Giải chi tiết:
Tại sao tác giả đề cập đến "một thiết bị màn hình phẳng nhỏ" trong đoạn đầu tiên?
A. để minh họa cho hệ thống tiền điện tử
B. để ca ngợi hệ thống tiền điện tử
C. chỉ trích hệ thống tiền điện tử
D. để thông báo cho hệ thống tiền điện tử
Thông tin: Even now, in Japan thousands of transactions, from paying rail tickets to picking up the
groceries, take place every day with customers passing their handsets across a small flat-screen device...
Tạm dịch: Ngay cả bây giờ, tại Nhật Bản, hàng ngàn giao dịch, từ việc trả vé đường sắt đến mua hàng tạp
hóa, diễn ra mỗi ngày với khách hàng đi qua thiết bị cầm tay của họ trên một thiết bị màn hình phẳng nhỏ.

Trang 100
Câu 37: Đáp án B
Kiến thức: Đọc hiểu
Giải chi tiết:
Điều nào sau đây KHÔNG đúng về điểm mạnh của tiền điện tử?
A. giảm chi phí
B. không gian lận
C. ít sai lầm
D. tốc độ nhanh hơn
Thông tin: Fraud and theft are also reduced and for the retailer, it reduces the cost of handling money.
Tạm dịch: Gian lận và trộm cắp chỉ được giảm chứ không phải là hoàn toàn không có.
Câu 38: Đáp án D
Kiến thức: Từ vựng
Giải chi tiết:
Từ "embedded” trong đoạn thứ hai có nghĩa gần nhất với ________.
embedded: gắn
A. isolated: cô lập
B. manufactured: sản xuất
C. generated: tạo ra
D. integrated: tích hợp, gắn
=> be embedded = be integrated
Thông tin: Sony's vision of having a chip embedded in computers, TVs and games consoles means that
films, music and games can be paid for easily without having to input credit card details.
Tạm dịch: Tầm nhìn của Sony về việc có một con chip gắn trên máy tính, TV và máy chơi game có nghĩa
là phim, nhạc và trò chơi có thể được thanh toán dễ dàng mà không cần phải nhập chi tiết thẻ tín dụng.
Câu 39: Đáp án A
Kiến thức: Từ vựng
Giải chi tiết:
Từ "grip” trong đoạn thứ hai có nghĩa gần nhất với ________.
grip (n): tầm kiểm soát, quyền lực
A. power (n): quyền lực
B. success (n): thành công
C. wealth (n): giàu có
D. range of branches: phạm vi của các chi nhánh
=> grip = power
Thông tin: Within their grip on the market, banks and creditcard firms want to be in a position to collect
most of the fees from the users of mobile and contactless-payment systems.

Trang 101
Tạm dịch: Trong tầm kiểm soát của họ trên thị trường, các ngân hàng và các công ty thẻ tín dụng muốn có
thể thu được hầu hết các khoản phí từ người dùng hệ thống thanh toán di động mà không cần sự tiếp xúc.
Câu 40: Đáp án B
Kiến thức: Đọc hiểu
Giải chi tiết:
Tác giả đề cập đến trường hợp của hành khách trong đoạn thứ ba để minh họa __________.
A. hợp tác của các ngân hàng với các công ty thẻ tín dụng
B. một nhược điểm có thể có của hệ thống
C. các công nghệ hiện đại của hệ thống tiền điện tử
D. sự liên thông của hệ thống
Thông tin: But the new system could prove to be a "disruptive technology" as far as the banks are
concerned. If payments for a few coffees, a train ticket and a newspaper are made every day by a
commuter with a mobile, this will not appear on their monthly credit card statements but on their mobile
phone statements.
Tạm dịch: Nhưng hệ thống mới có thể chứng minh là một "công nghệ gây rắc rối" theo như các ngân
hàng có liên quan. Nếu thanh toán cho một vài loại cà phê, vé tàu và báo được thực hiện mỗi ngày bởi
một người đi lại bằng vé tháng với một chiếc điện thoại di động, điều này sẽ không xuất hiện trên bảng
sao kê thẻ tín dụng hàng tháng của họ mà trên bảng sao kê điện thoại di động của họ.
Câu 41: Đáp án C
Kiến thức: Đọc hiểu
Giải chi tiết:
Từ “their” trong đoạn 3 thay thế cho __________.
A. thẻ tín dụng
B. ngân hàng trên đường cao tốc
C. nhân viên ngân hàng Mỹ
D. ngân hàng điện tử
Thông tin: However, quite a few American bankers are optimistic. They feel there is reason to be
suspicious of those who predict that high-street banks may be a thing of the past. They point out that
Internet banking did not result in the closure of their high-street branches as was predicted.
Tạm dịch: Tuy nhiên, khá nhiều nhân viên ngân hàng Mỹ lạc quan. Họ cảm thấy có lý do để nghi ngờ
những người dự đoán rằng các ngân hàng trên đường phố cao có thể lỗi thời. Họ chỉ ra rằng ngân hàng
Internet đã không dẫn đến việc đóng cửa các chi nhánh trên đường phố cao của họ như dự đoán.
Câu 42: Đáp án C
Kiến thức: Đọc hiểu
Giải chi tiết:
Nhà văn có vẻ cảm thấy thế nào về tương lai của các ngân hàng?

Trang 102
A. trung lập
B. lạc quan
C. không chắc chắn
D. bi quan
Thông tin: So, whether we'll become a totally cash-free society remains open to contention.
Tạm dịch: Vì vậy, chúng ta sẽ trở thành một xã hội hoàn toàn bằng tiền mặt miễn phí hay không vẫn còn
tranh cãi.
Câu 43: Đáp án D
Kiến thức: Đảo ngữ
Giải chi tiết:
Cấu trúc: Not only + tobe + S + adj + but S + (also) + V: … không những … mà còn …

A. Thật không công bằng khi phủ nhận rằng trường học này rất thành công, vì nó đã có chất lượng cao
trong nhiều năm nay. => sai về nghĩa
B. Chất lượng giáo dục trong trường này không cao, nhưng ít nhất tất cả học sinh đều ở cùng một trình
độ. => sai về nghĩa
C. Trình độ giáo dục tại trường này, thường khá là cao, cho thấy chỉ có sự thay đổi nhỏ theo từng năm. =>
sai về nghĩa
D. Không chỉ chất lượng đào tạo tốt ở trường này mà nó còn đã duy trì được chất lượng này qua nhiều
năm.
Tạm dịch: Trường này không chỉ có chất lượng giáo dục cao. Nó còn duy trì phong độ trong suốt nhiều
năm.
= Không chỉ chất lượng đào tạo tốt ở trường này mà nó còn đã duy trì được chất lượng này qua nhiều
năm.
Câu 44: Đáp án C
Kiến thức: Câu đồng nghĩa
Giải chi tiết:
Cấu trúc: S + V + so + adj/adv + that + S + V: quá … đến nỗi mà
A. Rõ ràng là chỉ có một người với tài nghệ xuất sắc mới có thể làm giả một bức tranh thành công như
vậy. => sai về nghĩa
B. Hầu như không thể để những người nghiệp dư nhận ra rằng bức tranh là giả, mặc dù các chuyên gia có
thể nhận ra nó khá dễ dàng. => sai về nghĩa
C. Bức tranh rất giống với bản gốc đến nỗi mà chỉ có các chuyên gia có thể biết đó không phải bản gốc.
D. Thật khó để những người nghiệp dư đánh giá giữa bức tranh giả và bức tranh thật, nhưng ngoại trừ các
chuyên gia. => sai về nghĩa

Trang 103
Tạm dịch: Không ai ngoại trừ các chuyên gia có thể nhận ra rằng bức tranh là đồ giả. Nó trông cực kỳ
giống với bản gốc.
= Bức tranh rất giống với bản gốc đến nỗi mà chỉ có các chuyên gia có thể biết đó không phải bản gốc.
Câu 45: Đáp án D
Kiến thức: So sánh với tính từ
Giải chi tiết:
Cấu trúc so sánh bằng: S1 + to be + as + adj + as + S2.
Cấu trúc so sánh hơn: S1 + to be + adj-er/ more adj + than + S2.
yours = your coffee
Tạm dịch: Cà phê của bạn không ngon bằng của tôi.
= Cà phê của tôi ngon hơn cà phê của bạn.
Câu 46: Đáp án B
Kiến thức: Câu tường thuật
Giải chi tiết:
be always + V_ing: lúc nào cũng làm gì đó (phàn nàn về hành động của ai khiến người nói thấy khó chịu)
Câu trực tiếp: “S + am/ is/ are + always + V_ing”, said S1.
Câu gián tiếp: S1 complained about + Ving: Ai đó phàn nàn về điều gì
Tạm dịch: “Em luôn mắc những sai lầm khủng khiếp.”, thầy giáo nói.
= Thầy giáo phàn nàn về việc học trò của minh mắc những sai lầm khủng khiếp.
A. Thầy giáo nhận ra rằng học sinh của mình luôn mắc sai lầm khủng khiếp. => sai về nghĩa
C. Thầy giáo hỏi học sinh của mình tại sao họ luôn mắc sai lầm khủng khiếp. => sai về nghĩa
D. Thầy giáo khiến học sinh của mình không phải lúc nào cũng mắc sai lầm khủng khiếp. => sai về nghĩa
Câu 47: Đáp án C
Kiến thức: Câu đồng nghĩa
Giải chi tiết:
Adj + though + S + tobe, S + V = Although S + tobe + adj, S + V: mặc dù
Tạm dịch: Mặc dù kỳ thi khó, nhưng anh ấy vẫn vượt qua với kết quả tốt.
= Dù bài thi có khó khăn đến đâu, anh vẫn vượt qua nó với kết quả tốt.
A. Vì kỳ thi khó khăn, anh ấy đã vượt qua nó với kết quả tốt. => sai về nghĩa
B. Sai vì có “although” thì không dùng “but”
D. Bài kiểm tra không quá khó, vì vậy anh ấy có thể vượt qua nó với kết quả tốt. => sai về nghĩa
Câu 48: Đáp án D
Kiến thức: Đại từ quan hệ
Giải chi tiết:
Trọng mệnh đề quan hệ:
- who + V/S + V: người mà => thay thế cho một danh từ chỉ người; đóng vai trò chủ ngữ/ tân ngữ

Trang 104
- which + V/S + V: cái mà => thay thế cho một danh từ chỉ vật; đóng vai trò chủ ngữ/ tân ngữ
“students” (những học sinh) => chỉ người
Sửa: which => who/that
Tạm dịch: Tom là một trong những học sinh có kết quả tồi tệ nhất trong kỳ thi vừa qua.
Câu 49: Đáp án B
Kiến thức: Đảo ngữ
Giải chi tiết:
Cấu trúc: No sooner + had + S + PII + than + S + V_ed: … vừa mới … thì
Sửa: when => than
Tạm dịch: Gió vừa mới ngừng thổi thì mưa bắt đầu rơi nặng hạt.
Câu 50: Đáp án D
Kiến thức: Liên từ
Giải chi tiết:
Although S + V, S + V: mặc dù
S + V, but S + V: nhưng
Trong câu đã có “Although” thì không dùng “but”
Sửa: but => bỏ
Tạm dịch: Mặc dù họ luôn tranh luận với nhau nhưng họ là những người bạn tốt.

TRƯỜNG THPT ĐỀ THI THỬ LẦN 1 NĂM HỌC: 2019 – 2020


LÝ THÁI TỔ MÔN: TIẾNG ANH
Thời gian làm bài: 60 phút; không kể thời gian phát đề

Mark the letter A, B, C, or D on your answer sheet to indicate the correct answer to each of the following
questions.
Câu hỏi 1 (NB): Mike came across his old friend - Maria while he _______ along the street.
A. had walked B. would walk C. was walking D. walked
Câu hỏi 2 (VDC): I don’t mind walking long distances, because I know it ________ me good.
A. does B. plays C. gets D. gives
Câu hỏi 3 (TH): Nowadays, most people rely ________ the mass media as their main source of
information.
A. on B. in C. to D. with
Câu hỏi 4 (NB): Human beings are ________ greatest threat to survival of endangered species.
A. Ø B. an C. the D. a
Câu hỏi 5 (TH): _________, we couldn’t make out the gist of the passage.
A. Due to Romeo’s carefully explanations B. Though carefully explained by Romeo

Trang 105
C. Carefully as Romeo explained D. As Romeo carefully explained
Câu hỏi 6 (TH): Josh would never forget ________ by his parent through no fault of his own.
A. having scold B. to have scolded C. to be scolded D. being scolded
Câu hỏi 7 (TH): All his companies had been successful and he was known to be ________ rich.
A. excessively B. completely C. extremely D. thoroughly
Câu hỏi 8 (NB): It is of great ________ to show your interest and politeness when you are having a job
interview.
A. important B. importance C. importantly D. unimportant
Câu hỏi 9 (VDC): I looked the word up in the dictionary to refresh my ________ of its exact meaning.
A. brain B. mind C. thoughts D. memory
Câu hỏi 10 (TH): If ________ access to education, children living in poverty will have a brighter future.
A. providing B. are provided C. having provided D. provided
Câu hỏi 11 (VD): Andrew is a very disciplined employee. He ________ great importance to coming to
work in time.
A. affixes B. attaches C. admits D. attributes
Câu hỏi 12 (TH): Jane never gives her mother a hand in housework _________ she has a lot of free time.
A. although B. since C. in spite of D. because of
Câu hỏi 13 (TH): Rita is not _________ of doing this work - she should change her class.
A. capable B. possible C. suitable D. habitual
Câu hỏi 14 (NB): If you eat too much, you _________ the price by putting on weight.
A. will pay B. paid C. would pay D. were paying
Mark the letter A, B, C, or D on your answer sheet to indicate the most suitable response to complete each
of the following exchanges.
Câu hỏi 15 (NB): Joana and David, two students, are talking about studying abroad.
Joana: "I think studying abroad is the only way to get a well-paid job."
David: “___________. There’re still many different ways to get it."
A. You're exactly right B. I don’t think so C. That’s what I think D. There’s no
doubt about it
Câu hỏi 16 (NB): Peter is having dinner at Wendy’s house.
Peter: "This steamed crab is so delicious."
Wendy:" ________.''
A. Sure. I’d love to. B. I’m afraid not. C. No, don’t worry D. I’m glad you like it
Mark the letter A, B, C, or D on your answer sheet to indicate the word whose underlined part differs
from the other three in pronunciation in each of the following questions.
Câu hỏi 17 (NB): A. mouth B. founder C. about D. country
Câu hỏi 18 (NB): A. accessed B. searched C. recorded D. developed

Trang 106
Mark the letter A, B, C, or D on your answer sheet to indicate the word that differs from the other three in
the position of primary stress in each of the following questions.
Trả lời cho các câu 19, 20 dưới đây:
Câu hỏi 19 (NB): A. interpret B. chemistry C. attention D. romantic
Câu hỏi 20 (NB): A. attract B. discuss C. follow D. confide
Mark the letter A, B, C, or D on your answer sheet to indicate the word(s) CLOSEST in meaning to the
underlined word(s) in each of the following questions.
Câu hỏi 21 (VD): His second venture, a software firm, is booming and will soon be ready to take on more
staff.
A. adopt B. dismiss C. prepare D. employ
Câu hỏi 22 (TH): The smell of chalk always evokes memories of my school days.
A. revises B. brings C. recalls D. catches
Mark the letter A, B, C, or D on your answer sheet to indicate the word(s) OPPOSITE in meaning to the
underlined word(s) in each of the following questions.
Câu hỏi 23 (TH): You won’t be penalized for venturing a guess, so be sure not to leave any questions
unanswered.
A. rewarded B. punished C. motivated D. discouraged
Câu hỏi 24 (VDC): I think we cannot purchase this device this time as it costs an arm and a leg.
A. is cheap B. is painful C. is confusing D. is expensive
Read the following passage and mark the letter A, B, C, or D on your answer sheet to indicate the correct
word or phrase that best fits each of the numbered blanks.
Adult Learners
Adult learners have many characteristics that distinguish them from school or college learners. Firstly,
they come to courses with experiences and knowledge in diverse areas. They often prefer (25) ________
learning activities that enable them to draw on the skills and experience they have acquired. In the main,
adults have realistic aims and have valuable (26) ________ into what is likely to be successful. (27)
________, they are readily able to relate new facts to past experiences and enjoy having their talents
explored in leaning situations. Adults have established opinions and beliefs, (28) ________ have been
formed over time through their experience of families, work, community, or politics.
A majority of adults also have (29) ________ motivation and their effort increases because of their desire
to learn. they are also motivated by the relevance of the material to be addressed, and learn better when
material is related to their own needs and interests.
(Adopted from “Complete Advanced” by Guy Brook-Hart and Simon Haines)
Câu hỏi 25 (TH): A. practicality B. practical C. practically D. practice
Câu hỏi 26 (TH): A. ability B. grasp C. uptake D. insight
Câu hỏi 27 (TH): A. In addition B. In contrast C. As a result D. However

Trang 107
Câu hỏi 28 (NB): A. where B. which C. who D. that
Câu hỏi 29 (TH): A. instinctive B. intrinsic C. spiritual D. perceptive
Read the following passage and mark the letter A, B, C, or D on your answer sheet to indicate the correct
answer to each of the questions from 38 to 42.
Table manners differ around the world. If you visit a friend’s home for a meal, it’s good to know about
the customs they follow.
If you are invited to a Moroccan’s home, bring a gift of sweet pastries, nuts, figs, dates, or flowers to the
hostess. In many traditional homes, people often sit on the floor to eat a meal. Often, everyone shares
food from the same 10 plate. The plate is put in the centre of the table. Usually, everyone also drinks
water from the same glass. Only eat the food in front of you. Don’t reach across someone for food. It is
common to use you your fingers and small pieces of bread to eat food. Use only your right hand to eat.
Don’t say “no” to food. If the host of the meal offers you food or drink, take some and try a little. In many
Moroccan homes, it is common to take off your shoes.
Tanzania is known for its fresh fish and spices. Don’t act scared if your meal comes with its head still on
the plate. It is polite to try a bite of everything. In many Tanzania homes, people sit at a table on a small
stools to eat. It is OK to eat with your right hand, using bread or chapati to pick up food. Everyone may
take food from the same plate., but it is not usual for people to share drinks. You may compliment the
cook on a delicious meal, but don’t exaggerate. In some parts of Tanzania, men and women still sit at
different tables. Also remember: it is common to take off your shoes in the home, but it is rude to show
the bottom of your foot.
(Adopted from Active Skills for Reading Intro by Neil J Anderson)
Câu hỏi 30 (TH): Which best serves as the title for the passage?
A. Table Manners across some cultures in the world.
B. Tips for foreigners when having meal in Tanzania
C. Table Manners in Morocco, Things to do for everyone
D. Table Manners - Ultimate Guide to Dining Etiquette
Câu hỏi 31 (NB): The word "they" in paragraph 1 refers to ________.
A. customs B. manners C. meals D. friends
Câu hỏi 32 (TH): According to the passage, why does the writer think it is good to know about customs in
other countries?
A. You can know how to eat healthily. B. You can act in a proper way.
C. You can order food in a restaurant. D. You can learn the language.
Câu hỏi 33 (TH): The word "offers" in paragraph 2 is closest in meaning to ________.
A. exchanges B. attends C. receives D. provides
Câu hỏi 34 (TH): According to paragraph 3, which of the following should NOT do in Tanzania?
A. eat with your hands B. take food from the same plate

Trang 108
C. show the bottom of your foot D. take off your shoes
Read the following passage and mark the letter A, B, C, or D on your answer sheet to indicate the correct
answer to each of the questions.
Nepal has made important progress over the past few years to promote equality, but the country still has
one of the highest rates of child marriage in the world. 41% of Nepalese girls are married before the age
of 18.
Poverty is both a cause and consequence of child marriage in Nepal. Girls from the wealthiest families
marry 2 years later than those from the poorest, who are seen as an economic burden, and who drop out of
school and earn little money.
Food insecurity plays an important role too. Nepalese families that do not have enough food to eat are
more likely to marry their daughters at a young age to decrease the financial burden. One study shows
that 91% of people who had secure access to food married over the age of 19.
Dowry is also common practice in many communities. Parents marry their daughters as soon as possible
because the money they have to pay to the groom’s family is higher if their daughter is older. Since 2010,
the legal age of marriage is 20 for both men and women, or 18 with parental consent, according to the
Nepalese Country Code.
The law states that punishment for child marriage is imprisonment for up to three years and a fine of up to
10,000 rupees (£102). But reports suggest that this law is rarely applied. There has been quite a lot of
progress in Nepal over the past 3 years with a clear government commitment to ending child marriage and
civil society cooperation. The Ministry of Women, Children and Social Welfare is currently developing
Nepal’s first national strategy on child marriage in collaboration with UNICEF Nepal and Girls Not
Brides Nepal. However, the post-earthquake and post-fuel crisis environment has meant progress is slow
and the national strategy has been delayed.
(Adapted from http://www.girlsnotbrides.org/)
Câu hỏi 35 (TH): Which of the following best serves as the title for the passage?
A. Nepal Government in an attempt to reduce social inequality.
B. Child marriage - A completely tractable problem in Nepal.
C. Nepal’s strategies to child protection programming.
D. One type of child abuse in Nepal: Child Marriage.
Câu hỏi 36 (TH): According to paragraph 2, in Nepal, girls from needy families ________.
A. play an important role in bolstering their household finances
B. have to get married earlier than those are from affluent families
C. can get married two years later than those are from poor families
D. are forced to drop out of school because their parents have no money
Câu hỏi 37 (NB): The word "those" in paragraph 2 refers to ________.
A. girls B. families C. years D. rates

Trang 109
Câu hỏi 38 (TH): The word "burden” in paragraph 3 is closest in meaning to ________.
A. issue B. potency C. load D. cargo
Câu hỏi 39 (TH): According to the last paragraph, how many years will a rule-breaker spend in jail?
A. exactly three years B. over three years C. from one to three years D. about two years
Câu hỏi 40 (TH): The word "imprisonment” in the last paragraph could be best replaced by _________.
A. detention B. custody C. salvation D. emblem
Câu hỏi 41 (TH): According to the passage, which of the following statements is NOT true?
A. Nepalese families have a tendency to marry their daughters at a young age.
B. Parents who marry their children before the allowed age normally pay a fine.
C. The age of marriage in Nepal depends on the money the families have.
D. According to the law, if people have their parents’ permission, they are allowed to marry at 18.
Câu hỏi 42 (VD): It can be inferred from the passage _________.
A. The government is not doing much to put an end to child marriage
B. Nepal had the highest overall prevalence of child marriage in the world
C. Families want to marry their daughters soon to earn money
D. The recent earthquake was negative for the eradication of child marriage
Mark the letter A, B, C, or D on your answer sheet to indicate the underlined part that needs correction in
each of the following questions.
Câu hỏi 43 (NB): My brother and I go sometimes swimming together with our uncle’s family.
A. My brother B. go sometimes C. together D. uncle’s family
Câu hỏi 44 (TH): Her father used to be a distinguishable professor at the university. Many students
worshipped him.
A. used to B. distinguishable C. at D. worshipped
Câu hỏi 45 (TH): Those famous pictures hanging on the gallery wall painted in France.
A. Those B. hanging C. the D. painted
Mark the letter A, B, C, or D on your answer sheet to indicate the sentence that is closest in meaning to
each of the following questions.
Câu hỏi 46 (VD): "You should have informed me of the change in advance.” The boss said to his
secretary.
A. The boss blamed his secretary for not informing him of the change in advance.
B. The boss thanked his secretary for informing him of the change in advance.
C. The boss encouraged his secretary not to inform him of the change in advance.
D. The boss advised his secretary to inform him of the change in advance.
Câu hỏi 47 (VD): In Venezuela, beauty contests are more popular than football.
A. In Venezuela, beauty contests are as popular as football.
B. In Venezuela, football is more popular than beauty contests.

Trang 110
C. In Venezuela, football is not as popular as beauty contests.
D. In Venezuela, beauty contests are not as popular as football.
Câu hỏi 48 (VD): Joey put all his money on the lottery, which was his big mistake.
A. Joey shouldn’t have put all his money on the lottery.
B. Joey needn’t have put all his money on the lottery.
C. Joey may not have put all his money on the lottery.
D. Joey can’t have put all his money on the lottery.
Mark the letter A, B, C, or D on your answer sheet to indicate the sentence that best combines each pair
of sentences in the following questions.
Câu hỏi 49 (VD): Susan didn’t apply for the job in the library. She regrets it now.
A. Susan wishes she had applied for the job in the library.
B. If only Susan applied for the job in the library.
C. Susan regrets applying for the job in the library.
D. Susan regrets to apply for the job in the library.
Câu hỏi 50 (VDC): Hans told us about his investment in the company. He did it on his arrival at the
meeting.
A. Hardly had he informed us about his investment in the company when Hans arrived at the
meeting.
B. Only after investing in the company did Hans informs us of his arrival at the meeting.
C. No sooner had Hans arrived at the meeting than he told us about his investment in the
company.
D. Not until Hans told us that he would invest in the company did he arrive at the meeting.

Đáp án
1-C 2-A 3-A 4-C 5-B 6-D 7-C 8-B 9-D 10-D
11-B 12-A 13-A 14-A 15-B 16-D 17-D 18-C 19-B 20-C
21-D 22-C 23-A 24-A 25-B 26-D 27-A 28-B 29-B 30-A
31-D 32-B 33-D 34-C 35-D 36-B 37-A 38-C 39-C 40-A
41-B 42-D 43-B 44-B 45-D 46-A 47-C 48-A 49-A 50-C

LỜI GIẢI CHI TIẾT


Câu 1: Đáp án C
Kiến thức: Thì quá khứ tiếp diễn
Giải chi tiết:
Cách dùng:

Trang 111
- Thì quá khứ tiếp diễn dùng khi muốn nhấn mạnh diễn biến hay quá trình của sự vật sự việc hoặc thời
gian sự vật hay sự việc đó diễn ra.
Công thức chung: S + was/ were + Ving.
- Thì quá khứ tiếp diễn diễn tả một hành động đang diễn ra (chia thì quá khứ tiếp diễn) thì có một hành
động khác xem vào (chia thì quá khứ đơn)
Công thức: S + V_ed/V cột 2 + while + S + was/ were + V_ing.
Tạm dịch: Mike tình cờ gặp người bạn cũ của mình - Maria khi anh đang đi trên đường.
Câu 2: Đáp án A
Kiến thức: Thành ngữ
Giải chi tiết:
do sbd good: có lợi cho ai, tốt cho ai
Chủ ngữ “it” => động từ chia “does”
Tạm dịch: Tôi không ngại đi bộ đường dài, vì tôi biết nó tốt cho tôi.
Câu 3: Đáp án A
Kiến thức: Giới từ
Giải chi tiết:
rely on: tin vào, dựa vào, phụ thuộc vào
Tạm dịch: Ngày nay, hầu hết mọi người đều dựa vào các phương tiện truyền thông đại chúng như là
nguồn thông tin chính của họ.
Câu 4: Đáp án C
Kiến thức: Mạo từ
Giải chi tiết:
great (adj): tốt, lớn => greatest: lớn/tốt nhất
Mạo từ “the” dùng trong câu so sánh hơn nhất: the + adj-est/ most adj
Tạm dịch: Con người là mối đe dọa lớn nhất đối với sự sống còn của các loài có nguy cơ tuyệt chủng.
Câu 5: Đáp án B
Kiến thức: Rút gọn mệnh đề
Giải chi tiết:
Khi hai mệnh đề có cùng chủ ngữ có thể rút gọn một mệnh đề về dạng:
- V_ing: khi mệnh đề dạng chủ động và hai hành động xảy ra liên tiếp
- Having V_ed/ V3: khi mệnh đề dạng chủ động, hành động được rút gọn xảy ra trước và là nguyên nhân
dẫn đến hành động còn lại.
- V_ed/ V3: khi mệnh đề dạng bị động (giữ nguyên liên từ)
Câu đầy đủ: Though we are explained carefully by Romeo, we couldn’t make out the gist of the passage.
Câu rút gọn: Though carefully explained by Romeo, we couldn’t make out the gist of the passage.
Tạm dịch: Mặc dù được Romeo giải thích cẩn thận, chúng tôi không thể tìm ra ý chính của đoạn văn.

Trang 112
Câu 6: Đáp án D
Kiến thức: to V/ Ving
Giải chi tiết:
forget to V_nguyên thể: quên làm việc gì
forget V_ing: quên chuyện đã làm
forget being V_ed/ V3: quên việc bị/được làm gì
Tạm dịch: Josh sẽ không bao giờ quên việc bị cha mẹ mắng mà không phải do lỗi của mình.
Câu 7: Đáp án C
Kiến thức: Từ vựng
Giải chi tiết:
A. excessively (adv): quá chừng, quá đáng
B. completely (adv): hoàn toàn, đầy đủ, trọn vẹn
C. extremely (adv): vô cùng, cực độ, cực kỳ
D. thoroughly (adv): kỹ lưỡng, thấu đáo
Tạm dịch: Tất cả các công ty của ông đã thành công và ông được biết đến là người vô cùng giàu có.
Câu 8: Đáp án B
Kiến thức: Từ loại
Giải chi tiết:
A. important (adj): quan trọng, hệ trọng
B. importance (n): sự quan trọng, tầm quan trọng
C. importantly (adv): quan trọng
D. unimportant (adj): không quan trọng
Dấu hiệu: sau tính từ “great” (to lớn) cần một danh từ
Tạm dịch: Điều rất quan trọng là thể hiện sự quan tâm và lịch sự của bạn khi bạn đang có một cuộc phỏng
vấn xin việc.
Câu 9: Đáp án D
Kiến thức: Thành ngữ
Giải chi tiết:
refresh one’s memory: gợi nhớ lại, nhớ lại cái gì
Tạm dịch: Tôi đã tra từ trong từ điển để nhớ lại ý nghĩa chính xác của nó.
Câu 10: Đáp án D
Kiến thức: Rút gọn mệnh đề
Giải chi tiết:
Khi hai mệnh đề có cùng chủ ngữ có thể rút gọn một mệnh đề về dạng:
- Ving: khi mệnh đề dạng chủ động và hai hành động xảy ra liên tiếp

Trang 113
- Having Ved/ V3: khi mệnh đề dạng chủ động, hành động được rút gọn xảy ra trước và là nguyên nhân
dẫn đến hành động còn lại.
- Ved/ V3: khi mệnh đề dạng bị động
Câu đầy đủ: If children living in poverty are provided access to education, children will have a brighter
future.
Câu rút gọn: If provided access to education, children living in poverty will have a brighter future.
Tạm dịch: Nếu được cung cấp sự tiếp cận với giáo dục, trẻ em sống trong nghèo đói sẽ có một tương lai
tươi sáng hơn.
Câu 11: Đáp án B
Kiến thức: Sự kết hợp từ
Giải chi tiết:
A. affixes (v): in vào
B. attaches (v): gắn, dán, gia nhập, coi
C. admits (v): thừa nhận, thú nhận
D. attributes (v): quy cho
=> attach importance to sth/doing sth: coi việc gì là quan trọng
Tạm dịch: Andrew là một nhân viên rất kỷ luật. Ông rất coi trọng việc đến làm việc đúng giờ.
Câu 12: Đáp án A
Kiến thức: Liên từ
Giải chi tiết:
A. although S + V: mặc dù
B. since S + V: vì
C. in spite of + N/ Ving: mặc dù
D. because of + N/ Ving: vì
“she has a lot of free time” là một mệnh đề gồm S + V => loại C, D
Tạm dịch: Jane không bao giờ giúp mẹ làm việc nhà mặc dù cô có rất nhiều thời gian rảnh.
Câu 13: Đáp án A
Kiến thức: Từ vựng
Giải chi tiết:
A. capable (adj): có khả năng
B. possible (adj): có thể
C. suitable (adj): phù hợp, thích hợp
D. habitual (adj): thường lệ, quen thuộc
=> to be capable of + V_ing: có khả năng làm gì
Tạm dịch: Rita không có khả năng làm việc này - cô ấy nên chuyển lớp.
Câu 14: Đáp án A

Trang 114
Kiến thức: Câu điều kiện loại 1
Giải chi tiết:
- Dấu hiệu: vế chứa “if” chia “eat” ở thì hiện tại đơn
- Cách dùng: Câu điều kiện loại 1 dùng để diễn tả điều có thể xảy ra ở hiện tại hoặc tương lai:
- Công thức: If + S + V(s/es) + O, S + will + V(nguyên thể) + O.
Tạm dịch: Nếu bạn ăn nhiều, bạn sẽ trả giá bằng việc tăng cân.
Câu 15: Đáp án B
Kiến thức: Ngôn ngữ giao tiếp
Giải chi tiết:
Hai sinh viên Joana và David đang nói về việc đi du học.
- Joana: "Mình nghĩ du học là cách duy nhất để có được một công việc với mức lương cao."
- David: “________. Vẫn có nhiều cách khác nhau để có được công việc đó.”
A. Cậu hoàn toàn đúng
B. Mình không nghĩ vậy
C. Đó là những gì mình nghĩ
D. Chắc chắn là vậy
Các phản hồi A, C, D không phù hợp với ngữ cảnh
Câu 16: Đáp án D
Kiến thức: Ngôn ngữ giao tiếp
Giải chi tiết:
Peter đang ăn tối tại nhà của Wendy.
- Peter: "Món cua hấp này rất ngon."
- Wendy: "________ .''
A. Chắc chắn. Mình cũng thích vậy.
B. Mình sợ rằng không được.
C. Không, đừng lo lắng.
D. Mình vui vì bạn thích nó.
Các phản hồi A, B, C không phù hợp với ngữ cảnh
Câu 17: Đáp án D
Kiến thức: Phát âm “-ou”
Giải chi tiết:
A. mouth /maʊθ/
B. founder /ˈfaʊndər/
C. about /əˈbaʊt/
D. country /ˈkʌntri/
Phần gạch chân đáp án D phát âm là /ʌ/, còn lại là /aʊ/

Trang 115
Câu 18: Đáp án C
Kiến thức: Phát âm “-ed”
Giải chi tiết:
A. accessed /ˈæksest/
B. searched /sɜːtʃt/
C. recorded /rɪˈkɔːdɪd/
D. developed /dɪˈveləpt/
Quy tắc:
Cách phát âm đuôi “ed”:
- Đuôi “ed” được phát âm là /ɪd/ khi động từ có phát âm kết thúc là /t/ hay /d/
- Đuôi “ed” được phát âm là /t/ khi động từ có phát âm kết thúc là /s/,/f/,/p/,/ʃ/,/tʃ/,/k/
- Đuôi “ed” được phát âm là /d/ với các trường hợp còn lại
Phần gạch chân đáp án C phát âm là /ɪd/, còn lại là /t/
Câu 19: Đáp án B
Kiến thức: Trọng âm từ có 3 âm tiết
Giải chi tiết:
A. interpret /ɪnˈtɜːprət/
B. chemistry /ˈkemɪstri/
C. attention /əˈtenʃn/
D. romantic /rəʊˈmæntɪk/
Quy tắc: Những từ có tận cùng là đuôi “-tion”, “-ic” có trọng âm rơi vào âm tiết đứng ngay trước nó.
Trọng âm đáp án B rơi vào âm tiết thứ nhất, còn lại là âm thứ hai
Câu 20: Đáp án C
Kiến thức: Trọng âm từ có 2 âm tiết
Giải chi tiết:
A. attract /əˈtrækt/
B. discuss /dɪˈskʌs/
C. follow /ˈfɒləʊ/
D. confide /kənˈfaɪd/
Quy tắc: Động từ có 2 âm tiết thường có trọng âm rơi vào âm tiết thứ hai. Ngoại lệ: ‘follow
Trọng âm đáp án C rơi vào âm tiết thứ nhất, còn lại là âm thứ hai
Câu 21: Đáp án D
Kiến thức: Cụm động từ
Giải chi tiết:
take on: tuyển dụng, thuê
A. adopt (v): chấp nhận, thông qua

Trang 116
B. dismiss (v): sa thải, đuổi
C. prepare (v): chuẩn bị
D. employ (v): thuê, tuyển dụng
=> take on = employ
Tạm dịch: Dự án kinh doanh thứ hai của ông, một công ty phần mềm, đang thành công rực rỡ và sẽ sớm
sẵn sàng tuyển dụng thêm nhân viên.
Câu 22: Đáp án C
Kiến thức: Từ vựng
Giải chi tiết:
evokes (v): gợi lại
A. revises (v): ôn thi, xem lại
B. brings (v): mang lại, đem lại
C. recalls (v): gợi lại
D. catches (v): bắt lấy, nắm lấy
=> evoke = recall
Câu 23: Đáp án A
Kiến thức: Từ vựng
Giải chi tiết:
penalize - penalized - penalized: trừng phạt
A. reward - rewarded - rewarded: thưởng
B. punish - punished - punished: trừng phạt
C. motivate - motivated - motivated: thúc đẩy
D. discourage - discouraged - discouraged: làm nản lòng
=> penalize: trừng phạt >< reward: thưởng
Tạm dịch: Bạn sẽ không bị phạt vì đoán bừa, vì vậy hãy chắc chắn đừng quên trả lời câu hỏi nào.
Câu 24: Đáp án A
Kiến thức: Thành ngữ
Giải chi tiết:
costs an arm and a leg: giá rất đắt
A. is cheap: rẻ
B. is painful: đau
C. is confusing: hỗn độn
D. is expensive: đắt
=> costs an arm and a leg: giá rất đắt >< is cheap: rẻ
Tạm dịch: Tôi nghĩ rằng chúng ta không thể mua thiết bị này lần này vì giá của nó rất đắt.
Câu 25: Đáp án B

Trang 117
Kiến thức: Từ loại
Giải chi tiết:
A. practicality (n): tính thực tiễn, tính thực tế
B. practical (adj): thực hành, thiết thực
C. practically (adv): về mặt thực hành
D. practice (v): luyện tập, thực hành
Dấu hiệu: trước cụm danh từ “learning activities” (hoạt động học tập) cần một tính từ
They often prefer (25) practical learning activities that enable them to draw on the skills and experience
they have acquired.
Tạm dịch: Họ thường thích các hoạt động học tập thực tế cho phép họ rút ra các kỹ năng và kinh nghiệm
họ có được.
Câu 26: Đáp án D
Kiến thức: Từ vựng
Giải chi tiết:
A. ability (n): năng lực, khả năng
B. grasp (n): quyền lực, sự hiểu
C. uptake (n): sự hấp thu, sự thu hút
D. insight (n): sự hiểu thấu được bên trong sự vật
In the main, adults have realistic aims and have valuable (26) insight into what is likely to be successful.
Tạm dịch: Nhìn chung, người lớn có mục tiêu thực tế và có cái nhìn giá trị về những gì có khả năng thành
công.
Câu 27: Đáp án A
Kiến thức: Liên từ
Giải chi tiết:
A. In addition: Ngoài ra,, Thêm vào đó
B. In contrast: Ngược lại
C. As a result: Vì vậy, kết quả là
D. However: Tuy nhiên
(27) In addition, they are readily able to relate new facts to past experiences and enjoy having their talents
explored in leaning situations.
Tạm dịch: Ngoài ra, họ có thể dễ dàng liên hệ các sự kiện mới với kinh nghiệm trong quá khứ và thích
khám phá tài năng của họ trong các tình huống học tập.
Câu 28: Đáp án B
Kiến thức: Đại từ quan hệ
Giải chi tiết:
Trong mệnh đề quan hệ:

Trang 118
- where + S + V: nơi mà => thay thế cho trạng từ, cụm trạng từ chỉ nơi chốn
- which + V / S + V: cái mà => thay thế cho một danh từ chỉ vật; đóng vai trò chủ ngữ/ tân ngữ
- who + V / S + V: người mà => thay thế cho một danh từ chỉ người; đóng vai trò chủ ngữ/ tân ngữ
- that + V/S + V: người mà, cái mà => thay thế cho một danh từ chỉ người/vật, đóng vai trò chủ ngữ/tân
ngữ và không đứng sau dấu phẩy
“opinions and beliefs” (quan điểm và niềm tin) là cụm từ chỉ vật, trước nó có dấu phẩy => dùng “which”
Adults have established opinions and beliefs, (28) which have been formed over time through their
experience of families, work, community, or politics.
Tạm dịch: Người lớn đã thiết lập quan điểm và niềm tin, được hình thành theo thời gian thông qua kinh
nghiệm của họ về gia đình, công việc, cộng đồng hoặc chính trị.
Câu 29: Đáp án B
Kiến thức: Từ vựng
Giải chi tiết:
A. instinctive (adj): thuộc bản năng, theo bản năng
B. intrinsic (adj): thuộc bản chất, thực chất, bên trong
C. spiritual (adj): thuộc tinh tần, thuộc tâm hồn
D. perceptive (adj): sâu sắc, nhận thức
A majority of adults also have (29) intrinsic motivation and their effort increases because of their desire to
learn.
Tạm dịch: Phần lớn người trưởng thành cũng có động lực bên trong và nỗ lực của họ tăng lên vì mong
muốn học hỏi.
Câu 30: Đáp án A
Kiến thức: Đọc hiểu
Giải chi tiết:
Tiêu đề nào là phù hợp nhất cho bài đọc?
A. Phép tắc ăn uống trên một số nền văn hóa thế giới.
B. Lời khuyên cho người nước ngoài khi dùng bữa tại Tanzania
C. Cách cư xử ở Ma-rốc, những điều cần làm cho mọi người
D. Phép tắc ăn uống - Hướng dẫn cơ bản về nghi thức ăn uống
Thông tin: Table manners differ around the world.
Tạm dịch: Phép tắc ăn uống khác nhau trên khắp thế giới.
Câu 31: Đáp án D
Kiến thức: Đọc hiểu
Giải chi tiết:
Từ “they” trong đoạn 1 thay thế cho ________.
A. phong tục

Trang 119
B. cách cư xử
C. bữa ăn
D. bạn bè
Thông tin: If you visit a friend’s home for a meal, it’s good to know about the customs they follow.
Tạm dịch: Nếu bạn đến thăm nhà của một người bạn để dùng bữa, thì thật tốt khi biết về phong tục mà họ
tuân theo.
Câu 32: Đáp án B
Kiến thức: Đọc hiểu
Giải chi tiết:
Theo đoạn văn, tại sao người viết nghĩ rằng thật tốt khi biết về phong tục ở các nước khác?
A. Bạn có thể biết cách ăn uống lành mạnh.
B. Bạn có thể hành động một cách thích hợp.
C. Bạn có thể gọi đồ ăn trong một nhà hàng.
D. Bạn có thể học ngôn ngữ.
Thông tin: If you visit a friend’s home for a meal, it’s good to know about the customs they follow.
Tạm dịch: Nếu bạn đến thăm nhà của một người bạn để dùng bữa, thì thật tốt khi biết về phong tục mà họ
tuân theo.
Câu 33: Đáp án D
Kiến thức: Từ vựng
Giải chi tiết:
Từ “offers” trong đoạn 2 có nghĩa gần nhất với ________.
offer (v): cho, cung cấp, đề nghị
A. exchanges (v): trao đổi
B. attends (v): tham dự
C. receives (v): nhận
D. provides (v): cung cấp
=> offer = provide
Thông tin: If the host of the meal offers you food or drink, take some and try a little.
Tạm dịch: Nếu chủ nhà của bữa ăn cho bạn đồ ăn hoặc đồ uống, hãy lấy một ít và thử một chút.
Câu 34: Đáp án C
Kiến thức: Đọc hiểu
Giải chi tiết:
Theo đoạn 3, điều nào sau đây KHÔNG nên làm ở Tanzania?
A. ăn bằng tay
B. lấy thức ăn từ cùng một đĩa
C. khoe lòng bàn chân

Trang 120
D. cởi giày ra
Thông tin: Also remember: it is common to take off your shoes in the home, but it is rude to show the
bottom of your foot.
Tạm dịch: Cũng nên nhớ rằng: việc cởi giày trong nhà là điều thường thấy, nhưng thật thô lỗ khi để lộ
lòng bàn chân của bạn.
Câu 35: Đáp án D
Kiến thức: Đọc hiểu
Giải chi tiết:
Tiêu đề nào là phù hợp nhất cho bài đọc?
A. Chính phủ Nepal trong nỗ lực giảm bất bình đẳng xã hội.
B. Hôn nhân trẻ em - Một vấn đề khó giải quyết ở Nepal.
C. Chiến lược Nepal để thiết lập bảo vệ trẻ em.
D. Một loại lạm dụng trẻ em ở Nepal: Hôn nhân trẻ em.
Thông tin: Nepal has made important progress over the past few years to promote equality, but the
country still has one of the highest rates of child marriage in the world. 41% of Nepalese girls are married
before the age of 18… Nepalese families that do not have enough food to eat are more likely to marry
their daughters at a young age to decrease the financial burden.
Tạm dịch: Nepal đã đạt được những tiến bộ quan trọng trong vài năm qua để thúc đẩy bình đẳng, nhưng
đất nước này vẫn có một trong những tỷ lệ kết hôn ở trẻ em cao nhất thế giới. 41% các cô gái Nepal đã
kết hôn trước 18 tuổi… Các gia đình Nepal mà không có đủ thức ăn để ăn có nhiều khả năng cho con gái
của họ kết hôn khi còn trẻ để giảm gánh nặng tài chính.
Câu 36: Đáp án B
Kiến thức: Đọc hiểu
Giải chi tiết:
Theo đoạn 2, ở Nepal, các cô gái từ các gia đình nghèo khó ____.
A. đóng một vai trò quan trọng trong việc củng cố tài chính gia đình của họ
B. phải kết hôn sớm hơn các cô gái trong gia đình giàu có
C. có thể kết hôn muộn hơn hai năm so với các cô gái trong gia đình nghèo
D. bị buộc thôi học vì bố mẹ không có tiền
Thông tin: Girls from the wealthiest families marry 2 years later than those from the poorest, …
Tạm dịch: Những cô gái từ những gia đình giàu có nhất kết hôn muộn hơn 2 năm so với các cô gái từ
những gia đình nghèo nhất, …
Câu 37: Đáp án A
Kiến thức: Đọc hiểu
Giải chi tiết:
Từ “those” trong đoạn 2 thay thế cho ________.

Trang 121
A. cô gái
B. gia đình
C. năm
D. tỷ lệ
Thông tin: Girls from the wealthiest families marry 2 years later than those from the poorest, who are
seen as an economic burden, and who drop out of school and earn little money.
Tạm dịch: Những cô gái từ những gia đình giàu có nhất kết hôn muộn hơn 2 năm so với các cô gái từ
những gia đình nghèo nhất, những người được coi là gánh nặng kinh tế, và họ bỏ học và kiếm được ít
tiền.
Câu 38: Đáp án C
Kiến thức: Từ vựng
Giải chi tiết:
Từ “burden” trong đoạn 3 có nghĩa gần nhất với từ ________.
burden (n): gánh nặng
A. issue (n): đề tài, vấn đề
B. potency (n): hiệu lực, quyền thế
C. load (n): vật nặng, gánh nặng
D. cargo (n): hàng hóa
=> burden = load
Thông tin: Nepalese families that do not have enough food to eat are more likely to marry their daughters
at a young age to decrease the financial burden.
Tạm dịch: Các gia đình Nepal mà không có đủ thức ăn để ăn có nhiều khả năng cho con gái của họ kết
hôn khi còn trẻ để giảm gánh nặng tài chính.
Câu 39: Đáp án C
Kiến thức: Đọc hiểu
Giải chi tiết:
Theo đoạn cuối, một kẻ phá luật sẽ phải ngồi tù bao nhiêu năm?
A. chính xác ba năm
B. hơn ba năm
C. từ một đến ba năm
D. khoảng hai năm
Thông tin: The law states that punishment for child marriage is imprisonment for up to three years and a
fine of up to 10,000 rupees (£102).
Tạm dịch: Luật pháp quy định rằng hình phạt cho hôn nhân ở trẻ em là phạt tù tới ba năm và phạt tiền lên
tới 10.000 rupee (102 bảng).
Câu 40: Đáp án A

Trang 122
Kiến thức: Từ vựng
Giải chi tiết:
Từ “imprisonment” trong đoạn 3 có nghĩa gần nhất với từ ________.
imprisonment (n): phạt tù
A. detention (n): sự giam cầm, sự cầm tù
B. custody (n): sự chăm sóc, sự giám hộ
C. salvation (n): sự bảo vệ, sự cứu giúp
D. emblem (n): cái biểu tượng
=> imprisonment = detention
Thông tin: The law states that punishment for child marriage is imprisonment for up to three years and a
fine of up to 10,000 rupees (£102).
Tạm dịch: Luật pháp quy định rằng hình phạt cho hôn nhân ở trẻ em là phạt tù tới ba năm và phạt tiền lên
tới 10.000 rupee (102 bảng).
Câu 41: Đáp án B
Kiến thức: Đọc hiểu
Giải chi tiết:
Theo đoạn văn, phát biểu nào sau đây KHÔNG đúng?
A. Các gia đình Nepal có xu hướng cho con gái của họ kết hôn khi còn trẻ.
B. Cha mẹ cho con của họ kết hôn trước tuổi cho phép thường phải nộp phạt.
C. Tuổi kết hôn ở Nepal phụ thuộc vào số tiền mà các gia đình có.
D. Theo luật, nếu mọi người được cha mẹ của họ cho phép, họ được phép kết hôn năm 18 tuổi.
Thông tin: The law states that punishment for child marriage is imprisonment for up to three years and a
fine of up to 10,000 rupees (£102). But reports suggest that this law is rarely applied.
Tạm dịch: Luật pháp quy định rằng hình phạt cho việc hôn nhân ở trẻ em là phạt tù tới ba năm và phạt
tiền lên tới 10.000 rupee (102 bảng). Nhưng các báo cáo cho thấy luật này hiếm khi được áp dụng.
Câu 42: Đáp án D
Kiến thức: Đọc hiểu
Giải chi tiết:
Điều gì có thể được suy ra từ bài đọc _________.
A. Chính phủ không làm gì nhiều để chấm dứt hôn nhân trẻ em
B. Nepal có tỷ lệ kết hôn trẻ em cao nhất thế giới
C. Gia đình muốn cưới con gái sớm để kiếm tiền
D. Trận động đất gần đây thì tiêu cực tới việc việc xóa bỏ hôn nhân ở trẻ em
Thông tin: However, the post-earthquake and post-fuel crisis environment has meant progress is slow and
the national strategy has been delayed.

Trang 123
Tạm dịch: Tuy nhiên, hoàn cảnh sau động đất và sau khủng hoảng nhiên liệu có nghĩa là tiến độ (chấm
dứt hôn nhân ở trẻ em) chậm và chiến lược quốc gia đã bị trì hoãn.
Câu 43: Đáp án B
Kiến thức: Từ loại
Giải chi tiết:
Trong thì hiện tại đơn: trạng từ chỉ tần suất đứng trước động từ thường
Sửa: go sometimes => sometimes go
Tạm dịch: Anh trai tôi và tôi thỉnh thoảng đi bơi cùng với gia đình chú của chúng tôi.
Câu 44: Đáp án B
Kiến thức: Từ vựng
Giải chi tiết:
distinguishable (adj): có thể phân biệt
distinguished (adj): đặc biệt, khác biệt, ưu tú, xuất sắc
Sửa: distinguishable => distinguished
Tạm dịch: Cha cô đã từng là một giáo sư nổi tiếng tại trường đại học. Nhiều sinh viên tôn thờ ông.
Câu 45: Đáp án D
Kiến thức: Câu bị động
Giải chi tiết:
Câu bị động thì quá khứ đơn: S + was/ were + Ved/ V3.
Chủ ngữ “Those famous pictures…” số nhiều => tobe chia “were”
Sửa: painted => were painted
Tạm dịch: Những bức tranh nổi tiếng treo trên tường phòng trưng bày đã được vẽ ở Pháp.
Câu 46: Đáp án A
Kiến thức: Câu tường thuật
Giải chi tiết:
Câu trực tiếp: “S + should have + Ved/ V3 + O.”, S1 said to S2.: Ai đó đáng lẽ nên làm gì trong quá khứ
(nhưng họ đã không làm)
Câu gián tiếp: S1 + blamed + S2 + for + not Ving + O.: Ai đó đổ lỗi cho ai vì đã không làm gì
Tạm dịch: “Cô đáng lẽ nên thông báo cho tôi về sự thay đổi trước đó.” Ông chủ nói với thư ký của mình.
= Ông chủ đổ lỗi cho thư ký của mình vì đã không thông báo trước cho ông về sự thay đổi.
B. Ông chủ cảm ơn thư ký của mình đã thông báo trước cho ông về sự thay đổi. => sai về nghĩa
C. Ông chủ khuyến khích thư ký của mình không thông báo trước cho ông về sự thay đổi. => sai về nghĩa
D. Ông chủ khuyên thư ký của mình thông báo cho anh ta về sự thay đổi trước. => sai về nghĩa
Câu 47: Đáp án C
Kiến thức: So sánh hơn, so sánh bằng
Giải chi tiết:

Trang 124
So sánh hơn: S + to be + more adj/ adj-er + than + N.
So sánh bằng: S + to be + (not) as + adj + as + N.
Tạm dịch: Ở Venezuela, các cuộc thi sắc đẹp phổ biến hơn bóng đá.
= Ở Venezuela, bóng đá không phổ biến bằng các cuộc thi sắc đẹp.
A. Ở Venezuela, các cuộc thi sắc đẹp cũng phổ biến bằng bóng đá. => sai về nghĩa
B. Ở Venezuela, bóng đá phổ biến hơn các cuộc thi sắc đẹp. => sai về nghĩa
D. Ở Venezuela, các cuộc thi sắc đẹp không phổ biến bằng bóng đá. => sai về nghĩa
Câu 48: Đáp án A
Kiến thức: Câu đồng nghĩa
Giải chi tiết:
shouldn’t have Ved/ V3: đáng lẽ không nên làm nhưng đã làm
needn’t have Ved/ V3: đáng lẽ không cần làm nhưng đã làm
may not have Ved/ V3: chắc là đã không làm gì
can’t have Ved/ V3: chắc đã không làm gì
Tạm dịch: Joey đặt tất cả tiền của mình vào xổ số, đó là sai lầm lớn của anh ấy.
= Joey đáng lẽ không nên đặt tất cả tiền của mình vào xổ số.
B. Joey đáng lẽ không cần đặt tất cả tiền của mình vào xổ số. => sai về nghĩa
C. Joey chắc là đã không Joe đặt tất cả tiền của mình vào xổ số. => sai về nghĩa
D. Joey không thể nào đã đặt tất cả tiền của mình vào xổ số. => sai về nghĩa
Câu 49: Đáp án A
Kiến thức: Câu ước
Giải chi tiết:
- Dấu hiệu: “didn’t apply” – chia thì quá khứ đơn, “regret it now” (bây giờ hối hận)
- Cách dùng: câu ước “wish” ở quá khứ: diễn tả những mong ước về một sự việc không có thật ở quá khứ,
hay giả định một điều ngược lại so với thực tại đã xảy ra ở quá khứ.
- Công thức chung: S + wish(es)/ If only + S + had + Ved/ PII + O.
Tạm dịch: Susan không nộp đơn xin việc trong thư viện. Bây giờ cô thấy hối hận.
= Susan ước cô ấy đã nộp đơn xin việc trong thư viện.
B. Giá mà Susan nộp đơn xin việc trong thư viện. => sai vì đây là câu ước ở hiện tại
C. Susan hối tiếc khi đã xin việc trong thư viện. => sai về nghĩa
D. Susan hối hận khi xin việc ở thư viện. => sai về nghĩa
Câu 50: Đáp án C
Kiến thức: Đảo ngữ
Giải chi tiết:
No sooner had + S + Ved/ V3 + than S + Ved. = Hardly had + S + Ved/ V3 + when S + Ved.: vừa mới …
thì

Trang 125
Only after + N/ Ving/ S + V + V(trợ) + S + V(chính): chỉ sau khi
Not until + S + V + V(trợ) + S + V(chính): mãi đến khi
Tạm dịch: Hans đã nói với chúng tôi về việc đầu tư vào công ty. Ông đã làm điều đó ngay khi đến cuộc
họp.
= Hans vừa mới đến cuộc họp thì ông ấy đã nói với chúng tôi về việc đầu tư vào công ty.
A. Ngay sau khi thông báo cho chúng tôi về việc đầu tư của mình vào công ty thì Hans đến cuộc họp. =>
sai về nghĩa
B. Chỉ sau khi đầu tư vào công ty, Hans mới thông báo cho chúng tôi về việc anh ấy đến cuộc họp. => sai
về nghĩa
D. Mãi cho đến khi Hans nói với chung tôi rằng ông sẽ đầu tư vào công ty, ông mới đến cuộc họp. => sai
về nghĩa

SỞ GD & ĐT BẮC NINH ĐỀ THI ĐỊNH KÌ LẦN 2 NĂM HỌC 2019-2020


TRƯỜNG THPT CHUYÊN MÔN: TIẾNG ANH 12
Thời gian làm bài: 60 phút; không kể thời gian phát đề

Mark the letter A, B, C, or D on your answer sheet to indicate the word whose underlined part differs
from the other three in pronunciation in each of the following questions from 1 to 2.
Câu hỏi 1 (NB): A. passed B. wished C. touched D. moved
Câu hỏi 2 (NB): A. blood B. pool C. food D. tool
Mark the letter A, B, C, or D on your answer sheet to indicate the word(s) OPPOSITE in meaning to the
underlined word(s) in each of the following questions from 3 to 4.
Câu hỏi 3 (TH): He was so insubordinate that he lost his job within a week.
A. understanding B. obedient C. fresh D. disobedient
Câu hỏi 4 (VDC): I had no idea that you and he were on such intimate terms. I thought you were only
casual acquaintances.
A. were hostile to each other B. behaved well toward each other
C. hardly knew each other D. were such close friends
Mark the letter A, B, C, or D on your answer sheet to indicate the word that differs from the rest in the
position of the primary stress in each of the following questions from 5 to 6.
Câu hỏi 5 (NB): A. reflect B. contain C. purchase D. suggest
Câu hỏi 6 (NB): A. possession B. politics C. decision D. refusal
Read the passage and mark the letter A, B, C, or D on your answer sheet to indicate the correct answer to
each of the questions from 7 to 14.
The sculptural legacy that the new United States inherited from its colonial predecessors was far from a
rich one, and in fact, in 1776 sculpture as an art form was still in the hand of artisans and craftspeople.

Trang 126
Stone carvers engraved their motifs of skulls and crossbones and other religious icons of death into the
gray slabs that we still see standing today in old burial grounds. Some skilled craftspeople made
intricately carved wooden ornamentations for furniture or architectural decorations, while others carved
wooden shop signs and ships' figureheads. Although they often achieved expression and formal
excellence in their generally primitive style, they remained artisans skilled in the craft of carving and
constituted a group distinct from what we normally think of as "sculptors" in today's use of the word.
On the rare occasion when a fine piece of sculpture was desired, Americans turned to foreign sculptors, as
in the 1770's when the cities of New York and Charleston, South Carolina, commissioned the Englishman
Joseph Wilton to make marble statues of William Pitt. Wilton also made a lead equestrian image of King
George III that was created in New York in 1770 and torn down by zealous patriots six years later. A few
marble memorials with carved busts, urns, or other decorations were produced in England and brought to
the colonies to be set in the walls of churches - as in King's Chapel in Boston. But sculpture as a high art,
practiced by artists who knew both the artistic theory of their Renaissance-Baroque-Rococo predecessors
and the various technical procedures of modeling, casting, and carving rich three-dimensional forms, was
not known among Americans in 1776. Indeed, for many years thereafter, the United States had two
groups from which to choose - either the local craftspeople or the imported talent of European sculptors.
The eighteenth century was not one in which powered sculptural conceptions were developed. Add to this
the timidity with which unschooled artisans originally trained as stonemasons, carpenters, or
cabinetmakers - attacked the medium from which they sculpture made in the United States in the late
eighteenth century.
Câu hỏi 7 (TH): What is the main idea of the passage?
A. There was a great demand for the work of eighteenth-century artisans.
B. American sculptors were hampered by a lack of tools and materials.
C. Skilled sculptors did not exist in the US in the 1770’s.
D. Many foreign sculptors worked in the US after 1776.
Câu hỏi 8 (TH): It is stated in the first paragraph that the sculptural legacy that the new United States had
from colonial times was ______.
A. not great B. plentiful C. very rich D. not countable
Câu hỏi 9 (VD): The phrase “turned to” in paragraph 2 is closest in meaning to ______.
A. castigated B. censored C. consulted D. hired
Câu hỏi 10 (TH): The work of which of the following could be seen in burial ground?
A. stone carves B. carpenters C. cabinetmakers D. European sculptors
Câu hỏi 11 (NB): The word “they” in the passage refers to ______.
A. wooden ornamentations B. skilled craftspeople
C. architectural decorations D. wooden shop signs
Câu hỏi 12 (TH): The word "commissioned" in paragraph 2 refers to _________.

Trang 127
A. enabled B. allowed C. conferred D. empowered
Câu hỏi 13 (VD): What can be inferred about the importation of marble memorials from England?
A. Such sculpture was expensive to produce locally than to import.
B. Such sculpture was as prestigious as those made locally.
C. Such sculpture was not available in the US.
D. The materials found abroad were superior.
Câu hỏi 14 (VD): How did the work of American carvers in 1776 differ from that of contemporary
sculptors?
A. It was less time-consuming. B. It was more expensive.
C. It was less refined. D. It was more dangerous.
Mark the letter A, B, C, or D on your answer sheet to indicate the word(s) CLOSEST in meaning to the
underlined word(s) in each of the following questions from 15 to 16.
Câu hỏi 15 (TH): Such problems as haste and inexperience are a universal feature of youth.
A. separated B. shared C. marked D. hidden
Câu hỏi 16 (VD): Many inhabitants in the village have been inflicted with cancer due to the polluted
source of water.
A. have fought against B. have prevented C. have suffered from D. have avoided
Mark the letter A, B, C, or D on your answer sheet to indicate the sentence that is closest in meaning to
each of the following questions from 17 to 19.
Câu hỏi 17 (VD): Fansipan is the highest mountain in the Indochinese Peninsula.
A. There are some mountains in the Indochinese Peninsula higher than Fansipan.
B. The Indochinese Peninsula includes one of the highest mountains on earth.
C. The highest mountain in the Indochinese Peninsula is exclusive Fansipan.
D. No mountains in the Indochinese Peninsula are higher than Fansipan.
Câu hỏi 18 (VDC): “If I were you, I wouldn’t read the job advertisement and position description
carelessly.”, Helen said.
A. Helen advised me on reading the job advertisement and position description carelessly.
B. Helen recommended that I take no notice of the job advertisement and position description.
C. I was blamed for not reading the job advertisement and position description carefully by Helen.
D. Helen advised me against reading the job advertisement and position description carelessly.
Câu hỏi 19 (VDC): It’s possible that Joanna didn’t receive my message.
A. Joanna can’t have received my message. B. Joanna might not have received my message.
C. Joanna may have received my message. D. Joanna might have received my message.
Read the passage and mark the letter A, B, C, or D on your answer sheet to indicate the correct answer to
each of the questions from 20 to 24.

Trang 128
Tsunami is a Japanese word that means harbor wave and is used as the scientific term for seismic sea
wave generated by an undersea earthquake or possibly an undersea landslide or volcanic eruption. When
the ocean floor is tilted or offset during an earthquake, a set of waves is created similar to the concentric
waves generated by an object dropped into the water. Most tsunamis originate along the Ring of Fire, a
zone of volcanoes and seismic activity, 32,500 km long that encircles the Pacific Ocean. Since 1819,
about 40 tsunamis have struck the Hawaiian Islands.
tsunami can have wavelengths, or widths, of 100 to 200 km, and may travel hundreds of kilometers across
the deep ocean, reaching speeds of about 725 to 800 kilometres an hour. Upon entering shallow coastal
waters, the wave, which have been only about half a metre high out at sea, suddenly grows rapidly. When
the wave reaches the shore, it may be 15 m high or more. Tsunamis have tremendous energy because of
the great volume of water affected. They are capable of obliterating coastal settlements.
Tsunamis should not be confused with storm surges, which are domes of water that rise underearth
hurricanes or cyclones and cause extensive coastal flooding when the storms reach land. Storm surges are
particularly devastating if they occur at high tide. A cyclone and accompanying storm surges killed an
estimated 500,000 people in Bangladesh in 1970. The tsunami which struck south and southeast Asia in
late 2004 killed over 200 thousand people.
Câu hỏi 20 (TH): What does the word “concentric” in paragraph 1 mean?
A. Having a common centre B. Having wavy centres
C. Having wavy movements D. Having many centres
Câu hỏi 21 (NB): What is the greatest speed of tsunami travelling across the deep ocean?
A. 150,000 kilometres an hour B. 200 kilometres an hour
C. 700 kilometres an hour D. 800 kilometres an hour
Câu hỏi 22 (NB): The word “they” in paragraph 2 refers to ________.
A. Volumes of water B. Coastal waters C. Coastal settlements D. Tsunamis
Câu hỏi 23 (TH): Which of the following is NOT true?
A. Storm surges are domes of water rising underearth hurricanes or cyclones.
B. Storm surges cause extensive coastal flooding.
C. Tsunami only occurs in Asia.
D. A cyclone along with storm surges happened in Asia in 1970.
Câu hỏi 24 (TH): What is the passage mainly about?
A. Where tsunamis originate. B. How tremendous is the energy of a tsunami.
C. Damage caused by tsunamis. D. Facts about tsunamis.
Mark the letter A, B, C, or D on your answer sheet to indicate the most suitable response to complete each
of the following exchanges from 25 to 26.
Câu hỏi 25 (NB): Jane offered Jim some more chicken; however, Jim was full and seemed not to have
another serving. Choose the best response.

Trang 129
Jane: “Would you like some more chicken?”
Jim: “___________. I’m full.”
A. Never mind. B. No, I wouldn’t. C. No, I can’t. D. No, thanks.
Câu hỏi 26 (TH): An old gentleman, who is not sure where to go for the summer holiday, is asking a
travel agent for advice. Select the most suitable response to fill in the blank.
Gentleman: “Can you recommend any places for this summer holiday?”
Agent: “________.”
A. Yes, please go to other agencies
B. I don’t think you could afford a tour to Singapore, sir
C. No. You cannot recommend any places
D. A package tour to the Spratly Islands would be perfect, sir
Mark the letter A, B, C, or D on your answer sheet to indicate the underlined part that needs correction in
each of the following questions.
Câu hỏi 27 (TH): Facebook.com’s server IP address could not find in Google Chrome browser because of
the error of Internet connection.
A. Facebook.com’s B. could not find C. because of D. Internet connection
Câu hỏi 28 (NB): My girlfriend and I drink sometimes coffee in the morning in a café near my school.
A. girlfriend B. drink sometimes C. in D. near my school
Câu hỏi 29 (VD): There were inconsiderate amounts of money wasted on large building projects.
A. inconsiderate B. amounts C. wasted D. building
Read the following passage and mark the letter A, B, C, or D on your answer sheet to indicate the correct
word or phrase that best fits each of the numbered blanks from 30 to 34.
Having a relationship with someone is important to people around the world; (30) _______, dating is
different from culture to culture. Here are some examples.
Group dating is popular among young people in Europe and Australia. Groups as large as 30 people take
part in events, such as going camping or having a party. This is seen as a (31) _______ way to spend time
together, and to help to (32) _______ tension because people feel more comfortable in the company of
friends before deciding whether to go on a one-to-one date.
In Singapore, since many young people stay single, the government has tried its best to encourage dating.
Dating services are offered to single people. One of them is speed dating, in (33) _______ singles will
spend a few minutes talking to one person before moving on to meet the next one.
Online dating is a common way of (34) _______ in the United States. Internet companies are now
offering a service called ‘online dating assistant’ to help busy people to find a partner. An assistant helps
customers to build their profiles, selects potential matches, and then sends several emails to the possible
matches until the two people agree to meet face to face.
Câu hỏi 30 (TH): A. although B. and C. however D. despite

Trang 130
Câu hỏi 31 (NB): A. safe B. savings C. safely D. save
Câu hỏi 32 (VD): A. deteriorate B. ease C. refrain D. escape
Câu hỏi 33 (NB): A. when B. that C. which D. whom
Câu hỏi 34 (VD): A. matchtaking B. matchgoing C. matchcatching D.
matchmaking
Mark the letter A, B, C, or D on your answer sheet to indicate the correct answer to each of the following
questions from 35 to 48.
Câu hỏi 35 (NB): Of all the world’s major oceans, ________ Arctic Ocean is ________ shallowest.
A. the - the B. an - the C. Ø - the D. a - the
Câu hỏi 36 (NB): If you do what you tell others, they ________ in you.
A. believe B. won’t believe C. will believe D. would believe
Câu hỏi 37 (NB): We ________ in silence when he suddenly ________ me to help him.
A. walked - asked B. were walking - asked
C. walked - was asking D. were walking - was asking
Câu hỏi 38 (TH): Make sure you mix the ingredients well, _______ you might get up lumps in your cake.
A. otherwise B. supposing C. unless D. provided
Câu hỏi 39 (TH): To protect _______ hackers, security experts advise longer passwords _______
combinations of upper and lowercase letters, as well as numbers.
A. against - in B. from - to C. on - between D. against - with
Câu hỏi 40 (VD): _______ as the representative at the conference, she felt extremely proud of herself.
A. On choosing B. Having been chosen
C. Be chosen D. Having chosen
Câu hỏi 41 (TH): Applications _______ in after 30 April will not be considered.
A. sent B. send C. that is sent D. which sent
Câu hỏi 42 (VDC): I won’t buy that car because it has too much _______ on it.
A. ups and downs B. white tie C. wear and tear D. odds and ends
Câu hỏi 43 (TH): Don’t worry! Our new product will keep you bathroom clean and _______.
A. odour B. odourless C. odourlessly D. odourful
Câu hỏi 44 (TH): It’s important to project a(n) _______ image during the interview.
A. optimistic B. cheerful C. positive D. upbeat
Câu hỏi 45 (VD): Part time jobs give us freedom to _______ our own interest.
A. pursue B. chase C. seek D. catch
Câu hỏi 46 (VD): Van Gogh suffered from depression _______ by overwork and ill-health.
A. taken up B. pull through C. coming about D. brought on
Câu hỏi 47 (TH): Jane would never forget _______ first prize in such a prestigious competition.
A. to have awarded B. being awarded C. having awarded D. to be awarded

Trang 131
Câu hỏi 48 (TH): There has been a widespread _______ about whether North Korea has successfully
miniaturized a nuclear weapon and whether it has a working H-bomb.
A. gossip B. rumour C. challenge D. doubt
Mark the letter A, B, C, or D on your answer sheet to indicate the sentence that best combines each of the
pair of sentences in the following questions from 49 to 50.
Câu hỏi 49 (VDC): The basketball team knew they lost the match. They soon started to blame each other.
A. Not only did the basketball team lose the match but they blamed each other as well.
B. Hardly had the basketball team known they lost the match when they started to blame each
other.
C. No sooner had the basketball team started to blame each other than they knew they lost the
match.
D. As soon as they blamed each other, the basketball team knew they lost the match.
Câu hỏi 50 (VDC): I invited Rachel to my party, but she couldn’t come. She had arranged to do
something else.
A. Without having arranged to do something else, Rachel would have come to my party as invited.
B. If it hadn’t been for her arrangement for something else, Rachel would come to my party as
invited.
C. Rachel would have come to my party, unless she hadn’t arranged to so something else.
D. If Rachel hadn’t arranged to do something else, she would come to my party as invited.

Đáp án
1-D 2-A 3-B 4-A 5-C 6-B 7-C 8-A 9-D 10-A
11-B 12-D 13-C 14-C 15-B 16-C 17-D 18-D 19-B 20-A
21-D 22-D 23-C 24-D 25-D 26-D 27-B 28-B 29-A 30-C
31-A 32-B 33-C 34-D 35-A 36-C 37-B 38-A 39-A 40-B
41-A 42-C 43-B 44-C 45-A 46-D 47-B 48-B 49-B 50-A

LỜI GIẢI CHI TIẾT


Câu 1: Đáp án D
Kiến thức: Phát âm “-ed”
Giải chi tiết:
A. passed /pɑːst/
B. wished /wɪʃt/
C. touched /tʌtʃt/
D. moved /muːvd/
Quy tắc:

Trang 132
Cách phát âm đuôi “ed”:
- Đuôi “ed” được phát âm là /ɪd/ khi động từ có phát âm kết thúc là /t/ hay /d/
- Đuôi “ed” được phát âm là /t/ khi động từ có phát âm kết thúc là /s/,/f/,/p/,/ʃ/,/tʃ/,/k/
- Đuôi “ed” được phát âm là /d/ với các trường hợp còn lại
Phần gạch chân đáp án D phát âm là /d/, còn lại là /t/
Câu 2: Đáp án A
Kiến thức: Phát âm “oo”
Giải chi tiết:
A. blood /blʌd/
B. pool /puːl/
C. food /fuːd/
D. tool /tuːl/
Phần gạch chân đáp án A phát âm là /ʌ/, còn lại là /uː/
Câu 3: Đáp án B
Kiến thức: Từ vựng, từ trái nghĩa
Giải chi tiết:
insubordinate (adj): ngỗ nghịch, không vâng lời
A. understanding (adj): am hiểu, hiểu biết
B. obedient (adj): tuân theo, vâng lời
C. fresh (adj): tươi, tươi mới
D. disobedient (adj): không vâng lời, không tuân theo
=> insubordinate (adj): ngỗ nghịch, không vâng lời >< obedient (adj): tuân theo, nghe lời
Tạm dịch: Anh ấy quá ngỗ nghịch đến nỗi mất việc trong vòng một tuần.
Câu 4: Đáp án A
Kiến thức: Thành ngữ
Giải chi tiết:
be on intimate terms: có mối quan hệ thân thiết
A. were hostile to each other: thù địch với nhau
B. behaved well toward each other: cư xử tốt với nhau
C. hardly knew each other: hầu như không biết nhau
D. were such close friends: là bạn thân
=> were on such intimate terms: có mối quan hệ thân thiết >< were hostile to each other: thù địch với
nhau
Tạm dịch: Tôi không biết rằng bạn và anh ấy lại thân thiết đến vậy. Tôi nghĩ bạn chỉ là quen biết thông
thường.
Câu 5: Đáp án C

Trang 133
Kiến thức: Trọng âm từ có 2 âm tiết
Giải chi tiết:
A. reflect /rɪˈflekt/
B. contain /kənˈteɪn/
C. purchase /ˈpɜːtʃəs/
D. suggest /səˈdʒest/
Quy tắc:
- Những động từ có 2 âm tiết thường có trọng âm rơi vào âm tiết thứ hai. Ngoại lệ: ‘purchase
- Những danh từ, tính từ có 2 âm tiết thường có trọng âm rơi vào âm tiết thứ nhất.
Phương án C trọng âm rơi vào âm tiết thứ nhất, còn lại là âm thứ hai
Câu 6: Đáp án B
Kiến thức: Trọng âm từ có 3 âm tiết
Giải chi tiết:
A. possession /pəˈzeʃn/
B. politics /ˈpɒlətɪks/
C. decision /dɪˈsɪʒn/
D. refusal /rɪˈfjuːzl/
Quy tắc:
- Những từ có tận cùng là đuôi “-ion”, “-ics” thường có trọng âm rơi vào âm tiết đứng ngay trước nó.
Ngoại lệ: ‘politics
- Hậu tố “-al” không nhận trọng âm và không làm thay đổi trọng âm từ gốc.
Phương án B trọng âm rơi vào âm tiết thứ nhất, còn lại là âm thứ hai
Câu 7: Đáp án C
Kiến thức: Đọc hiểu
Giải chi tiết:
Ý chính của đoạn văn là gì?
A. Có nhu cầu lớn cho công việc của các nghệ nhân thế kỷ thứ mười tám.
B. Các nhà điêu khắc người Mỹ bị cản trở vì thiếu công cụ và vật liệu.
C. Các nhà điêu khắc lành nghề không tồn tại ở Mỹ vào năm 1770.
D. Nhiều nhà điêu khắc nước ngoài làm việc ở Mỹ sau năm 1776.
Thông tin: in 1776 sculpture as an art form was still in the hand of artisans and craftspeople… On the rare
occasion when a fine piece of sculpture was desired, Americans turned to foreign sculptors… The
eighteenth century was not one in which powered sculptural conceptions were developed.
Tạm dịch: vào năm 1776, điêu khắc như một hình thức nghệ thuật vẫn còn trong tay của các nghệ nhân và
thợ thủ công… Trong những dịp đặc biệt cần đến tác phẩm điêu khắc, người Mỹ tìm đến những nghệ
nhân nước ngoài… Thế kỉ XVIII không phải thời kì tạo điều kiện cho điêu khắc phát triển.

Trang 134
Câu 8: Đáp án A
Kiến thức: Đọc hiểu
Giải chi tiết:
Được nêu ra trong đoạn đầu tiên rằng di sản điêu khắc mà Hoa Kỳ mới có từ thời thuộc địa ______.
A. không vĩ đại, không lớn/nhiều
B. dồi dào
C. rất giàu
D. không đếm được
Thông tin: The sculptural legacy that the new United States inherited from its colonial predecessors was
far from a rich one
Tạm dịch: Di sản điêu khắc mà nước Mĩ kế thừa từ tổ tiên thời kì thuộc địa không thể coi là dồi dào
Câu 9: Đáp án D
Kiến thức: Đọc hiểu
Giải chi tiết:
Cụm từ “turned to” trong đoạn 2 có nghĩa gần nhất với ______.
A. castigated: trau chuốt, gọt giũa
B. censored: bị thiếu
C. consulted: tra cứu, tham khảo
D. hired: thuê, mướn
turn to: tìm đến, chuyển sang
Thông tin: On the rare occasion when a fine piece of sculpture was desired, Americans turned to foreign
sculptors, as in the 1770's when the cities of New York and Charleston, South Carolina, commissioned
the Englishman Joseph Wilton to make marble statues of William Pitt.
Tạm dịch: Trong những dịp đặc biệt cần đến tác phẩm điêu khắc, người Mỹ tìm đến những nghệ nhân
nước ngoài, như vào những năm 1770 khi các thành phố New York và Charleston thuộc Nam Carolina,
ủy quyền cho một người Anh tên Joseph Wilton khắc tượng William Pitt bằng cẩm thạch.
Câu 10: Đáp án A
Kiến thức: Đọc hiểu
Giải chi tiết:
Công việc nào sau đây có thể được thấy ở bãi chôn lấp?
A. chạm khắc đá
B. thợ mộc
C. thợ đóng đồ gỗ
D. nhà điêu khắc châu Âu
Thông tin: Stone carvers engraved their motifs of skulls and crossbones and other religious icons of death
into the gray slabs that we still see standing today in old burial grounds.

Trang 135
Tạm dịch: Thợ khắc đá đã chạm khắc các thiết kế đầu lâu, xương chéo và các biểu tượng tôn giáo khác
liên quan đến cái chết lên các phiến đá màu xám mà ngày nay chúng ta vẫn thấy trong những di tích chôn
vùi dưới lòng đất.
Câu 11: Đáp án B
Kiến thức: Đọc hiểu
Giải chi tiết:
Từ “they” trong bài đọc ám chỉ ______.
A. trang trí bằng gỗ
B. thợ thủ công lành nghề
C. trang trí kiến trúc
D. bảng hiệu cửa hàng gỗ
Thông tin: Some skilled craftspeople made intricately carved wooden ornamentations for furniture or
architectural decorations, while others carved wooden shop signs and ships' figureheads. Although they
often achieved expression and formal excellence in their generally primitive style, …
Tạm dịch: Một số thợ thủ công lành nghề đã tạo ra những tác phẩm khắc gỗ tinh xảo để trang trí nội thất
hoặc kiến trúc, trong khi đó, những người khác khắc gỗ thành biển hiệu cửa hàng và tượng gắn mũi tàu.
Mặc dù họ đạt được sự biểu đạt và tính ưu tú về hình thức với phong cách nhìn chung còn thô sơ của
mình, …
Câu 12: Đáp án D
Kiến thức: Đọc hiểu
Giải chi tiết:
Từ "commissioned" trong đoạn 2 đề cập đến _________.
A. enabled: đã cho phép
B. allowed: đã cho phép
C. conferred: đã phong, đã ban
D. empowered: đã trao quyền
Thông tin: On the rare occasion when a fine piece of sculpture was desired, Americans turned to foreign
sculptors, as in the 1770's when the cities of New York and Charleston, South Carolina, commissioned
the Englishman Joseph Wilton to make marble statues of William Pitt.
Tạm dịch: Trong những dịp đặc biệt cần đến tác phẩm điêu khắc, người Mỹ tìm đến những nghệ nhân
nước ngoài, như vào những năm 1770 khi các thành phố New York và Charleston thuộc Nam Carolina,
ủy quyền cho một người Anh tên Joseph Wilton khắc tượng William Pitt bằng cẩm thạch.
Câu 13: Đáp án C
Kiến thức: Đọc hiểu
Giải chi tiết:
Có thể được suy luận điều gì về việc nhập khẩu đài tưởng niệm bằng đá cẩm thạch từ Anh?

Trang 136
A. Tác phẩm điêu khắc như vậy để sản xuất tại địa phương thì tốn kém hơn là nhập khẩu.
B. Tác phẩm điêu khắc như vậy có uy tín như những sản phẩm làm tại địa phương.
C. Tác phẩm điêu khắc như vậy không có ở Mỹ.
D. Các vật liệu được tìm thấy ở nước ngoài thì cao cấp.
Thông tin: A few marble memorials with carved busts, urns, or other decorations were produced in
England and brought to the colonies to be set in the walls of churches - as in King's Chapel in Boston.
Tạm dịch: Một số ít các tác phẩm bằng cẩm thạch gồm tượng bán thân, bình hoặc các vật trang trí khác
được sản xuất tại Anh và mang tới các thuộc địa để đặt trong tường các nhà thờ, ví dụ như nhà thờ King’s
Chapel ở Boston (địa điểm này ở Mỹ).
Câu 14: Đáp án C
Kiến thức: Đọc hiểu
Giải chi tiết:
Tác phẩm của thợ chạm khắc người Mỹ năm 1776 khác với tác phẩm của các nhà điêu khắc đương đại
như thế nào?
A. Nó ít tốn thời gian hơn.
B. Nó đắt hơn.
C. Nó ít lỗi lạc, tinh chế hơn.
D. Nó nguy hiểm hơn.
Thông tin: But sculpture as a high art, practiced by artists who knew both the artistic theory of their
Renaissance-Baroque-Rococo predecessors and the various technical procedures of modeling, casting,
and carving rich three-dimensional forms, was not known among Americans in 1776. Indeed, for many
years thereafter, the United States had two groups from which to choose - either the local craftspeople or
the imported talent of European sculptors.
Tạm dịch: Nhưng nghệ thuật điêu khắc cấp cao, thực hiện bởi các nghệ nhân có hiểu biết về cả lý thuyết
kiến trúc Baroque-Rococo thời Phục Hưng và các kĩ năng chế tác như làm mẫu vật, đổ khuôn và khắc 3D,
lại không được biết đến bởi người Mỹ những năm 1776. Thực tế, nhiều năm sau đó, Mỹ vẫn có hai nhóm
người để chọn lựa: thợ thủ công địa phương và những nghệ nhân điêu khắc tài năng đến từ châu Âu.
Câu 15: Đáp án B
Kiến thức: Từ vựng, từ đồng nghĩa
Giải chi tiết:
universal (adj): phổ biến, của chung
A. separated (adj): bị tách ra
B. shared (adj): được chia sẻ chung
C. marked (adj): được đánh dấu
D. hidden (adj): bị giấu đi
=> universal (adj): phổ biến, của chung = shared (adj): được chia sẻ chung

Trang 137
Tạm dịch: Những vấn đề như nông nổi và thiếu kinh nghiệm là đặc điểm chung của tuổi trẻ.
Câu 16: Đáp án C
Kiến thức: Từ vựng, từ đồng nghĩa
Giải chi tiết:
be inflicted with: đã bị hứng chịu
A. have fought against: đã chiến đấu với
B. have prevented: đã ngăn cản
C. have suffered from: đã chịu đựng
D. have avoided: đã tránh
=> have been inflicted with: đã bị hứng chịu = have suffered from: đã chịu đựng
Tạm dịch: Nhiều người dân trong làng đã bị ung thư do nguồn nước bị ô nhiễm.
Câu 17: Đáp án D
Kiến thức: Câu đồng nghĩa, câu so sánh
Giải chi tiết:
Công thức so sánh hơn nhất: S + to be + the adj-est/ most + long adj + …
Công thức so sánh hơn: S + to be + adj-er/ more adj + than …
Tạm dịch: Fansipan là ngọn núi cao nhất ở bán đảo Đông Dương.
= Không có ngọn núi nào ở bán đảo Đông Dương cao hơn Fansipan.
A. Có một số ngọn núi ở Bán đảo Đông Dương cao hơn Fansipan. => sai về nghĩa
B. Bán đảo Đông Dương bao gồm một trong những ngọn núi cao nhất trên trái đất. => sai về nghĩa
C. Ngọn núi cao nhất ở bán đảo Đông Dương ngoại trừ Fansipan. => sai về nghĩa
Câu 18: Đáp án D
Kiến thức: Câu đồng nghĩa, câu tường thuật
Giải chi tiết:
Câu trực tiếp: “If I were you, I wouldn’t + V + O.”, S1 said to S2.: Nếu tôi là bạn, tôi sẽ không …
Câu tường thuật: S1 advised S2 + against + V_ing + O: Ai đó khuyên ai chống lại việc làm gì (không nên
làm gì)
Tạm dịch: “Nếu tôi là bạn, tôi sẽ không đọc quảng cáo việc làm và mô tả công việc một cách bất cẩn như
vậy.”, Helen nói.
= Helen khuyên tôi không nên đọc quảng cáo việc làm và mô tả công việc một cách bất cẩn.
A. Helen khuyên tôi đọc quảng cáo việc làm và mô tả công việc một cách bất cẩn. => sai về nghĩa
B. Helen khuyên tôi không nên chú ý đến quảng cáo việc làm và mô tả công việc. => sai về nghĩa
C. Tôi đã bị đổ lỗi vì đã không đọc quảng cáo công việc và mô tả công việc một cách cẩn thận bởi Helen.
=> sai về nghĩa
Câu 19: Đáp án B
Kiến thức: Câu đồng nghĩa, cấu trúc phỏng đoán

Trang 138
Giải chi tiết:
Cấu trúc: It’s possible that S + didn’t + V + O.: Có thể là ai đó đã không làm gì
= S + mightn’t + have V_ed/ V3: Ai đó chắc là đã không làm gì (trong quá khứ)
can’t have V_ed/ V3: chắc chắn là đã không
mightn’t + have V_ed/ V3: chắc là đã không
may/ might have V_ed/ V3: có lẽ là đã
Tạm dịch: Có thể Joanna không nhận được tin nhắn của tôi.
= Joanna có lẽ là đã không nhận được tin nhắn của tôi.
A. Joanna chắc chắn là đã không nhận được tin nhắn của tôi. => sai về nghĩa
C. Joanna có lẽ đã nhận được tin nhắn của tôi. => sai về nghĩa
D. Joanna chắc là đã nhận được tin nhắn của tôi. => sai về nghĩa
Câu 20: Đáp án A
Kiến thức: Đọc hiểu
Giải chi tiết:
Từ “concentric” trong đoạn 1 có nghĩa là gì?
A. Having a common centre: Có cùng tâm
B. Having wavy centres: Có các trung tâm lượn sóng
C. Having wavy movements: Có chuyển động lượn sóng
D. Having many centres: Có nhiều trung tâm
Thông tin: When the ocean floor is tilted or offset during an earthquake, a set of waves is created similar
to the concentric waves generated by an object dropped into the water.
Tạm dịch: Khi đáy đại dương bị nghiêng hoặc trôi dạt trong trận động đất, một tập hợp sóng được tạo ra
tương tự như sóng đồng tâm được tạo ra bởi một vật thể rơi xuống nước.
Câu 21: Đáp án D
Kiến thức: Đọc hiểu
Giải chi tiết:
Tốc độ lớn nhất của sóng thần khi đi qua đại dương sâu là gì?

A. 150.000 km một giờ


B. 200 km một giờ
C. 700 km một giờ
D. 800 km một giờ
Thông tin: A tsunami can have wavelengths, or widths, of 100 to 200 km, and may travel hundreds of
kilometers across the deep ocean, reaching speeds of about 725 to 800 kilometres an hour.
Tạm dịch: Một cơn sóng thần có thể có bước sóng hoặc chiều rộng từ 100 đến 200 km và có thể di
chuyển hàng trăm km trên đại dương sâu, đạt tốc độ khoảng 725 đến 800 km một giờ.

Trang 139
Câu 22: Đáp án D
Kiến thức: Đọc hiểu
Giải chi tiết:
Từ “they” trong đoạn 2 thay thế cho _________.

A. Khối lượng nước


B. Vùng nước ven biển
C. Khu định cư ven biển
D. Sóng thần
Thông tin: Tsunamis have tremendous energy because of the great volume of water affected. They are
capable of obliterating coastal settlements.
Tạm dịch: Sóng thần có năng lượng rất lớn vì khối lượng nước lớn bị ảnh hưởng. Chúng có khả năng xóa
sổ các khu định cư ven biển.
Câu 23: Đáp án C
Kiến thức: Đọc hiểu
Giải chi tiết:
Điều nào sau đây là không đúng?

A. Nước dâng do bão là những vòm nước dâng lên dưới những cơn bão hoặc lốc xoáy.
B. Nước dâng do bão gây ra lũ lụt trên bờ biển.
C. Sóng thần chỉ xảy ra ở châu Á.
D. Một cơn bão cùng với nước dâng do bão đã xảy ra ở châu Á vào năm 1970.
Thông tin: Most tsunamis originate along the Ring of Fire, a zone of volcanoes and seismic activity,
32,500 km long that encircles the Pacific Ocean. Since 1819, about 40 tsunamis have struck the Hawaiian
Islands.
Tạm dịch: Hầu hết các cơn sóng thần bắt nguồn dọc theo vành đai lửa, một khu vực núi lửa và hoạt động
địa chấn, dài 32.500 km bao quanh Thái Bình Dương. Kể từ năm 1819, khoảng 40 cơn sóng thần đã tấn
công quần đảo Hawaii.
Câu 24: Đáp án D
Kiến thức: Đọc hiểu
Giải chi tiết:
Ý chính của bài là gì?

A. Sóng thần bắt nguồn từ đâu.


B. Năng lượng của sóng thần khủng khiếp đến mức nào.
C. Tác hại do sóng thần.

Trang 140
D. Sự thật về sóng thần.
Thông tin: Tsunami is a Japanese word that means harbor wave and is used as the scientific term for
seismic sea wave generated by an undersea earthquake or possibly an undersea landslide or volcanic
eruption… Tsunamis should not be confused with storm surges, which are domes of water that rise
underearth hurricanes or cyclones and cause extensive coastal flooding when the storms reach land.
Tạm dịch: Sóng thần là một từ tiếng Nhật có nghĩa là sóng ngầm và được sử dụng như một thuật ngữ
khoa học cho sóng địa chấn được tạo ra bởi một trận động đất dưới đáy biển hoặc có thể là một vụ lở đất
hoặc phun trào núi lửa dưới đáy biển… Sóng thần không nên bị nhầm lẫn với nước dâng do bão, đó là
những vòm nước dâng lên dưới những cơn bão hoặc lốc xoáy và gây ra lũ lụt ven biển trên diện rộng khi
bão đổ bộ vào đất liền.
Câu 25: Đáp án D
Kiến thức: Ngôn ngữ giao tiếp
Giải chi tiết:
Jane đã mời Jim ăn thêm một ít thịt gà; tuy nhiên, Jim đã no và dường như không ăn thêm nữa. Chọn
phản hồi tốt nhất.
Jane: “Bạn có muốn ăn thêm một chút thịt gà không?”
Jim: “___________. Tôi no rồi.”
A. Đừng bận tâm đến.
B. Không, tôi sẽ không.
C. Không, tôi không thể.
D. Không, cảm ơn.
Các phản hồi A, B, C không phù hợp với ngữ cảnh.
Câu 26: Đáp án D
Kiến thức: Ngôn ngữ giao tiếp
Giải chi tiết:
Một người đàn ông lớn tuổi, người không chắc chắn sẽ đi đâu vào kỳ nghỉ hè, đang nhờ một đại lý du lịch
tư vấn. Chọn phản hồi phù hợp nhất để điền vào chỗ trống.
Quý ông: “Bạn có thể giới thiệu địa điểm nào đó cho kỳ nghỉ hè này không?”
Người tư vấn: “________.”
A. Vâng, vui lòng đến các cơ quan khác
B. Tôi không nghĩ rằng ông có thể đủ khả năng đi tour đến Singapore, thưa ông
C. Không. Bạn không thể giới thiệu bất kỳ địa điểm nào
D. Một tour du lịch trọn gói đến Quần đảo Trường Sa sẽ rất hoàn hảo, thưa ông
Các phản hồi A, B, C không phù hợp với ngữ cảnh.
Câu 27: Đáp án B
Kiến thức: Câu bị động

Trang 141
Giải chi tiết:
Câu bị động với động từ khuyết thiếu: S + modal verb + V (nguyên thể).
Sửa: could not find => could not be found
Tạm dịch: Địa chỉ IP của máy chủ Facebook.com không thể được tìm thấy trên Google Chrome do lỗi kết
nối Internet.
Câu 28: Đáp án B
Kiến thức: Trạng từ tần suất
Giải chi tiết:
Trong thì hiện tại đơn: trạng từ tấn suất đứng trước động từ thường và sau động từ “to be”
Sửa: drink sometimes => sometimes drink
Tạm dịch: Tôi và bạn gái thỉnh thoảng uống cà phê vào buổi sáng trong quán cà phê gần trường.
Câu 29: Đáp án A
Kiến thức: Từ vựng
Giải chi tiết:
inconsiderate (adj): thiếu suy nghĩa, khinh suất
inconsiderable (adj): không đáng kể
Theo nghĩa của câu thì “inconsiderate” không phù hợp.
Sửa: inconsiderate => inconsiderable
Tạm dịch: Có những khoản tiền không đáng kể đã bị lãng phí cho các dự án xây dựng lớn.
Câu 30: Đáp án C
Kiến thức: Liên từ
Giải chi tiết:
A. although S + V: mặc dù
B. and: và
C. ; however, S + V: tuy nhiên
D. despite + N/V_ing: mặc dù
Having a relationship with someone is important to people around the world; (30) however, dating is
different from culture to culture.
Tạm dịch: Có mối quan hệ với ai đó rất quan trọng đối với mọi người trên khắp thế giới; tuy nhiên, hẹn
hò thì khác nhau giữa văn hóa này và văn hóa khác.
Câu 31: Đáp án A
Kiến thức: Từ loại, từ vựng
Giải chi tiết:
A. safe (adj): an toàn, chắc chắn
B. savings (n): tiền tiết kiệm
C. safely (adv): an toàn, chắc chắn

Trang 142
D. save (v): cứu vớt, dành dụm
Trước tính từ “way” (cách, con đường) cần một tính từ
This is seen as a (31) safe way to spend time together,
Tạm dịch: Đây được coi là một cách an toàn để dành thời gian cho nhau,
Câu 32: Đáp án B
Kiến thức: Từ vựng
Giải chi tiết:
A. deteriorate (v): làm hư hỏng
B. ease (v): làm dễ chịu, làm giảm bớt
C. refrain (v): kiềm chế, kiềm lại
D. escape (v): trốn thoát, thoát khỏi
This is seen as a safe way to spend time together, and to help to (32) ease tension because people feel
more comfortable in the company of friends before deciding whether to go on a one-to-one date.
Tạm dịch: Đây được xem là một cách an toàn để dành thời gian cho nhau, và để giảm bớt căng thẳng bởi
vì mọi người cảm thấy thoải mái hơn trong mối quan hệ là bạn bè trước khi quyết định có nên hẹn hò một
lần không.
Câu 33: Đáp án C
Kiến thức: Đại từ quan hệ
Giải chi tiết:
Trong mệnh đề quan hệ:
- when: thay thế cho trạng từ, trạng ngữ chỉ thời gian
- that: thay thế cho “who”, “whom”, “which” hoặc lược bỏ khi nó đóng vai trò làm tân ngữ trong mệnh đề
quan hệ xác định
- which: thay thế cho một danh từ chỉ vật; đóng vai trò chủ ngữ/ tân ngữ
- whom: thay thế cho một danh từ chỉ người; đóng vai trò tân ngữ
Lưu ý: Đại từ quan hệ “when”, “that” không đứng sau giới từ
One of them is speed dating, in (33) which singles will spend a few minutes talking to one person before
moving on to meet the next one.
Tạm dịch: Một trong số đó là hẹn hò tốc độ, trong đó người độc thân sẽ dành vài phút để nói chuyện với
một người trước khi chuyển sang gặp người tiếp theo.
Câu 34: Đáp án D
Kiến thức: Từ vựng
Giải chi tiết:
A. matchtaking (không tồn tại từ này)
B. matchgoing (không tồn tại từ này)
C. matchcatching (không tồn tại từ này)

Trang 143
D. matchmaking (n): sự mai mối
Online dating is a common way of (34) matchmaking in the United States.
Tạm dịch: Hẹn hò trực tuyến là một cách mai mối phổ biến ở Hoa Kỳ.
Câu 35: Đáp án A
Kiến thức: Mạo từ
Giải chi tiết:
Dùng mạo từ “the” trước các tên đại dương và trong công thức so sánh hơn nhất: the + most adj/ adj-est
Arctic Ocean: Bắc Băng Dương
shallowest (adj): nông nhất (dạng so sánh nhất của “shallow”)
Tạm dịch: Trong tất cả các đại dương lớn trên thế giới, Bắc Băng Dương là đại dương nông nhất.
Câu 36: Đáp án C
Kiến thức: Câu điều kiện loại 1
Giải chi tiết:
- Dấu hiệu: mệnh đề chứa “if” chia thì hiện tại đơn (do want)
- Cách dùng: Câu điều kiện loại 1 diễn điều có thể xảy ra ở hiện tại hoặc tương lai.
- Công thức chung: If + S + V (thì hiện tại đơn) + O, S + will/won’t + V(nguyên thể) + O.
Tạm dịch: Nếu bạn làm những gì bạn nói với người khác, họ sẽ tin bạn.
Câu 37: Đáp án B
Kiến thức: Thì quá khứ tiếp diễn và thì quá khứ đơn
Giải chi tiết:
- Cách dùng: Thì quá khứ tiếp diễn dùng để diễn tả hành động đang xảy ra trong quá khứ (chia thì quá
khứ tiếp diễn) thì có một hành động khác xen vào (chia thì quá khứ đơn).
- Công thức: S1 + was/ were V-ing + when + S2 + Ved/V2.
Tạm dịch: Khi chúng tôi đang nhẹ nhàng thì anh ta đột nhiên hỏi tôi để giúp anh ta.
Câu 38: Đáp án A
Kiến thức: Từ vựng
Giải chi tiết:
A. otherwise: nếu không thì
B. supposing: giả sử, nếu như
C. unless: nếu … không
D. provided that: giả sử là, nếu
Tạm dịch: Hãy chắc chắn rằng bạn trộn đều các thành phần, nếu không thì bạn có thể làm vón cục trong
bánh.
Câu 39: Đáp án A
Kiến thức: Giới từ
Giải chi tiết:

Trang 144
protect (sb/sth) from/ against: bảo vệ ai khỏi, chống lại
in combination of: với sự kết hợp của
Tạm dịch: Để bảo vệ chống lại tin tặc, các chuyên gia bảo mật khuyên rằng nên mật khẩu dài hơn với sự
kết hợp của chữ hoa và chữ thường, cũng như số.
Câu 40: Đáp án B
Kiến thức: Rút gọn mệnh đề đồng ngữ
Giải chi tiết:
Khi hai mệnh đề có cùng chủ ngữ có thể rút gọn một mệnh đề về dạng:
- V_ing: khi mệnh đề dạng chủ động và hai hành động xảy ra liên tiếp
- Having Ved/ V3: khi mệnh đề dạng chủ động, hành động được rút gọn xảy ra trước và là nguyên nhân
dẫn đến hành động còn lại.
- V_ed/ V3: khi mệnh đề dạng bị động
- Having been Ved/ V3: khi mệnh đề dạng bị động, hành động được rút gọn xảy ra trước và là nguyên
nhân dẫn đến hành động còn lại.
Câu đầy đủ: She had been chosen as the representative at the conference. She felt extremely proud of
herself.
Câu rút gọn: Having been chosen as the representative at the conference, she felt extremely proud of
herself.
Tạm dịch: Được chọn làm đại diện tại hội nghị, cô ấy cảm thấy vô cùng tự hào về bản thân.
Câu 41: Đáp án A
Kiến thức: Rút gọn mệnh đề quan hệ
Giải chi tiết:
Mệnh đề quan hệ được rút gọn về dạng (bỏ đại từ quan hệ):
- Ved/ V3: khi mệnh đề đó ở dạng bị động
- to V: khi trước danh từ là “the only/ first/ second/ so sánh nhất …) => dạng bị động: to + be + Ved/ V3.
Câu đầy đủ: Applications which are sent in after 30 April will not be considered.
Câu rút gọn: Applications sent in after 30 April will not be considered.
Tạm dịch: Các đơn xin việc được gửi sau ngày 30 tháng 4 sẽ không được xem xét.
Câu 42: Đáp án C
Kiến thức: Thành ngữ
Giải chi tiết:
A. ups and downs: sự thăng trầm
B. white tie: lời nói dối vô hại
C. wear and tear: sự hao mòn và hư hỏng
D. odds and ends: đầu thừa đuôi vẹo
Tạm dịch: Tôi sẽ không mua chiếc xe đó vì nó bị hao mòn và hỏng nhiều quá.

Trang 145
Câu 43: Đáp án B
Kiến thức: Từ loại, từ vựng
Giải chi tiết:
A. odour (n): mùi
B. odourless (adj): không mùi
C. odourlessly (adv): không mùi
D. odourful (adj): đầy mùi
Cấu trúc song hành: A and B (A, B cùng một dạng thức: danh từ, động từ, tính từ,…)
“clean” (sạch sẽ) là tính từ nên vị trí cần điền phải là một tính từ
Tạm dịch: Đừng lo lắng! Sản phẩm mới của chúng tôi sẽ giữ cho phòng tắm của bạn sạch sẽ và không
mùi.
Câu 44: Đáp án C
Kiến thức: Từ vựng
Giải chi tiết:
A. optimistic (adj): lạc quan
B. cheerful (adj): vui vẻ, phấn khởi
C. positive (adj): tích cực
D. upbeat (adj): lạc quan, vui vẻ
Tạm dịch: Rất quan trọng để xây dựng một hình ảnh tích cực trong cuộc phỏng vấn.
Câu 45: Đáp án A
Kiến thức: Từ vựng
Giải chi tiết:
A. pursue (v): theo đuổi
B. chase (v): đuổi, xua đuổi
C. seek (v): tìm kiếm
D. catch (v): đuổi, bắt
=> pursue one’s own interest: theo đuổi sở thích của riêng mình
Tạm dịch: Công việc bán thời gian cho tôi tự do theo đuổi sở thích của riêng mình.
Câu 46: Đáp án D
Kiến thức: Cụm động từ
Giải chi tiết:
A. taken up: chiếm thời gian, không gian, bắt đầu một sở thích, thói quen
B. pull through: bình phục, hồi phục
C. coming about: xảy đến, xảy ra
D. brought on: bị xảy ra
Tạm dịch: Van Gogh bị trầm cảm do làm việc quá sức và sức khỏe kém.

Trang 146
Câu 47: Đáp án B
Kiến thức: to V/V_ing
Giải chi tiết:
forget to V: nhớ phải làm gì
forget V_ing: nhớ đã làm gì (đã làm nhưng lại quên)
forget being V_ed/ V3: nhớ đã được làm gì
Tạm dịch: Jane sẽ không bao giờ quên được trao giải nhất trong một cuộc thi danh tiếng như vậy.
Câu 48: Đáp án B
Kiến thức: Từ vựng
Giải chi tiết:
A. gossip (n): chuyện tầm phào, tin đồn nhảm
B. rumour (n): tin đồn, lời đồn
C. challenge (n): thử thách
D. doubt (n): sự nghi ngờ
Tạm dịch: Đã có một lời đồn lan rộng về việc Triều Tiên có thu nhỏ thành công vũ khí hạt nhân hay
không và liệu nó có bom H hoạt động hay không.
Câu 49: Đáp án B
Kiến thức: Đảo ngữ
Giải chi tiết:
Đảo ngữ với “Hardly … when”: Hardly had + S + Ved/ V3 + when S + Ved: vừa mới … thì

Tạm dịch: Đội bóng rổ biết họ thua trận đấu. Ngay sau đó họ bắt đầu đổ lỗi cho nhau.
= Đội bóng rổ vừa mới biết rằng họ thua trận đấu thì họ bắt đầu đổ lỗi cho nhau.
A. Đội bóng rổ không chỉ thua trận mà họ còn đổ lỗi cho nhau. => sai về nghĩa
C. Ngay sau khi đội bóng rổ bắt đầu đổ lỗi cho nhau thì họ biết mình thua trận. => sai về nghĩa
D. Ngay khi họ đổ lỗi cho nhau, đội bóng rổ biết rằng họ thua trận đấu. => sai về nghĩa
Câu 50: Đáp án A
Kiến thức: Câu điều kiện loại 3
Giải chi tiết:
- Cách dùng: Diễn tả một giả thiết trái ngược với thực tế đã xảy ra ở quá khứ
Công thức chung: If + S + had (not) + Ved/ V3, S + would/ could (not) + have + Ved/ V3.
- Trong câu điều kiện loại 3, có thể sử dụng “but for” và “without” “had it not been for” thay cho “if”
Công thức chung: But for/ Without/ Had it not been for + N/Ving, S + would/ could (not) + have + Ved/
V3.
Tạm dịch: Tôi đã mời Rachel đến bữa tiệc của tôi, nhưng cô ấy không thể đến. Cô đã sắp xếp để làm việc
khác.

Trang 147
= Nếu sắp xếp để làm việc khác, Rachel sẽ đến bữa tiệc của tôi như được mời rồi.
B. Sai công thức. “would come” => “would have come”
C. Sai công thức. Mệnh đề sau “unless” (nếu … không) không để dạng phủ định
D. Sai công thức. “would come” => “would have come”

SỞ GD & ĐT VĨNH PHÚC ĐỀ KTCL THI THPT QUỐC GIA NĂM 2020 LẦN 1
TRƯỜNG THPT YÊN LẠC 2 MÔN: TIẾNG ANH
Thời gian làm bài: 60 phút; không kể thời gian phát đề

Read the following passage and mark the letter A, B, C or D on your answer sheet to indicate the best
answer for each question
We first learn about loving and caring relationships from our families. Family is defined as a domestic
group of people with some degree of kinship - whether through blood, marriage, or adoption. Ideally,
each child is nurtured, respected, and grows up to care for others and develop strong and healthy
relationships. This does not mean that it is always easy to make and keep friends; it just means that we
share the goal of having strong relationships.
"Family" includes your siblings and parents, as well as relatives who you may not interact with every day,
such as your cousins, aunts, uncles, grandparents, and stepparents. These are probably the people you are
closest to and with whom you spend the most time. Having healthy relationships with your family
members is both important and difficult.
Families in the 21st century come in all shapes and sizes: traditional, single parent, blended (more than
one family together in the same house], and gay and lesbian parents -just to name a few. No matter the
"type" of family you have, there are going to be highs and lows - good times and bad. Many times,
however, families become blocked in their relationships by hurt, anger, mistrust, and confusion. These are
natural and normal, and few families do not have at least a few experiences with them. The worst time for
most families, is during a divorce. By making a few simple changes in the way we look at the world and
deal with other people, it is possible to create happier, more stable relationships. Families need to be units
of mutual caring and support; they can be sources of lifelong strength for all individuals.
(Adapted from http://www.pamf.org)
Câu hỏi 1 (TH): What is the main idea of the passage?
A. The importance of sharing housework in a family.
B. The importance of having children in a family.
C. The role of members in family.
D. The healthy relationships among members in family.
Câu hỏi 2 (TH): Ideally, each child is nurtured, respected, and grows up________?
A. to keep in touch with the people around them.

Trang 148
B. to keep a track of the people around them.
C. to be familiar with the people around them.
D. to be concerned about the people around them.
Câu hỏi 3 (NB): What does the word "These" in paragraph 2 refer to?
A. good and bad times B. feelings C. relationships D. high sand lows
Câu hỏi 4 (NB): Which of the following is closest in meaning to "siblings" in paragraph 2?
A. brothers & sisters B. husband & wife C. children D. friends
Câu hỏi 5 (TH): According to the passage, which example below is probably NOT TRUE about the
definition of family?
A. wife &husband relationship B. step- father &daughter relationship
C. nanny &baby relationship D. god- mother & god-son relationship
Mark the letter A, B, C, or D on your answer sheet to indicate the word whose underlined part is
pronounced differently from that of the others.
Câu hỏi 6 (NB): A. finished B. noticed C. approached D. supported
Câu hỏi 7 (NB): A. final B. identity C. applicant D. decide
Mark the letter A, B, C, or D on your answer sheet to indicate the most suitable response to complete each
of the following exchanges.
Câu hỏi 8 (NB): Marry and Janet are at the dancing club.
Marry: "You are a great dancer. I wish I could do half as well as you."
Janet: "_______. I'm an awful dancer."
A. You've got to be kidding! B. You're too kind
C. Oh, thank you very much D. That's a nice compliment!
Câu hỏi 9 (NB): Hoa and Laura are talking about their volunteer work.
Hoa: "Thank you very much for helping the disadvantaged children here."
Laura: "________."
A. What a pity! B. It's our pleasure. C. That's nice of you! D. Sorry, we don't know.
Mark the letter A, B, C, or D on your answer sheet to indicate the word(s) CLOSEST in meaning to the
underlined word(s) in each of the following questions.
Câu hỏi 10 (TH): Her style of dress was conservative. She never wears items that are too tight, short or
low-cut.
A. high-fashion B. up to date C. traditional D. trendy
Câu hỏi 11 (VD): There has been a hot debate among the scientists relating to the pros and cons of using
robotic probes to study distant objects in space.
A. problems and solutions B. causes and effects
C. solutions and limitations D. advantages and disadvantages

Trang 149
Read the following passage and mark the letter A, B, C, or D on your answer sheet to indicate the correct
word or phrase that best fits each of the numbered blanks.
In the early twentieth century, an American woman named Emily Post wrote a book on etiquette.
This book explained the proper behavior Americans should follow in many different social (12) ______,
from birthday parties to funerals. But in modern society, it is not simply to know the proper rules for
behavior in your own country. It is necessary for people (13) ______ work or travel abroad to understand
the rules of etiquette in other cultures as well.
Cultural (14) _______ can be found in such simple processes as giving or receiving a gift. In Western
cultures, a gift can be given to the receiver with relatively little ceremony. When a gift is offered, the
receiver usually takes the gift and expresses his or her thanks. (15) ______, in some Asian countries, the
act of gift-giving may appear confusing to Westerners. In Chinese culture, both the giver and receiver
understand that the receiver will typically refuse to take the gift several times before he or she finally
accepts it. In addition, to (16) _______respect for the receiver, it is common in several Asian cultures to
use both hands when offering a gift to another person.
(Source: Reading Advantage by Casey Malarcher)
Câu hỏi 12 (TH): A. locations B. situations C. positions D. conditions
Câu hỏi 13 (NB): A. whom B. where C. whose D. who
Câu hỏi 14 (TH): A. different B. differently C. differences D. differ
Câu hỏi 15 (TH): A. However B. Moreover C. Otherwise D. Therefore
Câu hỏi 16 (VD): A. take B. show C. feel D. get
Mark the letter A, B, C, or D on your answer sheet to indicate the word(s) OPPOSITE in meaning to the
underlined word(s) in each of the following questions.
Câu hỏi 17 (TH): This restaurant was highly recommended for good service, delicious food and kind-
hearted boss.
A. generous and gracious B. ambitious and greedy
C. attentive and helpful D. polite and friendly
Câu hỏi 18 (TH): Vietnamese have a strong sense of hospitality and feel embarrassed if they cannot show
their guests full respect by preparing for their interval.
A. friendliness B. unfriendliness C. generosity D. politeness
Mark the letter A, B, C, or D on your answer sheet to indicate the sentence that is CLOSEST in meaning
to each of the following questions.
Câu hỏi 19 (VDC): Despite his early retirement, he found no peace in life.
A. Although he retired early, but he found no peace in life.
B. He found no peace in life because he retired early.
C. Early as he retired, he found no peace in life.
D. His early retirement has brought him peace in life.

Trang 150
Câu hỏi 20 (VD): "Why don't you participate in the volunteer work in summer?", said Sophie.
A. Sophie asked me why not participate in the volunteer work in summer.
B. Sophie made me participate in the volunteer work in summer.
C. Sophie suggested my participating in the volunteer work in summer.
D. Sophie suggested me to participate in the volunteer work in summer.
Câu hỏi 21 (VD): He visited London three years ago.
A. He hasn’t visited London for three years. B. He was in London for three years.
C. He didn’t visit London three years ago. D. He has been in London for three years.
Read the following passage and mark the letter A, B, C, or D on your answer sheet to indicate the correct
answer to each of the questions.
It is an undeniable fact that a woman's place was once in the home. In the past, women were merely
required to fulfill the role of mother and housewife. Today, this situation has changed tremendously. If a
woman possesses the attributes and qualities of her male counterpart, she will definitely be given equal
opportunities in the career world without much bias. Therefore, women began to make the scene and,
eventually, there was a steady flow of women leaving their homes. A certain vacuum or emptiness was
thus created in the households. No more could the husbands turn to their wives after a hard day's work.
Society marveled at the ability of women, but it also suffered at the realization of the important role that
women play in their homes. Should women be allowed to work after marriage then? The answer is
undoubtedly positive although this issue is highly debatable in terms of the nature of the professions
involved. If a woman pursues her career but is at the same time able to care for her home and children,
one simply cannot find any reason why she should not be allowed to do so.
A woman's influence is greatly needed in the home, on the children. What a child needs most is his
mother's care because how the child is molded depends greatly on her. It is a real pity that women who
leave their homes solely in search of a career seldom give a thought to this. The children, being helpless
and dependent creatures, may have nobody to turn to at home, except servants or relatives. With the
mother's back only after a hard day's toil, the children surely do not get much attention.
Whether a woman should continue to work after marriage would depend on the nature of her profession.
It is a waste of resources if women, after seeking higher education, immediately abandon their careers
after matrimony. A woman's effort can also contribute to the well- being and development of society. In
the Malaysian context, a teacher is only required to work a five-day week with term holidays every now
and then. Moreover, she is in school for only half of the day. The other half of the day can be devoted to
her home. A teacher, besides educating the society, can fulfill the role of both mother and housewife.
There are many other careers like those of nurses, clerks and typists where women can fulfill the double
role.
Nevertheless, there are many professions which would not be advisable for women to indulge in after
marriage. A public relations officer spends almost three-quarters of her time in her career. She has heavy

Trang 151
tasks to shoulder which might require her to entertain others till odd hours at night. Women who venture
into the business world should think twice before plunging into it. It would be beyond their means to
fulfill two demanding roles at the same time.
As it is, a woman's most important responsibility still lies in her home. Without her around in the house,
one just cannot bear to think of the consequences. Unless and until she can fulfill the basic role of a
housewife and mother, she should not make a career her sole responsibility.
Câu hỏi 22 (TH): The word "marveled" in paragraph 1 could be best replaced by ______.
A. was excited B. was amazed C. was frustrated D. was fascinated
Câu hỏi 23 (TH): According to the passage, what job is NOT suitable for women after getting married?
A. typists B. public relations officers
C. nurses D. clerks
Câu hỏi 24 (TH): Which of the following about women is NOT true in paragraph 1?
A. Women play an important role in their homes.
B. Women will have equality of job opportunities if they are as qualified as men.
C. Women's going out to work didn't change anything in the life of their families.
D. The main role of women in bygone days was childbearing and homemaking.
Câu hỏi 25 (TH): Why should a businesswoman have the second thought before starting her career?
A. Because she has heavy tasks to shoulder till very late at night.
B. Since she has to spend three quarters of her time on business.
C. Because she has to plunge into this job.
D. As it would be hard for her to fulfill the double role at the same time.
Câu hỏi 26 (TH): The word "this" in paragraph 2 refers to _______.
A. the woman's influence at her home
B. the career that the woman wants to do
C. the child of the woman
D. the mother's care which is the most necessary for a child
Câu hỏi 27 (TH): What is the best title of the passage?
A. The necessary characteristics for women to work after marriage.
B. Should women be allowed to work after marriage?
C. What jobs are suitable for women after marriage?
D. The role of women in society.
Câu hỏi 28 (TH): The word "matrimony" in paragraph 3 is closest in meaning to _______.
A. marriage B. divorce C. celibate D. remarriage
Câu hỏi 29 (VD): Which can be inferred from the last paragraph?
A. Women should fulfill their primary roles at home before making their own careers.
B. Women should not have their own careers.

Trang 152
C. Women should stay at home to fulfill their responsibilities.
D. Women should think of the consequences in their houses.
Mark the letter A, B, C or D on your answer sheet to indicate the correct sentence that best combines each
pair of sentences in the following questions.
Câu hỏi 30 (VD): George graduated with a good degree. However, he joined the ranks of the
unemployed.
A. That George graduated with a good degree helped him join the ranks of the unemployed.
B. George joined the ranks of the unemployed because he graduated with a good degree.
C. If George graduated with a good degree, he would join the ranks of the unemployed.
D. Although George graduated with a good degree, he joined the ranks of the unemployed.
Câu hỏi 31 (VD): Electronic devices are bad for your eyes. Their radiation is very harmful.
A. Electronic devices, which are bad for your eyes, their radiation is very harmful.
B. Electronic devices, whose radiation is very harmful, are bad for your eyes.
C. Electronic devices that their radiation is very harmful are bad for your eyes.
D. Electronic devices which their radiation is very harmful are bad for your eyes.
Mark the letter A, B, C, or D on your answer sheet to indicate the correct answer to each of the following
questions.
Câu hỏi 32 (TH): Both husband and wife should be responsible ________doing the household chores.
A. for B. with C. of D. to
Câu hỏi 33 (NB): They ________ enthusiastically when their teacher_______ in.
A. will discuss/ will come B. will have discussed/ comes
C. were discussing/ came D. discuss/ comes
Câu hỏi 34 (TH): Gestures such as waving and handshaking are ________forms of communication.
A. regular B. non-verbal C. direct D. verbal
Câu hỏi 35 (TH): His brother refuses to even listen to anyone else's point of view. He is very _______.
A. narrow-minded B. absent-minded C. open-minded D. kind-hearted
Câu hỏi 36 (VD): I finally ______the cold that I had had all week.
A. get over B. pass out C. pull through D. come down with
Câu hỏi 37 (TH): Some people are concerned with physical _______when choosing a wife or husband.
A. attract B. attractiveness C. attractive D. attractively
Câu hỏi 38 (NB): Everyone likes him, _____?
A. doesn't she B. don't they C. doesn't he D. don't you
Câu hỏi 39 (NB): If I ______ it was a formal party, I wouldn't have worn my old jeans and a jumper.
A. had been knowing B. could know C. knew D. had known
Câu hỏi 40 (TH): _______ his physical disability, he managed to finish the course with good results.
A. Despite B. Since C. Because of D. Although

Trang 153
Câu hỏi 41 (VDC): Mr. Park Hang Seo, a Korean coach, is considered a big _____in Vietnam football.
A. sandwich B. bread C. egg D. cheese
Câu hỏi 42 (TH): I deeply regret _______ to her so severely last night. She was badly hurt.
A. being spoken B. having spoken C. to be speaking D. to speak
Câu hỏi 43 (TH): Why don't you _______ these photos into your computer, because then you would have
your own digital copies.
A. change B. put C. give D. scan
Câu hỏi 44 (TH): _______ in large quantities in the Middle East, oil became known as black gold because
of the large profit it brought.
A. That when discovered B. Discovering
C. Discovered D. Which was discovered
Câu hỏi 45 (TH): Mark Zuckerberg's enormous success has taken a lot of hard work and _______.
A. dedication B. loyalty C. reputation D. indifference
Mark the letter A, B, C or D on your answer sheet to indicate the word which is stressed differently from
the rest.
Câu hỏi 46 (NB): A. relevant B. attitude C. assistant D. argument
Câu hỏi 47 (NB): A. promote B. profile C. prefer D. regret
Mark the letter A, B, C or D on your answer sheet to indicate the underlined word or phrase in each
sentence that needs correcting.
Câu hỏi 48 (VD): Jane's friends insist that she will stay at their house when she visits Toronto next
weekend.
A. visits B. that C. next weekend D. will stay at
Câu hỏi 49 (TH): The woman is famous not only for her beauty, intelligent but also for her hardworking.
A. not only B. for C. intelligent D. hardworking
Câu hỏi 50 (TH): Most people consider it women's responsible to take care of children and do housework.
A. responsible B. Most C. do D. of

Trang 154
Đáp án
1-D 2-D 3-B 4-A 5-C 6-D 7-C 8-A 9-B 10-C
11-D 12-B 13-D 14-C 15-A 16-B 17-B 18-B 19-C 20-C
21-A 22-B 23-B 24-C 25-D 26-D 27-B 28-A 29-A 30-D
31-B 32-A 33-C 34-B 35-A 36-A 37-B 38-B 39-D 40-A
41-D 42-B 43-D 44-C 45-A 46-C 47-B 48-D 49-C 50-A

LỜI GIẢI CHI TIẾT


Câu 1: Đáp án D
Kiến thức: Đọc hiểu
Giải chi tiết:
Ý chính của đoạn văn là gì?
A. Tầm quan trọng của việc chia sẻ công việc nhà trong gia đình
B. Tầm quan trọng của việc có trẻ nhỏ trong một gia đình
C. Vai trò của các thành viên trong gia đình
D. Mối quan hệ lành mạnh giữa các thành viên trong gia đình
Thông tin: We first learn about loving and caring relationships from our families...Ideally, each child is
nurtured, respected, and grows up to care for others and develop strong and healthy relationships. This
does not mean that it is always easy to make and keep friends; it just means that we share the goal of
having strong relationships.
Tạm dịch: Chúng ta đầu tiên học được từ gia đình của mình về tình yêu và sự quan tâm... Một cách lí
tưởng, mỗi đứa trẻ được nuôi dưỡng, được tôn trọng và lớn lên để quan tâm đến người khác và phát triển
những mối quan hệ bền chặt và lành mạnh. Điều này không có nghĩa rằng luôn dễ dàng để làm quen và
duy trì được những người bạn; nó chỉ có nghĩa là chúng ta có chung mục đích là có những mối quan hệ
bền vững.
Câu 2: Đáp án D
Kiến thức: Đọc hiểu
Giải chi tiết:
Một cách lí tưởng, mỗi đứa trẻ được nuôi dưỡng, được tôn trọng và lớn lên _______
A. để giữ liên lạc với những người xung quanh họ
B. để theo dấu những người xung quanh họ
C. để trở nên thân thuộc với những người xung quanh họ
D. để quan tâm đến những người xung quanh họ
Thông tin: Ideally, each child is nurtured, respected, and grows up to care for others...

Trang 155
Tạm dịch: Một cách lí tưởng, mỗi đứa trẻ được nuôi dưỡng, được tôn trọng và lớn lên để quan tâm người
khác…
Câu 3: Đáp án B
Kiến thức: Đọc hiểu
Giải chi tiết:
Từ “These” trong đoạn 2 đề cập đến cái gì?
A. những lúc tốt và xấu
B. những cảm giác
C. các mối quan hệ
D. những thăng trầm
“These” đề cập đến “hurt, anger, mistrust, and confusion” ở câu trước => là tên những cảm giác
Thông tin: Many times, however, families become blocked in their relationships by hurt, anger, mistrust,
and confusion. These are natural and normal, ...
Tạm dịch: Tuy nhiên thì nhiều lần gia đình trở nên bế tắc trong mối quan hệ của họ bởi nỗi đau, sự tức
giận, sự nghi ngờ, và nhầm lẫn. Những điều này thì tự nhiên và bình thường,...
Câu 4: Đáp án A
Kiến thức: Đọc hiểu
Giải chi tiết:
Từ nào dưới đây gần nghĩa nhất với “siblings” trong đoạn 2?
A. anh em trai và chị em gái
B. chồng và vợ
C. những đứa trẻ
D. những người bạn
siblings = brothers and sisters: anh em trai và chị em gái
Thông tin: "Family" includes your siblings and parents, as well as relatives…
Tạm dịch: “Gia đình” bao gồm anh chị em và bố mẹ bạn, cũng như là những người họ hàng…
Câu 5: Đáp án C
Kiến thức: Đọc hiểu
Giải chi tiết:
Theo đoạn văn, ví dụ nào dưới đây không đúng về định nghĩa của gia đình?
A. mối quan hệ vợ chồng
B. quan hệ bố dượng và con gái
C. quan hệ vú nuôi và trẻ nhỏ
D. quan hệ mẹ và con trai đỡ đầu
Thông tin:

Trang 156
Family is defined as a domestic group of people with some degree of kinship - whether through blood,
marriage, or adoption.
"Family" includes your siblings and parents, as well as relatives who you may not interact with every day,
such as your cousins, aunts, uncles, grandparents, and stepparents.
Tạm dịch:
Gia đình được định nghĩa như là một nhóm người cùng một nhà với một mức độ gắn bó – hoặc quan hệ
ruột thịt, hôn nhân, hoặc việc nhận nuôi.
“Gia đình” bao gồm anh chị em và bố mẹ bạn, cũng như là những người họ hàng bạn có thể không tiếp
xúc với mỗi ngày, như là anh chị em họ, cô dì, chú bác, ông bà và bố mẹ kế.
Câu 6: Đáp án D
Kiến thức: Phát âm “ed”
Giải chi tiết:
A. finished /ˈfɪnɪʃt/
B. noticed /ˈnəʊtɪst/
C. approached /əˈprəʊtʃt/
D. supported /səˈpɔːtɪd/
Quy tắc:
Cách phát âm đuôi “-ed”:
- Đuôi “-ed” được phát âm là /ɪd/ khi động từ có phát âm kết thúc là /t/ hay /d/
- Đuôi “-ed” được phát âm là /t/ khi động từ có phát âm kết thúc là /s/,/f/,/p/,/ʃ/,/tʃ/,/k/
- Đuôi “-ed” được phát âm là /d/ với các trường hợp còn lại
Phần gạch chân phương án D được phát âm là /ɪd/, còn lại là /t/
Câu 7: Đáp án C
Kiến thức: Phát âm “i”
Giải chi tiết:
A. final /ˈfaɪnl/
B. identity /aɪˈdentəti/
C. applicant /ˈæplɪkənt/
D. decide /dɪˈsaɪd/
Phần gạch chân phương án C được phát âm là /ɪ/, còn lại là /aɪ/
Câu 8: Đáp án A
Kiến thức: Ngôn ngữ giao tiếp
Giải chi tiết:
Marry và Janet đang ở câu lạc bộ múa
Marry: “Bạn là một vũ công giỏi. Mình ước mình có thể làm được bằng một nửa của bạn.”
Janet: “_________.Mình là một vũ công kém lắm."

Trang 157
A. Chắc bạn đùa!
B. Bạn quá tốt bụng.
C. Ồ, cảm ơn bạn rất nhiều.
D. Đó là một lời khen tốt!
Các phản hồi B, C, D không phù hợp ngữ cảnh.
Câu 9: Đáp án B
Kiến thức: Ngôn ngữ giao tiếp
Giải chi tiết:
Hoa và Laura đang nói về công việc tình nguyện của họ.
Hoa: “Cảm ơn các bạn rất nhiều vì đã giúp đỡ những trẻ em khiếm khuyết ở đây”
Laura: “_________________.”
A. Thật đáng tiếc!
B. Đó là vinh hạnh của chúng tôi.
C. Bạn thật tốt bụng!
D. Xin lỗi, chúng tôi không biết.
Các phản hồi A, C, D không phù hợp ngữ cảnh.
Câu 10: Đáp án C
Kiến thức: Từ vựng, từ đồng nghĩa
Giải chi tiết:
conservative (adj): truyền thống
A. high-fashion: thời trang sành điệu
B. up to date: kịp thời
C. traditional (adj): truyền thống
D. trendy (adj): theo xu hướng
=> conservative = traditional : truyền thống
Tạm dịch: Phong cách ăn mặc của cô ấy đã mang tính truyền thống. Cô ấy không bao giờ mặc những đồ
mà quá chật, ngắn, hoặc cắt ngắn.
Câu 11: Đáp án D
Kiến thức: Thành ngữ
Giải chi tiết:
pros and cons: những mặt lợi và hại
A. problems and solutions: những vấn đề và giải pháp
B. causes and effects: nguyên nhân và ảnh hưởng
C. solutions and limitations: các giải pháp và hạn chế
D. advantages and disadvantages: những thuận lợi và khó khăn
=> pros and cons = advantages and disadvantages: những mặt lợi và hại

Trang 158
Tạm dịch: Đã có một cuộc tranh cãi nảy lửa giữa các nhà khoa học liên quan đến những lợi ích và tác hại
của việc sử dụng các thăm dò robot để nghiên cứu các vật thể ở xa trong không gian.
Câu 12: Đáp án B
Kiến thức: Từ vựng
Giải chi tiết:
A. locations (n): vị trí
B. situations (n): các tình huống
C. positions (n): vị trí
D. conditions (n): những điều kiện
This book explained the proper behavior Americans should follow in many different social (12)
situations, from birthday parties to funerals.
Tạm dịch: Quyển sách này đã giải thích cách ứng xử phù hợp mà người Mĩ nên làm theo ở trong nhiều
tình huống xã hội khác nhau từ bữa tiệc sinh nhật cho đến các đám tang.
Câu 13: Đáp án D
Kiến thức: Mệnh đề quan hệ
Giải chi tiết:
Trong mệnh đề quan hệ:
A. whom: người mà => thay thế cho từ chỉ người, giữ vai trò tân ngữ (có thể đứng sau giới từ)
B. where: nơi mà => thay thế cho từ chỉ nơi chốn
C. whose + N: của ai, cái gì đó => thay thế cho từ chỉ người, vật (đứng trước danh từ)
D. who: người mà => thay thế cho từ chỉ người, giữ vai trò chủ ngữ
Sau chỗ trống là động từ “work” => cần điền một chủ ngữ
Trước chỗ trống là “people” (người) => dùng “who”
It is necessary for people (13) who work or travel abroad to understand the rules of etiquette in other
cultures as well.
Tạm dịch: Cũng cần thiết cho những người mà làm việc hoặc du lịch nước ngoài hiểu các qui tắc lễ nghi ở
các văn hóa khác.
Câu 14: Đáp án C
Kiến thức: Từ loại
Giải chi tiết:
Sau tính từ “cultural” cần một danh từ
A. different (adj): khác nhau
B. differently (adv): một cách khác nhau
C. differences (n): những sự khác nhau
D. differ (v): khác
Cultural (14) differences can be found in such simple processes as giving or receiving a gift.

Trang 159
Tạm dịch: Sự khác biệt văn hóa có thể tìm thấy ở các quá trình đơn giản như tặng hoặc nhận một món
quà.
Câu 15: Đáp án A
Kiến thức: Liên từ
Giải chi tiết:
A. However: Tuy nhiên
B. Moreover: Hơn nữa
C. Otherwise: Nếu không thì
D. Therefore: Do đó
When a gift is offered, the receiver usually takes the gift and expresses his or her thanks. (15) However,
in some Asian countries, the act of gift-giving may appear confusing to Westerners.
Tạm dịch:
Khi một món quà được tặng, người nhận thường lấy quà và bày tỏ sự cảm ơn của anh hoặc cô ấy. Tuy
nhiên, ở một vài nước Châu Á, hành động đưa quà dường như có thể gây bối rối cho người phương Tây.
Câu 16: Đáp án B
Kiến thức: Sự kết hợp từ
Giải chi tiết:
Ta có cụm từ “have/show respect for sb”: bày tỏ sự kính trọng đối với ai đó
In addition, to (16) show respect for the receiver, it is common in several Asian cultures to use both hands
when offering a gift to another person.
Tạm dịch: Thêm vào đó, để bày tỏ sự tôn trọng với người nhận, thông thường ở một vài nền văn hóa
Châu Á họ sử dụng cả hai tay khi đưa quà cho một người khác.
Câu 17: Đáp án B
Kiến thức: Từ vựng, từ trái nghĩa
Giải chi tiết:
kind-hearted (adj): nhân hậu, tốt bụng
A. generous and gracious: rộng lượng và thoải mái
B. ambitious and greedy: tham vọng và tham lam
C. attentive and helpful: cẩn thận và tốt bụng
D. polite and friendly: lịch sự và thân thiện
=> kind-hearted >< ambitious and greedy
Tạm dịch: Nhà hàng đã được đánh giá cao vì phục vụ tốt, thức ăn ngon và ông chủ tốt bụng.
Câu 18: Đáp án B
Kiến thức: Từ vựng, từ trái nghĩa
Giải chi tiết:
hospitality (n): sự hiếu khách, thân thiện

Trang 160
A. friendliness (n): sự thân thiện
B. unfriendliness (n): sự không thân thiện
C. generosity (n): sự rộng lượng
D. politeness (n): sự lịch sự
=> hospitality >< unfriendliness
Tạm dịch: Người Việt Nam có tinh thần hiếu khách cao và cảm thấy lúng túng nếu họ không thể bày tỏ
hết sự tôn trọng cho những vị khách của mình bằng việc chuẩn bị cho những giờ nghỉ của họ.
Câu 19: Đáp án C
Kiến thức: Câu đồng nghĩa
Giải chi tiết:
Despite + N, S + V: Mặc dù việc gì, ai đó vẫn làm gì
= Adv + as + S + V, S + V: Mặc dù ai đó làm cái này, người đó vẫn làm cái kia
A. Mặc dù ông ấy đã nghỉ hưu sớm, nhưng ông ta đã không cảm thấy bình yên trong cuộc sống. => sai vì
có 2 liên từ nối trong một câu.
B. Ông ấy không cảm thấy sự bình yên trong cuộc sống bởi vì ông ấy nghỉ hưu sớm. => sai nghĩa
C. Mặc dù ông ấy nghỉ hưu sớm, ông ta đã không cảm thấy sự bình yên trong cuộc sống.
D. Việc nghỉ hưu sớm của ông ấy đã mang lại cho ông sự bình yên trong cuộc sống. => sai nghĩa
Tạm dịch: Mặc dù ông ấy nghỉ hưu sớm, ông ta đã không cảm thấy sự bình yên trong cuộc sống.
Câu 20: Đáp án C
Kiến thức: Lời nói gián tiếp
Giải chi tiết:
Câu trực tiếp: “Why don’t you + V_nguyên thể?”, said S: “Tại sao bạn không làm gì đó?”, ai đó đã nói.
Câu gián tiếp: S suggested sth/doing sth: Ai đó đã gợi ý cái gì/làm gì
A. Sophie đã hỏi tôi tại sao không tham gia công việc tình nguyện vào mùa hè. => sai nghĩa
B. Sophie đã khiến tôi tham gia công việc tình nguyện vào mùa hè. => sai nghĩa
C. Sophie đã gợi ý việc tham gia công việc tình nguyện vào mùa hè của tôi.
D. Sophie đã gợi ý tôi tham gia công việc tình nguyện vào mùa hè. => sai cấu trúc với “suggest”
Tạm dịch: “Tại sao bạn không tham gia công việc tình nguyện vào mùa hè?” Sophie đã nói.
= Sophie đã gợi ý việc tham gia công việc tình nguyện vào mùa hè của tôi.
Câu 21: Đáp án A
Kiến thức: Câu đồng nghĩa
Giải chi tiết:
S + V_ed + khoảng thời gian + ago: Ai đó đã làm gì cách đây bao lâu trước.
= S + haven’t/ hasn’t + P2 + for + khoảng thời gian: Ai đó đã không làm gì trong bao lâu.
A. Anh ấy đã không đến London trong 3 năm rồi.
B. Anh ấy đã ở London trong 3 năm. => sai nghĩa

Trang 161
C. Anh ấy đã không đến London 3 năm trước. => sai nghĩa
D. Anh ấy đã ở London trong 3 năm. => sai nghĩa
Tạm dịch: Anh ấy đã đến thăm London 3 năm trước.
= Anh ấy đã không đến London trong 3 năm rồi.
Câu 22: Đáp án B
Kiến thức: Đọc hiểu
Giải chi tiết:
Từ “marveled” trong đoạn 1 có thể thay được thay thế tốt nhất bởi _______.
marveled (adj): đầy bất ngờ, kinh ngạc
A. was excited: phấn khích
B. was amazed: ngạc nhiên
C. was frustrated: sợ hãi
D. was fascinated: say mê
=> marveled at = be amazed at : ngạc nhiên
Thông tin: Society marveled at the ability of women, but it also suffered at the realization of the important
role that women play in their homes.
Tạm dịch: Xã hội sẽ ngạc nhiên bởi khả năng của phụ nữ, nhưng nó cũng phải chịu đựng trong quá trình
nhận thức hóa về vai trò quan trọng mà phụ nữ trong nhà của họ.
Câu 23: Đáp án B
Kiến thức: Đọc hiểu
Giải chi tiết:
Theo đoạn văn, công việc nào không phù hợp với phụ nữ sau khi kết hôn?
A. người đánh máy
B. nhân viên phòng quan hệ công chúng
C. y tá
D. nhân viên bán hàng
Thông tin: Nevertheless, there are many professions which would not be advisable for women to indulge
in after marriage. A public relations officer spends almost three-quarters of her time in her career.
Tạm dịch: Tuy nhiên, có rất nhiều ngành nghề mà sẽ không được khuyến khích cho phụ nữ để làm sau
khi kết hôn. Một nhân viên quan hệ công chúng dành gần ba phần tư thời gian của mình cho sự nghiệp.
Câu 24: Đáp án C
Kiến thức: Đọc hiểu
Giải chi tiết:
Câu nào sau đây không đúng về phụ nữ trong đoạn 1?
A. Phụ nữ đóng vai trò quan trọng trong ngôi nhà của họ.
B. Phụ nữ sẽ có sự công bằng về cơ hội việc làm nếu họ có chất lượng như nam giới.

Trang 162
C. Việc phụ nữ ra ngoài đi làm đã không thay đổi điều gì trong cuộc sống của gia đình họ.
D. Vai trò chính của người phụ nữ trong quá khứ là chăm con và làm việc nhà.
Thông tin: ...there was a steady flow of women leaving their homes. A certain vacuum or emptiness was
thus created in the households. No more could the husbands turn to their wives after a hard day's work.
Tạm dịch: ...đã có một số lượng ổn định phụ nữ rời khỏi nhà của họ. Một khoảng không nhất định hoặc sự
trống trải do đó được tạo ra trong các hộ gia đình. Không còn nhiều người chồng có thể dựa vào vợ của
họ sau một ngày làm việc vất vả.
Câu 25: Đáp án D
Kiến thức: Đọc hiểu
Giải chi tiết:
Tại sao một nữ doanh nhân nên suy nghĩ kĩ trước khi bắt đầu một sự nghiệp?
A. Bởi vì cô ấy có nhiều nhiệm vụ nặng nề phải gánh vác cho tới tận đêm muộn.
B. Bởi vì cô ấy phải dành 3/4 thời gian của mình vào việc kinh doanh.
C. Bởi vì cô ấy phải đắm chìm vào công việc
D. Bởi vì sẽ khó cho cô ấy để làm tốt cả hai vai trò cùng một lúc.
Thông tin: Women who venture into the business world should think twice before plunging into it. It
would be beyond their means to fulfill two demanding roles at the same time.
Tạm dịch: Phụ nữ mà mạo hiểm vào thế giới kinh doanh nên cân nhắc kĩ trước khi đắm chìm vào nó. Sẽ
vượt quá ý định của họ là hoàn thành hai vai trò khó khăn cùng một lúc.
Câu 26: Đáp án D
Kiến thức: Đọc hiểu
Giải chi tiết:
Từ “this” trong đoạn 2 đề cập đến ______.
A. ảnh hưởng của người phụ nữ trong căn nhà của họ
B. sự nghiệp mà phụ nữ muốn làm
C. đứa con của người phụ nữ
D. sự chăm sóc của người mẹ cái mà cần thiết nhất đối với một đứa trẻ
Từ “this” đề cập đến “mother’s care” ở câu trước.
Thông tin: What a child needs most is his mother's care because how the child is molded depends greatly
on her. It is a real pity that women who leave their homes solely in search of a career seldom give a
thought to this.
Tạm dịch: Những gì một đứa trẻ cần nhất là sự chăm sóc của mẹ bởi vì cách đứa trẻ hình thành phụ thuộc
rất nhiều vào họ. Một điều thực đáng tiếc rằng người phụ nữ mà rời khỏi nhà một mình để tìm kiếm sự
nghiệp hiếm khi suy nghĩ về điều này.
Câu 27: Đáp án B
Kiến thức: Đọc hiểu

Trang 163
Giải chi tiết:
Đâu là tiêu đề phù hợp nhất cho đoạn văn?
A. Những đặc tính cần thiết với phụ nữ để làm việc sau khi kết hôn.
B. Phụ nữ có nên được phép đi làm sau hôn nhân?
C. Những công việc nào phù hợp với phụ nữ sau hôn nhân?
D. Vai trò của người phụ nữ trong xã hội.
Câu 28: Đáp án A
Kiến thức: Đọc hiểu
Giải chi tiết:
Từ “matrimony” trong đoạn 3 gần nghĩa nhất với _______
matrimony (n): hôn nhân
A. marriage (n): hôn nhân
B. divorce (n): li hôn
C. celibate (adj): độc thân
D. remarriage (n): tái hôn
=> matrimony = marriage: hôn nhân
Thông tin: It is a waste of resources if women, after seeking higher education, immediately abandon their
careers after matrimony.
Tạm dịch: Đó là một sự lãng phí tài nguyên nếu phụ nữ, sau khi tìm bậc học cao hơn, ngay lập tức từ bỏ
sự nghiệp sau hôn nhân.
Câu 29: Đáp án A
Kiến thức: Đọc hiểu
Giải chi tiết:
Có thể suy ra gì từ đoạn văn cuối?
A. Phụ nữ nên làm tốt vai trò chính của họ trong gia đình trước khi bắt đầu sự nghiệp của chính mình.
B. Phụ nữ không nên có sự nghiệp của riêng mình.
C. Phụ nữ nên ở nhà để làm tốt nhiệm vụ của họ.
D. Phụ nữ nên suy nghĩ về những hậu quả ở nhà họ.
Thông tin: Unless and until she can fulfill the basic role of a housewife and mother, she should not make
a career her sole responsibility.
Tạm dịch: Trừ khi và cho đến khi cô ấy có thể hoàn thành vai trò cơ bản của người nội trợ và người mẹ,
cô ấy không nên coi sự nghiệp là trách nhiệm duy nhất của mình.
Câu 30: Đáp án D
Kiến thức: Mệnh đề nhượng bộ
Giải chi tiết:
S +V1. However, S + V2. Ai đó làm cái này. Tuy nhiên, họ làm cái kia.

Trang 164
= Although S + V1, S + V2: Mặc dù ai làm cái này, họ làm cái kia.
A. Việc George đã tốt nghiệp với một tấm bằng giỏi giúp anh ấy gia nhập hội những người thất nghiệp.
=> sai nghĩa
B. George đã gia nhập hội những người thất nghiệp bởi vì anh ấy đã tốt nghiệp với bằng giỏi. => sai
nghĩa
C. Nếu George đã tốt nghiệp với bằng giỏi, anh ấy sẽ gia nhập hội những người thất nghiệp. => sai nghĩa
D. Mặc dù George đã tốt nghiệp với bằng giỏi, anh ấy đã gia nhập hội những người thất nghiệp.
Tạm dịch: George đã tốt nghiệp với bằng giỏi. Tuy nhiên, anh ấy đã gia nhập hội những người thất
nghiệp.
= Mặc dù George đã tốt nghiệp với bằng giỏi, anh ấy đã gia nhập hội những người thất nghiệp.
Câu 31: Đáp án B
Kiến thức: Mệnh đề quan hệ
Giải chi tiết:
Trong mệnh đề quan hệ:
“whose + N”: cái mà … => thay thế cho tính từ sở hữu chỉ người, vật
that + V / S + V : cái mà/người mà => thay thế cho từ chỉ người/vật, đóng vai trò làm chủ ngữ/tân ngữ
which + V / S + V : cái mà => thay thế cho từ chỉ vật, đóng vai trò làm chủ ngữ/tân ngữ
“radiation” (sóng bức xạ) là danh từ, thể hiện mối quan hệ phụ thuộc – sở hữu với “Electronic devices”
(Các thiết bị điện tử) => không thể dùng “which, that”
Tạm dịch:
Các thiết bị điện tử gây hại cho mắt của bạn. Sóng của chúng thì rất có hại.
= Các thiết bị điện tử, sóng bức xạ của chúng rất có hại, gây hại cho mắt của bạn.
Câu 32: Đáp án A
Phương pháp giải:
Kiến thức: Giới từ
Giải chi tiết:
Cấu trúc “be responsible for sth/ sb”: có trách nhiệm cho cái gì/ ai
Tạm dịch: Cả chồng và vợ nên có trách nhiệm làm các công việc nhà.
Câu 33: Đáp án C
Kiến thức: Thì quá khứ tiếp diễn
Giải chi tiết:
- Cách dùng: Thì quá khứ tiếp diễn diễn tả hành động đang diễn ra trong quá khứ (chia quá khứ tiếp diễn)
thì có hành động khác cắt ngang (chia quá khứ đơn).
- Công thức: S + was/were + V_ing when + S + V_ed/V cột 2
Chủ ngữ “They” => tobe chia “were”; come => came
Tạm dịch: Họ đang thảo luận nhiệt tình khi giáo viên bước vào lớp.

Trang 165
Câu 34: Đáp án B
Kiến thức: Từ vựng
Giải chi tiết:
Các cử chỉ như vẫy tay và bắt tay là những dạng _______ của giao tiếp.
A. regular (adj): thường xuyên
B. non-verbal (adj): không lời nói
C. direct (adj): trực tiếp
D. verbal (adj): lời nói
Tạm dịch: Các cử chỉ như vẫy tay và bắt tay là dạng không lời nói của giao tiếp.
Câu 35: Đáp án A
Kiến thức: Từ vựng
Giải chi tiết:
A. narrow-minded (adj): hẹp hòi
B. absent-minded (adj): mất trí
C. open-minded (adj): phóng khoáng
D. kind-hearted (adj): nhân hậu
Tạm dịch: Anh trai của anh ấy từ chối lắng nghe quan điểm của bất cứ ai. Anh ta rất hẹp hòi.
Câu 36: Đáp án A
Kiến thức: Cụm động từ
Giải chi tiết:
A. get over: vượt qua, hồi phục (sau trận ốm, sau cú sốc,…)
B. pass out: bất tỉnh
C. pull through: phục hồi (từ trận ốm nặng, ca phẫu thuật,…)
D. come down with: mắc, nhiễm
Cảm cúm là một trận ốm nhẹ, không nghiêm trọng.
Tạm dịch: Tôi cuối cùng đã vượt qua bệnh cúm cái mà tôi đã chịu cả tuần.
Câu 37: Đáp án B
Kiến thức: Từ loại
Giải chi tiết:
Sau tính từ “physical” cần một danh từ
A. attract (v): thu hút
B. attractiveness (n): sự thu hút
C. attractive (adj): cuốn hút
D. attractively (adv): một cách thu hút
Tạm dịch: Một vài người quan tâm về sự thu hút ngoại hình khi chọn vợ hoặc chồng.
Câu 38: Đáp án B

Trang 166
Kiến thức: Câu hỏi đuôi
Giải chi tiết:
Vế trước khẳng định => câu hỏi đuôi sẽ là phủ định
Chủ ngữ là “everyone” => câu hỏi đuôi: “them”
Động từ vế trước chia thì hiện tại đơn “likes’ => câu hỏi đuôi mượn trợ động từ “do/does”
Tạm dịch: Mọi người thích anh ấy phải không?
Câu 39: Đáp án D
Kiến thức: Câu điều kiện loại 3
Giải chi tiết:
- Dấu hiệu: mệnh đề chính “wouldn’t have worn”
- Cách dùng: Câu điều kiện loại 3 diễn tả một điều kiện không có thực ở quá khứ dẫn đến kết quả trái với
sự thật ở quá khứ.
- Công thức: If + S + had + P2, S + would (not) + have + P2
Tạm dịch: Nếu tôi đã biết đó là một bữa tiệc trang trọng, tôi sẽ không mặc chiếc quần bò cũ và một cái áo
thụng của tôi.
Câu 40: Đáp án A
Kiến thức: Liên từ
Giải chi tiết:
A. Despite: Mặc dù => Despite + N/N phr.: Mặc dù cái gì
B. Since + S + V: Bởi vì
C. Because of + N/N phr.: Bởi vì
D. Although + S + V: Mặc dù
“his physical disability” (khuyết tật thể chất) là một cụm danh từ => Loại B, D
Tạm dịch: Mặc cho sự khuyết tật thể chất của mình, anh ấy đã xoay sở để hoàn thành khóa học với kết
quả tốt.
Câu 41: Đáp án D
Kiến thức: Thành ngữ
Giải chi tiết:
“a big cheese”: chỉ một người quan trọng, có quyền lực
Tạm dịch: Ông Park Hang Seo, một huấn luyện viên người Hàn Quốc, được coi là người quan trọng nhất
cho nền bóng đá Việt Nam.
Câu 42: Đáp án B
Kiến thức: to V/ V_ing
Giải chi tiết:
regret + to V_nguyên thể: tiếc khi phải thông báo (tin buồn)
regret + V_ing: tiếc/hối hận vì đã làm gì

Trang 167
“speak” là động từ thường, câu mang nghĩa chủ động => không dùng với “be”
Tạm dịch: Tôi vô cùng hối hận khi đã nói chuyện với cô ấy nặng nề như vậy. Cô ấy đã rất tổn thương.
Câu 43: Đáp án D
Kiến thức: Từ vựng
Giải chi tiết:
change (v): thay đổi
put (v): đặt
give (v): đưa, gửi
scan (v): quét
“scan sth into computer”: quét cái gì vào máy tính
Tạm dịch: Tại sao bạn không quét những tấm ảnh này vào máy tính của bạn, bởi vì sau đó bạn sẽ có các
bản sao chép kĩ thuật số riêng của mình.
Câu 44: Đáp án C
Kiến thức: Mệnh đề rút gọn
Giải chi tiết:
Hai mệnh đề cùng chủ ngữ => có thể rút gọn mệnh đề
Câu mang nghĩa bị động => sử dụng P2
Dạng đầy đủ: Oil was discovered in large quantities in the Middle East, …
Dạng rút gọn: Discovered in large quantities in the Middle East, …
Tạm dịch: Được tìm thấy với số lượng lớn ở Trung Đông, dầu đã được biết đến như là vàng đen bởi vì lợi
nhuận khổng lồ mà nó mang lại.
Câu 45: Đáp án A
Kiến thức: Từ vựng
Giải chi tiết:
A. dedication (n): sự tận tụy, cống hiến
B. loyalty (n): sự trung thành
C. reputation (n): danh tiếng
D. indifference (n): sự thờ ơ
Tạm dịch: Thành công to lớn của Mark Zuckerberg đã lấy đi rất nhiều sự nỗ lực vất vả và cống hiến của
anh ấy. (Mark Zuckerberg đã phải nỗ lực và cống hiến rất nhiều để có được thành công to lớn này.)
Câu 46: Đáp án C
Kiến thức: Trọng âm từ có 3 âm tiết
Giải chi tiết:
A. relevant /ˈreləvənt/
B. attitude /ˈætɪtjuːd/
C. assistant /əˈsɪstənt/

Trang 168
D. argument /ˈɑːɡjumənt/
Đáp án C trọng âm rơi vào âm tiết thứ 2, còn lại là âm tiết thứ nhất
Câu 47: Đáp án B
Kiến thức: Trọng âm từ có 2 âm tiết
Giải chi tiết:
A. promote /prəˈməʊt/
B. profile /ˈprəʊfaɪl/
C. prefer /prɪˈfɜː(r)/
D. regret /rɪˈɡret/
Đáp án B trọng âm rơi vào âm tiết thứ nhất, còn lại là âm tiết thứ 2.
Câu 48: Đáp án D
Kiến thức: Thức giả định
Giải chi tiết:
Câu giả định thường dùng với một số động từ: propose, advise, recommend, suggest, insist, …
Công thức: S1 + động từ giả định + that + S2 + V_nguyên thể
Sửa: “will stay at” => “stay at”
Tạm dịch: Bạn của Jane khăng khăng rằng cô ấy ở lại nhà của họ khi cô ấy đến Toronto cuối tuần tới.
Câu 49: Đáp án C
Kiến thức: Từ loại
Giải chi tiết:
Sau tính từ sở hữu “her” cần một danh từ
intelligent (adj): thông minh
intelligence (n): sự thông minh
Sửa: intelligent => intelligence
Tạm dịch: Người phụ nữ không chỉ nổi tiếng vì sắc đẹp, trí thông minh mà còn vì sự nỗ lực vất vả của cô
ấy.
Câu 50: Đáp án A
Kiến thức: Từ loại
Giải chi tiết:
Sau tính từ sở hữu “women’s” cần một danh từ
responsible (adj): có trách nhiệm
responsibility (n): trách nhiệm
Sửa: responsible => responsibility
Tạm dịch: Phần lớn mọi người cho rằng trách nhiệm của phụ nữ là phải chăm sóc con cái và làm việc
nhà.

Trang 169
SỞ GD & ĐT VĨNH PHÚC ĐỀ THI KHẢO SÁT THPT QUỐC GIA LẦN 1 KHỐI 12
TRƯỜNG THPT QUANG HÀ MÔN: TIẾNG ANH
Thời gian làm bài: 60 phút; không kể thời gian phát đề

Mark the letter A, B, C, or D on your answer sheet to indicate the word whose underlined part is
pronounced differently from that of the others.
Câu hỏi 1 (NB): A. facial B. communicate C. contractual D. carpet
Câu hỏi 2 (NB): A. sometimes B. gestures C. examples D. glances
Mark the letter A, B, C or D on your answer sheet to indicate the word which is stressed differently from
the rest.
Câu hỏi 3 (NB): A. romantic B. marriage C. majority D. attractiveness
Câu hỏi 4 (NB): A. nervous B. verbal C. polite D. body
Mark the letter A, B, C, or D on your answer sheet to indicate the correct answer to each of the following
questions.
Câu hỏi 5 (VD): To show the differences, a survey was ________ among American, Chinese and Indian
students to determine their attitudes towards love and marriage.
A. carried out B. carried about C. carried away D. carried on
Câu hỏi 6 (TH): My mother is the only one that I can absolutely confide ________.
A. in B. up C. with D. for
Câu hỏi 7 (NB): When I ______ up this morning, my roommate _______ already.
A. am waking / leaves B. woke / had left C. was waking / left D. wake / has left
Câu hỏi 8 (NB): Robert wanted to know if I was leaving the ______ Friday.
A. following B. ago C. previous D. before
Câu hỏi 9 (NB): ______ in that company?
A. Are shoes made B. Are shoes be made C. Do shoes make D. Shoes are made
Câu hỏi 10 (VD): Doctors are supposed to _______ for human life.
A. take notice B. take place C. take responsibility D. take advantage
Câu hỏi 11 (TH): John looked up at Jane and tried a smile of love, of ______, of gratitude.
A. dislike B. anger C. unreliability D. trust
Câu hỏi 12 (NB): John asked me _______ that film the night before.
A. that I saw B. had I seen C. if I had seen D. if had I seen
Câu hỏi 13 (TH): His achievements were partly due to the _______ of his wife.
A. assist B. assistant C. assisted D. assistance
Câu hỏi 14 (TH): My grandmother is used to _______ up early in the morning
A. get B. got C. getting D. have gotten
Câu hỏi 15 (NB): In ______ most social situations, _______ informality is appreciated.

Trang 170
A. Ø / Ø B. a / a C. the / Ø D. the / the
Câu hỏi 16 (TH): It _______ it is parents' responsibility to take good care of their children.
A. is commonly saying B. commonly to be said that
C. is commonly said that D. commonly says that
Câu hỏi 17 (TH): _______ my parents are busy at work, they try to find time for their children.
A. Despite B. However C. Because D. Although
Câu hỏi 18 (NB): Everyday Mary types on the computer, but today she _______ a typewriter because
there is no power.
A. has used B. is using C. uses D. will use
Mark the letter A, B, C, or D on your answer sheet to indicate the word(s) CLOSEST in meaning to the
underlined word(s) in each of the following questions.
Câu hỏi 19 (TH): American children customarily go trick-or-treating on Halloween.
A. readily B. happily C. traditionally D. responsibly
Câu hỏi 20 (TH): This is the instance where big, obvious non-verbal signals are appropriate.
A. attention B. situation C. place D. matter
Mark the letter A, B, C, or D on your answer sheet to indicate the word(s) OPPOSITE in meaning to the
underlined word(s) in each of the following questions.
Câu hỏi 21 (VDC): At first, no one believed she was a pilot, but her documents lent colour to her
statements.
A. provided evidence for B. got information from
C. borrowed colour from D. gave no proof of
Câu hỏi 22 (TH): She decided to remain celibate and devote her life to helping the homeless and orphans.
A. divorced B. separated C. married D. single
Mark the letter A, B, C, or D on your answer sheet to indicate the most suitable response to complete each
of the following exchanges.
Câu hỏi 23 (TH): Alice and Mary are talking about women’s roles.
Alice: In my opinion, women should go to work to share household financial burden with their husband.
Mary: __________.
A. That’s a good idea. Women are good at raising children
B. I can’t agree with you more
C. I like going to work
D. No, I don’t think so. Many of them are successful businesswomen
Câu hỏi 24 (TH): Tom and Tony are talking about a tennis game.
Tom: I thought your tennis game was a lot better today, Tony.
Tony: ________! I thought it was terrible.
A. Thanks! Same to you B. You’ve got to be kidding

Trang 171
C. You can say that again! D. No, I think so
Read the following passage and mark the letter A, B, C, or D on your answer sheet to indicate the correct
word or phrase that best fits each of the numbered blanks.
The relationship between students and teachers is less formal in the USA than in many other countries,
especially at the college level. American college students do not stand up when their teachers (25) ______
the room. Students are generally encouraged to ask questions in class, to stop in the professor's office for
extra help, and to phone if they are absent. Most teachers (26) _______ students to enter the class late or
leave early, if necessary. (27) ________ the lack of formality, students are still expected to be polite to
their teacher and fellow classmate.
When students want to ask questions, they usually (28) _______ a hand and wait to be called on. When a
test is being given, talking to a classmate is not only rude but also risky. Most American teachers consider
that students who are talking to each other (29) ________ a test are cheating.
(Source: https://goo.gl/DV77C2)
Câu hỏi 25 (NB): A. enter B. arrive C. come D. go
Câu hỏi 26 (TH): A. encourage B. let C. allow D. make
Câu hỏi 27 (TH): A. Despite B. In spite C. Though D. Because of
Câu hỏi 28 (TH): A. raise B. put C. rise D. arise
Câu hỏi 29 (TH): A. take B. show C. in D. during
Read the following passage and mark the letter A, B, C or D on your answer sheet to indicate the best
answer for each question
When people plan to marry, they expect to find in their partner not only a lover but a friend also. They
find a person with whom they can share their opinions, their emotions, thoughts and fears. In marriage we
are looking for a partner who will be able to understand our values, our likes and dislikes.
If a man and a woman are born and raised in the same country, most likely they are familiar with the
same songs, movies, jokes, books and life in general. They basically have the same roots. In the case of a
western man and foreign woman family, everything is more complicated and requires much more
patience and understanding from both spouses. On one hand each of the partners has an opportunity to
learn a great deal about the other's country, culture, traditions and lifestyles which can be very interesting.
On the other hand, it can be very disappointing if there is the inability to understand your partner's
excitement and frustrations. For example, you are watching the television and suddenly you see a famous
actor or singer, or other types of an artist whose name you have grown up with. Maybe this artist was an
idol for your parents and the music of this artist was often played in your house when you were a child.
Unfortunately, you realize that your wife is unable to understand your feelings because she has no idea
who this artist is. Her eyes are absolutely empty because she has never even heard the song before. You
feel rather disappointed! Remember that your wife has the same situation as you. You do not know her
country's songs, her country's famous actors, her books. She has her own memories and in actuality, for

Trang 172
her, everything is much more difficult than it is for you. At least, you live in your own country where
everybody can understand you. She lives in completely strange surroundings, where she has nobody to
share her feelings with, except you.
Do some research and learn about your wife's country, culture and lifestyles. Talk with her, ask her
questions, get to know what songs she likes, what movies and books are of interest to her. The Internet
will give you a great opportunity to find anything! Tell her about your country's culture, let her listen to
the music that you like, rent a movie for her that left you with great impression. Let her understand you
better through the things that you like. Patience and time will help you to fight cultural differences.
Source: https://vi4.ilovetranslation)
Câu hỏi 30 (TH): spouse should .
A. not interfere with what the partner's likes and dislikes
B. not share the feelings with the partner
C. let the partner do everything alone
D. be not only a lover but also a friend
Câu hỏi 31 (TH): According to the passage, ____________
A. spouses who are from different cultures can never share anything together.
B. spouses who have the same nationality need more patience and understanding in their marriage
than those who are from different cultures.
C. spouses who have the same roots go not share anything together.
D. spouses who are from different cultures need more patience and understanding in their
marriage than those who have the same roots.
Câu hỏi 32 (TH): If there is the inability to understand a spouse's excitements and frustrations the
marriage, he or she may feel _______.
A. faithful B. disappointed C. happy D. hopeless
Câu hỏi 33 (VD): The passage is ________.
A. convincing B. advisory C. apologetic D. critical
Câu hỏi 34 (TH): To overcome cultural differences in marriage needs ________.
A. books and the Internet B. patience and time
C. movies and music D. time and money
Read the following passage and mark the letter A, B, C, or D on your answer sheet to indicate the correct
answer to each of the questions.
For hundreds of years, giving flowers has been a social means of communication. In the United States,
flowers are often given during rites of passage, for commemorating special occasions or as a heartfelt gift
between loved ones and friends. Flower gifting also occurs in most countries around the world. However,
the meanings and traditions often vary.

Trang 173
While students traditionally gave their favorite teacher an apple in past years, in China, teachers are given
flowers. Peonies are by far the flower most often given in China. They are also quite popularly used for
weddings. Strangely, potted plants are not considered a pleasant gift among Asian cultures. The people
believe that like a plant confined by a pot, the gift symbolizes a binding or restriction.
In Russia, in lieu of giving birthday presents, the guest of honor receives a single flower or an unwrapped
bouquet. Floral arrangements or baskets are not given. Russians celebrate a holiday known as Woman's
Day. Traditional gifts include red roses, hyacinths or tulips. When there is a funeral or other occasion
where someone wishes to express sympathy, carnations, lilies or roses are given in circular
configurations, which signify the transition of birth, life and death to rebirth. In this instance, the color of
choice is commonly yellow. For joyous occasions, arrangements and bouquets generally contain an odd
number of flowers.
In the times of ancient Rome, brides carried flowers to scare away evil spirits and encourage fertility. The
Dutch believed that flowers were food for the soul. When invited to someone's home in Great Britain, it is
tradition to bring a gift of flowers. All types are acceptable except white lilies, which are usually seen at
funerals. Unlike the United States, red roses are a symbol of love. Flowers are generally gifted in odd
numbered increments regardless of the occasion. However, the Brits also have superstitions regarding the
number 13, so the number is avoided.
In the southern region of the continent, flowers are traditionally given during Christmas. Egyptians are
much more conservative and restrict flower gifting to funerals and weddings. While certain flowers may
have significant meanings for some, flowers in Las Vegas and across the United States flowers are an
accepted gift for any reason desired.
(Source: http://www.fJowersofthefieldIv.com/)
Câu hỏi 35 (TH): What does the topic mainly discuss?
A. The different meanings of flowers in different cultures.
B. The comparison of giving flowers between Asian and European cultures.
C. The kinds of flowers people often give each other in different cultures.
D. The fascinating tradition of giving flowers.
Câu hỏi 36 (NB): What does the word “They" in paragraph 2 refer to?
A. Flowers B. Teachers C. Peonies D. Students
Câu hỏi 37 (TH): Why should not you give a potted plant to an Asian?
A. As this gift is often given at weddings in Asia.
B. Because the Asian prefer to be given flowers.
C. Since this gift is believed to symbolize an astriction and limitation in Asia.
D. Because Asian students like to give an apple or flowers to others.
Câu hỏi 38 (NB): According to the passage, the following flowers are given at Women's Day in Russia,
EXCEPT _______.

Trang 174
A. hyacinths B. tulips C. red roses D. yellow roses
Câu hỏi 39 (TH): What could the word "fertility" in paragraph 4 best be replaced by?
A. fecundity B. happiness C. good spirit D. loyalty
Câu hỏi 40 (TH): The word “superstitions" in paragraph 4 is closest in meaning to _______.
A. deep-seated belief B. religious belief C. traditional belief D. unfounded belief
Câu hỏi 41 (NB): In which country should not people bring white lilies to other houses?
A. Russia B. China C. Great Britain D. United States
Câu hỏi 42 (VD): It can be inferred from the passage that _______.
A. Egyptians are rather comfortable when receiving flowers at funerals and weddings.
B. At the funerals in any cultures, flowers are gifted in circular configurations.
C. People can give flowers to the American on any occasion.
D. Flowers given in Britain are in even numbers in any case.
Mark the letter A, B, C or D on your answer sheet to indicate the underlined word or phrase in each
sentence that needs correcting.
Câu hỏi 43 (TH): No sooner I had got out of the burning building than it collapsed.
A. I had B. than C. burning D. collapsed
Câu hỏi 44 (NB): Last month, while my friend travelled around England by car, he crashed the car into a
tree.
A. while B. travelled C. the D. a
Câu hỏi 45 (TH): There are dangerous times in a marriage, especially when the wife can come to feel
such overburdened that she decides to end the relationship.
A. such B. dangerous times C. especially D. decides
Mark the letter A, B, C, or D on your answer sheet to indicate the sentence that is CLOSEST in meaning
to each of the following questions.
Câu hỏi 46 (VD): “Shall I help you do the dishes, Carol”? said Robert.
A. Robert suggested help Carol with the dishes.
B. Robert offered to help Carol do the dishes.
C. Robert suggested to help Carol do the dishes.
D. Robert offered Carol to help do the dishes.
Câu hỏi 47 (VD): This is the first time I have attended such an enjoyable wedding party.
A. The first wedding party I attended was enjoyable.
B. I have never attended such an enjoyable wedding party before.
C. I had the first enjoyable wedding party.
D. My attendance at the first wedding party was enjoyable.
Câu hỏi 48 (VDC): He expected us to offer him the job.
A. He is expected that we should offer him the job.

Trang 175
B. He expected to be offered the job.
C. We were expected to be offered him the job.
D. He was offered the job without expectation.
Mark the letter A, B, C or D on your answer sheet to indicate the correct sentence that best combines each
pair of sentences in the following questions.
Câu hỏi 49 (VD): We spend about one-third of our lives sleeping. We know relatively little about sleep.
A. We know relatively little about sleep; as a result, we spend about one-third of our lives
sleeping.
B. We shall know more about sleep if we spend more than one-third of our lives sleeping.
C. We spend about one-third of our lives sleeping so that we know relatively little about sleep.
D. Despite spending about one-third of our lives sleeping, we know relatively little about sleep.
Câu hỏi 50 (VDC): My motorbike cannot start in the mornings. I think I will get the garage to repair it.
A. My motorbike I will get it repaired which cannot start in the mornings.
B. My motorbike cannot start in the mornings, so I will have it repaired.
C. My motorbike which I will get it repaired cannot start in the mornings.
D. My motorbike which I will have it repaired cannot start in the mornings.

Trang 176
Đáp án
1-A 2-D 3-B 4-C 5-A 6-A 7-B 8-A 9-A 10-C
11-D 12-C 13-D 14-C 15-A 16-C 17-D 18-B 19-C 20-
21-D 22-C 23-B 24-B 25-A 26-C 27-A 28-A 29-D 30-
31-D 32-B 33-B 34-B 35-D 36-C 37-C 38-D 39-A 40-
41-C 42-C 43-A 44-B 45-A 46-B 47-B 48-B 49-D 50-

LỜI GIẢI CHI TIẾT


Câu 1: Đáp án A
Kiến thức: Phát âm “c”
Giải chi tiết:
A. facial /ˈfeɪʃl/
B. communicate /kəˈmjuːnɪkeɪt/
C. contractual /kənˈtræktʃuəl/
D. carpet /ˈkɑːpɪt/
Phần gạch chân phương án A được phát âm là /ʃ/, còn lại là /k/
Câu 2: Đáp án D
Kiến thức: Phát âm “s”
Giải chi tiết:
A. sometimes /ˈsʌmtaɪmz/
B. gestures /ˈdʒestʃə(r)z/
C. examples /ɪɡˈzɑːmplz/
D. glances /ˈɡlɑːnsɪz/
Quy tắc: Có 3 cách phát âm phụ âm cuối “s”:
- /s/: Khi từ có phát âm tận cùng là các phụ âm vô thanh /p/, k/, /f/, /ð/, /t/
- /ɪz/: Khi từ có phát âm tận cùng là các âm /s/, /ʃ/, /tʃ/, /z/, /ʒ/, /dʒ/ (thường có tận cùng là các chữ cái ce,
x, z, sh, ch, s, ge, ss)
- /z/: Khi từ có tận cùng là các nguyên âm và các phụ âm còn lại.
Phần gạch chân phương án D được phát âm là /ɪz/, còn lại là /z/
Câu 3: Đáp án B
Kiến thức: Trọng âm của từ có 3, 4 âm tiết
Giải chi tiết:
A. romantic /rəʊˈmæntɪk/
B. marriage /ˈmærɪdʒ/
C. majority /məˈdʒɒrəti/

Trang 177
D. attractiveness /əˈtræktɪvnəs/
Quy tắc:
Những từ kết thúc bằng đuôi _ic, _ity, _ive thì trọng âm thường rơi vào âm tiết đứng ngay trước các đuôi
này.
Hậu tố _ness không nhận trọng âm và không làm thay đổi trọng âm của từ gốc.
Phương án B trọng âm rơi vào âm tiết thứ nhất, còn lại là âm thứ 2
Câu 4: Đáp án C
Kiến thức: Trọng âm của từ có 2 âm tiết
Giải chi tiết:
A. nervous /ˈnɜːvəs/
B. verbal /ˈvɜːbl/
C. polite /pəˈlaɪt/
D. body /ˈbɒdi/
Quy tắc: Danh từ, tính từ có 2 âm tiết thường có trọng âm rơi vào âm tiết thứ nhất. Ngoại lệ: po‘lite
Phương án C trọng âm rơi vào âm tiết thứ 2, còn lại là âm thứ nhất
Câu 5: Đáp án A
Kiến thức: Cụm động từ
Giải chi tiết:
A. carried out: tiến hành, thực hiện
B. carried about: đưa đi vòng quanh
C. carried away: cuỗm, lấy đi
D. carried on: tiếp tục
Tạm dịch: Để thể hiện những điểm khác nhau, một cuộc khảo sát đã được thực hiện giữa các sinh viên
Mĩ, Trung Quốc và Ấn Độ để xác định thái độ của họ về tình yêu và hôn nhân.
Câu 6: Đáp án A
Phương pháp giải:
Kiến thức: Giới từ
Giải chi tiết:
confide in (sb): giãi bày (với ai)
Tạm dịch: Mẹ là người duy nhất mà tôi có thể giãi bày.
Câu 7: Đáp án B
Kiến thức: Thì quá khứ đơn, quá khứ hoàn thành
Giải chi tiết:
- Dấu hiệu: “this morning” (sáng nay), “already” (đã...rồi)
- Cách dùng: Thì quá khứ hoàn thành diễn tả hành động đã xảy ra trước một hành động khác trong quá
khứ.

Trang 178
- Công thức: When S1 + V_ed, S2 + had + P2 + already: Khi ai làm gì, ai đó đã làm gì trước rồi.
wake => woke (V cột 2); leave => left (P2/ V cột 3)
Tạm dịch: Khi tôi thức dậy vào sáng nay, bạn cùng phòng tôi đã rời đi rồi.
Câu 8: Đáp án A
Kiến thức: Lời nói gián tiếp
Giải chi tiết:
Khi chuyển từ lời nói trực tiếp sang lời nói gián tiếp ta cần thay đổi một số trạng từ chỉ thời gian:
next => the next + N (time) / the following + N (time)
Câu trực tiếp: “Are you leaving next Friday?”, Robert asked me.
Câu gián tiếp: Robert wanted to know if I was leaving the next/following Friday.
Tạm dịch: Robert đã muốn biết liệu tôi sẽ rời đi vào thứ Sáu tới không.
Câu 9: Đáp án A
Kiến thức: Câu bị động
Giải chi tiết:
- Công thức câu bị động thì hiện tại đơn: S + are/ is/ am + P2
- Dạng nghi vấn/câu hỏi: Are/ Is/ Am + S + P2?
Tạm dịch: Những đôi giày được làm ở công ty đó phải không?
Câu 10: Đáp án C
Kiến thức: Sự kết hợp từ
Giải chi tiết:
“take responsibility for sb/ sth”: chịu trách nhiệm về ai/ cái gì
Các phương án khác:
take notice of …: chú ý
take place: diễn ra
take advantage of ...: tận dụng lợi thế
Tạm dịch: Bác sĩ được mong đợi là sẽ chịu trách nhiệm cho sự sống của loài người.
Câu 11: Đáp án D
Kiến thức: Từ vựng
Giải chi tiết:
A. dislike (n): sự không thích
B. anger (n): sự tức giận
C. unreliablity (n): sự không đáng tin cậy
D. trust (n): niềm tin, sự tin tưởng
Tạm dịch: John đã nhìn vào Jane và cố nở một nụ cười của tình yêu, của sự tin tưởng, của lòng vị tha.
Câu 12: Đáp án C
Kiến thức: Lời nói gián tiếp

Trang 179
Giải chi tiết:
Dấu hiệu: “the night before” (tối hôm trước)
Công thức câu gián tiếp với câu hỏi Yes/No: S1 asked S2 + if/whether + S2 + V_lùi thì
saw (quá khứ đơn) => had seen (quá khứ hoàn thành)
last night => the night before
Tạm dịch: John đã hỏi tôi rằng tôi đã xem bộ phim đó tối qua chưa.
Câu 13: Đáp án D
Kiến thức: Từ loại
Giải chi tiết:
Sau mạo từ “the” cần một danh từ
A. assist (v): hỗ trợ
B. assistant (n): trợ lí (danh từ chỉ người)
C. assisted (v_ed): đã hỗ trợ
D. assistance (n): sự hỗ trợ
Theo ngữ cảnh câu, đáp án D là phù hợp
Tạm dịch: Các thành tựu của anh ấy một phần là nhờ sự hỗ trợ của người vợ.
Câu 14: Đáp án C
Phương pháp giải:
Kiến thức: to V/ V_ing
Giải chi tiết:
be used to + V_ing: quen làm gì
Tạm dịch: Bà tôi quen dậy sớm vào buổi sáng.
Câu 15: Đáp án A
Kiến thức: Mạo từ
Giải chi tiết:
most + N số nhiều: phần lớn, hầu hết => không dùng mạo từ
informality (n): sự không trang trọng => danh từ chỉ chung, không đếm được => không cần mạo từ
Tạm dịch: Trong hầu hết các tình huống xã hội, sự không trang trọng được đánh giá cao.
Câu 16: Đáp án C
Kiến thức: Câu bị động
Giải chi tiết:
Cấu trúc: It + tobe + said/ believed/ thought that + S + V
Vế sau tobe chia “is” => động từ tobe vế trước cũng chia hiện tại đơn là “is”
Tạm dịch: Thường được nói rằng trách nhiệm của cha mẹ là chăm sóc con cái.
Câu 17: Đáp án D
Kiến thức: Liên từ

Trang 180
Giải chi tiết:
A. Despite + N/N phr.: Mặc dù
B. However, S + V: Tuy nhiên
C. Because + S + V: Bởi vì
D. Although + S + V: Mặc dù
“my parents are busy at work” là một cụm S + V => loại A
Tạm dịch: Mặc dù bố mẹ tôi bận làm việc, họ vẫn cố gắng tìm thời gian dành cho con cái của mình.
Câu 18: Đáp án B
Kiến thức: Thì hiện tại tiếp diễn
Giải chi tiết:
- Dấu hiệu: “but today”: nhưng hôm nay (nhấn mạnh vào hành động ngắn, chỉ xảy ra vào hôm nay)
- Cách dùng: Thì hiện tại tiếp diễn dùng để diễn tả một sự việc đang diễn ra tại thời điểm nói.
- Công thức: S + are/ is/ am + V_ing
Tạm dịch: Mọi ngày, Mary đánh chữ trên máy tính, nhưng hôm nay cô ấy đang sử dụng máy đánh chữ
bởi vì mất điện.
Câu 19: Đáp án C
Kiến thức: Từ vựng, từ đồng nghĩa
Giải chi tiết:
customarily (adv): một cách truyền thống
A. readily (adv): một cách sẵn sàng
B. happily (adv): một cách vui vẻ
C. traditionally (adv): một cách truyền thống
D. responsibly (adv): một cách có trách nhiệm
=> customarily = traditionally
Tạm dịch: Trẻ em Mĩ theo truyền thống chơi trò bị ghẹo hay cho kẹo vào đêm Halloween.
Câu 20: Đáp án B
Kiến thức: Từ vựng, từ đồng nghĩa
Giải chi tiết:
instance (n): trường hợp
A. attention (n): sự chú ý
B. situation (n): tình huống
C. place (n): nơi
D. matter (n): vấn đề
=> instance = situation
Tạm dịch: Đây là trường hợp mà ở đó những kí hiệu phi ngôn ngữ to, rõ ràng thì phù hợp.
Câu 21: Đáp án D

Trang 181
Kiến thức: Thành ngữ
Giải chi tiết:
lent colour to (idiom): đưa ra bằng chứng
A. provided evidence for: cung cấp bằng chứng cho
B. got information from: lấy thông tin từ
C. borrowed colour from: mượn màu từ
D. gave no proof of: không đưa ra bằng chứng
=> lent colour to >< gave no proof of
Tạm dịch: Đầu tiên, không một ai tin cô ấy đã là một phi công, nhưng các tài liệu của cô ấy đã đưa ra
bằng chứng cho khẳng định của cô.
Câu 22: Đáp án C
Kiến thức: Từ vựng, từ trái nghĩa
Giải chi tiết:
celibate (adj): độc thân
A. divorce (v): li hôn
B. seperated (adj): li thân
C. married (adj): đã kết hôn
D. single (adj): độc thân
=> celibate >< married
Tạm dịch: Cô ấy đã quyết định duy trì tình trạng độc thân và dành cuộc đời mình để giúp đỡ trẻ mồ côi và
những người vô gia cư.
Câu 23: Đáp án B
Kiến thức: Ngôn ngữ giao tiếp
Giải chi tiết:
Alice và Mary đang nói về vai trò của người phụ nữ.
Alice: Theo tôi, phụ nữ nên đi làm để chia sẻ gánh nặng tài chính trong gia đình với chồng của họ.
Mary: _________.
A. Đó là một ý kiến hay. Phụ nữ giỏi việc nuôi con
B. Tôi hoàn toàn đồng ý!
C. Tôi thích đi làm
D. Không, tôi không nghĩ vậy. Nhiều người trong số họ là các nữ doanh nhân thành đạt
Các phản hồi A, C, D không phù hợp ngữ cảnh.
Câu 24: Đáp án B
Phương pháp giải:
Kiến thức: Ngôn ngữ giao tiếp
Giải chi tiết:

Trang 182
Tom và Tony đang nói về trò chơi tennis.
Tom: “Mình nghĩ hôm nay cậu chơi tennis đã tốt hơn nhiều rồi đó, Tony”
Tony: “________. Mình nghĩ nó thật tồi tệ”
A. Cảm ơn! Cậu cũng vậy!
B. Cậu chắc đùa.
C. Cậu nói rất đúng.
D. Không, mình nghĩ vậy
Các phản hồi A, C, D không phù hợp ngữ cảnh.
Câu 25: Đáp án A
Kiến thức: Từ vựng
Giải chi tiết:
A. enter (v): đi vào
B. arrive in/at … (v): đến
C. come + to … (v): đến
D. go + to …. (v): đi
American college students do not stand up when their teachers (25) enter the room.
Tạm dịch: Các sinh viên Mĩ không đứng dậy khi giáo viên vào lớp.
Câu 26: Đáp án C
Kiến thức: Từ vựng
Giải chi tiết:
A. encourage (v): khuyến khích => encourage sb + to V_nguyên thể: khuyến khích ai đó làm gì
B. let (v): để => let sb + V_nguyên thể: để ai đó làm gì
C. allow (v): cho phép => allow sb + to V_nguyên thể: cho phép ai đó làm gì
D. make (v): khiến, làm => make sb + to V_nguyên thể: khiến ai đó phải làm gì
Most teachers (26) allow students to enter the class late or leave early, if necessary.
Tạm dịch: Phần lớn giáo viên cho phép sinh viên vào lớp muộn hoặc tan học sớm, nếu cần thiết.
Câu 27: Đáp án A
Kiến thức: Liên từ
Giải chi tiết:
A. Despite + N: Mặc dù việc gì
B. In spite + of + N: Mặc dù việc gì
C. Though + S + V: Mặc dù ai làm gì
D. Because of + N: Bởi vì
“the lack of formality” (sự thiếu trang trọng) là một cụm từ, không phải S + V => loại C
(27) Despite the lack of formality, students are still expected to be polite to their teacher and fellow
classmate.

Trang 183
Tạm dịch: Mặc dù việc thiếu trang trọng, sinh viên vẫn được mong là sẽ lịch sự với giáo viên và bạn cùng
lớp.
Câu 28: Đáp án A
Kiến thức: Từ vựng
Giải chi tiết:
raise + N: giơ, đưa cái gì đó lên (cần tân ngữ)
put (v): đưa, đặt
rise (v): mọc, tăng lên (không cần tân ngữ)
arise (v): tự tăng lên
“raise a hand”: giơ tay
When students want to ask questions, they usually (28) raise a hand and wait to be called on.
Tạm dịch: Khi sinh viên muốn đặt câu hỏi, họ thường giơ tay và đợi để được gọi.
Câu 29: Đáp án D
Kiến thức: Giới từ + Liên từ
Giải chi tiết:
A. when + S +V: khi …
B. while + S + V: trong khi …
C. in (prep.): ở trong
D. during + N : trong suốt
Most American teachers consider that students who are talking to each other (29) during a test are
cheating.
Tạm dịch: Hầu hết giáo viên ở Mĩ cho rằng các sinh viên nói chuyện với nhau trong suốt một bài kiểm tra
là đang gian lận.
Câu 30: Đáp án D
Kiến thức: Đọc hiểu
Giải chi tiết:
Một người chồng/ vợ nên ________.
A. không can thiệp vào sở thích và không thích của bạn đời
B. không chia sẻ cảm xúc với bạn đời
C. để bạn đời làm mọi thứ một mình
D. không chỉ là người yêu mà còn là một người bạn
Thông tin: When people plan to marry, they expect to find in their partner not only a lover but a friend
also.
Tạm dịch: Khi mọi người có kế hoạch kết hôn, họ hy vọng sẽ tìm thấy ở người bạn đời của họ không chỉ
là một người yêu mà còn là một người bạn.
Câu 31: Đáp án D

Trang 184
Kiến thức: Đọc hiểu
Giải chi tiết:
Theo đoạn văn, __________
A. vợ chồng người mà đến từ nền văn hóa khác nhau không bao giờ có thể chia sẻ điều gì cùng nhau.
B. vợ chồng người mà có cùng quốc tịch cần nhiều sự kiên nhẫn và thấu hiểu hơn trong hôn nhân của họ.
C. vợ chồng người mà có cùng nguồn gốc không chia sẻ điều gì cùng nhau.
D. vợ chồng người mà đến từ nền văn hóa khác nhau cần nhiều sự kiên nhẫn và thấu hiểu hơn trong hôn
nhân
của họ so với những người có cùng nguồn gốc.
Thông tin: In the case of a western man and foreign woman family everything is more complicated and
requires much more patience and understanding from both spouses.
Tạm dịch: Trong trường hợp gia đình một người đàn ông phương Tây và người phụ nữ nước ngoài mọi
thứ phức tạp hơn và đòi hỏi sự kiên nhẫn nhiều hơn và sự thấu hiểu nhiều hơn đến từ cả hai vợ chồng.
Câu 32: Đáp án B
Kiến thức: Đọc hiểu
Giải chi tiết:
Nếu không có khả năng thấu hiểu niềm vui và sự khó chịu của vợ/chồng trong hôn nhân, anh hoặc cô ấy
có thể cảm thấy ________.
A. trung thành
B. thất vọng
C. hạnh phúc
D. vô vọng
Thông tin: On the other hand it can be very disappointing if there is the inability to understand your
partner's excitements and frustrations.
Tạm dịch: Mặt khác, có thể rất đáng thất vọng nếu không có khả năng hiểu những niềm vui và sự khó
chịu của người bạn đời.
Câu 33: Đáp án B
Kiến thức: Đọc hiểu
Giải chi tiết:
Đoạn văn thì _______.
A. có tính thuyết phục
B. có tính khuyên nhủ
C. thể hiện sự tiếc nuối
D. có tính chỉ trích
Thông tin: Do some research and learn about your wife's country, culture and lifestyles. Talk with her,
ask her questions, get to know what songs she likes, what movies and books are of interest to her. The

Trang 185
Internet will give you a great opportunity to find anything! Tell her about your country's culture, let her
listen to the music that you like, rent a movie for her that left you with great impression. Let her
understand you better through the things that you like.
Tạm dịch: Hãy thực hiện một vài nghiên cứu và tìm hiểu về đất nước, văn hóa và lối sống của vợ bạn. Nói
chuyện với cô ấy, hỏi câu hỏi của cô, biết những bài hát cô thích, những bộ phim và sách cô ấy có hứng
thú. Internet sẽ cung cấp cho bạn một cơ hội tuyệt vời để tìm thấy bất cứ điều gì! Hãy nói với cô ấy về
văn hóa của đất nước bạn, hãy để cô ấy nghe nhạc mà bạn thích, thuê cho cô ấy bộ phim mà để lại ấn
tượng tuyệt vời với bạn. Hãy để cô ấy hiểu bạn hơn thông qua những điều mà bạn thích.
=> Đưa ra những lời khuyên, những việc nên làm
Câu 34: Đáp án B
Kiến thức: Đọc hiểu
Giải chi tiết:
Để vượt qua sự khác biệt văn hóa trong hôn nhân cần ______.
A. sách và Internet
B. sự kiên nhẫn và thời gian
C. các bộ phim và nhạc
D. thời gian và tiền bạc
Thông tin: Patience and time will help you to fight cultural differences.
Tạm dịch: Sự kiên nhẫn và thời gian sẽ giúp bạn chống lại sự khác biệt văn hóa.
Câu 35: Đáp án D
Kiến thức: Đọc hiểu
Giải chi tiết:
Bài đọc chủ yếu bàn luận cái gì?
A. Các ý nghĩa khác nhau của hoa ở các nền văn hóa khác nhau.
B. Sự so sánh về việc tặng hoa giữa văn hóa Châu Á và Châu Âu.
C. Các loại hoa mọi người thường tặng cho người khác ở các nền văn hóa khác nhau.
D. Truyền thống thú vị của việc tặng hoa.
Thông tin: Flower gifting also occurs in most countries around the world. However, the meanings and
traditions often vary.
Tạm dịch: Việc tặng hoa cũng diễn ra ở hầu hết các nước trên thế giới. Tuy nhiên, ý nghĩa và truyền
thống thường khác nhau.
Câu 36: Đáp án C
Kiến thức: Đọc hiểu
Giải chi tiết:
Từ “They” trong đoạn 2 đề cập đến cái gì?
A. Những bông hoa

Trang 186
B. Giáo viên
C. Hoa mẫu đơn
D. Học sinh
Thông tin: Peonies are by far the flower most often given in China. They are also quite popularly used
for weddings.
Tạm dịch: Hoa mẫu đơn là loài hoa thường được tặng nhất ở Trung Quốc. Chúng cũng được sử dụng khá
phổ biến cho đám cưới.
Câu 37: Đáp án C
Kiến thức: Đọc hiểu
Giải chi tiết:
Tại sao không nên tặng một cây được trồng trong chậu cho người Châu Á?
A. Bởi vì món quà thường được tặng trong các đám cưới ở Châu Á.
B. Bởi vì người Châu Á thích được tặng hoa.
C. Bởi vì món quà này được tin là biểu tượng cho sự giới hạn và hạn chế ở Châu Á.
D. Bởi vì sinh viên Châu Á thích tặng táo hoặc hoa cho người khác.
Thông tin: The people believe that like a plant confined by a pot, the gift symbolizes a binding or
restriction.
Tạm dịch: Mọi người tin rằng giống như một cái cây bị giới hạn bởi một cái chậu, món quà tượng trưng
cho một ràng buộc hoặc hạn chế.
Câu 38: Đáp án D
Kiến thức: Đọc hiểu
Giải chi tiết:
Theo bài văn, loài hoa nào được tặng vào ngày phụ nữ ở Nga ngoại trừ ______.
A. hoa lan dạ hương
B. hoa tulip
C. hoa hồng đỏ
D. hoa hồng vàng
Thông tin: Russians celebrate a holiday known as Women's Day. Traditional gifts include red roses,
hyacinths or tulips.
Tạm dịch: Nga kỷ niệm một ngày lễ được biết đến như là ngày phụ nữ. Quà tặng truyền thống bao gồm
hoa hồng đỏ, hoa lan dạ hương hoặc hoa tulip.
Câu 39: Đáp án A
Kiến thức: Đọc hiểu
Giải chi tiết:
Từ nào có thể thay thế phù hợp nhất cho từ “fertility” trong đoạn 4?
fertility (n): khả năng sinh sản, sinh sôi

Trang 187
A. fecundity (n): sự màu mỡ, sinh sôi nảy nở
B. happiness (n): sự vui vẻ
C. good spirit (n): tinh thần tốt
D. loyalty (n): sự trung thành
=> fertility = fecundity
Thông tin: In the times of ancient Rome, brides carried flowers to scare away evil spirits and encourage
fertility.
Tạm dịch: Trong thời của La Mã cổ đại, cô dâu mang hoa để xua đuổi linh hồn ma quỷ và
khuyến khích sự sinh sản.
Câu 40: Đáp án D
Kiến thức: Đọc hiểu
Giải chi tiết:
Từ “superstitions” trong đoạn 4 gần nghĩa nhất với _______
superstitions (n): sự mê tín
A. deep-seated belief: niềm tin chắc chắn
B. religious belief: niềm tin tôn giáo
C. traditional belief: niềm tin truyền thống
D. unfounded belief: niềm tin vô căn cứ
=> superstitions = unfounded belief
Thông tin: However, the Brits also have superstitions regarding the number 13, so the number is avoided.
Tạm dịch: Tuy nhiên, người Anh cũng có những mê tín liên quan đến số 13, do đó, số này bị tránh.
Câu 41: Đáp án C
Kiến thức: Đọc hiểu
Giải chi tiết:
Ở đất nước nào, mọi người không nên đem hoa huệ trắng đến nhà người khác?
A. Nga
B. Trung Quốc
C. Anh
D. Mĩ
Thông tin: When invited to someone's home in Great Britain, it is tradition to bring a gift of flowers. All
types are acceptable except white lilies, which are usually seen at funerals.
Tạm dịch: Khi được mời đến nhà của một ai đó ở Anh, đó là truyền thống khi đem tặng hoa làm quà. Tất
cả các loại đều được chấp nhận ngoại trừ hoa huệ trắng, cái mà thường được thấy ở đám tang.
Câu 42: Đáp án C
Kiến thức: Đọc hiểu
Giải chi tiết:

Trang 188
Có thể suy ra từ đoạn văn rằng ______.
A. Người Ai Cập khá thoải mái khi nhận hoa trong đám tang và tiệc cưới.
B. Trong đám tang ở bất kì nền văn hóa nào, hoa được tặng theo dạng vòng tròn.
C. Mọi người có thể tặng hoa cho người Mĩ vào bất cứ dịp nào.
D. Hoa được tặng ở Anh thì là lượng số chắn trong bất kì hoàn cảnh nào.
Thông tin: ... flowers in Las Vegas and across the United States flowers are an accepted gift for any
reason desired.
Tạm dịch: ... hoa ở Las Vegas và trên toàn nước Mĩ hoa là một món quà được chấp nhận cho bất kỳ lý do
mong muốn nào.
Câu 43: Đáp án A
Kiến thức: Đảo ngữ với “No sooner”
Giải chi tiết:
Công thức: No sooner + had + S + P2 than S + V_ed: Ngay sau khi việc gì xảy ra, việc kia xảy ra
Sửa: “I had” => “had I”
Tạm dịch: Ngay sau khi tôi ra khỏi tòa nhà đang cháy thì nó sập
Câu 44: Đáp án B
Kiến thức: Thì quá khứ tiếp diễn
Giải chi tiết:
- Cách dùng: thì quá khứ tiếp diễn diễn tả hành động đang xảy ra trong quá khứ (chia quá khứ tiếp diễn)
thì có hành động khác cắt ngang (chia quá khứ đơn)
- Công thức: While + S + was/were +V_ing, S + V_ed/V cột 2
Sửa: travelled => was travelling
Tạm dịch: Tháng trước, trong khi bạn tôi đang đi vòng quanh nước Anh bằng xe hơi, anh ấy đã đâm xe
vào một cái cây.
Câu 45: Đáp án A
Kiến thức: Cấu trúc “so...that”
Giải chi tiết:
Công thức: S + V + so + adj/ adv that + S + V: Cái gì đó quá … đến nỗi mà …
= S + V + such + a/an + adj + N + that + S + V
overburdened (adj): quá gánh nặng
Sửa: such => so
Tạm dịch: Có những thời gian nguy hiểm trong một cuộc hôn nhân, đặc biệt khi người vợ có thể cảm thấy
quá gánh nặng đến nỗi mà cô ấy quyết định kết thúc mối quan hệ.
Câu 46: Đáp án B
Kiến thức: Lời nói gián tiếp
Giải chi tiết:

Trang 189
Câu trực tiếp: “Shall I + V?” S1 said.
Câu gián tiếp: S1 offered to V_nguyên thể: Ai đó đề nghị làm gì
A. Sai ngữ pháp: suggest + V_ing
B. Robert đã đề nghị giúp Carol rửa bát đĩa.
C. Sai ngữ pháp: suggest + V_ing
D. Robert đã đề nghị Carol giúp (Robert) rửa bát đĩa. => sai nghĩa
Tạm dịch:
“Tôi có thể giúp bạn rửa bát đĩa không, Carol?” Robert nói.
= Robert đã đề nghị giúp Carol rửa bát đĩa.
Câu 47: Đáp án B
Kiến thức: Cấu trúc đồng nghĩa
Giải chi tiết:
This is the first time + S + have/has + P2: Đây là lần đầu tiên ai làm việc gì
= S + have/ has + never/ ever + P2 before: Ai đó chưa từng làm gì trước đây
A. Tiệc cưới đầu tiên tôi đã tham dự thì vui. => sai nghĩa
B. Tôi chưa từng tham gia một tiệc cưới nào vui như vậy trước đây.
C. Tôi đã có tiệc cưới vui vẻ đầu tiên. => sai nghĩa
D. Sự tham gia của tôi ở tiệc cưới đầu tiên thì vui. => sai nghĩa
Tạm dịch: Đây là lần đầu tiên tôi tham dự một tiệc cưới vui như vậy.
= Tôi chưa từng tham gia một tiệc cưới nào vui như vậy trước đây.
Câu 48: Đáp án B
Kiến thức: Câu bị động
Giải chi tiết:
Công thức câu bị động: S + be + P2
S + expect + O + to V_nguyên thể: Ai đó hi vọng ai làm gì
= S + expect + to + be + P2: Ai đó hi vọng được làm gì
A. Anh ấy được hi vọng rằng chúng tôi nên giao công việc cho anh ấy. => sai nghĩa
B. Anh ấy đã hi vọng được giao cho công việc.
C. Chúng tôi đã được hi vọng được giao công việc cho anh ấy. => sai nghĩa
D. Anh ấy đã được giao công việc mà không phải mong đợi.
Tạm dịch: Anh ấy đã hi vọng chúng tôi giao cho anh ấy công việc.
= Anh ấy đã mong được giao cho công việc.
Câu 49: Đáp án D
Kiến thức: Mệnh đề nhượng bộ
Giải chi tiết:
Công thức: Despite + V_ing, S + V: Mặc dù làm gì, ai đó vẫn làm gì

Trang 190
A. Chúng ta biết tương đối ít về giấc ngủ; do đó, chúng ta dành một phần ba cuộc đời mình để ngủ. => sai
nghĩa
B. Chúng ta sẽ biết nhiều hơn về giấc ngủ nếu chúng ta dành nhiều hơn một phần ba cuộc đời mình để
ngủ. => sai nghĩa
C. Chúng ta dành khoảng một phần ba cuộc sống của mình để ngủ để mà chúng ta biết tương đối ít về
giấc ngủ. => sai nghĩa
D. Mặc dù dành khoảng một phần ba cuộc sống của mình để ngủ, chúng ta biết tương đối ít về giấc ngủ.
Tạm dịch: Chúng ta dành khoảng một phần ba cuộc sống của mình để ngủ. Chúng ta biết tương đối ít về
giấc ngủ.
= Mặc dù dành khoảng một phần ba cuộc sống của mình để ngủ, chúng ta biết tương đối ít về giấc ngủ.
Câu 50: Đáp án B
Kiến thức: Mệnh đề kết quả, mệnh đề quan hệ, câu bị động
Giải chi tiết:
Cấu trúc mệnh đề kết quả: S + V, so + S + V: …., vì vậy …
Cấu trúc: get sb + to V_nguyên thể = have sb + V_nguyên thể: nhờ ai đó làm gì
= have/get sth + P2: có cái gì được làm bởi ai đó
A. Sai vị trí mệnh đề quan hệ. Mệnh đề quan hệ đứng ngay sau “My motorbike” để giải thích thêm cho
nó.
B. Xe máy của tôi không thể khởi động vào các buổi sáng, vì vậy tôi sẽ nhờ người sửa nó.
C, D Sai vị trí mệnh đề quan hệ. Hành động chính trong câu này là đi sửa xe, mệnh đề quan hệ đưa ra
thông tin bổ sung, giải thích rõ xem đó là cái xe nào.
Tạm dịch: Xe máy của tôi không thể khởi động vào các buổi sáng. Tôi nghĩ tôi sẽ đến nhà xe để sửa nó.
= Xe máy của tôi không thể khởi động vào các buổi sáng, vì vậy tôi sẽ nhờ người sửa nó.

SỞ GD & ĐT THÁI BÌNH ĐỀ THI THỬ THPT QUỐC GIA LẦN 2 LỚP 12
TRƯỜNG THPT CHUYÊN NĂM HỌC: 2019 – 2020
Mã đề 404 MÔN: TIẾNG ANH
Thời gian làm bài: 60 phút; không kể thời gian phát đề

Mark the letter A, B, C, or D on your answer sheet to indicate the most suitable response to complete each
of the following exchanges.
Câu 1 (TH): Jenny and Jimmy are talking about university education.
Jenny: “I think a university degree is the only way to succeed in life”.
Jimmy: “__________. There are successful people without a degree.”
A. That’s life. B. That’s all right. C. I don’t quite agree. D. I can’t agree more.

Trang 191
Câu 2 (NB): Laura is telling Bob about her exam results.
Laura: “ ____________________”
Bob: “That’s great. Congratulations.
A. I’ve passed the exam with an A. B. I hope I’ll pass the exam tomorrow.
C. I’ll get the exam result tomorrow. D. I didn’t do well in the exam.
Mark the letter A, B, C, or D on your answer sheet to indicate the word whose underlined part differs
from the other three in pronunciation in each of the following questions.
Câu 3 (NB): A. called B. passed C. talked D. washed
Câu 4 (TH): A. rise B. advise C. raise D. practise
Mark the letter A, B, C, or D on your answer sheet to indicate the correct answer to each of the following
questions.
Câu 5 (TH): Because they fly, bats are often mistaken for birds; ________, bats are mammals, not birds.
A. otherwise B. as if C. in case D. however
Câu 6 (TH): ________ you plan to be walking after dark or not, it is a good idea to take a torch along
when you are hiking in rough terrain.
A. Provided B. In case C. Even if D. Whether
Câu 7 (TH): While ________the historical building, they had to get some of the missing tiles
specially______.
A. to restore - making B. restore - being made
C. restoring – made D. being restored - make
Câu 8 (NB): Some roads in Ho Chi Minh City don’t provide enough pavements for the
pedestrians________ on safely.
A. to be walking B. having walked C. to have walked D. to walk
Câu 9 (TH): If you ________ up before the journey, we wouldn’t have lost half an hour looking for a
petrol station in an unknown city.
A. had remembered filling B. had remembered to fill
C. remember to fill D. remembered filling
Câu 10 (NB): We can only win if we remain united, and so we must support them the moment they
_________ on strike.
A. will have gone B. will go C. go D. went
Câu 11 (NB): I have bought ________ new shirt which matches ________ jacket I bought last week.
A. some-a B. the-a C. a-the D. some-the
Câu 12 (TH): All the inhabitants in the area have been asked to _________ at home if the storm returns.
A. settle B. dwell C. remain D. occupy
Câu 13 (TH): This fund has been set up to help protect those________ of animals that are endangered
with complete extinction.

Trang 192
A. kinds B. species C. races D. breeds
Câu 14 (VD): In most countries, photocopying books without the publisher’s permission is clearly a
copyright ________.
A. interference B. interpretation C. infringement D. infliction
Câu 15 (TH): Marine reptiles are among the few creatures that are known to have a possible life span
greater than _______.
A. man B. the man’s C. the one of mans D. that of man
Câu 16 (VD): According to some, one way to cut ______ a school system’s budget with minimal impact
on the rest of the system is to do ______ the twelfth grade.
A. down- away with B. out- with C. back- out of D. off- by
Câu 17 (VDC): The car swerved to avoid a cyclist and just missed hitting a passer-by by _____.
A. a slight edge B. a narrow escape C. a close thing D. a hair’s breadth
Câu 18 (VDC): You’d better not place a bet on Stalion. In my opinion, the horse doesn’t ______ a chance
of winning the race.
A. win B. stand C. rise D. play
Mark the letter A, B, C, or D on your answer sheet to indicate the underlined part that needs correction in
each of the following questions.
Câu 19 (VD): Drawing on her own experience in psychology, the writer successfully portrayed a volatile
character with dramatic alternatives of mood.
A. in psychology B. portrayed C. character D. dramatic alternatives
Câu 20 (NB): In England, as early as the twelfth century, young boys enjoyed to play football.
A. In England B. as early as C. twelfth century D. to play
Câu 21 (NB): Some methods to prevent soil erosion are plowing parallel with the slopes of hills, to plant
trees on unproductive land, and rotating crops.
A. Some B. to prevent C. to plant D. rotating
Mark the letter A, B, C, or D on your answer sheet to indicate the word(s) CLOSEST in meaning to the
underlined word(s) in each of the following questions.
Câu 22 (TH): It is firmly believed that books are a primary means for disseminating knowledge and
information.
A. classifying B. distributing C. adopting D. inventing
Câu 23 (VD): Moving to a new town brought about many changes in his life.
A. prevented B. speeded up C. resulted in D. resulted from
Mark the letter A, B, C, or D on your answer sheet to indicate the word(s) OPPOSITE in meaning to the
underlined word(s) in each of the following questions.
Câu 24 (VD): She has always been honest with me, and I respect her for that.
A. look through B. look forward to C. look up to D. look down on

Trang 193
Câu 25 (VDC): With price increases on most necessities, many people have to tighten their belt for fear of
getting into financial difficulties.
A. dress in loose clothes B. spend money freely
C. save on daily expenses D. put on tighter belts
Mark the letter A, B, C, or D on your answer sheet to indicate the word that differs from the rest in the
position of the primary stress in each of the following questions.
Câu 26 (NB): A. president B. opponent C. assistant D. companion
Câu 27 (NB): A. accuse B. admire C. enter D. deny
Mark the letter A, B, C, or D on your answer sheet to indicate the sentence that best combines each pair
of sentences in the following questions.
Câu 28 (VDC): Laura practiced playing the instrument a lot. She could hardly improve her performance.
A. Hardly had Laura practiced playing the instrument a lot when she could improve her
performance.
B. Had Laura practiced playing the instrument a lot, she could have performed much better.
C. However much Laura practiced playing the instrument, she could hardly perform any better.
D. As soon as Laura practiced playing the instrument a lot, she could perform much better.
Câu 29 (VDC): Lucy paid a visit to the local orphanage. She then decided to donate part of her savings to
the children there.
A. Lucy had hardly decided to donate part of her savings to the children when she paid a visit to
the local orphanage.
B. It was only when Lucy had decided to donate part of her savings to the children that she paid a
visit to the local orphanage.
C. Before she paid a visit to the local orphanage, Lucy decided to donate part of her savings to the
children there.
D. Having paid a visit to the local orphanage, Lucy decided to donate part of her savings to the
children there.
Mark the letter A, B, C, or D on your answer sheet to indicate the sentence that is CLOSEST in meaning
to each of the following questions.
Câu 30 (TH): My friend has stopped eating fast food.
A. My friend sometimes eats fast food. B. My friend doesn’t like eating fast food.
C. My friend no longer eats fast food. D. My friend has never eaten fast food.
Câu 31 (VD): “You should take better care of your health” said Tom’s mother.
A. Tom’s mother promised to take better care of his health.
B. Tom’s mother ordered to take better care of his health.
C. Tom’s mother required to take better care of his health.
D. Tom’s mother advised to take better care of his health.

Trang 194
Câu 32 (VDC): They report that soil pollution has seriously threatened the livelihood of many local
farmers.
A. The livelihood of many local farmers was reported to be seriously endangered by soil pollution.
B. It is reported that the livelihood of many local farmers has led to serious soil pollution.
C. Soil pollution is reported to have posed a serious threat to the livelihood of many local farmers.
D. It has been reported that soil pollution has actually put many local farmers’ lives at risk.
Read the following passage and mark the letter A, B, C, or D on your answer sheet to indicate the correct
word or phrase that best fits each of the numbered blanks.
Global Warming
Few people now question the reality of global warming and its effects on the world’s climate. Many
scientists (33) ________the blame for recent natural disasters on the increase in the world’s temperatures
and are convinced that, more than ever before, the Earth is at risk from the forces of the wind, rain and
sun. (34) _________to them, global warming is making extreme weather events, such as hurricanes and
droughts, even more (35) ________and causing sea levels all around the world to rise.
Environmental groups are putting pressure on governments to take action to reduce the amount of carbon
dioxide which is given (36) _________by factories and power plants, thus attacking the problem at its
source. They are in favour of more money being spent on research into solar, wind and wave energy
devices, which could the replace existing power stations. Some scientists, (37) _______, believe that even
if we stopped releasing carbon dioxide and other gases into the atmosphere tomorrow, we would have to
wait several hundred years to notice the results. Global warming, it seems, is here to stay
Câu 33 (VD): A. give B. put C. take D. have
Câu 34 (TH): A. Concerning B. Regarding C. Depending D. According
Câu 35 (TH): A. strict B. severe C. strong D. healthy
Câu 36 (VD): A. off B. away C. up D. over
Câu 37 (TH): A. but B. although C. despite D. however
Read the passage and mark the letter A, B, C, or D on your answer sheet to indicate the correct answer to
each of the questions.
When we meet people for the first time, we often make decisions about them based entirely on how they
look. And of course, we too are being judged on our appearance. Undoubtedly, it’s what’s inside that’s
important but sometimes we can send out the wrong signals and so get a negative reaction, simply by
wearing inappropriate clothing.
When selecting your clothes each day, it is therefore important to think about who you’re likely to meet,
where you are going to be spending most of your time and what tasks you are likely to perform. Clearly,
on a practical level, some outfits will be more appropriate to different sorts of activity this will dictate
your choice to an extent. However, there’s no need to abandon your individual taste completely. After all,

Trang 195
if your dress to please somebody else’s idea of what looks good, you may end up feeling uncomfortable
and not quite yourself.
Some colors bring your natural coloring to life and others can give you a washed-out appearance. Try out
new ones by all means but remember that dressing in bright colors when you really subtle neutral tones or
vice versa will make you feel self-conscious and uncomfortable. You know deep down where your taste
boundaries lie. It may be fun to cross these sometimes, but do take care not to go too far all at once.
Reappraising your image isn’t selfish because everyone who comes into contact with you will benefit.
You’ll look better and you’ll feel better person all round. And if in doubt, you only need to read Professor
Albert Mehrabian’s book Silent Messages, which showed that the impact we make on each other depends
55 percent on how we look and behave, 38 percent on how we speak, and only seven percent on what we
actually say.
Câu 38 (VD): Which could be the best title for the passage?
A. Making Your Image Work For You
B. Choosing Approriate Business Suits
C. Making Judgements about People’s Appearance
D. Creating a Professional Image
Câu 39 (TH): According to paragraph 1, people can get a negative reaction from others by______.
A. wearing inappropriate clothes B. expressing too strong emotions
C. sending out right signals D. talking about other people’s behaviors
Câu 40 (TH): The word "outfits" in paragraph 2 mostly means________.
A. types of gestures B. sets of equipments C. sets of clothes D. types of signals
Câu 41 (NB): Which of the following is NOT mentioned in paragraph 2 as a factor to be considered when
choosing clothes?
A. Places you spend time in B. Kinds of tasks you perform
C. Other people’s views on beauty D. People you meet
Câu 42 (TH): According to Professor Albert Mehrabian, the impact we make on each other depends
mainly on _________.
A. how we look and behave B. what we read
C. what we actually say D. how we speak
Read the following passage and mark the letter A, B, C, or D on your answer sheet to indicate the correct
answer to each of the questions.
Glaciers are large masses of ice on land that show evidence of past or present movement. They grow by
the gradual transformation of snow into glacier ice. A fresh snowfall is a fluffy mass of loosely packed
snowflakes, small delicate ice crystals grown in the atmosphere. As the snow ages on the ground for
weeks or months, the crystals shrink and become more compact to form firm, a much whole mass
becomes squeezed together into a more dense form, granular snow. As new snow falls and buries the

Trang 196
older snow, the layers of granular snow further compact to form firm, a much denser kind of snow,
usually a year or more old, which has little pore space. Further burial and slow cementation- a process by
which crystals become bound together in a mosaic of intergrown ice crystals- finally produce solid glacial
ice. In this process of recrystallization, the growth of new crystals at the expense of old ones, the
percentage of air is reduced from about 90 percent for snowflakes to less than 20 percent for glacier ice.
The whole process may take as little as a few years, but more likely ten or twenty years or longer. The
snow is usually many meters deep by the time the lower layers are converted into ice.
In cold glaciers those formed in the coldest regions of the Earth, the entire mass of ice is at temperatures
below the melting point and no free water exists. In temperate glaciers, the ice is at the melting point at
every pressure level within the glaciers, and free water is present as small drops or as larger
accumulations, in tunnels within or beneath the ice. Formation of a glacier is complete when ice has
accumulated to a thickness (and thus weight) sufficient to make it move slowly under pressure, in much
the same way that solid rock deep within the Earth can change shape without breaking. Once that point is
reached, the ice flows downhill, either as a tongue of ice filling a valley or as thick ice cap that flows out
in directions from the highest central area where the most snow accumulates. The up down leads to the
eventual melting of ice.
Câu 43 (TH): Which of the following does the passage mainly discuss?
A. The effect of glaciers on climate B. Damage from glaciers
C. Glacier formation D. The location of glaciers
Câu 44 (TH): Which of the following will cause density within the glacier to increase?
A. Increased water and air content
B. Pressure from the weight of new snow
C. Long periods of darkness and temperature variations
D. Movement of the glacier
Câu 45 (TH): The word "bound" is closest in meaning to ________.
A. covered B. chosen C. planned D. held
Câu 46 (TH): Which of the following will be lost is a glacier forms?
A. Air B. Pressure C. Weight D. Rocks
Câu 47 (TH): According to the passage which of the following is the LEAST amount of time necessary
for glacial ice to form?
A. Several months B. Several years C. At least fifty years D. A century
Câu 48 (TH): The word "converted" is closest in meaning to ________.
A. changed B. delayed C. promoted D. dissolved
Câu 49 (TH): In temperate glaciers, where is water found?
A. Only near the surface B. In pools at various depths
C. In a thin layer below the firm D. In tunnels

Trang 197
Câu 50 (VD): It can be inferred from the last paragraph that a glacier ________.
A. can revert to a fluffy mass
B. maintains the same shape throughput the glacial process
C. is too cold to be thoroughly studied
D. can contribute water to lakes, rivers, or oceans

Trang 198
Đáp án
1-C 2-A 3-A 4-D 5-D 6-D 7-C 8-D 9-B 10-C
11-C 12-C 13-B 14-C 15-D 16-A 17-D 18-B 19-D 20-D
21-C 22-B 23-C 24-D 25-B 26-A 27-C 28-C 29-D 30-C
31-D 32-C 33-B 34-D 35-B 36-A 37-D 38-A 39-A 40-C
41-C 42-A 43-C 44-B 45-D 46-A 47-B 48-A 49-D 50-D

LỜI GIẢI CHI TIẾT


Câu 1: Đáp án C
Tạm dịch:
Jenny và Jimmy đang nói về giáo dục bậc đại học.
Jenny: “Tôi nghĩ một tấm bằng đại học là cách duy nhất để thành công trong cuộc sống.”
Jimmy: “_________. Có nhiều người thành công mà không có bằng."
A. Cuộc sống mà
B. Đúng vậy
C. Tôi khá không đồng ý
D. Tôi hoàn toàn đồng ý
Các phản hồi A, B, D không phù hợp ngữ cảnh.
Câu 2: Đáp án A
Kiến thức: Ngôn ngữ giao tiếp
Giải chi tiết:
Tạm dịch:
Laura đang kể cho Bob về kết quả kì thi của cô ấy.
Laura: “_________."
Bob: “Thật tuyệt. Chúc mừng bạn”
A. Tôi đã vượt qua kì thi với một điểm A.
B. Tôi hi vọng mình sẽ vượt qua kì thi ngày mai.
C. Tôi sẽ có kết quả thi vào ngày mai.
D. Tôi đã làm bài không tốt trong kì thi.
Các phản hồi B, C, D không phù hợp ngữ cảnh.
Câu 3: Đáp án A
Kiến thức: Phát âm “ed”
Giải chi tiết:
A. called /kɔːld/
B. passed /pɑːst/
C. talked /tɔːkt/

Trang 199
D. washed /wɒʃt/
Quy tắc:
Cách phát âm đuôi “-ed”:
- Đuôi “-ed” được phát âm là /ɪd/ khi động từ có phát âm kết thúc là /t/ hay /d/
- Đuôi “-ed” được phát âm là /t/ khi động từ có phát âm kết thúc là /s/,/f/,/p/,/ʃ/,/tʃ/,/k/
- Đuôi “-ed” được phát âm là /d/ với các trường hợp còn lại
Phần gạch chân phương án A được phát âm là /d/, còn lại là /t/
Câu 4: Đáp án D
Kiến thức: Phát âm “s”
Giải chi tiết:
A. rise /raɪz/
B. advise /ədˈvaɪz/
C. raise /reɪz/
D. practice /ˈpræktɪs/
Phần gạch chân phương án D được phát âm là /s/, còn lại là /z/
Câu 5: Đáp án D
Kiến thức: Liên từ
Giải chi tiết:
A. otherwise, S + V: nếu không thì
B. as if + S + V: như thể là
C. in case + S + V: phòng khi
D. ; however, S + V: tuy nhiên
Tạm dịch: Bởi vì chúng bay, dơi thường bị nhầm lẫn với chim; tuy nhiên, dơi là động vật có vú, không
phải chim.
Câu 6: Đáp án D
Kiến thức: Liên từ
Giải chi tiết:
A. Provided: Miễn là
B. In case: Phòng khi
C. Even if: Ngay cả khi
D. Whether: Liệu => Whether... or: Dù... hay...
Tạm dịch: Dù bạn có định đi bộ sau khi trời tối hay là không, vẫn sẽ tốt khi mang theo một cái đèn pin khi
bạn đi qua những địa hình gồ ghề.
Câu 7: Đáp án C
Kiến thức: Mệnh đề rút gọn, câu bị động với ‘get’
Giải chi tiết:

Trang 200
- Hai mệnh đề cùng chủ ngữ (they) => có thể rút gọn mệnh đề
Chủ ngữ “they”, động từ “restore” (khôi phục) => câu chủ động => rút gọn mệnh đề dùng V_ing
- get sth + P2: có cái gì được làm bởi ai
Tạm dịch: Trong khi khôi phục các tòa nhà lịch sử, họ đã phải đặc biệt làm xong những mặt hổng.
Câu 8: Đáp án D
Kiến thức: to V / V_ing
Giải chi tiết:
provide sth for sb: cung cấp cái gì cho ai
enough + N + to V_nguyên thể: đủ cái gì để làm gì
Tạm dịch: Vài con đường ở thành phố Hồ Chí Minh không cung cấp đủ vỉa hè dành cho người đi bộ để đi
lại một cách an toàn.
Câu 9: Đáp án B
Kiến thức: Câu điều kiện loại 3, to V/ V_ing
Giải chi tiết:
- Dấu hiệu: chứa “if”, mệnh đề chính “wouldn’t have lost”
- Cách dùng: Câu điều kiện loại 3 diễn tả một điều kiện không có thực trong quá khứ dẫn đến kết quả trái
với quá khứ.
- Công thức: If + S + had (not) + P2, S + would (not) + have + P2
remember + to V_nguyên thể: nhớ phải làm gì
remember + V_ing: nhớ đã làm gì (nhưng quên mất là mình đã làm)
Tạm dịch: Nếu bạn đã nhớ phải đổ đầy bình trước chuyến đi, chúng ta sẽ không mất hàng nửa giờ để tìm
một trạm xăng dầu ở một thành phố xa lạ.
Câu 10: Đáp án C
Kiến thức: Mệnh đề trạng ngữ chỉ thời gian
Giải chi tiết:
Sau “the moment” mệnh đề không chia thì tương lai.
Động từ “remain” chia hiện tại đơn => động từ mệnh đề trạng ngữ chia hiện tại đơn
Tạm dịch: Chúng ta chỉ có thể thắng nếu chúng ta giữ đoàn kết, và vì vậy chúng ta phải ủng hộ họ lúc mà
họ đình công.
Câu 11: Đáp án C
Kiến thức: Mạo từ
Giải chi tiết:
a/an + danh từ số ít, chưa xác định
the + danh từ đã được xác định rõ
“new shirt” chưa xác định, là danh từ số ít => dùng “a”

Trang 201
“jacket” đã được xác định xác định nhờ vào mệnh đề “I bought last week” (tôi đã mua tuần trước) =>
dùng “the”
Tạm dịch: Tôi đã vừa mua một cái áo mới, cái mà hợp với áo khoác tôi đã mua tuần trước.
Câu 12: Đáp án C
Kiến thức: Từ vựng
Giải chi tiết:
A. settle (v): định, đặt
B. dwell (v): trú, sống, ngụ, ở
C. remain (v): ở lại, ở nguyên chỗ trước đó
D. occupy (v): chiếm, giữ
Tạm dịch: Tất cả dân cư trong khu vực được yêu cầu ở yên trong nhà nếu cơn bão trở lại.
Câu 13: Đáp án B
Kiến thức: Từ vựng
Giải chi tiết:
A. kinds (n): loại
B. species (n): loài
C. races (n): chủng tộc
D. breeds (n): giống, chủng
Tạm dịch: Quỹ này được thành lập để giúp bảo vệ các loài động mà đang có nguy cơ tuyệt chủng hoàn
toàn.
Câu 14: Đáp án C
Kiến thức: Sự kết hợp từ
Giải chi tiết:
copyright infringement (n): vi phạm bản quyền
Các phương án khác:
A. interference (n): sự giao thoa, can thiệp
B. interpretation (n): sự giải thích, thuyết minh
D. infliction (n): sự tra khảo, hình phạt
Tạm dịch: Ở hầu hết các nước, việc sao in một cuốn sách mà không có sự cho phép của nhà xuất bản thì
rõ ràng là vi phạm bản quyền.
Câu 15: Đáp án D
Kiến thức: So sánh hơn
Giải chi tiết:
have + N1 + adj + er + than + N2: có cái gì …. hơn so với cái gì
“a possible life span” là đối tượng dùng để so sánh => thay thế bằng “that” tránh lặp lại
that of man = a possible life span of man: vòng đời của loài người

Trang 202
Tạm dịch: Nhiều loài bò sát biển là số ít các sinh vật mà được biết có một vòng đời sống lâu hơn vòng đời
của loài người.
Câu 16: Đáp án A
Kiến thức: Cụm động từ
Giải chi tiết:
A. down- away with => cut down: cắt giảm; do away with: xóa bỏ
B. out- with => cut out: ngưng hoạt động đột ngột (máy móc); do with: liên quan
C. back- out of => cut back: giảm, do out of: ngăn chặn (một cách gian lận)
D. off- by => cut off: ngừng cung cấp; do by: giải quyết
Tạm dịch: Theo một vài người, một cách để cắt giảm ngân sách hệ thống trường học mà ảnh hưởng nhỏ
với phần còn lại của hệ thống là xóa bỏ khối lớp 12.
Câu 17: Đáp án D
Kiến thức: Thành ngữ
Giải chi tiết:
A. a slight edge: ít lợi thế
B. a narrow escape => have a narrow escape: trong gang tấc
C. a close thing: sát nút
D. a hair’s breadth => by a hair’s breadth: trong gang tấc
Tạm dịch: Xe ô tô quẹo tay lái để tránh một người đi xe đạp và vừa lỡ đâm vào một người qua đường chỉ
trong gang tấc.
Câu 18: Đáp án B
Kiến thức: Sự kết hợp từ
Giải chi tiết:
stand a chance of sth: có cơ hội thành công
Tạm dịch: Bạn tốt hơn không nên đặt cược vào con Stalion. Tôi nghĩ, con ngựa đó không có cơ hội chiến
thắng cuộc đua.
Câu 19: Đáp án D
Kiến thức: Từ vựng
Giải chi tiết:
alternatives (n): sự thay thế
alternations of mood: cảm xúc thất thường
Sửa: “dramatic alternatives” => “dramatic alternations”
Tạm dịch: Bằng những trải nghiệm của chính mình trong tâm lí học, nhà văn đã khắc họa một nhân vật
nhẹ dạ với sự biến hóa cảm xúc kịch tính.
Câu 20: Đáp án D
Kiến thức: to V/ V_ing

Trang 203
Giải chi tiết:
enjoy + V_ing: thích làm gì
Sửa: “to play” => “playing”
Tạm dịch: Ở Anh, đầu thế kỉ 12, các cậu bé đã thích chơi bóng đá.
Câu 21: Đáp án C
Kiến thức: Cấu trúc song hành
Giải chi tiết:
Liên từ “and” kết nối các từ cùng tính chất/cùng loại từ/cùng dạng
“plowing”, “rotating” đang ở ở dạng V_ing
Sửa: “to plant” => “planting”
Tạm dịch: Một số phương pháp chống xói mòn đất là cày song song với sườn đồi, trồng cây trên đất kém
năng suất và luân canh.
Câu 22: Đáp án B
Kiến thức: Từ vựng, từ đồng nghĩa
Giải chi tiết:
disseminate (v): lan truyền, chia sẻ => disseminating (V_ing)
A. classifying (v): phân loại
B. distributing (v): phân phối
C. adopting(v): chấp nhận
D. inventing (v): phát minh
=> disseminating = distributing: phân phối, lan truyền
Tạm dịch: Được tin chắc rằng sách là một phương tiện để lan truyền kiến thức và thông tin.
Câu 23: Đáp án C
Kiến thức: Cụm động từ
Giải chi tiết:
bring about (v): dẫn đến => brought about (V_ed)
A. prevented (v): ngăn chặn
B. speeded up (v): thúc đẩy
C. resulted in (v): dẫn đến, gây ra
D. resulted from (v): do
=> brought about = resulted in: dẫn đến, gây ra
Tạm dịch: Chuyển đến một thị trấn mới đã gây ra nhiều thay đổi trong cuộc sống của anh ấy.
Câu 24: Đáp án D
Kiến thức: Cụm động từ, từ trái nghĩa
Giải chi tiết:
respect (v): tôn trọng

Trang 204
A. look through: xem xét kĩ lưỡng
B. look forward to: trông chờ
C. look up to: kính trọng
D. look down on: coi thường
=> respect >< look down on
Tạm dịch: Cô ấy luôn luôn thành thật với tôi, và tôi tôn trọng cô ấy vì điều đó.
Câu 25: Đáp án B
Kiến thức: Thành ngữ, từ trái nghĩa
Giải chi tiết:
tighten one’s belt: chi tiêu tiết kiệm, thắt lưng buộc bụng
A. dress in loose clothes: mặc quần áo rộng
B. spend money freely: tiêu tiền thoải mái
C. save on daily expenses: tiết kiệm chi tiêu hàng ngày
D. put on tighter belts: đeo dây cài chặt hơn
=> tighten their belt >< spend money freely
Tạm dịch: Với việc tăng giá trên hầu hết các mặt hàng thiết yếu, nhiều người phải chi tiêu tiết kiệm vì sợ
lâm vào tình trạng khó khăn về tài chính.
Câu 26: Đáp án A
Kiến thức: Trọng âm của từ có 3 âm tiết
Giải chi tiết:
A. president /ˈprezɪdənt/
B. opponent /əˈpəʊnənt/
C. assistant /əˈsɪstənt/
D. companion /kəmˈpænjən/
Phương án A trọng âm rơi vào âm tiết thứ nhất, còn lại là âm thứ 2
Câu 27: Đáp án C
Kiến thức: Trọng âm của từ có 2 âm tiết
Giải chi tiết:
A. accuse /əˈkjuːz/
B. admire /ədˈmaɪə(r)/
C. enter /ˈentə(r)/
D. deny /dɪˈnaɪ/
Phương án C trọng âm rơi vào âm tiết thứ nhất, còn lại là âm thứ 2
Câu 28: Đáp án C
Kiến thức: Đảo ngữ, liên từ
Giải chi tiết:

Trang 205
Hardly + had + S + P2 when + S + V_ed: Ngay sau khi ai làm gì thì làm gì
Had + S + P2, S + V_ed: Việc gì xảy ra trước, việc xảy ra sau
However + adj/ adv + S + be/ V, S + V: Dù làm gì như thế nào, cũng không làm gì được
As soon as + S+ V…: Ngay sau khi…
Tạm dịch: Laura đã luyện tập chơi nhạc cụ rất nhiều. Cô ấy hầu như không thể cải thiện trình diễn của
mình.
A. Ngay sau khi Laura đã luyện tập chơi nhạc cụ rất nhiều thì cô ấy đã có thể cải thiện trình diễn của
mình. => sai nghĩa
B. Đã luyện tập chơi nhạc cụ rất nhiều, cô ấy đã có thể cải thiện trình diễn của mình. => sai nghĩa
C. Dù luyện tập chơi nhạc cụ rất nhiều, cô ấy hầu như không thể trình diễn tốt hơn.
D. Ngay sau khi Laura đã luyện tập chơi nhạc cụ rất nhiều, cô ấy đã có thể trình diễn tốt hơn nhiều. => sai
nghĩa
Câu 29: Đáp án D
Kiến thức: Liên từ
Giải chi tiết:
hardly...when: ngay sau khi
only when... that: chỉ khi... thì
Before: Trước khi
Having + S + P2, S + V_ed: Ai đó làm gì trước, sau đó thì làm gì
Tạm dịch: Lucy đã đến thăm một trại trẻ mồ côi của địa phương. Sau đó cô ấy đã quyết định quyên góp
một phần tiền tiết kiệm của mình cho những đứa trẻ ở đó.
A. Ngay sau khi Lucy quyết định quyên góp một phần tiền tiết kiệm của mình cho những đứa trẻ, cô ấy
đến thăm trại trẻ mồ côi địa phương. => sai nghĩa
B. Chỉ khi Lucy đã quyết định quyên góp một phần tiền tiết kiệm của mình cho những đứa trẻ, cô ấy đến
thăm trại trẻ mồ côi địa phương. => sai nghĩa
C. Trước khi cô ấy đến thăm trại trẻ mồ côi địa phương, Lucy đã quyết định quyên góp một phần tiền tiết
kiệm của mình cho những đứa trẻ ở đó. => sai nghĩa
D. Đã đến thăm một trại trẻ mồ côi của địa phương, Lucy quyết định quyên góp một phần tiền tiết kiệm
của mình cho những đứa trẻ ở đó.
Câu 30: Đáp án C
Kiến thức: Câu đồng nghĩa
Giải chi tiết:
stop + V_ing: ngừng hẳn làm việc gì (sau đó không còn tiếp tục nữa)
= no longer + V: không còn làm gì nữa
Tạm dịch: Bạn của tôi đã dừng ăn thức ăn nhanh.
A. Bạn của tôi thỉnh thoảng ăn thức ăn nhanh. => sai nghĩa

Trang 206
B. Bạn của tôi không thích ăn thức ăn nhanh. => sai nghĩa
C. Bạn của tôi không còn ăn thức ăn nhanh nữa.
D. Bạn của tôi chưa từng ăn thức ăn nhanh. => sai nghĩa
Câu 31: Đáp án D
Kiến thức: Lời nói gián tiếp
Giải chi tiết:
“S + should + V_nguyên thể”: Ai đó nên làm gì
= S + advised + + O + to V_nguyên thể: Ai đó đã khuyên nên làm gì
Tạm dịch:
“Con nên chăm sóc sức khỏe của mình tốt hơn” mẹ của Tom đã nói.
A. Mẹ Tom đã hứa sẽ chăm sóc tốt hơn cho sức khỏe của cậu ấy. => sai nghĩa
B. Mẹ Tom đã yêu cầu Tom chăm sóc tốt hơn cho sức khỏe của cậu ấy. => sai nghĩa
C. Mẹ Tom đã đòi hỏi Tom phải chăm sóc tốt hơn cho sức khỏe của cậu ấy. => sai nghĩa
D. Mẹ Tom đã khuyên Tom chăm sóc tốt hơn cho sức khỏe của cậu ấy.
Câu 32: Đáp án C
Kiến thức: Câu bị động kép
Giải chi tiết:
S1 + report + that + S2 + V2…
= It is reported that + S2 + V2…
= S2 + is/ are/ am + reported + to + V_nguyên thể
Tạm dịch: Họ báo cáo rằng ô nhiễm đất đã đe dọa nghiêm trọng đến đời sống của nhiều nông dân địa
phương.
A. Sai thì. Sửa “was” => “is”
B. Được báo cáo rằng cuộc sống của nhiều nông dân địa phương đã dẫn đến ô nhiễm đất nghiêm trọng.
=> sai nghĩa
C. Ô nhiễm đất được báo cáo là gây đe dọa nghiêm trọng đến đời sống của nhiều nông dân địa phương.
D. Sai thì. Sửa “has been” => “is”
Câu 33: Đáp án B
Kiến thức: Sự kết hợp từ
Giải chi tiết:
put the blame for sth: đổ lỗi cho cái gì
Many scientists (33) put the blame for recent natural disasters on the increase in the world’s
temperatures…
Tạm dịch: Nhiều nhà khoa học đổ lỗi cho các thảm họa tự nhiên gần đây về sự gia tăng nhiệt độ của toàn
cầu...
Câu 34: Đáp án D

Trang 207
Kiến thức: Từ vựng
Giải chi tiết:
Kiến thức: Từ vựng
Giải thích:
A. Concerning sth: Về cái gì đó
B. Regarding sth…: Xét về/theo…
C. Depending on … : Theo như …
D. According to…: Theo như…
(34) According to them, global warming is making extreme weather events...
Tạm dịch: Theo họ, hiện tượng nóng lên toàn cầu đang làm cho các sự kiện thời tiết cực đoan...
Câu 35: Đáp án B
Kiến thức: Từ vựng
Giải chi tiết:
A. strict (adj): nghiêm khắc
B. severe (adj): khắc nghiệt
C. strong (adj): mạnh mẽ
D. healthy (adj): khỏe mạnh
...global warming is making extreme weather events, such as hurricanes and droughts, even more (35)
severe and causing sea levels all around the world to rise.
Tạm dịch: ...sự nóng lên toàn cầu đang làm cho các sự kiện thời tiết cực đoan, như bão và hạn hán thậm
chí khắc nghiệt hơn và khiến mực nước biển trên toàn thế giới tăng lên.
Câu 36: Đáp án A
Kiến thức: Cụm động từ
Giải chi tiết:
A. off => give off: tỏa ra, thải ra
B. away => give away: tiết lộ
C. up => give up: từ bỏ
D. over => give over: ngừng làm gì
Environmental groups are putting pressure on governments to take action to reduce the amount of carbon
dioxide which is given (36) off by factories and power plants, thus attacking the problem at its source.
Tạm dịch: Các nhóm môi trường đang gây áp lực buộc các chính phủ phải hành động để giảm lượng CO2
được thải ra từ các nhà máy và nhà máy điện, do đó nhìn nhận vấn đề tại nguồn gốc của nó.
Câu 37: Đáp án D
Kiến thức: Liên từ
Giải chi tiết:
A. but + S + V: nhưng

Trang 208
B. although + S + V: mặc dù
C. despite + N phr. / V_ing: mặc dù
D. , however, : tuy nhiên
Some scientists, (37) however, believe that even if we stopped releasing carbon dioxide and other gases
into the atmosphere tomorrow, we would have to wait several hundred years to notice the results.
Tạm dịch: Tuy nhiên, một số nhà khoa học, tin rằng ngay cả khi chúng ta ngừng thải CO2 và các loại khí
khác vào khí quyển trong tương lai, chúng ta vẫn sẽ phải chờ vài trăm năm để nhận thấy kết quả.
Câu 38: Đáp án A
Kiến thức: Đọc hiểu
Giải chi tiết:
Đâu có thể là tiêu đề phù hợp nhất cho đoạn văn?
A. Tạo ra hình ảnh phù hợp với bản thân
B. Chọn trang phục công sở phù hợp
C. Đánh giá về diện mạo của người khác
D. Tạo một hình ảnh chuyên nghiệp
Câu 39: Đáp án A
Kiến thức: Đọc hiểu
Giải chi tiết:
Theo đoạn văn 1, mọi người có thể nhận phản ứng tiêu cực từ người khác bởi_______.
A. mặc trang phục không phù hợp
B. bày tỏ cảm xúc quá mãnh liệt
C. phát ra những tín hiệu đúng
D. nói về thái độ của người khác
Thông tin: Undoubtedly, it’s what’s inside that’s important but sometimes we can send out the wrong
signals and so get a negative reaction, simply by wearing inappropriate clothing.
Tạm dịch: Không còn nghi ngờ gì nữa, có rất nhiều thứ mà bên trong đó rất quan trọng nhưng đôi khi
chúng ta có thể phát ra những tín hiệu sai và do đó nhận được phản ứng tiêu cực, đơn giản bằng việc mặc
quần áo không phù hợp.
Câu 40: Đáp án C
Kiến thức: Đọc hiểu
Giải chi tiết:
Từ “outfits” trong đoạn 2 gần nghĩa với_________.
A. những kiểu cử chỉ
B. những bộ dụng cụ
C. những bộ quần áo, trang phục
D. các loại tín hiệu

Trang 209
Thông tin: Clearly, on a practical level, some outfits will be more appropriate to different sorts of activity
this will dictate your choice to an extent.
Tạm dịch: Rõ ràng, ở mức độ thực tế, một số trang phục sẽ phù hợp hơn với các loại hoạt động khác
nhau, điều này sẽ quyết định sự lựa chọn của bạn đến một mức độ nào đó.
Câu 41: Đáp án C
Kiến thức: Đọc hiểu
Giải chi tiết:
Điều nào dưới đây không được đề cập trong đoạn 2 như là một yếu tố được cân nhắc khi chọn quần áo?
A. Nơi mà bạn dành thời gian (bạn ở đó)
B. Kiểu nhiệm vụ bạn thực hiện
C. Quan điểm của người khác về cái đẹp
D. Người mà bạn gặp
Thông tin: When selecting your clothes each day, it is therefore important to think about who you’re
likely to meet, where you are going to be spending most of your time and what tasks you are likely to
perform.
Tạm dịch: Khi chọn quần áo mỗi ngày, điều quan trọng là bạn phải suy nghĩ về việc bạn sẽ gặp ai, nơi
bạn sẽ dành phần lớn thời gian ở đó và những nhiệm vụ bạn có thể sẽ thực hiện.
Các phương án A, B, D đã được đề cập.
Câu 42: Đáp án A
Kiến thức: Đọc hiểu
Giải chi tiết:
Theo giáo sư Albert Mehrabian, tác động mà chúng ta đem lại cho người khác chủ yếu là _______.
A. cách chúng ta nhìn nhận và cư xử
B. điều chúng ta đọc
C. điều chúng ta thực sự nói
D. cách chúng ta nói
Thông tin: And if in doubt, you only need to read Professor Albert Mehrabian’s book Silent Messages,
which showed that the impact we make on each other depends 55 percent on how we look and behave, 38
percent on how we speak, and only seven percent on what we actually say.
Tạm dịch: Và nếu nghi ngờ, bạn chỉ cần đọc cuốn sách Tin nhắn im lặng của giáo sư Albert Mehrabian,
sách mà đã cho thấy tác động mà chúng ta gây ra cho nhau phụ thuộc 55% vào cách chúng ta nhìn và cư
xử, 38% về cách chúng ta nói và chỉ 7% về những gì chúng ta thực sự nói.
Câu 43: Đáp án C
Kiến thức: Đọc hiểu
Giải chi tiết:
Đoạn văn chủ yếu bàn luận điều nào dưới đây?

Trang 210
A. Ảnh hưởng của sông băng đến khí hậu
B. Sự phá hủy từ sông băng
C. Sự hình thành sông băng
D. Vị trí của sông băng
Câu 44: Đáp án B
Kiến thức: Đọc hiểu
Giải chi tiết:
Cái nào dưới đây sẽ khiến sự đặc trong sông băng tăng lên?
A. Lượng nước và không khí tăng
B. Áp lực từ sức nặng của tuyết mới
C. Khoảng thời gian dài của bóng tối và sự thay đổi nhiệt độ
D. Sự chuyển động của sông băng
Thông tin: As new snow falls and buries the older snow, the layers of granular snow further compact to
form firm, a much denser kind of snow...
Tạm dịch: Khi tuyết mới rơi và chôn vùi tuyết cũ, các lớp tuyết dạng hạt nhỏ gọn hơn để tạo thành một
khối tuyết đặc hơn...
Câu 45: Đáp án D
Kiến thức: Đọc hiểu
Giải chi tiết:
Từ “bound” gần nghĩa nhất với _______.
A. được che phủ
B. được chọn
C. được sắp xếp
D. được giữ
=> bound = held: giữ lại, ràng buộc
Thông tin: Further burial and slow cementation- a process by which crystals become bound together in a
mosaic of intergrown ice crystals- finally produce solid glacial ice.
Tạm dịch: Chôn chặt hơn và sự đông cứng chậm - một quá trình mà các tinh thể liên kết với nhau trong
một khối khảm gồm các tinh thể băng xen kẽ - cuối cùng tạo ra băng rắn.
Câu 46: Đáp án A
Kiến thức: Đọc hiểu
Giải chi tiết:
Cái nào dưới đây sẽ bị mất đi là sông băng hình thành?
A. Không khí
B. Áp suất
C. Trọng lượng

Trang 211
D. Đá
Thông tin: In this process of recrystallization, the growth of new crystals at the expense of old ones, the
percentage of air is reduced from about 90 percent for snowflakes to less than 20 percent for glacier ice.
Tạm dịch: Trong quá trình kết tinh lại này, sự phát triển của các tinh thể mới gây hại cho các tinh thể cũ,
phần trăm không khí bị giảm từ khoảng 90% với những bông tuyết xuống dưới 20% với băng hà.
Câu 47: Đáp án B
Kiến thức: Đọc hiểu
Giải chi tiết:
Theo bài văn, đâu là lượng thời gian ít nhất cần cho một băng hà hình thành?
A. Vài tháng
B. Vài năm
C. Ít nhất 50 năm
D. Một thế kỉ
Thông tin: The whole process may take as little as a few years, but more likely ten or twenty years or
longer.
Tạm dịch: Toàn bộ quá trình có thể mất ít nhất là vài năm, nhưng nhiều khả năng là mười hoặc hai mươi
năm hoặc lâu hơn.
Câu 48: Đáp án A
Kiến thức: Đọc hiểu
Giải chi tiết:
Từ “converted” gần nghĩa nhất với _______.
converted (v): chuyển đổi, biến đổi
A. changed (v): thay đổi
B. delayed (v): trì hoãn
C. promoted (v): thúc đẩy
D. dissolved (v): phân hủy
=> converted = changed: biến đổi
Câu 49: Đáp án D
Kiến thức: Đọc hiểu
Giải chi tiết:
Ở các sông băng ôn đới, nước được tìm thấy ở đâu?
A. Chỉ ở gần bề mặt
B. Trong các hồ ở độ sâu khác nhau
C. Trong một lớp dày bên dưới tảng cứng
D. Trong các hầm

Trang 212
Thông tin: In temperate glaciers, the ice is at the melting point at every pressure level within the glaciers,
and free water is present as small drops or as larger accumulations, in tunnels within or beneath the ice.
Tạm dịch: Ở các sông băng ôn đới, băng ở điểm nóng chảy ở mọi mức áp suất trong sông băng, và nước
tự do xuất hiện dưới dạng giọt nhỏ hoặc sự tích tụ lớn hơn, trong các hầm bên trong hoặc bên dưới băng.
Câu 50: Đáp án D
Kiến thức: Đọc hiểu
Giải chi tiết:
Có thể suy ra từ đoạn cuối rằng một sông băng _________.
A. có thể trở lại thành một khối lỏng lẻo
B. duy trì hình dạng như nhau trong suốt quá trình sông băng
C. quá lạnh đến nỗi không được nghiên cứu kĩ lưỡng
D. có thể góp nước vào các hồ, sông và đại dương
Thông tin: Once that point is reached, the ice flows downhill, either as a tongue of ice filling a valley or
as thick ice cap that flows out in directions from the highest central area where the most snow
accumulates.
Tạm dịch: Khi đạt đến điểm đó, băng chảy xuống dốc, như một lưỡi băng lấp đầy thung lũng hoặc như
tảng băng dày chảy theo hướng từ khu vực trung tâm cao nhất nơi mà có nhiều tuyết nhất tích tụ.

SỞ GD & ĐT HẢI PHÒNG ĐỀ THI THỬ THPT QUỐC GIA LẦN 1


TRƯỜNG THPT CHUYÊN TRẦN NĂM HỌC: 2019 – 2020
PHÚ MÔN: TIẾNG ANH
Mã đề 113 Thời gian làm bài: 60 phút; không kể thời gian phát đề

Mark the letter A, B, C or D on your answer sheet to indicate the word(s) CLOSEST in meaning to the
underlined word(s) in each of the following questions.
Câu 1 (TH): Don’t sign your name on any document you have not had time to scrutinize!
A. spend B. read C. examine D. spare
Câu 2 (TH): The use of loud sounds to scare off predators serves as a viable strategy for survival.
A. attract B. locate C. terrify D. disappoint
Mark the letter A, B, C or D on your answer sheet to indicate the underlined part that needs correction in
each of the following questions.
Câu 3 (VD): I’ve given talks so many times that now I just make up them as I go along.
A. so many B. make up them C. as D. along
Câu 4 (VD): The problems of traffic congestion and pollution will continue to grow if the government
does not make any measures to reduce the negative effects of urbanization.
A. traffic congestion B. to grow C. make any measures D. effects

Trang 213
Câu 5 (TH): It was on a beautiful day in November when she accepted his proposal of marriage.
A. was B. in C. when D. accepted
Read the following passage and mark the letter A, B, C or D on your answer sheet to indicate the correct
answer to each of the questions from 6 to 10.
Almon Strowger, an American engineer, constructed the first automatic telephone switching system,
which had a horizontal, bladelike contact arm, in 1891. The first commercial switchboard based on his
invention opened in La Porte, Indiana, a year later and was an instant success with business users. To
access the system, the caller pressed button to reach the desired number and turned the handle to activate
the telephone ringer. During the same year, Strowger’s step-by-step call advancement technology was
implemented in the long-distance service between New York and Chicago when it proved to have the
capacity of carrying signals through cable-joint extensions.
The first actual dial telephone, patented by Lee De Forest in 1907, was installed in Milwaukee in 1906. In
1912, their sound transmittal apparatus adapted an electronic tube to function as an amplifier.
Transatlantic radio-telephone service linked New York and London in 1927. However, the long distances
coaxial cable, which was hailed as unprecedented, came on the scene in 1936 connecting New York and
Philadelphia. The Bell Laboratories research facility came up with the transistor to replace the
cumbersome vacuum tube, thus diminishing the size of the electronic switch system to about 10 percent
of that of the original. Crossbar switching, installed in terminals in 1938, operated on the principle of an
electromagnetic force, which rotated horizontal and vertical bars within a rectangular frame and brought
contacts together in a split second. A technological breakthrough in the form of undersea cables between
the United States and Hawaii was implemented almost twenty years later. An extension was connected to
Japan in 1964.
Câu 6 (VD): Which of the following would be the best title for the passage?
A. The Patent History of the Telephone
B. A link between Research and Technology
C. The Developing Sophistication of the Telephone
D. The Telephone: A Technological Fantasy
Câu 7 (VD): It can be inferred from the passage that initially telephones ________.
A. were limited to businesses B. did not have a bell
C. utilized human operators D. revitalized business in La Porte, Indiana
Câu 8 (TH): The word “implemented” in paragraph 1 is closest in meaning to _______.
A. used B. breached C. broken D. usurped
Câu 9 (NB): The word “that” in paragraph 2 refers to _________.
A. the system B. the tube C. the size D. the percent
Câu 10 (VD): The author of the passage implies that telephone networks expanded because of _______.
A. the work of a few inventors B. staunch public and private support

Trang 214
C. multiple technical blunders D. a series of breakthroughs
Mark the letter A, B, C, or D on your answer sheet to indicate the word that differs from te other three in
the position of primary stress in each of the following questions.
Câu 11 (NB): A. selfish B. famine C. police D. target
Câu 12 (NB): A. ancestor B. transistor C. equator D. compressor
Mark the letter A, B, C or D on your answer sheets to indicate the word whose underlined part differs
from the other three in pronunciation in each of the following questions.
Câu 13 (NB): A. continued B. increased C. diseased D. lightened
Câu 14 (NB): A. reason B. pleasure C. treasure D. endeavour
Mark the letter A, B, C or D on your answer sheet to indicate the correct answer to each of the following
questions.
Câu 15 (TH): You will have to go for an interview tomorrow, but don’t worry. It’s just a ______.
A. form B. formality C. formation D. format
Câu 16 (TH): _______ where he tried to hide his favourite jacket, Jethreo’s brother always managed to
find it.
A. Much as B. No matter C. Even though D. However
Câu 17 (TH): Sam thought that he could ________ across the water from the ferry to the quay, but he was
wrong and he fell in.
A. leap B. rise C. drop D. climb
Câu 18 (TH): I can’t give you a detailed description of the woman, as I only _______ at her briefly.
A. gazed B. glared C. stared D. glanced
Câu 19 (TH): This cheese isn’t fit for eating. It’s ________ all over after lying in the bin for so long.
A. moldy B. rusty C. spoiled D. sour
Câu 20 (TH): Paul asked Maria to ______ him to the dentist’s, because he didn’t want to go by himself.
A. interfere B. join C. unify D. accompany
Câu 21 (VDC): Mr. Jones knew who had won the contest, but he kept it under his _______ until it was
announced publicly.
A. umbrella B. cap C. hat D. tongue
Câu 22 (VD): Olivia is still the only student ________ the first prize four times for the best design.
A. to be awarding B. being awarded
C. to have been awarded D. to have awarded
Câu 23 (TH): When ________ to petrol, lead improves the car’s performance.
A. added B. adding C. it adds D. is added
Câu 24 (NB): “I called the police but by the time they arrived, the two men _________”, said the witness.
A. were disappearing B. have disappeared C. disappeared D. had disappeared

Trang 215
Câu 25 (NB): Broadly speaking, interpretations can be classified _______ two groups: judgements and
opinions.
A. into B. over C. in D. of
Câu 26 (NB): James visited _________ State Hermitage Museum, ______ famous tourist attraction in St.
Petersburg.
A. x – the B. x – a C. a – the D. the – a
Câu 27 (TH): ________, Carla stayed with her roommate’s family during the spring break.
A. Upon she’s away in college B. As soon as she’s away in college
C. While away in college D. Meanwhile being away in college
Câu 28 (NB): Even if I had known her address, I _______ it to him.
A. wouldn’t give B. hadn’t given C. didn’t give D. wouldn’t have given
Read the following passage and mark the letter A, B, C or D on your answer sheet to indicate the correct
answer to each of questions from 29 to 36.
Every day, millions of shoppers hit the stores in full force - both online and on foot - searching frantically
for the perfect gift. Last year, Americans spent over $30 billion at retail stores in the month of December
alone. Aside from purchasing holiday gifts, most people regularly buy presents for other occasions
throughout the year, including weddings, birthdays, anniversaries, graduations, and baby showers. This
frequent experience of gift-giving can engender ambivalent feelings in gift-givers. Many relish the
opportunity to buy presents because gift-giving offers a powerful means to build stronger bonds with
one’s closest peers. At the same time, many dread the thought of buying gifts; they worry that their
purchases will disappoint rather than delight the intended recipients.
Anthropologists describe gift-giving as a positive social process, serving various political, religious, and
psychological functions. Economists, however, offer a less favorable view. According to Waldfogel
(1993), gift-giving represents an objective waste of resources. People buy gifts that recipients would not
choose to buy on their own, or at least not spend as much money to purchase (a phenomenon referred to
as ‘‘the deadweight loss of Christmas”). To wit, givers are likely to spend $100 to purchase a gift that
receivers would spend only $80 to buy themselves. This ‘‘deadweight loss” suggests that gift-givers are
not very good at predicting what gifts others will appreciate. That in itself is not surprising to social
psychologists. Research has found that people often struggle to take account of others’ perspectives - their
insights are subject to egocentrism, social projection, and multiple attribution errors.
What is surprising is that gift-givers have considerable experience acting as both gift-givers and gift-
recipients, but nevertheless tend to overspend each time they set out to purchase a meaningful gift. In the
present research, we propose a unique psychological explanation for this - that is, that gift-givers equate
how much they spend with how much recipients will appreciate the gift (the more expensive the gift, the
stronger a gift-recipient’s feelings of appreciation). Although a link between gift price and feelings of
appreciation might seem intuitive to gift-givers, such an assumption may be unfounded. Indeed, we

Trang 216
propose that gift-recipients will be less inclined to base their feelings of appreciation on the magnitude of
a gift than givers assume.
Why do gift-givers assume that gift price is closely linked to gift-recipients’ feelings of appreciation?
Perhaps givers believe that bigger (that is, more expensive) gifts convey stronger signals of
thoughtfulness and consideration. According to Camerer (1988) and others, gift-giving represents a
symbolic ritual, whereby gift-givers attempt to signal their positive attitudes toward the intended recipient
and their willingness to invest resources in a future relationship. In this sense, gift-givers may be
motivated to spend more money on a gift in order to send a “stronger signal” to their intended recipient.
As for gift-recipients, they may not construe smaller and larger gifts as representing smaller and larger
signals of thoughtfulness and consideration.
Câu 29 (VD): What is the main idea discussed in the passage?
A. Gift-giving, despite its uneconomical downsides, cultivates a positive social process.
B. Gifts can serve as implicit signals of thoughtfulness and consideration.
C. Gift-giving may have certain drawbacks alongside its positive qualities.
D. Gift-recipients are widely acknowledged as considerably experienced in gift-giving.
Câu 30 (VD): In paragraph 1, the word “ambivalent” is closest in meaning to _____.
A. unrealistic B. conflicted C. apprehensive D. supportive
Câu 31 (TH): The author indicates that people value gift-giving because they feel it _______.
A. functions as a form of self-expression B. is an inexpensive way to show appreciation
C. requires the gift-recipient to reciprocate D. can serve to strengthen a relationship
Câu 32 (NB): The word “this” in paragraph 3 refers to gift-givers’ ________.
A. struggling B. overspending
C. buying meaningful gifts D. having considerable experience
Câu 33 (TH): The passage indicates that the assumption made by gift-givers in paragraph 3 may be
______.
A. incorrect B. unreasonable C. insincere D. substantiated
Câu 34 (TH): The word “convey” in paragraph 4 is closest in meaning to ______.
A. transport B. counteract C. communicate D. exchange
Câu 35 (VD): The authors refer to work by Camerer and others (paragraph 4) in order to _______.
A. offer an explanation B. introduce an argument
C. support a conclusion D. question a movie
Câu 36 (VD): Which of the following best describes the tone of the author?
A. criticizing B. pessimistic C. informative D. ironic
Mark the letter A, B, C or D on your answer sheet to indicate the word(s) OPPOSITE in meaning to the
underlined word(s) in each of the following questions.

Trang 217
Câu 37 (TH): Because of their microscopic size, particles can easily penetrate deep into the respiratory
system, causing serious health problems.
A. strange B. tiny C. giant D. queer
Câu 38 (VDC): The new sweater collection is so popular with teenagers all over the world that the
manufacturer is making money hand over fist.
A. making a lot of profit at a very fast pace B. making only a little money
C. making more sweaters in the future D. making more hand products
Mark the letter A, B, C or D on your answer sheet to indicate the sentence that best combines each pair of
sentences in the following questions.
Câu 39 (VDC): We couldn’t get a babysitter. That’s why we couldn’t join you at the restaurant.
A. If we could get a babysitter, we would have joined you at the restaurant.
B. We could have joined you at the restaurant, but we couldn’t get a babysitter.
C. The reason for our absence at the restaurant was not that we could get a babysitter.
D. We couldn’t get a babysitter so that we couldn’t join you at the restaurant.
Câu 40 (VDC): The president failed to explain the cause of the crisis. He did not offer any solutions.
A. The president couldn’t explain the cause of the crisis, nor did he offer any solutions.
B. The president was not able to explain the cause of the crisis as well as any solutions.
C. The president’s solution was not offered because he couldn’t give any explanation for the
crisis.
D. The president’s explanation for the crisis was such a failure that he couldn’t offer any
solutions.
Mark the letter A, B, C or D on your answer sheet to indicate the correct word or phrase that best fits the
blank space in the following passage from 41 to 45.
Paparazzi are freelance photographers (41) ______ take photographs of celebrities. Their actions are
sometimes criticized because they often go to extreme extents to get unusual shots. At times, their actions
may actually be (42) _______, and in some countries they may be prosecuted. Wherever celebrities and
stars are found, paparazzi are not usually far behind.
The term paparazzi comes from the name of the character in the Italian film La Dolce Vita. Signo
Paparazzi is a photographer who is (43) _______ looking for his next photo opportunity and, at one point,
photographs a woman who has recently (44) ______ a personal tragedy. His tactics are remarkably
similar to those used by real-life celebrity photographers today, which is why they came to be known as
paparazzi. A single representative of this intriguing business is known as a paparazzo, although it is rare
to see a paparazzo alone, (45) _____ these photographers often follow each other as well as famous
people.
Câu 41 (NB): A. who B. which C. what D. when
Câu 42 (TH): A. acceptable B. wicked C. defective D. dishonest

Trang 218
Câu 43 (TH): A. constantly B. consistently C. generally D. particularly
Câu 44 (TH): A. suffered B. felt C. underwent D. tolerated
Câu 45 (TH): A. so B. nevertheless C. but D. since
Mark the letter A, B, C or D on your answer sheet to indicate the sentence that is closest in meaning to
each of the following questions.
Câu 46 (VD): Every soldier will have to use a radio after landing.
A. It is a must of every soldier that they use a radio after they landed.
B. That every soldier needs a radio to use after landing will be necessary.
C. After landing, it will be vital that every soldier use a radio.
D. Every soldier’s using a radio will be needed once landed.
Câu 47 (VD): It’s hard to concentrate when you’re tired.
A. Your being tired makes you unable to concentrate.
B. You’re impossible to concentrate as a result of your being tired.
C. Your tiredness leads to your incapable of concentration.
D. The more tired you are, the harder it is to concentrate.
Câu 48 (VD): “You must never play truant again,” said their mother.
A. Their mother forbade them from playing truant again.
B. They are not allowed to play truant ever again by their mother.
C. Their mother commanded that they should not play truant again.
D. Never would their mother allow them to play truant again.
Mark the letter A, B, C or D on your answer sheet to indicate the most suitable response to complete each
of the following exchanges.
Câu 49 (TH): Brian: “Andrew won’t like it, you know.”
Tim: “___________ I don’t care what Andrew thinks.”
A. So what? B. Come what may. C. What for? D. I wouldn’t say no.
Câu 50 (TH): Guest: “Would you give this note to Mr. Wilson, please?”
Secretary: “______________”
A. Yeah. Why not? B. Sorry. Why do I have to do?
C. No, I wouldn’t. Suit yourself. D. Sorry, I can’t. He no longer works here.

Đáp án
1-C 2-C 3-B 4-C 5-C 6-C 7-C 8-A 9-C 10-D
11-C 12-A 13-B 14-A 15-B 16-B 17-A 18-D 19-A 20-D
21-C 22-C 23-A 24-D 25-A 26-D 27-C 28-D 29-C 30-B
31-D 32-B 33-A 34-A 35-A 36-C 37-C 38-B 39-B 40-A
41-A 42-B 43-A 44-A 45-D 46-C 47-D 48-A 49-A 50-D

Trang 219
LỜI GIẢI CHI TIẾT
Câu 1: Đáp án C
Kiến thức: Từ vựng
Giải chi tiết:
scrutinize (v): xem xét kĩ lưỡng
A. spend (v): dành, sử dụng
B. read (v): đọc
C. examine (v): kiểm tra, xem xét
D. spare (v): để dành, tiết kiệm
=> scrutinize = examine
Tạm dịch: Đừng ký tên vào bất kỳ tài liệu nào bạn chưa có thời gian để xem xét kỹ lưỡng!
Câu 2: Đáp án C
Kiến thức: Cụm động từ
Giải chi tiết:
scare off: hù dọa (khiến đối tượng đó bỏ đi)
A. attract (v): thu hút
B. locate (v): nằm, tọa lạc
C. terrify (v): làm cho sợ hãi
D. disappoint (v): làm thất vọng
=> scare off = terrify
Tạm dịch: Việc sử dụng âm thanh lớn để hù dọa những kẻ săn mồi đóng vai trò là một chiến lược khả thi
để sinh tồn.
Câu 3: Đáp án B
Kiến thức: Cụm động từ
Giải chi tiết:
make sth up: tạo ra cái gì
Sửa: make up them => make them up
Tạm dịch: Tôi đã diễn thuyết quá nhiều lần đến mức bây giờ tôi chỉ cần nghĩ về chúng trước khi tôi trình
bày là đủ rồi.
Câu 4: Đáp án C
Kiến thức: Sự kết hợp từ
Giải chi tiết:
take measure: thực hiện biện pháp, giải pháp
Sửa: make any measures => take any measures

Trang 220
Tạm dịch: Các vấn đề tắc nghẽn giao thông và ô nhiễm sẽ tiếp tục gia tăng nếu chính phủ không thực hiện
bất kỳ biện pháp nào để giảm các tác động tiêu cực của đô thị hóa.
Câu 5: Đáp án C
Kiến thức: Câu chẻ
Giải chi tiết:
Công thức: It + tobe + O + who/that + S + V: Chính là …. người/cái mà …
Thành phần được nhấn mạnh là “on a beautiful day” (vào một ngày đẹp trời), chỉ thời gian => dùng “that”
Sửa: when => that
Tạm dịch: Đó là vào một ngày đẹp trời tháng 11 khi mà cô chấp nhận lời cầu hôn của anh ấy.
Câu 6: Đáp án C
Kiến thức: Đọc hiểu
Giải chi tiết:
Đâu sẽ là tiêu đề tốt nhất cho đoạn văn?
A. Lịch sử bằng sáng chế của điện thoại
B. Sự liên kết giữa Nghiên cứu và Công nghệ
C. Sự tinh vi đang phát triển của điện thoại
D. Điện thoại: Ảo tưởng công nghệ
Câu 7: Đáp án C
Kiến thức: Đọc hiểu
Giải chi tiết:
Có thể suy ra được từ đoạn văn rằng ban đầu, gọi điện thoại ____.
A. bị giới hạn trong các doanh nghiệp
B. không có chuông
C. sử dụng sự điều khiển của con người
D. hồi sinh kinh doanh ở La Porte, Indiana
Thông tin: To access the system, the caller pressed button to reach the desired number and turned the
handle to activate the telephone ringer.
Tạm dịch: Để truy cập hệ thống, người gọi nhấn nút để chọn số mong muốn và xoay tay cầm để kích hoạt
cuộc gọi.
Câu 8: Đáp án A
Kiến thức: Đọc hiểu
Giải chi tiết:
Từ “implemented” trong đoạn 1 gần nghĩa nhất với ________.
A. used: sử dụng
B. breached: phá thủng
C. broken: làm vỡ

Trang 221
D. usurped: chiếm đoạt
implement (v): triển khai
Thông tin: During the same year, Strowger’s step-by-step call advancement technology was implemented
in the long-distance service between New York and Chicago…
Tạm dịch: Trong cùng năm đó, công nghệ tiến bộ cuộc gọi từng bước của Strowger đã được triển khai
trong dịch vụ đường dài giữa New York và Chicago
Câu 9: Đáp án C
Kiến thức: Đọc hiểu
Giải chi tiết:
Từ “that” trong đoạn 2 ám chỉ _______.
A. hệ thống
B. ống
C. kích cỡ, kích thước
D. tỉ lệ
Thông tin: The Bell Laboratories research facility came up with the transistor to replace the cumbersome
vacuum tube, thus diminishing the size of the electronic switch system to about 10 percent of that of the
original.
Tạm dịch: Cơ sở nghiên cứu của Phòng thí nghiệm Bell đã đưa ra bóng bán dẫn để thay thế ống chân
không cồng kềnh, do đó làm giảm kích thước của hệ thống chuyển mạch điện tử xuống còn khoảng 10%
của nó so với ban đầu.
Câu 10: Đáp án D
Kiến thức: Đọc hiểu
Giải chi tiết:
Tác giả ngụ ý rằng các mạng điện thoại được mở rộng vì _______.
A. công việc của một vài nhà phát minh
B. sự chân thành hỗ trợ công cộng và tư nhân
C. nhiều sai lầm kỹ thuật
D. một loạt các bước đột phá
Thông tin: The Bell Laboratories research facility came up with the transistor to replace the cumbersome
vacuum tube, thus diminishing the size of the electronic switch system to about 10 percent of that of the
original. Crossbar switching, installed in terminals in 1938, operated on the principle of an
electromagnetic force, which rotated horizontal and vertical bars within a rectangular frame and brought
contacts together in a split second. A technological breakthrough in the form of undersea cables between
the United States and Hawaii was implemented almost twenty years later. An extension was connected to
Japan in 1964.

Trang 222
Tạm dịch: Cơ sở nghiên cứu của Phòng thí nghiệm Bell đã đưa ra bóng bán dẫn để thay thế ống chân
không cồng kềnh, do đó làm giảm kích thước của hệ thống chuyển mạch điện tử xuống còn khoảng 10%
so với ban đầu. Chuyển đổi thanh ngang, được cài đặt trong các thiết bị đầu cuối vào năm 1938, hoạt
động theo nguyên tắc của một lực điện từ, quay các thanh ngang và dọc trong một khung hình chữ nhật và
đưa các tiếp điểm lại với nhau trong tích tắc. Một bước đột phá công nghệ dưới dạng cáp dưới biển giữa
Hoa Kỳ và Hawaii đã được thực hiện gần hai mươi năm sau đó. Một phần mở rộng đã được kết nối với
Nhật Bản vào năm 1964.
Câu 11: Đáp án C
Kiến thức: Trọng âm của từ có 2 âm tiết
Giải chi tiết:
A. selfish /ˈselfɪʃ/
B. famine /ˈfæmɪn/
C. police /pəˈliːs/
D. target /ˈtɑːɡɪt/
Phương án C trọng âm rơi vào âm tiết thứ 2, còn lại rơi vào âm 1
Câu 12: Đáp án A
Kiến thức: Trọng âm của từ có 3 âm tiết
Giải chi tiết:
A. ancestor /ˈænsestə(r)/
B. transistor /trænˈzɪstə(r)/
C. equator /ɪˈkweɪtə(r)/
D. compressor /kəmˈpresə(r)/
Phương án A trọng âm rơi vào âm tiết thứ 1, còn lại rơi vào âm 2
Câu 13: Đáp án B
Kiến thức: Phát âm “_ed”
Giải chi tiết:
A. continued /kənˈtɪnjuːd/
B. increased /ɪnˈkriːst/
C. diseased /dɪˈziːzd/
D. lightened /ˈlaɪtnd/
Quy tắc: Cách phát âm đuôi “-ed”:
- Đuôi “-ed” được phát âm là /ɪd/ khi động từ có phát âm kết thúc là /t/ hay /d/
- Đuôi “-ed” được phát âm là /t/ khi động từ có phát âm kết thúc là /s/,/f/,/p/,/ʃ/,/tʃ/,/k/
- Đuôi “-ed” được phát âm là /d/ với các trường hợp còn lại
Phần gạch chân phương án B được phát âm là /t/, còn lại là /d/.
Câu 14: Đáp án A

Trang 223
Kiến thức: Phát âm “ea”
Giải chi tiết:
A. reason /ˈriːzn/
B. pleasure /ˈpleʒə(r)/
C. treasure /ˈtreʒə(r)/
D. endeavour /ɪnˈdevə(r)/
Phần gạch chân phương án A được phát âm là /i:/, còn lại là /e/.
Câu 15: Đáp án B
Kiến thức: Từ vựng
Giải chi tiết:
A. form (n): mẫu
B. formality (n): hình thức
C. formation (n): hệ thống, sự hình thành
D. format (n): định dạng
Tạm dịch: Bạn sẽ phải đi phỏng vấn vào ngày mai, nhưng đừng lo lắng. Đó chỉ là hình thức thôi mà.
Câu 16: Đáp án B
Kiến thức: Liên từ
Giải chi tiết:
A. Much as + S + V: Dù cho … nhiều thế nào …
B. No matter + S + V: Bất kể là…
C. Even though + S + V: Mặc dù…
D. However, S + V: Tuy nhiên…
Tạm dịch: Bất kể anh ấy có cố giấu chiếc áo khoác yêu thích của mình ở đâu đi chăng nữa thì anh trai của
Jethreo luôn tìm được nó.
Câu 17: Đáp án A
Kiến thức: Từ vựng
Giải chi tiết:
A. leap (v): nhảy qua, nhảy vọt
B. rise (v): tăng, mọc
C. drop (v): rơi, thả
D. climb (v): trèo
Tạm dịch: Sam nghĩ rằng anh ta có thể nhảy qua dòng nước từ tàu đến bến tàu, nhưng anh ta đã sai và anh
ta ngã xuống nước.
Câu 18: Đáp án D
Kiến thức: Từ vựng
Giải chi tiết:

Trang 224
A. gazed: nhìn đăm đăm
B. glared: trừng mắt
C. stared: nhìn chằm chằm
D. glanced: liếc nhìn
Tạm dịch: Tôi không thể cung cấp cho bạn một mô tả chi tiết về người phụ nữ ấy vì tôi chỉ liếc nhìn qua
cô ấy thôi.
Câu 19: Đáp án A
Kiến thức: Từ vựng
Giải chi tiết:
A. moldy (adj): mốc
B. rusty (adj): gỉ
C. spoiled (adj): hỏng (không dùng với đồ ăn)
D. sour (adj): chua
Tạm dịch: Phô mai này không ăn được nữa. Nó bị mốc hết cả rồi sau khi nằm trong thùng quá lâu.
Câu 20: Đáp án D
Kiến thức: Từ vựng
Giải chi tiết:
A. interfere (v): can thiệp
B. join (v): tham gia, cùng làm gì
C. unify (v): thống nhất
D. accompany (v): đi cùng đến đâu
Tạm dịch: Paul bảo Maria đi cùng anh ta đến nha sĩ vì anh ta không muốn tự mình đi.
Câu 21: Đáp án C
Kiến thức: Thành ngữ
Giải chi tiết:
keep (something) under one’s hat: giữ bí mật chuyện gì

Tạm dịch: Ông Jones biết ai đã chiến thắng cuộc thi, nhưng ông bí mật cho đến khi nó được công bố với
mọi người.
Câu 22: Đáp án C
Kiến thức: Rút gọn mệnh đề quan hệ
Giải chi tiết:
Khi rút gọn mệnh đề quan hệ chứa “the only” => dùng “to + V_nguyên thể”
Cụm từ “four times” (4 lần rồi) => trong tương lai sẽ có thể còn nhiều lần nữa => dùng: “to + have + P2”
Câu mang nghĩa bị động (được người khác giao giải cho) => dùng “to have been + P2”
Tạm dịch: Olivia vẫn là sinh viên duy nhất được 4 lần trao giải nhất cho bản thiết kế tốt nhất.

Trang 225
Câu 23: Đáp án A
Kiến thức: Rút gọn mệnh đề đồng ngữ
Giải chi tiết:
Khi 2 mệnh đề của một câu có chung chủ ngữ => có thể rút gọn
Rút gọn mệnh đề trạng ngữ chỉ thời gian, mang nghĩa bị động => dùng P2
Dạng đầy đủ: When lead is added to petrol, it improves the car’s performance.
Dạng rút gọn: When added to petrol, lead improves the car’s performance.
Tạm dịch: Khi chì được thêm vào xăng, nó sẽ cải thiện hiệu suất của xe.
Câu 24: Đáp án D
Kiến thức: Thì quá khứ hoàn thành
Giải chi tiết:
- Dấu hiệu: by the time + hành động chia ở quá khứ (they arrived)
- Cách dùng: thì quá khứ hoàn thành được dùng để diễn tả một hành động xảy ra trước một hành động
khác trong quá khứ.
- Công thức: …by the time + S + V_ed (quá khứ đơn), S + had + P2: …vào lúc … thì đã …
Tạm dịch: "Tôi đã gọi cảnh sát nhưng khi họ đến nơi, hai người đàn ông đã biến mất rồi", nhân chứng
nói.
Câu 25: Đáp án A
Kiến thức: Giới từ
Giải chi tiết:
classify into sth: phân loại/phân chia thành …
Tạm dịch: Nói rộng hơn, diễn giải có thể được phân thành hai nhóm: đánh giá và quan điểm.
Câu 26: Đáp án D
Kiến thức: Mạo từ
Giải chi tiết:
the + tên riêng của bảo tàng
a/an + định nghĩa, giải thích thông tin về cái gì đó
Tạm dịch: James đã đến thăm Bảo tàng Hermitage, một điểm thu hút khách du lịch nổi tiếng ở phố
Petersburg.
Câu 27: Đáp án C
Kiến thức: Trạng từ, rút gọn mệnh đề trạng ngữ thời gian
Giải chi tiết:
Upon: trên
As soon as: ngay khi
While: trong khi, khi
Meanwhile: trong khi đó

Trang 226
Khi 2 mệnh đề có cùng chủ ngữ => có thể rút gọn bằng cách lược bỏ chủ ngữ và động từ "be"
Câu đầy đủ: While Carla was away in college, she stayed with her roommate’s family during the spring
break.
Câu rút gọn: While away in college, Carla stayed with her roommate’s family during the spring break.
Tạm dịch: Khi đi học đại học, Carla ở với gia đình bạn cùng phòng trong kỳ nghỉ xuân.
Câu 28: Đáp án D
Kiến thức: Câu điều kiện loại 3
Giải chi tiết:
- Dấu hiệu: mệnh đề chứa “if” chia quá khứ hoàn thành (had known)
- Cách dùng: Câu điều kiện loại 3 diễn tả điều kiện trái với quá khứ, dẫn đến kết quả trái với quá khứ.
- Công thức: If + S + had + P2, S + would (not) + have + P2
Tạm dịch: Thậm chí là nếu tôi có biết địa chỉ của cô ấy thì tôi sẽ không đưa nó cho anh ta đâu.
Câu 29: Đáp án C
Kiến thức: Đọc hiểu
Giải chi tiết:
Ý chính được thảo luận trong đoạn văn là gì?
A. Tặng quà, mặc dù có những nhược điểm không tiết kiệm, nuôi dưỡng tiến trình tích cực xã hội.
B. Quà tặng có thể đóng vai trò là tín hiệu ngầm của sự chu đáo và quan tâm.
C. Tặng quà có thể có những hạn chế nhất định bên cạnh những phẩm chất tích cực của nó.
D. Người nhận quà tặng được thừa nhận rộng rãi là có kinh nghiệm đáng kể trong việc tặng quà.
Thông tin: Anthropologists describe gift-giving as a positive social process, serving various political,
religious, and psychological functions. Economists, however, offer a less favorable view. According to
Waldfogel (1993), gift-giving represents an objective waste of resources.
Tạm dịch: Các nhà nhân chủng học mô tả việc tặng quà như một tiến trình tích cực trong xã hội, trong
phục vụ các nền chính trị, tôn giáo, và trong các chức năng tâm lý. Tuy nhiên, các nhà kinh tế học lại đưa
ra một cái nhìn ít thiện cảm hơn. Theo Waldfogel (1993), việc tặng quà là tiêu biểu cho một sự lãng phí
có mục đích các nguồn tài nguyên.
Câu 30: Đáp án B
Kiến thức: Đọc hiểu
Giải chi tiết:
Trong đoạn 1, từ “ambivalent” gần nghĩa nhất với_________.
A. unrealistic (adj): không thực tế
B. conflicted (adj): mâu thuẫn
C. apprehensive (adj): e ngại
D. supportive (adj): ủng hộ
Thông tin: This frequent experience of gift-giving can engender ambivalent feelings in gift-givers.

Trang 227
Tạm dịch: Việc tặng quà thường xuyên này có thể gây ra những cảm xúc mâu thuẫn cho người tặng quà.
Câu 31: Đáp án D
Kiến thức: Đọc hiểu
Giải chi tiết:
Tác giả chỉ ra rằng mọi người coi trọng việc tặng quà bởi vì họ cảm thấy nó ________.
A. có chức năng như một hình thức thể hiện bản thân
B. là một cách không đắt để thể hiện sự đánh giá cao
C. yêu cầu người nhận quà tặng phải đáp lại
D. có thể cùng để củng cố một mối quan hệ
Thông tin: Many relish the opportunity to buy presents because gift-giving offers a powerful means to
build stronger bonds with one’s closest peers.
Tạm dịch: Nhiều người tận hưởng cơ hội mua quà vì họ cho rằng tặng quà là một phương tiện hữu hiệu
trong việc xây dựng các mối quan hệ với các đồng nghiệp thân cận nhất thêm mạnh mẽ.
Câu 32: Đáp án B
Kiến thức: Đọc hiểu
Giải chi tiết:
Từ “this” trong đoạn 3 ám chỉ _______ của người tặng quà.
A. sự đấu tranh
B. bội chi (chi tiêu quá mức)
C. mua những món quà ý nghĩa
D. có nhiều kinh nghiệm
Thông tin: but nevertheless tend to overspend each time they set out to purchase a meaningful gift. In the
present research, we propose a unique psychological explanation for this …
Tạm dịch: tuy vậy họ vẫn có xu hướng chi quá tay mỗi lần đặt mục tiêu mua một món quà ý nghĩa. Trong
nghiên cứu hiện nay, chúng tôi đề xuất một giải thích mang tính tâm lý cho vấn đề bội chi này …
Câu 33: Đáp án A
Kiến thức: Đọc hiểu
Giải chi tiết:
Đoạn văn chỉ ra rằng giả định được đưa ra bởi những người tặng quà trong đoạn 3 có thể là ______.
A. không chính xác
B. không hợp lý
C. không thành thật
D. chứng xác, đã được chứng minh
Thông tin: Although a link between gift price and feelings of appreciation might seem intuitive to gift-
givers, such an assumption may be unfounded.

Trang 228
Tạm dịch: Mặc dù một mối liên kết giữa giá quà và cảm giác yêu thích nghe có vẻ trực quan với người
tặng, nhưng có thể lại là một giả định vô căn cứ.
Câu 34: Đáp án A
Kiến thức: Đọc hiểu
Giải chi tiết:
Từ “convey” trong đoạn 4 gần nghĩa nhất với ________.
A. truyền tải, vận chuyển
B. phản tác dụng, chống đối
C. giao tiếp
D. trao đổi
convey (v): truyền tải, chuyển đạt
Thông tin: Perhaps givers believe that bigger (that is, more expensive) gifts convey stronger signals of
thoughtfulness and consideration.
Tạm dịch: Phải chăng vì họ tin rằng món quà càng lớn (nghĩa là càng đắt tiền) sẽ truyền tải một thông
điệp mạnh mẽ hơn về sự chu đáo và quan tâm của họ (với đối phương).
Câu 35: Đáp án A
Kiến thức: Đọc hiểu
Giải chi tiết:
Các tác giả tham khảo nghiên cứu của Camerer và những người khác (đoạn 4) để _______.
A. đưa ra một lời giải thích
B. giới thiệu một lập luận
C. ủng hộ một kết luận
D. chất vấn một bộ phim
Đoạn văn xem xét cách người tặng quà tin rằng những món quà đắt tiền là chu đáo hơn những món quà rẻ
hơn và sẽ được người nhận đánh giá cao hơn. Nghiên cứu của Camerer và những người khác đưa ra lời
giải thích cho người tặng quà.
Thông tin: … gift-givers attempt to signal their positive attitudes toward the intended recipient and their
willingness to invest resources in a future relationship.
Tạm dịch: … người tặng quà cố gắng để gửi gắm thái độ tích cực của họ đối với người nhận và sự sẵn
lòng để đầu tư nhiều hơn vào một mối quan hệ lâu dài trong tương lai.
Câu 36: Đáp án C
Kiến thức: Đọc hiểu
Giải chi tiết:
Cái nào sau đây mô tả đúng nhất giọng điệu của tác giả?
A. chỉ trích
B. bi quan

Trang 229
C. cung cấp thông tin
D. mỉa mai
Câu 37: Đáp án C
Kiến thức: Từ vựng
Giải chi tiết:
microscopic (adj): thuộc về kính hiển vi => kích thước vô cùng nhỏ
A. strange (adj): lạ
B. tiny (adj): rất nhỏ
C. giant (adj): khổng lồ
D. queer (adj): lạ lùng, kì quặc
=> microscopic >< giant
Tạm dịch: Do kích thước siêu nhỏ của chúng, các hạt có thể dễ dàng xâm nhập sâu vào hệ hô hấp, gây ra
các vấn đề nghiêm trọng về sức khỏe.
Câu 38: Đáp án B
Kiến thức: Thành ngữ
Giải chi tiết:
making money hand over fist: kiếm nhiều tiền rất nhanh
A. kiếm được nhiều lợi nhuận với tốc độ rất nhanh
B. chỉ kiếm được một ít tiền
C. làm thêm áo len trong tương lai
D. làm thêm sản phẩm cầm tay
=> making money hand over fist >< making only a little money
Tạm dịch: Bộ sưu tập áo len mới rất phổ biến với thanh thiếu niên trên toàn thế giới đến nỗi nhà sản xuất
đang kiếm được rất nhiều tiền một cách nhanh chóng.
Câu 39: Đáp án B
Kiến thức: Động từ khuyết thiếu
Giải chi tiết:
could + have + P2: có thể đã làm gì trong quá khứ (nhưng thực tế đã không làm được)
A. Nếu chúng tôi có thể tìm được một người giữ trẻ, chúng tôi sẽ tham gia cùng bạn tại nhà hàng. (Sai
công thức câu điều kiện loại 3)
B. Chúng tôi có thể đã tham gia cùng bạn tại nhà hàng, nhưng chúng tôi không thể tìm được một người
giữ trẻ.
C. Lý do cho sự vắng mặt của chúng tôi tại nhà hàng không phải là chúng tôi có thể tìm một người giữ
trẻ. => sai nghĩa
D. Chúng tôi không thể tìm được người trông trẻ để mà chúng tôi không thể tham gia cùng bạn tại nhà
hàng. => sai nghĩa

Trang 230
Tạm dịch: Chúng tôi không thể tìm được một người giữ trẻ. Đó là lý do tại sao chúng tôi không thể tham
gia cùng bạn tại nhà hàng.
= Chúng tôi có thể đã tham gia cùng bạn tại nhà hàng, nhưng chúng tôi không thể tìm được một người giữ
trẻ.
Câu 40: Đáp án A
Kiến thức: Câu đồng nghĩa
Giải chi tiết:
A. Tổng thống không thể giải thích nguyên nhân của cuộc khủng hoảng, ông cũng không đưa ra bất kỳ
giải pháp nào.
B. Tổng thống không có khả năng giải thích nguyên nhân của cuộc khủng hoảng cũng như bất kỳ giải
pháp nào. => sai nghĩa
C. Giải pháp của tổng thống không được đưa ra vì ông không thể đưa ra bất kỳ lời giải thích nào cho cuộc
khủng hoảng. => sai nghĩa
D. Lời giải thích của tổng thống về cuộc khủng hoảng là một thất bại đến nỗi ông không thể đưa ra bất kỳ
giải pháp nào. => sai nghĩa
Tạm dịch: Tổng thống đã không giải thích được nguyên nhân của cuộc khủng hoảng. Ông không đưa ra
bất kỳ giải pháp nào.
= Tổng thống không thể giải thích nguyên nhân của cuộc khủng hoảng, ông cũng không đưa ra bất kỳ giải
pháp nào.
Câu 41: Đáp án A
Kiến thức: Đại từ quan hệ
Giải chi tiết:
who + S + V: người mà => thay cho từ chỉ người, đóng vai trò làm chủ ngữ/tân ngữ của MĐ quan hệ
which + S + V / V : cái mà => thay cho từ chỉ người, đóng vai trò làm chủ ngữ/tân ngữ của MĐ quan hệ
what: cái mà
when: khi mà
Phía trước chỗ trống là “freelance photographers” (nhiếp ảnh gia tự do) chỉ người => dùng “who”
Paparazzi are freelance photographers (41) who take photographs of celebrities.
Tạm dịch: Paparazzi là những nhiếp ảnh gia tự do chụp ảnh của những người nổi tiếng.
Câu 42: Đáp án B
Kiến thức: Từ vựng
Giải chi tiết:
A. được chấp nhận
B. nguy hiểm, có tính làm hại
C. khiếm khuyết, có tật
D. không thật thà

Trang 231
At times, their actions may actually be (42) wicked, and in some countries they may be prosecuted.
Tạm dịch: Đôi khi, hành động của họ thực sự có thể là nguy hiểm, và ở một số quốc gia, họ có thể bị truy
tố.
Câu 43: Đáp án A
Kiến thức: Từ vựng
Giải chi tiết:
A. liên tục, không ngừng nghỉ
B. nhất quán
C. nói chung
D. đặc biệt, cụ thể
Paparazzi is a photographer who is (43) constantly looking for his next photo opportunity and …
Tạm dịch: Paparazzi là một nhiếp ảnh gia không ngừng tìm kiếm cơ hội chụp ảnh và…
Câu 44: Đáp án A
Kiến thức: Từ vựng
Giải chi tiết:
A. chịu đựng
B. cảm thấy
C. trải qua (sai dạng từ, vì have/have + P2)
D. chịu đau đớn; tha thứ, khoan dung
at one point, photographs a woman who has recently (44) suffered a personal tragedy.
Tạm dịch: tại một thời điểm, hình ảnh một người phụ nữ gần đây đã chịu đựng một bi kịch cá nhân.
Câu 45: Đáp án D
Kiến thức: Liên từ/Mệnh đề nguyên nhân
Giải chi tiết:
so + S + V: vì thế, vì vậy…
nevertheless = however + S + V: tuy nhiên….
but + S + V: nhưng…
since + S + V: bởi vì
although it is rare to see a paparazzo alone, (45) since these photographers often follow each other as well
as famous people.
Tạm dịch: mặc dù rất hiếm khi nhìn thấy một tay săn ảnh, vì những nhiếp ảnh gia này thường theo dõi
nhau cũng như những người nổi tiếng.
Câu 46: Đáp án C
Kiến thức: Câu đồng nghĩa
Giải chi tiết:
A. Sai cấu trúc. be a must for sth/sb: điều bắt buộc …

Trang 232
B. Mỗi người lính cần một đài phát thanh để sử dụng sau khi hạ cánh sẽ là cần thiết. (không làm cũng
được) => sai nghĩa
C. Sau khi hạ cánh, điều quan trọng là mọi người lính sử dụng radio.
D. Sai cấu trúc. “using a radio” không phải một cụm từ (đây là V + O) => không đứng sau ‘s chỉ sở hữu
Tạm dịch: Mỗi người lính sẽ phải sử dụng radio sau khi hạ cánh.
= Sau khi hạ cánh, điều quan trọng là mọi người lính sử dụng radio.
Câu 47: Đáp án D
Kiến thức: So sánh kép
Giải chi tiết:
Công thức so sánh kép: The + more + tính từ dài + S + tobe, the + adj_ngắn + _er + S + tobe …:
…càng…càng…
A. Sự mệt mỏi của bạn khiến bạn không thể tập trung. => sai nghĩa
B. Bạn không thể tập trung do mệt mỏi. => sai nghĩa
C. Sự mệt mỏi của bạn dẫn đến việc bạn không có khả năng tập trung. => sai nghĩa
D. Bạn càng mệt mỏi, càng khó tập trung.
Tạm dịch: Thật khó để tập trung khi bạn mệt mỏi. (vẫn có thể tập trung nhưng không dễ dàng)
= Bạn càng mệt mỏi, càng khó tập trung.
Câu 48: Đáp án A
Kiến thức: Câu tường thuật (đặc biệt)
Giải chi tiết:
must never + V_nguyên thể : không được phép làm gì (cấm làm gì)
forbade sb from doing sth: cấm ai làm gì
be not allowed to V_nguyên thể: không được cho phép làm gì
command (v): ra lệnh
Never + trợ động từ + S + V_nguyên thể…: không bao giờ…
A. Mẹ của họ cấm họ trốn học một lần nữa.
B. Họ không được mẹ cho phép trốn học một lần nữa. => sai nghĩa
C. Mẹ của họ ra lệnh rằng họ không nên trốn học nữa. => sai nghĩa
D. Không bao giờ mẹ của họ cho phép họ trốn học một lần nữa. (Sai “would”)
Tạm dịch: “Các con không bao giờ được phép trốn học nữa”, mẹ nói.
= Mẹ của họ cấm họ trốn học một lần nữa.
Câu 49: Đáp án A
Kiến thức: Ngôn ngữ giao tiếp
Giải chi tiết:
Brian: "Andrew sẽ không thích nó, cậu biết đấy."
Tim: "______ Tớ không quan tâm Andrew nghĩ gì."

Trang 233
A. Vậy thì sao?
B. Dù khó khăn đến mấy/Dù có rắc rối gì đi nữa
C. Để làm gì?
D. Tớ sẽ không nói không.
Các phản hồi B, C, D không phù hợp.
Câu 50: Đáp án D
Kiến thức: Ngôn ngữ giao tiếp
Giải chi tiết:
Khách: "Cô có thể làm ơn đưa ghi chú này cho ông Wilson được không?"
Thư ký: "_________"
A. Ừ. Tại sao không? => không phù hợp với ngữ cảnh trang trọng, lịch sự
B. Xin lỗi. Tại sao tôi phải làm thế?
C. Không, tôi sẽ không đưa. Tùy bạn thôi.
D. Xin lỗi, tôi không thể. Ông ấy không còn làm việc ở đây nữa.
Các phản hồi A, B, C không phù hợp.

SỞ GD & ĐT QUẢNG NINH ĐỀ THI THỬ THPT QUỐC GIA LẦN 1


TRƯỜNG THPT CHUYÊN HẠ NĂM HỌC: 2019 – 2020
LONG MÔN: TIẾNG ANH
Mã đề 211 Thời gian làm bài: 60 phút; không kể thời gian phát đề

Mark the letter A, B, C, or D on your answer sheet to indicate the word that differs from the other three in
the position of primary stress in each of the following questions.
Câu 1 (NB): A. oblige B. promise C. notice D. project
Câu 2 (NB): A. compliment B. interview C. outstanding D. industry
Mark the letter A, B, C or D on your answer sheet to indicate the most suitable response to complete each
of the following exchanges.
Câu 3 (TH): Mary is going shopping with John.
Mary: “What do you think of fashion?”
John: “________________________”
A. Of course the fashion show is excellent. B. It’s none of my business.
C. You are very welcome. D. I am crazy about it.
Câu 4 (TH): Tom and John are talking about a tennis game.
Tom: “I thought your tennis game was a lot better today, John.”
John: “ ___________! I thought it was terrible.”
A. You can say that again B. You’ve got to be kidding

Trang 234
C. No, I think so D. Thanks! Same to you
Mark the letter A, B, C or D on your answer sheet to indicate the underlined part that needs correction in
each of the following questions.
Câu 5 (TH): The Complex of Hue Monuments was the first site in Viet Nam recognized as a World
Heritage Site by the UNESCO.
A. Monuments B. was C. the first D. recognized
Câu 6 (NB): For thousands of years, man has created sweet-smelling substances from wood, herbs, and
flowers and using them for perfume or medicine.
A. man B. sweet-smelling C. using them D. or
Câu 7 (TH): Two billion dollars are not enough for the victims of the tsunami.
A. billion dollars B. are C. enough for D. of the
Mark the letter A, B, C or D on your answer sheets to indicate the word whose underlined part differs
from the other three in pronunciation in each of the following questions.
Câu 8 (TH): A. supposed B. maintained C. sacred D. arrived
Câu 9 (NB): A. command B. comprise C. complete D. common
Read the following passage and mark the letter A, B, C or D on your answer sheet to indicate the correct
word or phrase that best fits each of the numbered blanks from 10 to 14.
As a small-business owner, you can avoid many problems simply by improving communication in your
office. By clarifying everyone's expectations and roles, you'll help to build greater trust and increased
productivity among employees. Here are a few tips for doing so.
Practice active listening. The art of active listening includes (10)_______ close attention to what another
person is saying, then paraphrasing (11)______ you've heard and repeating it back. Concentrate on the
conversation at hand and avoid unwanted interruptions (cell phone calls, others walking into your office,
etc.). Take note of how your own experience and values may color your perception.
Pay attention to non-verbal cues. We don't communicate with words alone. Every conversation comes
with a host of non-verbal cues-facial expressions, body language, etc. - that may (12)_______ contradict
what we're saying. (13)_______ addressing a staff member or leading a project conference, think
carefully about your tone of voice, how you make eye contact, and what your body is "saying". Be
consistent throughout.
Be clear and to the point. Don't cloud instructions or requests with irrelevant details, such as problems
with past projects or issues with long-departed personnel. State what you need and what you expect. Ask,
"Does anyone have any questions?" demonstrate that you prefer questions up-front as (14)_______ to
misinterpretation later on.
Câu 10 (VD): A. showing B. using C. paying D. spending
Câu 11 (NB): A. what B. when C. which D. all
Câu 12 (TH): A. intentional B. unintentional C. intentionally D. unintentionally

Trang 235
Câu 13 (TH): A. While B. When C. Before D. After
Câu 14 (TH): A. led B. opposed C. refrained D. confessed
Mark the letter A, B, C or D on your answer sheet to indicate the word(s) CLOSEST in meaning to the
underlined word(s) in each of the following questions.
Câu 15 (TH): The emblem of the Association of Southeast Asian Nations was designed basing on rice –
the most important crop for the Southeast Asian people.
A. banner B. motto C. slogan D. logo
Câu 16 (VDC): When the police arrived the thieves took flight leaving all the stolen things behind.
A. did away B. climbed on C. ran away D. took away
Mark the letter A, B, C or D on your answer sheet to indicate the correct answer to each of the following
questions.
Câu 17 (TH): In Britain, most shops close at 6 p.m., _________ in other countries they often open in the
evening too.
A. despite B. moreover C. whereas D. nevertheless
Câu 18 (TH): No one should be exempt ________ prosecution if they have committed a crime.
A. of B. about C. against D. from
Câu 19 (TH): Ms. Thompson is always willing to help, but she doesn’t want ________ at home unless
there is an emergency.
A. to call B. to be called C. being called D. calling
Câu 20 (TH): He had to explain the lesson very clearly ________.
A. in order that his student to understand it B. so that his students could understand it
C. so as to his students understand it D. so that his students can understand it
Câu 21 (TH): In order to _______ their goals in college, students need to invest the maximum amount of
time, money, and energy in their studies.
A. manage B. establish C. achieve D. catch
Câu 22 (VD): It is difficult to assess the full _______ of the damage caused by Hurricane Sandy, one of
the most destructive storms of the U.S. has had in quite some time.
A. range B. amount C. extent D. quality
Câu 23 (VD): Don’t be _______ by these slick – talking salesmen.
A. put aside B. taken in C. taken away D. put up
Câu 24 (VD): She was _______ out of 100 applicants for the position of managing director.
A. short-changed B. short-listed C. short-sighted D. short-handed
Câu 25 (VDC): One way to let off ________ after a stressful day is to take some vigorous exercise.
A. steam B. cloud C. tension D. sweat
Câu 26 (NB): One third of _______ world’s population consumes two thirds of ______ world’s
resources.

Trang 236
A. a / the B. the / (no article) C. the / the D. the / a
Câu 27 (TH): I didn’t know you were asleep. Otherwise, I _______ so much noise when I came in.
A. didn’t make B. wouldn’t have made
C. won’t make D. wouldn’t make
Câu 28 (TH): By the time I _______ this report, I will give you a ring.
A. typed B. will type C. have typed D. will have typed
Câu 29 (TH): ________ the stories about people who reduced their carbon footprint, we started to change
our daily consumption habits.
A. Being read B. Reading C. Having read D. Having been read
Câu 30 (TH): In order to avoid boredom, the most important thing is to keep oneself ________.
A. occupation B. occupant C. occupied D. occupational
Mark the letter A, B, C or D on your answer sheet to indicate the word(s) OPPOSITE in meaning to the
underlined word(s) in each of the following questions.
Câu 31 (TH): They were forced to leave as part of a government drive to expel illegal immigrants.
A. depart B. welcome C. help D. facilitate
Câu 32 (VDC): At first, no one believes she was a pilot, but her documents lent colour to her statement.
A. provided evidence for B. got information from
C. borrowed colour from D. gave no proof of
Mark the letter A, B, C or D on your answer sheet to indicate the sentence that best combines each pair of
sentences in the following questions.
Câu 33 (VD): I had never seen her before. However, I recognized her form a photograph.
A. I recognized her from a photograph before I had never seen her.
B. Although I had never seen her before but I recognized her from a photograph.
C. After I had seen her, I recognized her from a photograph.
D. Although I had never seen her before, I recognized her from a photograph.
Câu 34 (VDC): The substance is very toxic. Protective clothing must be worn at all times.
A. Since the substance is very toxic, so protective clothing must be worn at all times.
B. So toxic is the substance that protective clothing must be worn at all times.
C. The substance is such toxic that protective clothing must be worn at all times.
D. The substance is too toxic to wear protective clothing at all times.
Mark the letter A, B, C or D on your answer sheet to indicate the sentence that is closest in meaning to
each of the following questions.
Câu 35 (VDC): People say that at least ten applicants have been selected for the job interview.
A. People say that fewer than ten job interviews have been held so far.
B. I have heard that only ten people have been chosen to have the job interviewed.
C. It is said that well over ten people are interested in having an interview for the job.

Trang 237
D. It is said that no fewer than ten people are going to be interviewed for the job.
Câu 36 (VD): “You’d better work harder if you don’t want to retake the exam!” the teacher said to
Jimmy.
A. The teacher reminded Jimmy to work harder if he didn’t want to retake the exam.
B. The teacher advised Jimmy to work harder if he didn’t want to retake the exam.
C. The teacher ordered Jimmy to work harder if he didn’t want to retake the exam.
D. The teacher suggested Jimmy to work harder if he didn’t want to retake the exam.
Câu 37 (VDC): I’m sure Lusia was very disappointed when she failed the exam.
A. Lusia must be very disappointed when he failed the exam.
B. Lusia must have been very disappointed when she failed the exam.
C. Lusia may be very disappointed when she failed the exam.
D. Lusia could have been very disappointed when she failed the exam.
Read the following passage and mark the letter A, B, C or D on your answer sheet to indicate the correct
answer to each of questions from 38 to 42.
The organisation that today is known as the Bank of America did start out in America, but under quite a
different name. Italian American A.P. Giannini estabished this bank on October 17 1904, in a renovated
saloon in San Francisco’s Italian community of North Beach under the name Bank of Italy, with
immigrants and first – time bank customers comprising the majority of his first customers. During its
development, Giannini’s bank survived major crises in the form of a natural disaster and a major
economic upheaval that not all other banks were able to overcome.
One major test for Giannini’s bank occurred on April 18,1906, when a massive earthquake struck San
Francisco, followed by a raging fire that destroyed much of the city. Giannini obtained two wagons and
teams of horses, filled the wagons with the bank’s reserves, mostly in the form of gold, covered the
reserves with crates of oranges, and escaped from the chaos of the city with his clients’ funds protected.
In the aftermath of the disaster, Giannini’s bank was the first to resume operations. Unable to install the
bank in a proper office setting, Giannini opened up shop on the Washington Street Wharf on a makeshift
desk created from boards and barrels.
In the period following the 1906 fire, the Bank of Italy continued to prosper and expand. By 1918 there
were twenty–four branches of the Bank of Italy, and by 1928 Giannini had acquired numerous other
banks, including a Bank of America located in New York City. In 1930 he consolidated all the branches
of the Bank of Italy, the Bank of America in New York City, and another bank of America that had
formed in California into the Bank of America National Trust and Savings Association.
A second major crisis for the bank occurred during the Great Depression of the 1930s. Although Giannini
had already retired prior to the darkest days of Depression, he became incensed when his successor began
selling off banks during the bad economic times. Giannini resumed leadership of the bank at the age of

Trang 238
sixty-two. Under Giannini’s leadership, the bank weathered the storm of the Depression and subsequently
moved into a phase of overseas development.
Câu 38 (TH): The word “consolidated” is closest in meaning to ________.
A. harden B. moved C. sold D. merge
Câu 39 (VD): It can be inferred from the passage that Giannini used crates of oranges after the earthquake
______.
A. to hide the gold B. to fill up the wagons
C. to provide nourishment for his customers D. to protect the gold from the fire
Câu 40 (TH): According to the passage, which of the following is NOT true about the San Francisco
earthquake?
A. It happened in 1906. B. It occurred in the aftermath of a fire.
C. It caused problems for Giannini’s bank. D. It was a tremendous earthquake.
Câu 41 (VD): The paragraph following the passage most likely discusses ________.
A. bank failures during the Great Depression
B. a third major crisis of the Bank of America
C. the international development of the Bank of America
D. how Giannini spent his retirement
Câu 42 (NB): The word its refers to _________.
A. saloon B. customer C. immigrant D. bank
Read the following passage and mark the letter A, B, C or D on your answer sheet to indicate the correct
answer to each of questions from 43 to 50.
The human criterion for perfect vision is 20/20 for reading the standard lines on a Snellen eye chart
without a hitch. The score is determined by how well you read lines of letters of different sizes from 20
feet away. But being able to read the bottom line on the eye chart does not approximate perfection as far
as other species are concerned. Most birds would consider us very visually handicapped. The hawk, for
instance, has such sharp eyes that it can spot a dime on the sidewalk while perched on top of the Empire
State Building. It can make fine visual distinctions because it is blessed with one million cones per square
millimeter in its retina. And in water, humans are farsighted, while the kingfisher, swooping down to
spearfish, can see well in both the air and water because it is endowed with two foveae – areas of the eye,
consisting mostly of cones, that provide visual distinctions. One foveae permits the bird, while in the air,
to scan the water below with one eye at a time. This is called monocular vision. Once it hits the water, the
other fovea joins in, allowing the kingfisher to focus both eyes, like binoculars, on its prey at the same
time. A frog’s vision is distinguished by its ability to perceive things as a constant motion picture. Known
as “bug detectors”, a highly developed set of cells in a frog’s eyes responds mainly to moving objects. So,
it is said that a frog sitting in a field of dead bugs wouldn’t see them as food and would starve.

Trang 239
The bee has a “compound” eye, which is used for navigation. It has 15,000 facets that divide what it sees
into a pattern of dots, or mosaic. With this kind of vision, the bee sees the sun only as a single dot, a
constant point of reference. Thus, the eye is a superb navigational instrument that constantly measures the
angle of its line of flight in relation to the sun. A bee’s eye also gauges flight speed. And if that is not
enough to leave our 20/20 “perfect vision” paling into insignificance, the bee is capable of seeing
something we can’t – ultraviolet light. Thus, what humans consider to be “perfect vision” is in fact rather
limited when we look at other species. However, there is still much to be said for the human eye. Of all
the mammals, only humans and some primates can enjoy the pleasures of color vision.
Câu 43 (NB): The word “that” in line 8 refers to________.
A. foveae B. areas of the eye C. cones D. visual distinctions
Câu 44 (TH): The word “criterion” in line 1 is closest in meaning to________.
A. need B. expectation C. rule D. standard
Câu 45 (TH): According to the passage, “bug detectors” are useful for ________.
A. navigation B. seeing moving objects
C. avoiding bugs when getting food D. avoiding starvation
Câu 46 (TH): According to the passage, why might birds and animals consider humans very visually
handicapped?
A. humans can’t see very well in either air or water.
B. human eyes are not as well suited to our needs.
C. the main outstanding feature of human eyes is color vision.
D. human eyes can’t do what their eyes can do.
Câu 47 (VD): According to the passage, which of the following is NOT true?
A. kingfishers have monocular vision
B. bees see patterns of dots
C. hawks eyes consist mostly of cones that can allow it to scan with one eye at a time
D. humans are farsighted in water
Câu 48 (VD): The phrase “paling into insignificance” is closest in meaning to _____.
A. fading away B. of less importance
C. without colored light D. being reduced to little importance
Câu 49 (TH): Which of the following can be inferred from the passage?
A. eyes have developed differently in each species
B. bees have the most complex eye
C. humans should not envy what they don’t need
D. perfect vision is not perfect
Câu 50 (TH): What does the passage mainly discuss?
A. limits of the human eye B. perfect vision

Trang 240
C. different eyes for different uses D. eye variation among different species

Trang 241
Đáp án
1-A 2-C 3-D 4-B 5-D 6-C 7-B 8-C 9-D 10-C
11-A 12-D 13-C 14-A 15-D 16-C 17-C 18-D 19-B 20-B
21-C 22-C 23-B 24-B 25-A 26-C 27-B 28-C 29-C 30-C
31-B 32-D 33-D 34-B 35-D 36-B 37-B 38-D 39-D 40-B
41-C 42-D 43-C 44-D 45-B 46-D 47-A 48-D 49-A 50-D

LỜI GIẢI CHI TIẾT


Câu 1: Đáp án A
Kiến thức: Trọng âm của từ có 2 âm tiết
Giải chi tiết:
A. oblige /əˈblaɪdʒ/
B. promise /ˈprɒmɪs/
C. notice /ˈnəʊtɪs/
D. project /ˈprɒdʒekt/
Phương án A trọng âm rơi vào âm tiết thứ 2, còn lại là âm thứ nhất
Câu 2: Đáp án C
Kiến thức: Trọng âm của từ có 3 âm tiết
Giải chi tiết:
A. compliment /ˈkɒmplɪmənt/
B. interview /ˈɪntəvjuː/
C. outstanding /aʊtˈstændɪŋ/
D. industry /ˈɪndəstri/
Phương án C trọng âm rơi vào âm tiết thứ 2, còn lại là âm thứ nhất
Câu 3: Đáp án D
Kiến thức: Ngôn ngữ giao tiếp
Giải chi tiết:
Mary đang đi mua sắm với John.
Mary: “Bạn nghĩ gì về thời trang?”
John: “_________."
A. Đương nhiên các buổi trình diễn thời trang thì tuyệt vời.
B. Đó không phải là việc của tôi.
C. Không có chi.
D. Tôi thích nó điên đảo.
Các phản hồi A, B, C không phù hợp ngữ cảnh.
Câu 4: Đáp án B

Trang 242
Kiến thức: Ngôn ngữ giao tiếp
Giải chi tiết:
Tom và John đang nói về một trận chơi quần vợt.
Tom: “Tớ thấy hôm nay cậu đã chơi quần vợt tốt hơn nhiều đấy, John.”
John: “___________. Tớ nghĩ nó rất tệ."
A. Tớ hoàn toàn đồng ý.
B. Cậu chắc đùa.
C. Không, tớ nghĩ vậy.
D. Cảm ơn! Chúc bạn cũng vậy.
Các phản hồi A, C, D không phù hợp ngữ cảnh.
Câu 5: Đáp án D
Kiến thức: Rút gọn mệnh đề quan hệ
Giải chi tiết:
Mệnh đề quan hệ chứa các từ chỉ số thứ tự “first, second, ...” thì ta rút gọn sử dụng “to + V_nguyên thể”
Công thức rút gọn với câu bị động: S + be + the + first/ second... + N + to + be + P2
Sửa: “recognized” => “to be recognized”
Tạm dịch: Quần thể di tích Huế là địa danh đầu tiên được công nhận là di sản văn hóa thế giới bởi
UNESCO.
Câu 6: Đáp án C
Kiến thức: Cấu trúc song hành
Giải chi tiết:
Liên từ “and” kết nối các từ cùng tính chất, cùng loại, cùng dạng.
Động từ “created” chia dạng P2 => động từ sau “and” cũng phỉa chia dạng P2
Sửa: “using” => “used”
Tạm dịch: Hàng nghìn năm, con người đã sáng tạo ra các vật có mùi thơm như gỗ, thảo mộc và hoa và sử
dụng chúng làm nước hoa hoặc thuốc.
Câu 7: Đáp án B
Kiến thức: Sự hòa hợp giữa chủ ngữ và động từ
Giải chi tiết:
Danh từ chỉ tiền bạc “Two billion dollars” được coi như danh từ số ít => động từ chia theo chủ ngữ số ít
Sửa: “are” => “is”
Tạm dịch: Hai tỉ đô thì không đủ cho các nạn nhân trong trận sóng thần.
Câu 8: Đáp án C
Kiến thức: Phát âm “ed”
Giải chi tiết:
A. supposed /səˈpəʊzd/

Trang 243
B. maintained /meɪnˈteɪnd/
C. scared /skeəd/
D. arrived /əˈraɪvd/
Quy tắc:
Cách phát âm đuôi “-ed”:
- Đuôi “-ed” được phát âm là /ɪd/ khi động từ có phát âm kết thúc là /t/ hay /d/
- Đuôi “-ed” được phát âm là /t/ khi động từ có phát âm kết thúc là /s/,/f/,/p/,/ʃ/,/tʃ/,/k/
- Đuôi “-ed” được phát âm là /d/ với các trường hợp còn lại
Phần gạch chân phương án C được phát âm là /əd/, còn lại là /d/
Câu 9: Đáp án D
Kiến thức: Phát âm “o”
Giải chi tiết:
A. command /kəˈmɑːnd/
B. comprise /kəmˈpraɪz/
C. complete /kəmˈpliːt/
D. common /ˈkɒmən/
Phần gạch chân phương án D được phát âm là /ɒ/, còn lại là /ə/
Câu 10: Đáp án C
Kiến thức: Sự kết hợp từ
Giải chi tiết:
pay attention to sth: chú ý đến cái gì
Các phương án khác:
A. showing: thể hiện, cho thấy
B. using: sử udnjg
D. spending: dành, sử dụng
The art of active listening includes (10) paying close attention to what another person is saying...
Tạm dịch: Nghệ thuật lắng nghe chủ động bao gồm chú ý chặt chẽ đến những gì người khác đang nói...
Câu 11: Đáp án A
Kiến thức: Đại từ quan hệ
Giải chi tiết:
Trong mệnh đề quan hệ
A. what = the things that: cái mà => đứng trước một mệnh đề
B. when: khi mà => dùng để thay thế cho trạng từ chỉ thời gian => ...time, when + S + V
C. which: cái mà => dùng làm chủ ngữ/tân ngữ, thay thế cho danh từ chỉ vật: ...N(thing), which + V / S +
V
D. all: tất cả

Trang 244
... then paraphrasing (11) what you've heard and repeating it back.
Tạm dịch: ...sau đó diễn giải những điều mà bạn đã nghe và lặp lại nó.
Câu 12: Đáp án D
Kiến thức: Từ loại, từ vựng
Giải chi tiết:
Trước động từ “contradict” cần một trạng từ bổ nghĩa
A. intentional (adj): cố tình
B. unintentional (adj): vô tình
C. intentionally (adv): một cách cố tình
D. unintentionally (adv): một cách vô tình
Every conversation comes with a host of non-verbal cues-facial expressions, body language, etc. - that
may (12) unintentionally contradict what we're saying.
Tạm dịch: Mỗi cuộc trò chuyện đều đi kèm với một loạt các tín hiệu phi ngôn ngữ - biểu cảm trên khuôn
mặt, ngôn ngữ cơ thể, v.v - điều đó có thể vô tình mâu thuẫn với những gì chúng ta đang nói.
Câu 13: Đáp án C
Kiến thức: Liên từ, mệnh đề trạng ngữ thời gian
Giải chi tiết:
A. While: Trong khi
B. When: Khi
C. Before: Trước khi
D. After: Sau khi
(13) Before addressing a staff member or leading a project conference, think carefully about your tone of
voice, how you make eye contact, and what your body is "saying".
Tạm dịch: Trước khi nói chuyện với một nhân viên hay dẫn dắt một cuộc họp dự án, hãy suy nghĩ cẩn
thận về giọng nói của bạn, cách bạn giao tiếp bằng mắt và những gì cơ thể bạn đang "nói".
Câu 14: Đáp án A
Kiến thức: Từ vựng
Giải chi tiết:
A. led to sth (v): dẫn đến cái gì
B. opposed sb/sth (v): phản đối ai/cái gì
C. refrained from sth (v): ngăn cản
D. confessed to sb/sth (v): bày tỏ với ai/cái gì
demonstrate that you prefer questions up-front as (14) led to misinterpretation later on.
Tạm dịch: chứng tỏ rằng bạn thích các câu hỏi trực tiếp hơn là phải giải thích sai sau này.
Câu 15: Đáp án D
Kiến thức: Từ vựng, từ đồng nghĩa

Trang 245
Giải chi tiết:
emblem (n): biểu tượng

A. banner (n): biểu ngữ quảng cáo


B. motto (n): khẩu hiệu
C. slogan (n): khẩu hiệu
D. logo (n): biểu tượng
=> emblem = logo: biểu tượng
Tạm dịch: Biểu tượng của Hiệp hội các quốc gia Đông Nam Á được thiết kế dựa trên hình hạt gạo - cây
trồng quan trọng nhất đối với người Đông Nam Á.
Câu 16: Đáp án C
Kiến thức: Thành ngữ
Giải chi tiết:
take flight (v): bỏ trốn
A. did away (v): chấm dứt
B. climbed on (v): trèo lên
C. ran away (v): bỏ trốn
D. took away (v): lấy đi
=> took flight = ran away: bỏ trốn
Tạm dịch: Khi cảnh sát đến, những tên trộm đã bỏ trốn để lại toàn bộ đồ ăn cắp.
Câu 17: Đáp án C
Kiến thức: Liên từ
Giải chi tiết:
A. despite: mặc dù
B. moreover: hơn nữa
C. whereas: trong khi
D. nevertheless: tuy nhiên
Tạm dịch: Ở Anh, phần lớn các cửa hàng đóng cửa lúc 6 giờ chiều, trong khi ở các nước khác, họ thường
mở cửa vào buổi tối nữa.
Câu 18: Đáp án D
Kiến thức: Giới từ
Giải chi tiết:
be exempt from sth: miễn cái gì

Tạm dịch: Không ai nên được miễn truy tố nếu họ phạm tội.
Câu 19: Đáp án B

Trang 246
Phương pháp giải:
Kiến thức: to V/ V_ing, câu bị động
Giải chi tiết:
want + to V_nguyên thể: muốn làm gì
Câu mang nghĩa bị động (bị gọi khi đang ở nhà để đi đến đâu đó) => want to + be + P2
Tạm dịch: Cô Thompson luôn sẵn sàng giúp đỡ, nhưng cô ấy không muốn bị gọi lúc ở nhà nếu không có
trường hợp khẩn cấp.
Câu 20: Đáp án B
Kiến thức: Mệnh đề chỉ mục đích
Giải chi tiết:
in order that + S + V: để mà ai làm gì
so as to + V_nguyên thể: để làm gì
so that + S + V: để ai làm gì
Tạm dịch: Anh ấy đã phải giải thích bài học rất rõ ràng để mà học sinh của anh ấy có thể hiểu nó.
Câu 21: Đáp án C
Kiến thức: Từ vựng
Giải chi tiết:
A. manage (v): xoay sở
B. establish (v): thiết lập
C. achieve (v): đạt được
D. catch (v): bắt, nắm lấy
Tạm dịch: Để đạt được mục tiêu của mình ở đại học, sinh viên cần đầu tư tối đa thời gian, tiền bạc và
năng lượng vào việc học của họ.
Câu 22: Đáp án C
Kiến thức: Sự kết hợp từ
Giải chi tiết:
the extent of damage: mức độ thiệt hại
Các phương án khác:
A. range (n): phạm vi
B. amount (n): lượng (số lượng)
D. quality (n): chất lượng
Tạm dịch: Thật khó để định lượng được toàn bộ mức độ thiệt hại gây ra bởi cơn bão nhiệt đới Sandy, một
trong những cơn bão có tính phá hủy nhất mà nước Mĩ đã chịu trong một thời gian dài.
Câu 23: Đáp án B
Kiến thức: Cụm động từ
Giải chi tiết:

Trang 247
A. put aside (v): để dành
B. taken in (v): lừa gạt
C. taken away (v): lấy đi
D. put up (v): ở nhờ
be taken in by sb: bị lừa gạt bởi ai
Tạm dịch: Đừng để bị lừa bởi những kẻ bán hàng dẻo miệng.
Câu 24: Đáp án B
Kiến thức: Từ vựng
Giải chi tiết:
A. short-changed (v): trả thiếu tiền
B. short-listed (v): sàng lọc, chọn lựa
C. short-sighted (adj): cận thị
D. short-handed (adj): không đủ nhân lực
Tạm dịch: Cô ấy đã được sàng lọc từ 100 ứng viên cho chức vụ giám đốc điều hành.
Câu 25: Đáp án A
Kiến thức: Thành ngữ
Giải chi tiết:
let off steam: xả hơi, thư giãn

Tạm dịch: Một cách để thư giãn sau một ngày căng thẳng là tập vài bài thể dục mạnh mẽ.
Câu 26: Đáp án C
Kiến thức: Mạo từ
Giải chi tiết:
a/an + danh từ chưa xác dịnh
the + danh từ đã xác định, vật mà nó được coi là duy nhất, chỉ có một
“world” (thế giới) là danh từ chỉ vật mà chỉ có một
Tạm dịch: Một phần ba dân số thế giới tiêu thụ hai phần ba tài nguyên của thế giới.
Câu 27: Đáp án B
Kiến thức: Câu điều kiện loại 3
Giải chi tiết:
- Câu điều kiện loại 3 diễn tả một điều kiện không có thực trong quá khứ dẫn đến kết quả trái với sự thật
trong quá khứ.
- Công thức: S + would (not) + have + P2
Tạm dịch: Tôi đã không biết bạn đã ngủ. Nếu không, tôi sẽ không gây ra nhiều tiếng ồn như vậy khi tôi
bước vào.
Câu 28: Đáp án C

Trang 248
Kiến thức: Mệnh đề thời gian mang nghĩa tương lai
Giải chi tiết:
Sau liên từ chỉ thời gian “by the time”, động từ chia ở thì hiện tại đơn hoặc hiện tại hoàn thành (nếu mang
nghĩa tương lai)
Cấu trúc: By the time + S + have/ has P2, S + will + V_nguyên thể: Trước khi làm xong việc gì, ai sẽ làm
gì.
Tạm dịch: Trước khi tôi đánh xong bản báo cáo này, tôi sẽ gọi cho bạn.
Câu 29: Đáp án C
Kiến thức: Mệnh đề phân từ
Giải chi tiết:
Hai mệnh đề cùng chủ ngữ => có thể rút gọn mệnh đề
Câu mang nghĩa chủ động, hành động vế đầu xảy ra trước hành động vế sau
=> rút gọn theo công thức: Having + P2
Tạm dịch: Sau khi đọc câu chuyện về những người mà làm giảm dấu chân carbon, chúng tôi bắt đầu thay
đổi thói quen tiêu thụ hàng ngày.
Câu 30: Đáp án C
Kiến thức: Từ loại, từ vựng
Giải chi tiết:
keep + sb + adj: khiến/giữ cho ai đó như thế nào
A. occupation (n): công việc
B. occupant (n): người ở phòng
C. occupied (adj): bận rộn
D. occupational (adj): liên quan đến công việc
Tạm dịch: Để tránh buồn chán, điều quan trọng nhất là giữ cho bản thân bận rộn.
Câu 31: Đáp án B
Kiến thức: Từ vựng, từ trái nghĩa
Giải chi tiết:
expel (v): trục xuất
A. depart (v): khởi hành
B. welcome (v): chào đón
C. help (v): giúp đỡ
D. facilitate (v): đơn giản hóa
=> expel >< welcome
Tạm dịch: Họ đã bị buộc phải rời đi bởi vì đảng chính phủ yêu cầu trục xuất những người nhập cư bất
hợp pháp.
Câu 32: Đáp án D

Trang 249
Kiến thức: Thành ngữ
Giải chi tiết:
lend color to (v): làm sáng tỏ, chứng minh cái gì (là đúng)
A. provided evidence for (v): cung cấp bằng chứng cho
B. got information from (v): lấy thông tin từ
C. borrowed color from (v): mượn màu từ
D. gave no proof of (v): không đưa ra bằng chứng của
=> lent color to >< gave no proof of
Tạm dịch: Ban đầu, không ai tin cô ấy là phi công, nhưng các tài liệu của cô ấy đã chứng minh cho khẳng
định của cô ấy.
Câu 33: Đáp án D
Kiến thức: Mệnh đề nhượng bộ
Giải chi tiết:
However: Tuy nhiên
before: trước khi
Although: Mặc dù
After: Sau khi
A. Tôi đã nhận ra cô ấy từ một bức ảnh trước khi tôi chưa bao giờ nhìn thấy cô ấy. => sai nghĩa
B. Sai ngữ pháp. Đã có “although” thì không dùng “but”.
C. Sau khi tôi gặp cô ấy, tôi đã nhận ra cô ấy từ một bức ảnh. => sai nghĩa
D. Mặc dù tôi đã chưa từng nhìn thấy cô ấy trước đó, tôi nhận ra cô ấy từ một bức ảnh.
Tạm dịch: Tôi đã chưa từng nhìn thấy cô ấy trước đó. Tuy nhiên, tôi đã nhận ra cô ấy từ một bức ảnh.
= Mặc dù tôi đã chưa từng nhìn thấy cô ấy trước đó, tôi nhận ra cô ấy từ một bức ảnh.
Câu 34: Đáp án B
Kiến thức: Đảo ngữ
Giải chi tiết:
Since + S + V: bởi vì
so + S+ V: vì vậy
So + adj/ adv + be/ V + S that S + V: Cái gì quá ... đến nỗi mà ... (dạng đảo ngữ)
…too + adj + to + V_nguyên thể: …quá ... đến nỗi không thể làm gì
A. Bởi vì các chất rất độc, vì vậy đồ bảo hooj phải được mặc mọi lúc. => sai ngữ pháp vì dùng 2 liên từ
trong cùng 1 câu
B. Các chất thì quá độc đến nỗi mà đồ bảo hộ phải được mặc mọi lúc.
C. Sai ngữ pháp vì sau “such” là một danh từ hoặc cụm danh từ, mà “toxic” là một tính từ: độc, có độc
D. Các chất thì quá độc đến nỗi không thể mặc đồ bảo hộ mọi lúc. => sai nghĩa
Tạm dịch: Các chất rất độc. Đồ bảo hộ phải được mặc mọi lúc.

Trang 250
= Các chất thì quá độc đến nỗi mà đồ bảo hộ phải được mặc mọi lúc.
Câu 35: Đáp án D
Kiến thức: Câu bị động kép
Giải chi tiết:
People + say + that + clause: Người ta đó nói rằng
= It is said that + clause
at least: ít nhất = no fewer than: không ít hơn
A. Mọi người nói rằng ít hơn 10 cuộc phỏng vấn việc làm đã được tổ chức cho đến nay. => sai nghĩa
B. Tôi nghe được rằng chỉ 10 người được chọn để phỏng vấn việc làm. => sai nghĩa
C. Được nói rằng có hơn 10 người quan tâm đến việc có một cuộc phỏng vấn việc làm. => sai nghĩa
D. Được nói rằng có không ít hơn 10 người sẽ được phỏng vấn việc làm.
Tạm dịch: Mọi người nói rằng ít nhất 10 ứng viên được chọn để phỏng vấn việc làm.
= Được nói rằng có không ít hơn 10 người sẽ được phỏng vấn việc làm.
Câu 36: Đáp án B
Kiến thức: Lời nói gián tiếp
Giải chi tiết:
“S’d better + V”: Ai đó nên làm gì
= S + advised + sb + to V: Ai đó đã khuyên ai nên làm gì
A. Giáo viên đã gợi nhắc Jimmy học hành chăm chỉ hơn nếu cậu ấy không muốn thi lại. => sai nghĩa
B. Giáo viên đã khuyên Jimmy nên học hành chăm chỉ hơn nếu cậu ấy không muốn thi lại.
C. Giáo viên đã yêu cầu Jimmy học hành chăm chỉ hơn nếu cậu ấy không muốn thi lại. => sai nghĩa
D. Giáo viên đã đề nghị Jimmy học hành chăm chỉ hơn nếu cậu ấy không muốn thi lại. => sai nghĩa, sai
cấu trúc (suggest + V_ing)
Tạm dịch: “Em nên học tập chăm chỉ hơn nếu em không muốn thi lại.” giáo viên đã nói với Jimmy.
= Giáo viên đã khuyên Jimmy nên học hành chăm chỉ hơn nếu cậu ấy không muốn thi lại.
Câu 37: Đáp án B
Kiến thức: Câu phỏng đoán/ Động từ khuyết thiếu
Giải chi tiết:
S + must + V_nguyên thể: Ai đó chắc hẳn làm gì (ở hiện tại)
S + must + have P2: Ai đó chắc hẳn đã làm gì (trong quá khứ)
S + may + V_nguyên thể: Ai đó có thể làm gì (ở hiện tại)
S + could + have P2: Ai đó có thể đã làm gì (trong quá khứ)
Tạm dịch: Tôi chắc rằng Luisa đã rất thất vọng khi cô ấy trượt kì thi.
= Luisa chắc hẳn đã rất thất vọng khi cô ấy trượt kì thi.
Câu 38: Đáp án D
Kiến thức: Đọc hiểu

Trang 251
Giải chi tiết:
Từ “consolidated” gần nghĩa nhất với ________.
consolidated (v): sáp nhập, hợp nhất
A. harden (v): làm cứng hơn
B. moved (v): chuyển động
C. sold (v): bán
D. merge (v): hợp nhất
=> consolidated = merge
Thông tin: In 1930 he consolidated all the branches of the Bank of Italy, the Bank of America in New
York City, and another bank of America that had formed in California into the Bank of America National
Trust and Savings Association.
Tạm dịch: Năm 1930, ông hợp nhất tất cả các chi nhánh của Ngân hàng Ý, Ngân hàng Hoa Kỳ tại Thành
phố New York và một ngân hàng khác của Hoa Kỳ đã thành lập ở California thành Hiệp hội Tiết kiệm và
Ủy thác Quốc gia của Ngân hàng Hoa Kỳ.
Câu 39: Đáp án D
Kiến thức: Đọc hiểu
Giải chi tiết:
Có thể suy ra từ đoạn văn rằng Giannini đã dùng các thùng cam sau trận động đất _______.
A. để giấu vàng
B. để làm đầy xe kéo
C. để cung cấp nguồn dưỡng cho khách hàng của ông ấy.
D. để bảo vệ vàng khỏi đám cháy
Thông tin: One major test for Giannini’s bank occurred on April 18,1906, when a massive earthquake
struck San Francisco, followed by a raging fire that destroyed much of the city. Giannini obtained two
wagons and teams of horses, filled the wagons with the bank’s reserves, mostly in the form of gold,
covered the reserves with crates of oranges, and escaped from the chaos of the city with his clients’ funds
protected.
Tạm dịch: Một thử nghiệm lớn đối với ngân hàng Giannini, đã xảy ra vào ngày 18 tháng 4 năm 1906, khi
một trận động đất lớn xảy ra ở San Fransico, theo sau là một trận hỏa hoạn đã phá hủy phần lớn thành phố
Giannini có được hai xe kéo và đàn ngựa, chứa đầy các toa xe là dự trữ ngân hàng, chủ yếu dưới dạng
vàng, che kín số dự trữ bằng các thùng cam và thoát khỏi sự hỗn loạn của thành phố với tiền quĩ khách
hàng được bảo vệ của mình.
Câu 40: Đáp án B
Kiến thức: Đọc hiểu
Giải chi tiết:
Theo bài văn, điều nào sau đây không đúng về trận động đất ở San Fransico?

Trang 252
A. Nó đã xảy ra năm 1906.
B. Nó đã xảy ra sau hệ lụy của cuộc hỏa hoạn.
C. Nó đã gây ra nhiều vấn đề cho ngân hàng của Giannini.
D. Nó là một trận động đất lớn.
Thông tin: One major test for Giannini’s bank occurred on April 18,1906, when a massive earthquake
struck San Francisco, followed by a raging fire that destroyed much of the city.
Tạm dịch: Một thử nghiệm lớn đối với ngân hàng Giannini, đã xảy ra vào ngày 18 tháng 4 năm 1906, khi
một trận động đất lớn ập đến San Fransico, theo sau là một trận hỏa hoạn đã phá hủy phần lớn thành phố.
Các đáp án A, C, D đều được đề cập đến.
Câu 41: Đáp án C
Kiến thức: Đọc hiểu
Giải chi tiết:
Đoạn văn tiếp theo của bài văn phần lớn có thể bàn luận về ________.
A. sự thất bại của ngân hàng trong thời kì Đại khủng hoảng
B. cuộc khủng hoảng lớn thứ 3 của Ngân hàng Hoa Kỳ
C. sự phát triển mang tầm quốc tế của Ngân hàng Hoa Kỳ
D. Giannini sử dụng thời gian nghỉ hưu như thế nào
Thông tin: Câu văn cuối trong bài: “Under Giannini’s leadership, the bank weathered the storm of the
Depression and subsequently moved into a phase of overseas development.”
Tạm dịch: Dưới sự lãnh đạo của Giannini, ngân hàng đã vượt qua cơn bão suy thoái và sau đó chuyển
sang giai đoạn phát triển ở nước ngoài.
Câu 42: Đáp án D
Kiến thức: Đọc hiểu
Giải chi tiết:
Từ “its” đề cập đến _______.
A. quán rượu công cộng
B. khách hàng
C. người nhập cư
D. ngân hàng
Thông tin: During its development, Giannini’s bank survived major crises in the form of a natural disaster
and a major economic upheaval that not all other banks were able to overcome.
Tạm dịch: Trong suốt quá trình phát triển của nó, ngân hàng của Giannini đã tồn tại qua các cuộc khủng
hoảng lớn dưới dạng thảm họa thiên nhiên và một biến động kinh tế lớn mà không phải tất cả các ngân
hàng khác đều có thể vượt qua.
Câu 43: Đáp án C
Kiến thức: Đọc hiểu

Trang 253
Giải chi tiết:
Từ “that” ở dòng 8 đề cập đến ______.
A. hốc mắt
B. các khu vực của mắt
C. các tế bào hình nón
D. sự phân biệt thị giác
Thông tin: And in water, humans are farsighted, while the kingfisher, swooping down to spear fish, can
see well in both the air and water because it is endowed with two foveae – areas of the eye, consisting
mostly of cones, that provide visual distinctions.
Tạm dịch: Và khi ở dưới nước, con người thì bị viễn thị, trong khi chim bói cá, sà xuống để mò cá, có thể
nhìn rõ cả không khí và nước bởi vì nó có hai hố mắt – các khu vực của mắt, bao gồm hầu hết các tế bào
hình nón, cái mà cung cấp sự phân biệt thị giác.
Câu 44: Đáp án D
Kiến thức: Đọc hiểu
Giải chi tiết:
Từ “criterion” ở dòng 1 gần nghĩa nhất với ________.
criterion (n): tiêu chí
A. need (n): nhu cầu
B. expectation (n): sự trông đợi
C. rule (n): qui luật
D. standard (n): tiêu chuẩn
=> criterion = standard: tiêu chí
Thông tin: The human criterion for perfect vision is 20/20 for reading the standard lines on a Snellen eye
chart without a hitch.
Tạm dịch: Tiêu chí của con người về tầm nhìn hoàn hảo là 20/20 để đọc các dòng tiêu chuẩn trên biểu đồ
mắt Snellen mà không gặp trở ngại nào.
Câu 45: Đáp án B
Kiến thức: Đọc hiểu
Giải chi tiết:
Theo bài văn, “bug detectors” có ích cho _______.
A. việc định hướng
B. nhìn các vật thể chuyển động
C. tránh các con bọ cánh cứng khi lấy thức ăn
D. tránh việc chết đói
Thông tin: Known as “bug detectors”, a highly developed set of cells in a frog’s eyes responds mainly to
moving objects.

Trang 254
Tạm dịch: Được biết đến như là “máy phát hiện bọ cánh cứng”, một bộ tế bào phát triển cao trong mắt
ếch phản ứng chủ yếu với các vật thể chuyển động.
Câu 46: Đáp án D
Kiến thức: Đọc hiểu
Giải chi tiết:
Theo bài văn, tại sao nhiều loài chim và động vật có thể coi con người thực sự khiếm thị?
A. mắt người không thể nhìn tốt ở trong nước và trên không khí.
B. mắt người không đáp ứng tốt các nhu cầu của chúng ta.
C. điểm nổi trội của mắt người là tầm nhìn màu sắc.
D. mắt người không thể làm được những việc mà mắt chúng (chim & động vật) có thể.
Thông tin: ... the bee is capable of seeing something we can’t – ultraviolet light.
Tạm dịch: ... con ong có khả năng nhìn thấy thứ mà chúng ta không thể - tia cực tím.
Câu 47: Đáp án A
Kiến thức: Đọc hiểu
Giải chi tiết:
Theo bài văn, điều nào sau đây không đúng?
A. chim bói cá có tầm nhìn một mắt
B. ong nhìn thấy các vết chấm tròn
C. mắt của diều hâu gồm phần lớn các tế bào hình nón mà cho phép nó có thể quét bằng một mắt tại một
thời điểm
D. con người bị viễn thị khi ở trong nước
Thông tin:
And in water, humans are farsighted, while the kingfisher, swooping down to spear fish, can see well in
both the air and water because it is endowed with two foveae – areas of the eye, consisting mostly of
cones, that provide visual distinctions. One foveae permits the bird, while in the air, to scan the water
below with one eye at a time.
The bee has a “compound” eye, which is used for navigation. It has 15,000 facets that divide what it sees
into a pattern of dots, or mosaic.
Tạm dịch: Và khi ở dưới nước, con người thì bị viễn thị, trong khi chim bói cá, sà xuống để mò cá, có thể
nhìn rõ cả không khí và nước bởi vì nó có hai hố mắt – các khu vực của mắt, bao gồm hầu hết các tế bào
hình nón, cái mà cung cấp sự phân biệt thị giác. Một hốc mắt cho phép con chim, khi ở trên không, có thể
quét nước bên dưới bằng một mắt.
Con ong có con mắt “kép”, được sử dụng để điều hướng. Nó có 15.000 mắt đơn phân chia những gì nó
nhìn thấy thành một mô hình các chấm hoặc khảm.
Câu 48: Đáp án D
Kiến thức: Đọc hiểu

Trang 255
Giải chi tiết:
Cụm từ “paling into insignificance” gần nghĩa nhất với _______.
paling into insignificance: không còn quan trọng
A. fading away: mờ đi
B. of less importance: ít quan trọng hơn
C. without colored light: không có ánh sáng màu
D. being reduced to little importance: bị làm giảm xuống đến mức không còn quan trọng
=> paling into insignificance = being reduced to little importance: trở nên không còn quan trọng
Thông tin: And if that is not enough to leave our 20/20 “perfect vision” paling into insignificance, the bee
is capable of seeing something we can’t – ultraviolet light.
Tạm dịch: Và nếu điều đó không đủ để khiến tầm nhìn hoàn hảo 20/20 của chúng ta trở nên không còn
quan trọng, con ong có khả năng nhìn thấy thứ mà chúng ta không thể - tia cực tím.
Câu 49: Đáp án A
Kiến thức: Đọc hiểu
Giải chi tiết:
Điều gì có thể suy ra từ đoạn văn?
A. mắt phát triển khác nhau ở mỗi loài
B. ong có con mắt phức tạp nhất
C. con người không nên ghen tị với những gì họ không cần
D. tầm nhìn hoàn hảo thì không hoàn hảo
Câu 50: Đáp án D
Kiến thức: Đọc hiểu
Giải chi tiết:
Đoạn văn chủ yếu bàn luận cái gì?
A. những hạn chế của mắt người
B. tầm nhìn hoàn hảo
C. mắt khác nhau với việc sử dụng khác nhau của các loài
D. sự đa dạng mắt giữa các loài khác nhau

Trang 256
SỞ GD & ĐT VĨNH PHÚC ĐỀ THI KSCĐ LỚP 12 LẦN 1
TRƯỜNG THPT NGÔ GIA TỰ NĂM HỌC: 2019 – 2020
MÔN: TIẾNG ANH
Thời gian làm bài: 60 phút; không kể thời gian phát đề
Mã đề 135
Mark the letter A, B, C, or D on your answer sheet to indicate the underlined part that needs correction in
each of the following questions.
Câu 1 (TH): Food prices have raised so rapidly in the past few months that some families have been
forced to alter their eating habits.
A. have raised B. rapidly C. that D. their eating habits
Câu 2 (TH): Having finished his term paper before the deadline, it was delivered to the professor before
the class.
A. it was delivered B. Having finished C. before D. before the
Câu 3 (NB): Even though the extremely bad weather in the mountains, the climbers decided not to cancel
their climb.
A. Even though B. extremely C. not to cancel D. their climb
Read the following passage and mark the letter A, B, C, or D on your answer sheet to indicate the correct
answer to each of the questions.
long time ago, when ancient Rome was still an empire, people of that time used similar weights and
measures. The standards for these weights and measures were established by the Romans, who kept these
standards in a temple in Rome. All standards for measuring weight or distance were the same, whether in
Spain or in Syria. But then the Roman Empire fell, and these standards disappeared. Today, standards
vary from place to place throughout the world. Tourists who drive from the United States in Canada, for
example, are surprised when they buy gasoline for their cars. A gallon of gas costs more than they are
accustomed to paying. They complain that prices are much higher in Canada than in the United States.
Then they discover that they can drive farther on a Canadian gallon than on a United States gallon. Is it a
different kind of gas? No, it is a different kind of gallon. Canada uses the British, or imperial, gallon that
is about one-fifth larger than the United States gallon. Four quarts equal a gallon and two pints equal a
quart in both countries. But Canada’s quarts and pints are larger than quarts and pints in the United States.
The imperial gallon equals 277.42 cubic inches while the gallon in the United States is equal to 231 cubic
inches. Measured in ounces, Canada’s large gallon holds 160 fluid ounces, while the smaller United
States gallon holds no more than 128 fluid ounces. From these figures, it is easy to see why Americans
can drive farther on the Canadian gallon than on the American gallon. Someday, countries may follow the
example of the ancient Romans and make weights and measures the same for every nation.
Câu 4 (TH): Why are American tourists surprised?
A. The price of a Canadian gallon of gas seems high.

Trang 257
B. They weigh more in Canada than in the United States.
C. Canada has a shortage of gasoline.
D. They didn’t know Canada sold gasoline.
Câu 5 (NB): Canada has a different kind of _______.
A. gallon B. gasoline C. tourist D. driver
Câu 6 (TH): Which statement does the article lead you to believe?
A. Americans should not travel so much.
B. No one in Rome ever went to a temple
. C. It would be good to use the same measures everywhere.
D. Canada should reduce their price of gasoline.
Câu 7 (TH): gallon of gas in Canada costs more _______.
A. than the imperial British like B. than American tourists afford
C. than the ancient Romans charged D. than Americans are used to paying
Câu 8 (TH): The word “empire” in the first line refers to ______.
A. a tribe of residents B. a kind of a society C. a country D. a measurement
Mark the letter A, B, C, or D on your answer sheet to indicate the correct answer to each of the following
questions.
Câu 9 (NB): Let's have a party to celebrate our third successive win, ______?
A. don’t we B. will we C. shall we D. do we
Câu 10 (TH): After the flash flood, all the drains were overflowing _______ storm water.
A. from B. for C. by D. with
Câu 11 (TH): Her room is very large. She is dreaming of a _______.
A. wooden big round table B. round big wooden table
C. big round wooden table D. table big wooden round
Câu 12 (NB): Janet: “Do you like going to the cinema this evening?”
Susan: “_______.”
A. You’re welcome B. I feel very bored
C. That would be great D. I don’t agree, I’m afraid
Câu 13 (TH): Marta: What did you think of the book?
Jones: __________ the books I’ve read. It was the most interesting.
A. All B. From all C. All of D. Of all
Câu 14 (VD): Our boss would rather _______ during the working hours.
A. we didn’t chat B. we don’t chat C. us not chat D. us not chatting
Câu 15 (TH): The robber was made _________ where he had hidden the money.
A. confess B. confessing C. to confess D. to confessing
Câu 16 (TH): The fire was caused by an ________ fault in the television.

Trang 258
A. electricity B. electric C. electrician D. electrical
Câu 17 (TH): _______ that we all went for a picnic.
A. So fine the weather B. So fine was the weather
C. Such a fine weather was it D. It was such a fine weather
Câu 18 (VDC): He may be shy now, but he will soon come out of his _______ when he meets the right
girl.
A. shed B. shell C. shoe D. hole
Câu 19 (TH): He eventually _______ his disability to achieve some business success.
A. suffered B. overwhelmed C. destroyed D. overcame
Câu 20 (TH): Anna: “Were you involved in the accident?”
Maria: “Yes. But I wasn’t to ___________ for it”
A. blame B. apologize C. accuse D. charge
Câu 21 (TH): The sky was cloudy and foggy. We went to the beach, _________.
A. so B. yet C. however D. even though
Câu 22 (VD): The old houses were______ down to make way for a block of flats.
A. hit B. knocked C. banged D. put
Câu 23 (TH): If you ______ to my advice in the first place, you wouldn’t be in this mess now.
A. listen B. had listened C. listened D. will listen
Mark the letter A, B, C, or D on your answer sheet to indicate the sentence that is CLOSEST in meaning
to each of the following questions.
Câu 24 (VDC): “If you don’t pay the ransom, we’ll kill your boy” the kidnappers told us.
A. The kidnappers ordered to kill our boy if we did not pay the ransom.
B. The kidnappers pledged to kill our boy if we did not pay the ransom.
C. The kidnappers threatened to kill our boy if we refused to pay the ransom.
D. The kidnappers promised to kill our boy if we refused to pay the ransom.
Câu 25 (VD): “Don’t forget to tidy up the final draft before submission,” the team leader told us.
A. The team leader reminded us to tidy up the final draft before submission.
B. The team leader asked us to tidy up the final draft before submission.
C. The team leader simply wanted us to tidy up the final draft before submission.
D. The team leader ordered us to tidy up the final draft before submission.
Câu 26 (VDC): “You shouldn’t have leaked our confidential report to the press, Frank!” said Jane.
A. Jane suspected that Frank had leaked their confidential report to the press.
B. Jane accused Frank of having cheated the press with their confidential report.
C. Jane blamed Frank for having flattered the press with their confidential report.
D. Jane criticized Frank for having disclosed their confidential report to the press.

Trang 259
Câu 27 (VDC): “My company makes a large profit every year. Why don’t you invest more money in it?”
my friend said to me.
A. My friend instructed me how to put more money into his company.
B. I was asked to invest more money in my friend’s company.
C. My friend persuaded me to invest more money in his company.
D. My friend suggested his investing more money in his company.
Câu 28 (VDC): “Mum, please don’t tell dad about my mistake,” the boy said.
A. The mother was forced to keep her son’s mistake as a secret when he insisted.
B. The boy begged his mother not to tell his father about his mistake.
C. The boy earnestly insisted that his mother tell his father about his mistake.
D. The boy requested his mother not to talk about his mistake any more.
Mark the letter A, B, C, or D on your answer sheet to indicate the word(s) CLOSEST in meaning to the
underlined word(s) in each of the following questions.
Câu 29 (VD): Whenever problems come up, we discuss them frankly and find solutions quickly.
A. happen B. clean C. arrive D. encounter
Câu 30 (TH): Unselfishness is the very essence of friendship.
A. difficult part B. romantic part C. important part D. interesting part
Mark the letter A, B, C, or D on your answer sheet to indicate the word whose underlined part differs
from the other three in pronunciation in each of the following questions.
Câu 31 (NB): A. appalled B. forged C. noticed D. composed
Câu 32 (NB): A. honest B. vehicle C. hour D. happy
Read the following passage and mark the letter A, B, C, or D on your answer sheet to indicate the correct
word or phrase that best fits each of the numbered blanks.
Ask most people for their list of top ten of fears, and you’ll be sure to find (33) _______ burgled is fairly
high on the list. An informal survey I carried among friends at a party last week revealed that eight of
them had had their homes (34) ______ into more than twice, and two had been burgled five times. To put
the record straight, (35) _______ of my friend owns valuable paintings or a sideboard full of family
silverware. Three of them are students, in fact. The most typical burglar, it seems, involved the theft of
easily transportable items - the television, the video, even food from freezer.
This may have something to do with the (36) _______ that the average burglar is in his (or her) late teens,
and probably wouldn’t know what to do with a Picasso, whereas selling a Walkman or a vacuum cleaner
is a much easier matter. They are perhaps not so (37) ______ professional criminals, as hard-up young
people who need a few pounds and some excitement. (38) _____ that this makes your house turned
upside down and your favorite things stolen any easier to (39) ______. In most case, the police have no
luck recovering any of the stolen goods. Unless there is any definite (40) ______, they are probably
unable to do anything at all. And alarms or special locks don’t seem to help either. The only advice my

Trang 260
friends could (41) ______ was “Never live on the ground floor” and “Keep two or three fierce dogs”,
which reminded me of a case, I read about, where the burglars’ loot included the family’s pet poodle.
Câu 33 (TH): A. having B. being C. out D. been
Câu 34 (VD): A. taken B. broken C. robbed D. entered
Câu 35 (TH): A. all B. few C. some D. none
Câu 36 (TH): A. knowledge B. fact C. idea D. information
Câu 37 (VD): A. rarely B. much C. many D. that
Câu 38 (VD): A. given B. not C. despite D. so
Câu 39 (TH): A. attempt B. do C. believe D. accept
Câu 40 (TH): A. case B. burglary C. evidence D. investigation
Câu 41 (VD): A. put in with B. get by with C. bring up with D. come up with
Mark the letter A, B, C, or D on your answer sheet to indicate the word(s) OPPOSITE in meaning to the
underlined word(s) in each of the following questions.
Câu 42 (TH): He was one of the most outstanding performers at the live show last night.
A. humble B. easy-looking C. well-known D. impressive
Câu 43 (TH): Advanced students need to be aware of the importance of collocation.
A. Of high level B. Of low level C. Of great importance D. Reputation
Read the passage and mark the letter A, B, C, or D on your answer sheet to indicate the correct answer to
each of the questions.
The sea has always interested people. From it they can get food, minerals, and treasures. For thousands of
years, they sailed on the sea. But they could not go far beneath its surface. People want to explore deep
into the sea. However, they are not fishes. Because people must breathe air, they cannot stay under the
water’s surface for any length of time. To explore deep water, men and women face even more dangers
and problems. A diver who wants to stay under water for more than a few minutes must breathe air or a
special mixture of gases. Divers can wear diving suits and have air pumped to them from above. They can
carry tanks of air on their back and breathe through a hose and a mouthpiece. Water weighs 800 times as
much as air. Tons of water push against a diver deep in the sea. The diver’s body is under great pressure.
When divers are under great pressure, their blood takes in some of the gases they breathe. As they rise too
quickly, the gases in their blood form bubbles. Divers then suffer from the bends. The bends can cause
divers to double up in pain. Bends can even kill divers.
Câu 44 (VD): Which statement does the story lead you to believe?
A. Humans are not at home deep in the sea.
B. All divers get the bends sooner or later.
C. The sea began to interest humans in the last few years.
D. Diving suits do not help divers.
Câu 45 (VD): The story does not say so, but it makes you think that _______.

Trang 261
A. deep-sea divers should be in good health
B. diving under the water too quickly causes the bends
C. divers can wear diving suits to avoid the bends
D. divers explore the deep seas only for treasure
Câu 46 (TH): diver’s body in deep water ________.
A. is just like a fish’s body B. suffers from the bends
C. is under great pressure D. weighs very little
Câu 47 (TH): Why do drivers get the bends?
A. They come to the surface too quickly. B. They try to do exercises under the water.
C. Their diving suits weigh too much. D. Their blood takes some of the gases they breathe.
Câu 48 (NB): The gases in a diver’s blood can form ______.
A. great pressure B. oxygen C. bubbles D. air
Mark the letter A, B, C, or D on your answer sheet to indicate the word that differs from the rest in the
position of the primary stress in each of the following questions.
Câu 49 (NB): A. control B. contrary C. contour D. constant
Câu 50 (NB): A. evaluate B. investigate C. exception D. category

Trang 262
Đáp án
1-A 2-A 3-A 4-A 5-A 6-C 7-D 8-B 9-C 10-D
11-C 12-C 13-D 14-A 15-C 16-D 17-B 18-B 19-D 20-A
21-C 22-B 23-B 24-C 25-A 26-D 27-C 28-B 29-A 30-C
31-C 32-D 33-B 34-B 35-D 36-B 37-B 38-B 39-D 40-C
41-D 42-A 43-B 44-A 45-A 46-C 47-D 48-C 49-A 50-D

LỜI GIẢI CHI TIẾT


Câu 1: Đáp án A
Kiến thức: Động từ
Giải chi tiết:
“have raised” là ngoại động từ => cần có một tân ngữ phía sau
Sau nội động từ (rise) không cần tân ngữ
Sửa: “have raised” => “have risen”
Tạm dịch: Giá thức ăn đã tăng quá nhanh trong vài tháng cuối đây đến nỗi mà một vài gia đình đã buộc
phải thay đổi thói quen ăn uống của họ.
Câu 2: Đáp án A
Kiến thức: Mệnh đề rút gọn
Giải chi tiết:
Hai mệnh đề cùng chủ ngữ => có thể rút gọn mệnh đề
Chủ ngữ “his term paper” (bài kiểm tra có hạn nộp), động từ “finish” (hoàn thành) => câu bị động
Câu mang nghĩa bị động, hành động vế đầu xảy ra trước hành động vế sau
=> rút gọn theo công thức: Having + been + P2
Sửa: “Having finished” => “Having been finished”
Tạm dịch: Bài thi của ông ấy được hoàn thành trước thời hạn, nó đã được chuyển tới giáo sư trước giờ
học.
Câu 3: Đáp án A
Kiến thức: Mệnh đề nhượng bộ
Giải chi tiết:
Even though + S + V: Mặc dù ai làm gì
= Despite/ In spite of + N/ N phrase: Mặc dù cái gì
“the extremely bad weather in the mountains” là cụm danh từ
Sửa: “Even though” => “Despite” hoặc “In spite of”
Tạm dịch: Mặc dù thời tiết trên núi cực kì xấu, những người leo núi đã quyết định không hủy cuộc leo núi
của họ.
Câu 4: Đáp án A

Trang 263
Kiến thức: Đọc hiểu
Giải chi tiết:
Tại sao du khách Mĩ ngạc nhiên?
A. Giá của một bình ga ở Canada có vẻ cao.
B. Họ nặng hơn khi ở Canada so với khi ở Mĩ.
C. Canada thiếu xăng dầu.
D. Họ đã không biết Canada bán xăng dầu.
Thông tin: Tourists who drive from the United States in Canada, for example, are surprised when they
buy gasoline for their cars. A gallon of gas costs more than they are accustomed to paying.
Tạm dịch: Ví dụ những khách du lịch ở Canada mà đến từ Mĩ thì ngạc nhiên khi họ mua xăng dầu cho xe
hơi của mình. Một bình xăng đắt hơn nhiều số tiền họ đã quen trả.
Câu 5: Đáp án A
Kiến thức: Đọc hiểu
Giải chi tiết:
Canada có một loại ______ khác.
A. can
B. xăng dầu
C. du khách
D. tài xế
Thông tin: No, it is a different kind of gallon.
Tạm dịch: Không, đó là một kiểu can khác.
Câu 6: Đáp án C
Kiến thức: Đọc hiểu
Giải chi tiết:
Câu văn nào mà bài báo khiến bạn tin nó?
A. Người Mĩ không nên đi lại quá nhiều.
B. Không ai ở Rome đã từng đến một ngôi đền.
C. Sẽ tốt khi sử dụng cùng một kiểu đo lường ở mọi nơi.
D. Canada nên giảm giá xăng dầu.
Thông tin: Someday, countries may follow the example of the ancient Romans and make weights and
measures the same for every nation.
Tạm dịch: Một ngày nào đó, nhiều đất nước có thể theo gương La Mã cổ đại và thực hiện việc cân và đo
lường giống nhau cho mỗi quốc gia.
Câu 7: Đáp án D
Kiến thức: Đọc hiểu
Giải chi tiết:

Trang 264
Một can dầu ở Canada đắt hơn nhiều _______.
A. so với vương quốc Anh
B. so với chi tiêu của du khách Mĩ
C. so với La Mã cổ đại đã nạp
D. so với người Mĩ quen trả
Thông tin: A gallon of gas costs more than they are accustomed to paying.
Tạm dịch: Một bình xăng đắt hơn nhiều số tiền họ đã quen trả.
Câu 8: Đáp án B
Kiến thức: Đọc hiểu
Giải chi tiết:
Từ “empire” ở dòng đầu tiên đề cập đến ______.
empire (n): đế chế, một kiểu xã hội
A. một bộ tộc của cư dân
B. một kiểu xã hội
C. một đất nước
D. một đơn vị đo lường
Câu 9: Đáp án C
Kiến thức: Câu hỏi đuôi
Giải chi tiết:
Vế trước có “Let’s + V_nguyên thể” => câu hỏi đuôi sẽ là “shall we”
Tạm dịch: Chúng ta hãy tổ chức một bữa tiệc để kỉ niệm chiến thắng thứ ba liên tiếp của mình, được
không?
Câu 10: Đáp án D
Kiến thức: Giới từ
Giải chi tiết:
A. from (prep): từ
B. for (prep): cho
C. by (prep): bởi
D. with (prep): với
overflow with something: tràn ra ùng với cái gì
Tạm dịch: Sau cơn lũ qua nhanh, tất cả nước cống tràn ra cùng với nước bão.
Câu 11: Đáp án C
Kiến thức: Trật tự tính từ
Giải chi tiết:
Vị trí của tính từ trước danh từ:
Size (kích cỡ) - big + Shape (hình dáng) - round + Material (chất liệu) - wooden + Noun (danh từ) – table

Trang 265
=> big round wooden table
Tạm dịch: Phòng của cô ấy rất rộng. Cô ấy đang ước có một cái bàn to tròn bằng gỗ.
Câu 12: Đáp án C
Kiến thức: Ngôn ngữ giao tiếp
Giải chi tiết:
Tạm dịch:
Janet: “Bạn có thích đi xem phim tối nay không?”
Susan: “_______.”
A. Không có gì. (Đáp lại lời cảm ơn của ai đó)
B. Tôi cảm thấy rất chán. (Đáp lại câu hỏi cảm giác)
C. Thế thì thật tuyệt.
D. Tôi e rằng tôi không đồng ý. (Đáp lại quan điểm, ý kiến về việc gì đó)
Các phản hồi A, B, D không phù hợp ngữ cảnh.
Câu 13: Đáp án D
Kiến thức: Ngôn ngữ giao tiếp
Giải chi tiết:
Tạm dịch:
Marta: “Bạn nghĩ gì về quyển sách?”
Jones: “______ quyển sách tôi đã đọc, nó là quyển thú vị nhất.”
A. Tất cả
B. Từ tất cả
C. Tất cả
D. Trong tất cả
Các phương án A, B, C không phù hợp.
Câu 14: Đáp án A
Kiến thức: Thức giả định
Giải chi tiết:
Công thức: S + would rather + S + V_ed: Ai đó muốn ai làm gì ở hiện tại.
Tạm dịch: Ông chủ của chúng tôi muốn chúng tôi không nói chuyện trong suốt giờ làm việc.
Câu 15: Đáp án C
Kiến thức: to V/ V_ing
Giải chi tiết:
be made + to V_nguyên thể: bị bắt làm gì
Tạm dịch: Tên cướp đã bị bắt phải khai ra nơi hắn ta đã giấu tiền.
Câu 16: Đáp án D
Kiến thức: Từ loại, từ vựng

Trang 266
Giải chi tiết:
Trước danh từ “fault” (lỗi sai, sự trục trặc) cần một tính từ
A. electricity (n); điện
B. electric (adj): dùng bằng điện
C. electrician (n): thợ điện
D. electrical (adj): liên quan đến điện
Tạm dịch: Vụ cháy bị gây ra bởi sự trục trặc liên quan đến điện ở ti vi.
Câu 17: Đáp án B
Kiến thức: Đảo ngữ
Giải chi tiết:
So + adj/ adv + be/V + S + that + S + V
= Such + (a/an) + N + be that + S + V: Cái gì quá... đến nỗi mà...
Danh từ “weather” (thời tiết) không đếm được => không dùng với “a”
Tạm dịch: Thời tiết quá tốt đến nỗi mà tất cả chúng tôi đã đi dã ngoại.
Câu 18: Đáp án B
Kiến thức: Thành ngữ
Giải chi tiết:
“come out of one’s shell”: trở nên tự tin
Tạm dịch: Bây giờ anh ấy có thể ngại ngùng, nhưng anh ấy sẽ nhanh chóng trở nên tự tin khi anh ấy gặp
đúng cô gái.
Câu 19: Đáp án D
Kiến thức: Từ vựng
Giải chi tiết:
A. suffered (v): chịu đựng
B. overwhelmed (v): phấn khích
C. destroyed (v): phá hủy
D. overcame (v): vượt qua
Tạm dịch: Anh ấy cuối cùng đã vượt qua khiếm khuyết của mình để đạt được thành công trong kinh
doanh.
Câu 20: Đáp án A
Kiến thức: Từ vựng
Giải chi tiết:
A. blame for … (v): chịu trách nhiệm
B. apologize for … (v): xin lỗi
C. accuse of … (v): buộc tội
D. charge of …. (v): buộc tội

Trang 267
Tạm dịch:
Anna: “Bạn đã liên quan đến vụ tai nạn à?”
Maria: “Đúng. Nhưng tôi đã không phải chịu trách nhiệm về nó”
Câu 21: Đáp án C
Kiến thức: Liên từ
Giải chi tiết:
A. so + S + V: vì vậy
B. yet: tuy nhiên => đứng ở giữa câu/đầu câu hoặc cuối câu (khi đứng ở cuối câu, trước nó không có dấu
phẩy)
C. however: tuy nhiên => có thể đứng đầu, giữa và cuối câu, trước nó có dấu phẩy
D. even though + S + V: mặc dù
Tạm dịch: Bầu trời nhiều mây và sương mù. Tuy nhiên chúng tôi đã đến bãi biển.
Câu 22: Đáp án B
Kiến thức: Từ vựng
Giải chi tiết:
A. hit: đâm, đánh
B. knocked => knocked down: đánh đổ, dỡ bỏ
C. banged: đập
D. put => put down: chấm dứt
Tạm dịch: Những ngôi nhà cũ đã bị dỡ bỏ để lấy chỗ cho một dãy các căn hộ.
Câu 23: Đáp án B
Kiến thức: Câu điều kiện hỗn hợp loại 3+2
Giải chi tiết:
- Dấu hiệu: now (bây giờ), wouldn’t + V_nguyên thể ở MĐ chính => MĐ chính thuộc câu điều kiện loại
2
- Cách dùng: diễn tả điều kiện không có thực trong quá khứ dẫn đến kết quả trái với hiện tại
- Công thức: S + had(not) + P2, S would(not) + V_nguyên thể + now.
Tạm dịch: Nếu bạn đã nghe lời khuyên của tôi ở ngay từ đầu, bây giờ bạn sẽ không ở trong hoàn cảnh hỗn
độn này.
Câu 24: Đáp án C
Kiến thức: Lời nói gián tiếp
Giải chi tiết:
If S + don’t + V_nguyên thể, S + would + V_nguyên thể: Nếu ai đó không làm gì, người kia sẽ làm gì
= S + threatened + O + to V_nguyên thể + if + S + V_ed: Ai đó đã đe dọa ai làm gì nếu họ (không) làm gì
A. Những tên bắt cóc đã đề nghị giết con trai chúng tôi nếu chúng tôi không trả tiền chuộc. => sai nghĩa
B. Những tên bắt cóc đã van nài giết con trai chúng tôi nếu chúng tôi không trả tiền chuộc. => sai nghĩa

Trang 268
C. Những kẻ bắt cóc đã đe dọa giết con trai chúng tôi nếu chúng tôi từ chối trả tiền chuộc.
D. Những tên bắt cóc đã hứa sẽ giết con trai chúng tôi nếu chúng tôi không trả tiền chuộc. => sai nghĩa
Tạm dịch: “Nếu các người không trả tiền chuộc, chúng ta sẽ giết con trai các ngươi” những tên bắt cóc đã
nói với chúng tôi.
= Những kẻ bắt cóc đã đe dọa giết con trai chúng tôi nếu chúng tôi từ chối trả tiền chuộc.
Câu 25: Đáp án A
Kiến thức: Lời nói gián tiếp
Giải chi tiết:
“Don’t + forget + to V_nguyên thể”: Đừng quên làm gì
= S + reminded + S + to V_nguyên thể: Ai đó đã nhắc nhở ai làm gì
A. Đội trưởng nhắc chúng tôi chỉn chu bản thảo cuối trước khi nộp.
B. Đội trưởng yêu cầu chúng tôi chỉn chu bản thảo cuối trước khi nộp. => sai nghĩa
C. Đội trưởng đơn giản muốn chúng tôi chỉn chu bản thảo cuối trước khi nộp. => sai nghĩa
D. Đội trưởng yêu cầu chúng tôi chỉn chu bản thảo cuối trước khi nộp. => sai nghĩa
Tạm dịch: “Đừng quên chỉn chu bản thảo cuối cùng trước khi nộp” đội trưởng đã nói với chúng tôi.
= Đội trưởng nhắc chúng tôi chỉn chu bản thảo cuối trước khi nộp.
Câu 26: Đáp án D
Kiến thức: Lời nói gián tiếp
Giải chi tiết:
“S + shouldn’t have P2”: Ai đó lẽ ra không nên làm gì
= S + criticized + S + for + having P2: Ai đó đã chỉ trích ai vì đã làm gì
A. Jane đã nghi ngờ rằng Frank đã tiết lộ báo cáo mật của họ với chủ biên. => sai nghĩa
B. Jane đã buộc tội Frank vì đã tiết lộ báo cáo mật của họ với chủ biên. => sai nghĩa
C. Jane đã đổ lỗi cho Frank vì tiết lộ báo cáo mật của họ với chủ biên. => sai nghĩa
D. Jane đã chỉ trích Frank vì đã tiết lộ báo cáo mật của họ với chủ biên.
Tạm dịch: “Bạn lẽ ra không nên tiết lộ báo cáo mật của chúng ta với chủ biên, Frank” Jane đã nói.
= Jane đã chỉ trích Frank vì đã tiết lộ báo cáo mật của họ với chủ biên.
Câu 27: Đáp án C
Kiến thức: Lời nói gián tiếp
Giải chi tiết:
“Why don’t you + V_nguyên thể?”: Tại sao bạn không làm gì?
= S + persuaded + S + to V_nguyên thể: Ai đó đã thuyết phục ai làm gì
A. Bạn của tôi đã hướng dẫn tôi làm sao để đổ nhiều tiền vào công ty của anh ấy. => sai nghĩa
B. Tôi được yêu cầu để đầu tư nhiều tiền hơn vào công ty bạn của tôi. => sai nghĩa
C. Bạn của tôi đã thuyết phục tôi đầu tư nhiều tiền vào công ty của anh ấy.
D. Bạn của tôi đã đề nghị việc đầu tư nhiều tiền hơn vào công ty của anh ấy. => sai nghĩa

Trang 269
Tạm dịch: “Công ty của tôi đạt lợi nhuận rất lớn mỗi năm. Tại sao bạn không đầu tư vào nó?” bạn của tôi
đã nói với tôi.
= Bạn của tôi đã thuyết phục tôi đầu tư nhiều tiền vào công ty của anh ấy.
Câu 28: Đáp án B
Kiến thức: Lời nói gián tiếp
Giải chi tiết:
“Please, don’t + V_nguyên thể”: Làm ơn đừng làm gì
= S + begged + S + (not) to V_nguyên thể: Ai đó đã van nài ai (đừng) làm gì
A. Người mẹ bị buộc phải giữ lỗi lầm của con trai mình như là một bí mật khi cậu bé đã nài nỉ. => sai
nghĩa
B. Cậu bé đã van nài mẹ mình không nói với bố về lỗi lầm của cậu ta.
C. Cậu bé đã khăng khăng rằng mẹ của mình nói với bố về lỗi của cậu ta. => sai nghĩa
D. Cậu bé đã yêu cầu mẹ của mình không nói về lỗi lầm của cậu ta nữa. => sai nghĩa
Tạm dịch: “Mẹ, làm ơn đừng nói với bố về lỗi lầm của con” cậu bé đã nói.
= Cậu bé đã van nài mẹ mình không nói với bố về lỗi lầm của cậu ta.
Câu 29: Đáp án A
Kiến thức: Cụm động từ
Giải chi tiết:
come up: xuất hiện, nảy sinh
A. happen (v): xảy ra
B. clean (v): dọn dẹp
C. arrive (v): đến
D. encounter (v): đối đầu
=> come up = happen: xảy ra, nảy sinh
Tạm dịch: Bất cứ khi nào vấn đề phát sinh, chúng tôi thảo luận chúng một cách thẳng thắn và tìm ra giải
pháp nhanh chóng.
Câu 30: Đáp án C
Kiến thức: Từ vựng, từ đồng nghĩa
Giải chi tiết:
essence (n): sự quan trọng
A. difficult part: phần khó khan
B. romantic part: phần lãng mạn
C. important part: phần quan trọng
D. interesting part: phần thú vị
=> essence = important part: phần quan trọng
Tạm dịch: Sự không ích kỉ là một phần thực sự quan trọng của tình bạn.

Trang 270
Câu 31: Đáp án C
Kiến thức: Phát âm “ed”
Giải chi tiết:
A. appalled /əˈpɔːld/
B. forged /fɔːdʒd/
C. noticed /ˈnəʊtɪst/
D. composed /kəmˈpəʊzd/
Quy tắc:
Cách phát âm đuôi “-ed”:
- Đuôi “-ed” được phát âm là /ɪd/ khi động từ có phát âm kết thúc là /t/ hay /d/
- Đuôi “-ed” được phát âm là /t/ khi động từ có phát âm kết thúc là /s/,/f/,/p/,/ʃ/,/tʃ/,/k/
- Đuôi “-ed” được phát âm là /d/ với các trường hợp còn lại
Phần gạch chân phương án C được phát âm là /t/, còn lại là /d/
Câu 32: Đáp án D
Kiến thức: Phát âm “h”
Giải chi tiết:
A. honest /ˈɒnɪst/
B. vehicle /ˈviːəkl/
C. hour /ˈaʊə(r)/
D. happy /ˈhæpi/
Phần gạch chân phương án D được phát âm là /h/, còn lại âm “h” câm
Câu 33: Đáp án B
Kiến thức: Câu bị động
Giải chi tiết:
Dạng chủ động: find + V_ing
Dạng bị động: find + being P2
Ask most people for their list of top ten of fears, and you’ll be sure to find (33) being burgled is fairly
high on the list.
Tạm dịch: Hỏi phần lớn mọi người về danh sách mười điều họ sợ nhất, và bạn sẽ chắc chắn tìm ra bị trộm
đột nhập thì xếp khá cao trong danh sách.
Câu 34: Đáp án B
Kiến thức: Cụm động từ
Giải chi tiết:
A. taken => take into: đổ vào
B. broken => be broken into: bị đột nhập
C. robbed: cướp

Trang 271
D. entered: đi vào
An informal survey I carried among friends at a party last week revealed that eight of them had had their
homes (34) broken into more than twice, and two had been burgled five times.
Tạm dịch: Một cuộc khảo sát bình thường tôi đã thực hiện giữa những người bạn ở trong một bữa tiệc
tuần trước đã tiết lộ rằng tám người trong số họ đã để nhà của mình bị đột nhập nhiều hơn hai lần, và hai
người đã bị đột nhập năm lần.
Câu 35: Đáp án D
Kiến thức: Lượng từ
Giải chi tiết:
Động từ “owns” chia ngôi thứ 3 số ít
all + of + N_số nhiều + V_nguyên thể: tất cả …
few + N số nhiều + V_nguyên thể: vài (nhưng rất ít, dường như không có)
some + N_số nhiều + V_nguyên thể: một vài
none + of + N_số ít + V (s, es): không …
To put the record straight, (35) none of my friend owns valuable paintings or a sideboard full of family
silverware.
Tạm dịch: Để làm sáng tỏ điều này, không người bạn nào của tôi sở hữu những bức họa quý giá hay một
tủ đồ đầy những dụng cụ gia đình bằng bạc.
Câu 36: Đáp án B
Kiến thức: Từ vựng
Giải chi tiết:
A. knowledge (n): kiến thức
B. fact (n): thực tế
C. idea (n): ý kiến
D. information (n): thông tin
This may have something to do with the (36) fact that the average burglar is in his (or her) late teens
Tạm dịch: Việc này có thể có một vài thứ liên quan tới thực tế rằng các tên trộm trung bình ở độ tuổi cuối
thành niên của anh (cô) ấy...
Câu 37: Đáp án B
Kiến thức: Từ loại
Giải chi tiết:
Trước tính từ “professional” cần một trạng từ
Câu đã có “not” mang nghĩa phủ định => không dùng “rarely”
They are perhaps not so (37) much professional criminals, as hard-up young people who need a few
pounds and some excitement.

Trang 272
Tạm dịch: Họ có lẽ không phải là những tội phạm quá chuyên nghiệp, bởi vì những người trẻ túng thiếu
mà cần một vài đồng lẻ và một chút phấn khích.
Câu 38: Đáp án B
Kiến thức: Từ vựng
Giải chi tiết:
A. given => given that: cân nhắc rằng
B. not => not that: không hẳn là
C. despite => despite te fact that: mặc dù
D. so => so that: để mà
(38) not that this makes your house turned upside down and your favorite things stolen any easier ...
Tạm dịch: Không hẳn là việc khiến nhà bạn trở nên lộn xộn và những đồ vật bạn yêu thích bị trộm trở nên
dễ dàng hơn ...
Câu 39: Đáp án D
Kiến thức: Từ vựng
Giải chi tiết:
A. attempt (v): cố gắng
B. do (v): làm
C. believe (v): tin
D. accept (v): chấp nhận
...this makes your house turned upside down and your favorite things stolen any easier to (39) accept.
Tạm dịch: ...điều này khiến nhà bạn trở nên lộn xộn và những đồ vật bạn yêu thích bị trộm trở nên dễ
dàng hơn để chấp nhận.
Câu 40: Đáp án C
Kiến thức: Từ vựng
Giải chi tiết:
A. case (n): vụ án
B. burglary (n): vụ trộm
C. evidence (n): chứng cứ
D. investigation (n): cuộc điều tra
Unless there is any definite (40) evidence, they are probably unable to do anything at all.
Tạm dịch: Nếu không có bất kì chứng cứ nào rõ ràng, họ có lẽ không thể làm bất cứ điều gì hết.
Câu 41: Đáp án D
Kiến thức: Cụm động từ
Giải chi tiết:
A. put in with: chen vào
B. get by with: xoay sở

Trang 273
C. bring up with: bắt đầu nói đến
D. come up with: nghĩ ra, nảy ra ý nghĩ gì
The only advice my friends could (41) come up with was “Never live on the ground floor” and “Keep two
or three fierce dogs”, ...
Tạm dịch: Lời khuyên duy nhất mà bạn bè tôi có thể nghĩ là “Không bao giờ sống ở tầng trệt” và “Nuôi
hai hoặc ba con chó dữ”, ...
Câu 42: Đáp án A
Kiến thức: Từ vựng, từ trái nghĩa
Giải chi tiết:
outstanding (adj): nổi bật
A. humble (adj): khiêm tốn
B. easy-looking (adj): ưa nhìn
C. well-known (adj): nổi tiếng
D. impressive (adj): ấn tượng
=> outstanding >< humble
Câu 43: Đáp án B
Kiến thức: Từ vựng, từ trái nghĩa
Giải chi tiết:
Advanced (adj): nâng cao, giỏi
A. Of high level: Có trình độ cao
B. Of low level: Có trình độ thấp
C. Of great level: Có trình độ cao
D. Reputation (n): Danh tiếng
=> Advanced >< Of low level
Câu 44: Đáp án A
Kiến thức: Đọc hiểu
Giải chi tiết:
Câu văn nào câu chuyện khiến bạn tin?
A. Loài người không ở nhà sâu trong lòng biển.
B. Tất cả thợ lặn rồi cũng sẽ mắc bệnh khí ép.
C. Biển đã bắt đầu gây hứng thú cho con người trong vài năm gần đây.
D. Các bộ đồ lặn không giúp thợ lặn.
Thông tin: But they could not go far beneath its surface.
Tạm dịch: Nhưng họ không thể đi xa vào bên dưới bề mặt của nó.
Câu 45: Đáp án A
Kiến thức: Đọc hiểu

Trang 274
Giải chi tiết:
Câu chuyện không nói như vậy, nhưng nó khiến bạn nghĩ rằng ______.
A. các thợ lặn sâu dưới lòng biển nên có sức khỏe tốt
B. lặn dưới nước quá nhanh gây ra bệnh ép khí
C. thợ lặn có thể mặc bộ đồ lặn để tránh bệnh ép khí
D. thợ lặn khám phá biển sâu chỉ vì kho báu
Thông tin: People want to explore deep into the sea. However, they are not fishes. Because people must
breathe air, they cannot stay under the water’s surface for any length of time. To explore deep water, men
and women face even more dangers and problems.
Tạm dịch: Con người muốn khám phá sâu trong lòng biển. Tuy nhiên, chúng không phải là cá. Bởi vì con
người phải hít thở không khí, họ không thể ở dưới mặt nước trong khoảng thời gian dài. Để khám phá
vùng nước sâu, đàn ông và phụ nữ phải đối mặt với nhiều nguy hiểm và vấn đề hơn.
Câu 46: Đáp án C
Kiến thức: Đọc hiểu
Giải chi tiết:
Một cơ thể thợ lặn ở nước sâu ______.
A. thì chỉ giống như cơ thể cá
B. chịu đựng bệnh ép khí
C. thì chịu áp lực lớn
D. nặng không đáng kể
Thông tin: The diver’s body is under great pressure.
Tạm dịch: Cơ thể thợ lặn phải chịu áp lực rất lớn.
Câu 47: Đáp án D
Kiến thức: Đọc hiểu
Giải chi tiết:
Tại sao thợ lặn mắc bệnh ép khí?
A. Họ chạm mặt nước quá nhanh.
B. Họ cố tập thể dục dưới nước.
C. Bộ đồ lặn của họ quá nặng
D. Máu của họ nhận một vài khí khi họ thở.
Thông tin: When divers are under great pressure, their blood takes in some of the gases they breathe. As
they rise too quickly, the gases in their blood form bubbles. Divers then suffer from the bends.
Tạm dịch: Khi thợ lặn chịu áp lực lớn, máu của họ sẽ nhận một số khí mà họ thở. Khi chúng tăng quá
nhanh, các khí trong máu tạo thành bong bóng. Thợ lặn sau đó phải chịu bệnh khí ép.
Câu 48: Đáp án C
Kiến thức: Đọc hiểu

Trang 275
Giải chi tiết:
Khí trong máu thợ lặn có thể hình thành _____.
A. áp lực lớn
B. oxy
C. bong bóng
D. không khí
Thông tin: As they rise too quickly, the gases in their blood form bubbles.
Tạm dịch: Khi chúng tăng quá nhanh, các khí trong máu tạo thành bong bóng.
Câu 49: Đáp án A
Kiến thức: Trọng âm của từ có 2, 3 âm tiết
Giải chi tiết:
A. control /kənˈtrəʊl/
B. contrary /ˈkɒntrəri/
C. contour /ˈkɒntʊə(r)/
D. constant /ˈkɒnstənt/
Phương án A trọng âm rơi vào âm tiết thứ 2, còn lại rơi vào âm thứ nhất
Câu 50: Đáp án D
Kiến thức: Trọng âm của từ có 3, 4 âm tiết
Giải chi tiết:
A. evaluate /ɪˈvæljueɪt/
B. investigate /ɪnˈvestɪɡeɪt/
C. exception /ɪkˈsepʃn/
D. category /ˈkætəɡəri/
Phương án D trọng âm rơi vào âm tiết thứ nhất, còn lại là âm thứ 2

Trang 276

You might also like